Friendliest 1700 by DR - Khaled

You might also like

Download as docx, pdf, or txt
Download as docx, pdf, or txt
You are on page 1of 151

Friendliest 1700 files explained by Dr. Khalid Ans. The key is B. TFT. [The patient has 6.

has 6.A 67yo man after a stroke, presents with left


Saifullah (Question 1 to question 1707) Compiled paroxysmal atrial fibrillation That is why there is sided ptosis and constricted pupil. He also has
By Dr Malik & team. no arrhythmia in between attacks. From the given lossof pain and temp on the right side of his
option TFT is the appropriate test as body and left side of his face. Which part of the
New corrections [1. Q. 1226; 2. Q. 805; 3. Q. 197; thyrotoxycosis is a leading cause of paroxysmal brain ismost likely affected?
4. Q. 998; 5. Q. 1118; 6. Q. 687; 7. Q. 1635; 8. Q. atrial fibrillation and this ladies weight loss also a. Frontal cortex
1517; 9. Q. 1609; 10. Q. 232 11. Q. 1395; 12. Q. makes thyrotoxycosis as the probable cause b. Cerebellum
1397; 13. Q. 264; 14. Q. 332; 15. Q. 1525] ?1491? here]. c. Pons
1.A 65yo man presents with painless hematuria, d. Medulla
IVU is normal, prostate is mildly enlarged with 4.A 79yo anorexic male complains of thirst and e. Parietal cortex
mildfrequency. What is the most appropriate fatigue. He has symptoms of frequency, urgency
next step? and terminal dribbling. His urea and creatinine Ans. The key is D. Medulla. [The name of the
a. US Abdomen levels are high. His serum calcium is 1.9 and he is condition is “Lateral medullary syndrome”
b. Flexible cystoscopy anemic. His BP is 165/95 mmHg. What is the most [ipsilateral Horner syndrome and contralateral
c. MRI probable dx? loss of pain and temperature sense].
d. Nuclear imaging a. BPH
e. PSA b. Prostate carcinoma 7.A 60yo man presents with dysphagia and pain
Ans. The key is B. Flexible cystoscopy. [Painless c. Chronic pyelonephritis
on swallowing both solids and liquids. A barium
hematuria in an elderly (here 65 years old man) d. Benign nephrosclerosis
meal shows gross dilatation of the esophagus with
indicates carcinoma bladder for which flexible a smooth narrowing at the lower end of the
cystoscopy is done. Ans. The key is B. Prostate Carcinoma.
esophagus. What is the SINGLE most likely cause
Here BEP is not advanced to cause hemorrhage. Explanation for Question no. 4:
of dysphagia?
There is mild enlargement of prostate and mild First to say in this case (almost all features goes in
a. Achalasia
symptoms of prostration and hemorrhage is favour of prostatic carcinoma like- frequency,
b. Myasthenia gravis
unlikely at this initial stage of BEP which makes urgency and terminal dribbling are features of
c. Esophageal carcinoma
Bladder cancer as the likely cause of painless prostatism; Age, anorexia and anaemia are
d. Esophageal web
hematuria. constitutional features of carcinoma prostate and
e. Systemic sclerosis
It is also less likely to be prostate cancer as it would be accurate presentation if it was
Ans. The key is A. Achalasia. [Dysphagia for both
symptoms of prostration are mild (indicates hypercalcaemia. But given calcium level is of
solid and liquid or prominently liquid suggest
disease is not advanced). Moreover bleeding in hypocalcaemic level and it is the main cause of
achalasia where dysphagia to solid suggest
cancer prostate is less common]. discrepancy of this question). Renal failure can be
stricture. Also gross dilatation of oesophagus with
an association of malignant disease and can cause
smooth narrowing at lower end is seen in
2.A 74yo smoker presented to his GP with cough high BP. Thirst is a feature of hypercalcaemia
achalasia. In achalasia dysphagia is usually
and SOB. Exam revealed pigmentation of the (here may be erroneously calcium level is given in
described as progressive].
oral hypocalcaemic level ; probably a bad recall).
mucosa and also over the palms and soles. Tests Prostate biopsy is the confirmatory diagnosis and
8.A man undergoes a pneumonectomy. After
show that he is diabetic and hypokalemic. others like PSA is suggestive.
surgery, invs show hyponatremia. What could
What is the most probable dx? ***There are some suggestion that Renal Failure
be the
a. Pseudocushing syndrome may be the cause of hypocalcemia.
cause of the biochemical change?
b. Conns disease
5.A 64yo man has recently suffered from an MI a. Removal of hormonally active tumor
c. Ectopic ACTH
b. Excess dextrose
d. Cushings disease and is on aspirin, atorvastatin and ramipril. He
c. Excess colloid
e. Hypothyroidism has
d. Excessive K+
Ans. The key is C. Ectopic ACTH. [The patient is been having trouble sleeping and has been losing
e. Hemodilution
smoker and probably developed squamous cell weight for the past 4 months. He doesn’t feel
lung cancer which is working as a tumour like doing anything he used to enjoy and has
Ans. The key is A. Removal of harmonically active
producing ectopic ACTH causing pigmentation. stopped socializing. He says he gets tired easily
tumour. [Ectopic ACTH secreting tumour causes
Resulting raised cortisole is leading to diabetes and
hypernatremia and body's homeostatic
and hypokalemia (though small cell carcinoma is can’t concentrate on anything. What is the most
mechanism try to lower the level of high sodium
usual cause but squamous cell carcinoma can appropriate tx?
and do a lesser degree though sodium remains in
produce ectopic ACTH as paraneoplastic a. Lofepramine
hypernatremic level or even it may be normal
syndrome also)]. b. Dosulepin
(this question does not mention any preoperative
c. Citalopram
hypernatremia). Removal of that tumour results
3.A 44yo woman has lost weight over 12 months. d. Fluoxetine
in negative sodium balance for time being which
She has also noticed episodes where her heart e. Phenelzine
results hyponatremia while gradually it tends to
beats rapidly and strongly. She has a regular pulse Ans. The key is C. Citalopram. [Among SSRIs
rise again to normal level].
rate of 90bpm. Her ECG shows sinus rhythm. Sertraline is the drug of choice for ischemic heart
What is the most appropriate inv to be done? disease. Next choice is citalopram (as it is often
9.A pregnant lady came with pain in her calf
a. Thyroid antibodies related to torsades de pointes it is not 1st choice).
muscle with local rise in temp to the antenatal
b. TFT If SSRI cannot be used Mirtazapine is
clinic.
c. ECG recommended as next antidepressant].
What tx should be started?
d. Echocardiogram
a. Aspirin
e. Plasma glucose
b. LMWH c. Varicella
c. Paracetamol d. Rubella 16. A 22yo man has a reduced conscious level and
d. Cocodamol e. Measles a fixed dilated pupil after being involved in a MVC.
e. Aspirin and heparin Ans. The key is A. Shingles. [Here ophthalmic Choose the single most appropriate option?
Ans. The key is B. LMWH. [Injections with low division of trigeminal nerve is involved. Typically a. Facial nerve
molecular weight heparin (LMWH) are usually shingles are unilateral]. b. Oculomotor nerve
used to treat pregnant women with DVT. LMWH c. Olfactory nerve
is an anticoagulant, which means it prevents the 13. A 29yo lady who is a bank manager is referred d. Optic nerve
blood clot getting bigger. It does not affect the by the GP to the medical OPC due to a long hx of e. Trigeminal nerve
developing baby (www.nhs.uk)].  tiredness and pain in the joints. An autoimmune Ans. The key is B. Oculomotor nerve. [3rd nerve
screen result showed smooth muscle damage can cause fixed dilated pupil].
10. A 53yo female presents with an acute painful antibodies positive. What is the most appropriate
hot knee joint. She is a known case of RA. On next inv? 17. A man with suspected active TB wants to be
examination, the knee is red, tender and swollen. a. ECG treated at home. What should be done to prevent
The hamstring muscles are in spasm. Her b. TFT the spread of disease?
temp is 38.5C and BP is 120/80mmHg. What is the c. LFT a. Immediate start of the tx with Anti-TB drugs
SINGLE best next inv? d. Serum glucose b. All family members should be immediately
a. Joint aspiration for cytology and culture and e. Jejunal biopsy vaccinated with BCG vaccine
sensitivity Ans. The key is C. LFT. [A case of autoimmune c. Patient should be isolated in a negative
b. Joint aspiration for positive birefrengent hepatitis. Autoimmune hepatitis is an uncommon pressure chamber in his house
crystals cause of chronic hepatitis which if untreated can d. Universal prevention application protocol
c. Joint aspiration for negative birefrengent lead to cirrhosis. However with treatment outlook Ans. The key is D. Universal prevention application
crystals is very good. Smooth muscle antibody is positive protocol.
d. Blood culture in autoimmune hepatitis.Definitive investigation is
e. Serum uric acid liver biopsy. Treated with steroid [start with high 18. A 7yo child is brought to the ED with a 1 day
Ans. The likely key is A. Joint aspiration for dose prednisolone]. Azathioprine is commonly hx of being listless. On examination, the child is
cytology and culture and sensitivity. [Case of added with steroid to reduce its dose as steroid drowsy with an extensive non-blanching rash.
septic arthritis. Any chronically arthritic joint is has more side effects than azathioprine]. What advice would you give the parents?
predisposed to infection. Moreover chronic use of a. All family members need antibiotic therapy
steroid in Rh. arthritis is one of the important 14. A 5yo with recurrent chest pain, finger b. Only the mother should be given rifampicin
predisposing factor. In this age group likely clubbing with offensive stool. Choose the single prophylaxis
organism is Staphylococcus. In younger age group most c. All family members need isolation
Neisseria gonorrhea is more common]. likely inv? d. All family members should be given rifampicin
a. Endomyseal/Alpha glidin antibody prophylaxis
11. An 80yo man presented with pain in his b. Sweat test Ans. The key is D. All family members should be
lower back and hip. He also complains c. Barium meal given rifampicin prophylaxis. [Meningococcal
of waking up in d. ECG disease. Diagnosis is done with blood or CSF PCR.
the night to go to the washroom and has urgency e. Glucose tolerance test Initial prehospital management: Benzyl penicillin
as well as dribbling. What is the most likely dx? Ans. The key is B. Sweat test. [Recurrent chest or cefotaxime].
a. BPH pain from frequent lung infections including
b. Prostatitis pneumonia or bronchitis."Clubbing" of the fingers 19. A 47yo man has a temp of 39C and is delirious.
c. UTI is a classic features of Cystic Fibrosis, although not He has developed blisters mainly on his trunk,
d. Prostate carcinoma present in many patients. The digestive enzymes which appeared a few hours ago. He is well and
e. Bladder carcinoma are not being produced, food is not adequately not on any medications. He last travelled 5
Ans. The key is D. Prostate carcinoma. [Age, digested (malabsorption) and excess fat and months ago to Italy. Which of the following is the
nocturia, urgency and dribbling points towards protein is lost in the stools, making them bulky, most likely dx?
prostate pathology. Pain of lower back and hip oily, smelly and difficult to flush away]. a. Shingles
points towards bony metastases from prostate b. Chicken pox
cancer.Blood test for PSA; Prostate biopsy; MRI (if 15. A clinical picture of breast cancer originated c. Pemphigoid
initial biopsy is negative, to decide repeat biopsy). from the mammary duct. Biopsy was done and d. Bullous pemphigus
Treatment options: 1. Active treatment [i) radical there were neoplastic cells found. Choose the Ans. The key is B. Chicken pox. [Centripetal
prostatectomy ii) radical radiotherapy iii) histological picture of the cancer. distribution of blisters favours chickenpox. Adults
hormone therapy iv) brachytherapy v) pelvic a. Neoplastic cells are arranged in small clusters more commonly develop a more generalized
radiotherapy vi) orchidectomy occupying a space between collagen bundles brain inflammation ("encephalitis") whose
2. Active surveillance (Seirrhous carcinoma) symptoms may include delirium and seizures.
3. Watchful waiting b. Spindle cell neoplasms with margins, which Incubation period of chicken-pox is 10-21 days. So
4. Palliative care (Source: NICE)]. infiltrate adjacent structure, fat invaded (Breast this travel history is not significant].
sarcoma)
12. An 18yo female has peri-orbital blisters. Some c. Small cells with round nucleus and scant 20. A 64yo pt has been having freq episodes of
of them are crusted, others secreting pinkish fluid. indistinct cytoplasm (Lobular carcinoma) secretory diarrhea, which is extremely watery,
What is the most likely dx? Ans. The key is C. Small cells with round nucleus with large amounts of mucus. A dx of villous
a. Shingles and scant indistinct cytoplasm (Lobular adenoma was made after endoscopy. What
b. Chicken pox carcinoma) electrolyte
abnormality is most likely in this pt? 25.A resident of a nursing home presented with a. Median nerve
a. Hyperkalemia rashes in his finger webs and also on his b. Ulnar nerve
b. Hypernatremia abdomen, c. Radial nerve
c. Hyponatremia with complaints of itching which is severe at d. Long thoracic nerve
d. Hypokalemia night. He was dx with scabies. What the best tx e. Axillary nerve
Ans. Key not given. Correct key is both C and D! for Ans. The key is B. Ulner nerve. [Compression of
[Villous adenoma can cause both hyponatremia his condition? ulner nerve at the elbow, known as cubital tunnel
and hypokalemia]. a. 0.5% permethrin syndrome, causes numbness in the 5th (pinky)
b. Doxycycline finger, along the half (lengthwise) of the 4th (ring)
21. A pt with an acute gout attack came to the ED. c. 5% permethrin finger closest to the 5th finger, and the back half
What drug should be given to relieve symptoms? d. Reassure of the hand over the 5th finger].
a. NSAIDs e. Acyclovir 30. A young man complains of double vision on
b. Allopurinol Ans. The key is C. 5% permethrin. [Scabies seeing to the right. Which nerve is most likely to
c. Ibuprofen outbreaks in nursing homes and cases of crusted be
Ans.The key is A. NSAIDs. [Oral NSAIDs scabies may require combination therapy involved?
commenced immediately and continue for 1 – 2 consisting of topical application of permethrin and a. Left abducens
weeks; Colchicine can be effective alternative but 2 oral doses of ivermectin at 200 mcg/kg b. Right abducens
is slower to work than NSAIDs. Intra articular (administered 1 wk apart)]. c. Left trochlear
corticosteroids are highly effective in acute gouty 26. A 34yo alcoholic is found passed out in front d. Right trochlear
monoarthritis. of a local pub. The ambulance crew informs you e. Right oculomotor
22.A pt was lying down on the operating table in a that he was sweating when they found him and Ans. The key is B. Right abducens. [Diplopia on
position with his arms hanging down for 3 hours. there were cans of cider lying empty around him. seeing to right indicates right lateral rectus palsy
Soon after he woke up, he complains of numbness What is the initial stage of inv? which is supplied by right abducent nerve].
and weakness in that hand and has limited a. Capillary blood sugar 31. A 45yo man keeps having intrusive thoughts
wrist movement/wrist drop and sensory loss over b. CT head about having dirt under the bed. He can’t keep
dorsum of that hand, weakness of extension c. MRI head himself from thinking about these thoughts. If he
of the fingers and loss of sensation at the web of d. ABG tries to resist, he starts having palpitations.
the thumb. What structure is likely to be e. MCV What is the most likely dx?
damaged? Ans. The key is A. Capillary blood sugar. [Alcohol a. OC personality
a. Radial nerve induced hypoglycemia can present as this case]. b. OCD
b. Median nerve 27. A young boy fell on his outstretched hand and c. Schizophrenia
c. Ulnar nerve has presented with pain around the elbow. Hehas d. Panic disorder
d. Axillary nerve absent radial pulse on the affected hand. What is e. Phobia
e. Suprascapular nerve the most likely dx? Ans. The key is B. OCD. [Here patients thoughts
Ans. The key is A. Radial nerve. [Here arm hanging a. Dislocated elbow are obsession and though no compulsive act is
down compressing the radial nerve at the spiral b. Angulated supracondylar fx described (like repeated cleansing of dirt) but his
groove is the cause of given scenario]. c. Undisplaced fx of radial head nature of thought like inability to resist the
23.A pt who was previously on 120mg slow d. Posterior dislocation of shoulder thinking or getting palpitation on trying to avoid
release oral morphine has had his dose increased Ans. The key is B. Angulated supracondyllar fx. thinking can be regarded as compulsion of
to [Damage or occlusion of the bracheal artery is the thought. CBT 1st line. SSRIs].
200mg. He is still in significant pain. He complains cause of absent radial pulse.Often closed
of drowsiness and constipation. What is the reduction results in restoration of normal 32. A 33yo man presents with an itchy scaly
next step in the management? anatomy and correction of occlusion of bracheal annular rash on his thigh after a walk in the park.
a. Increase slow release morphine dose artery and establishes circulation again but in few Which
b. Fentanyl patch instances open reduction is required to fix the of the following drugs will treat his condition?
c. Replace morphine with oral hydromorphone occluded artery]. a. Erythromycin
d. Replace morphine with oxycodone 28. A 65yo woman presented with transient arm b. Doxycycline
e. Subcutaneous morphine and leg weakness as well as a sudden loss of c. Penicillin
Ans. The key is D.Replace morphine with vision d. Amoxicillin
oxycodone. in the left eye. Her symptoms resolved within the Ans. The key is B. Doxycycline. [Itchy scaly annular
24. A 40yo woman notices increasing lower next couple of hours. What is the most rash after a walk in the park indicates erythema
abdominal distention with little/no pain. On appropriate next inv? migrans caused by the spirochete Borrelia
examination, a lobulated cystic mass is felt and it a. CT brain Burgdorferi transmitted by bite of pinhead-sized
seems to be arising from the pelvis. What is b. Echo ixodes ticks leading to lyme disease].
the most appropriate inv? c. Doppler USG
a. CA 125 d. Arteriography 33. A pt with cerebral mets has polyuria and
b. CA 153 e. 24h ECG polydipsia. What part of the brain would be
c. CA 199 Ans. The key is c. [A case of TIA. Probable cause affected?
d. CEA carotid artery atherosclerosis]. a. Cerebral cortex
e. AFP 29. A man complains of loss of sensation in his b. Cerebellum
Ans. The key is A. CA 125. [Ovarian ca is the likely little and ring finger. Which nerve is most likely to c. Diencephalon
diagnosis for which tumour marker is CA 125]. be d. Pons
involved? e. Medulla
Ans. The key is C. Diencephalon. [Diencephalon is 37. A 3yo child has been brought with facial Ans. The key is C. Gilbert’s syndrome. [Only
the caudal (posterior) part of the forebrain, lacerations. On examination he has some cuts bilirubin is increased but not the liver enzymes.
containing the epithalamus, thalamus, over his Also positive nicotinic acid provocation test is in
hypothalamus, and ventral thalamus and the third right cheek and under the eye. The GCS on initial its favour].
ventricle. Hypothalamus produce ADH and hens evaluation is 15. What is the appropriate next
lesion of diencephalon (hypothalsamus) may inv? 41.A 24yo biker has been rescued after being
produce cranial diabetes insipidus. a. Skull XR trapped under rocks for almost 12h. He complains
b. Facial XR of
34. A 32yo man presented with painless c. CT scan reddish brown urine. His creatinine is 350umol/L
hematuria. He is hypertensive but the rest of the d. MRI and his urea is 15mmol/L. What is the most
exam is e. Observation imp step in the management of this patient?
unremarkable. What is the most likely dx? Ans. The key is B. Facial X-ray. [Normal GCS makes a. Dialysis
a. Polycystic kidneys intracranial lesion less likely. As there is facial b. IV NS
b. Ca bladder injury to exclude any facial bone fracture we can c. IV dextrose
c. Ca prostate do facial X-ray]. d. IV KCl
d. TTP e. Pain relief
e. HUS  38. A 73yo woman has lymphadenopathy and Ans. Key is B. IV NS. [It is a case of rhabdomyolysis
Ans. The key is A. Polycystic kidneys. [Painless splenomegaly. She feels well but has had which is initially treated with IV NS].
haematuria at an younger age with hypertension recurrent
is suggestive of polycystic kidney disease. Renal chest infections recently. Choose the single most 42. A 74yo man who has been a smoker since he
ultrasound is used to diagnose the condition]. likely blood film findings? was 20 has recently been dx with SCLC. What
a. Atypical lymphocytes serum electrolyte picture will confirm the
35. A 45yo female complains of pain in the inner b. Excess of mature lymphocytes presence of SIADH?
side of her right thigh. She was dx with benign c. Plasma cells a. High serum Na, low serum osmolarity, high
ovarian mass on the right. Which nerve is d. Multiple immature granulocytes with blast cells urine osmolarity
responsible for this pain? e. Numerous blast cells b. Low serum Na, low serum osmolarity, high
a. Femoral nerve Ans. The key is B. Excess of mature lymphocytes. urine osmolarity
b. Obturator nerve [Dx is CLL. Age of patient (usually above 50 yrs), c. Low serum Na, high serum osmolarity, high
c. Iliohypogastric nerve lymhadenopathy and splenomegaly, appearance urine osmolarity
d. Ovarian branch of splanchic nerve of lymphocytes (mature lymphocytes – but d. High serum Na, low serum osmolarity, low
e. Pudendal nerve functionally not normal). Repeated chest infection urine osmolarity
Ans. The key is B. Obturator nerve. [The Obturator points towards abnormal function of lymphocytes e. High serum Na, high serum osmolarity, low
nerve is responsible for the sensory innervation of against infection]. urine osmolarity
the skin of the medial aspect of the thigh]. Ans. . The key is B. Low serum Na, low serum
39. A lady presents with itching around the breast osmolarity, high urine osmolarity.
36. A 37yo lady strongly believes that a famous and greenish foul smelling discharge from the
politician has been sending her flowers every day nipple. She had a similar episode before. What is 43. A man brought into the ED after being stabbed
and is in love with her. However, this is not the the most likely dx? in the chest. Chest is bilaterally clear with muffled
case. What is the most likely dx? a. Duct papilloma heart sounds. BP is 60/nil. Pulse is 120bpm. JVP
a. Erotomania b. Duct ectasia raised. What is the most likely dx?
b. Pyromania c. Breast abscess a. Pulmonary embolism
c. Kleptomania d. Periductal mastitis b. Cardiac tamponade
d. Trichotillomania e. Mammary duct fistula c. Pericardial effusion
e. Grandiosity Ans. The key is B. Duct ectasia. [Duct ectasia of d. Hemothorax
Ans. 1. The key is A. Erotomania. [Erotomania is a the breast or mammary duct ectasia or plasma e. Pneumothorax
type of delusion in which the affected person cell mastitis is a condition in which the lactiferous Ans. The key is B. Cardiac tamponade. [chest is
believes that another person, usually a stranger, duct becomes blocked or clogged.This is the most clear, so there is no pneumothorax or pleural
high-status or famous person, is in love with common cause of greenish discharge. Mammary effusion. Muffled heart sound is due to fluid in
them]. duct ectasia can mimic breast cancer. It is a pericardial space, low BP from reduced chamber
Pyromania is an impulse control disorder in which disorder of peri- or post-menopausal age]. expansion due to pericardial fluid’s pressure and
individuals repeatedly fail to resist impulses to restricted right heart expansion causes raised
deliberately start fires, in order to relieve tension 40. A young male whose sclera was noted to be JVP].
or for instant gratification. yellow by his colleagues has a hx of taking OTC
Kleptomania is the inability to refrain from the drugs for some pain. Tests showed raised 44. A 50yo pt is admitted for elective
urge to steal items. bilirubin, ALT and AST normal. The provocation herniorraphy. Which of the following options will
Trichotillomania is an impulse disorder test with IV nicotinic acid is positive and produces lead to a postponement of the operation?
characterized by the compulsive urge to pull further rise in the serum bilirubin levels. What is a. SBP 110mmHg
out one's hair, leading to noticeable hair loss the most likely dx? b. MI 2 months ago
and balding. a. Acute hepatitis c. Hgb 12g/dl
Grandiosity refers to an unrealistic sense of b. Drug hypersensitivity d. Pain around hernia
superiority. c. Gilberts syndrome e. Abdominal distention
d. Acute pancreatitis Ans. The key is B. MI 2 months ago. [After MI
elective surgery should not be done before 6
months post MI, as operation in earlier than this Management: 1st line: Evacuation by barr hole [discrepancy like no previous infection is the only
time has significant increase in mortality]. craniostomy. 2nd line: Craniotomy if the clot is point in favour of congenital here and may be due
organized]. to a bad recall!!!]
45. A 32yo woman of 39wks gestation attends the
antenatal day unit feeling very unwell with 49. A 25yo female complains of intermittent pain 52. A female with T1DM would like to know about
sudden onset of epigastric pain associated with in her fingers. She describes episodes of a deficiency of vitamins in pregnancy that can
nausea and vomiting. Her temp is 36.7C. Exam: numbness and burning of the fingers. She wears be harmful. A deficiency of which vitamin can lead
she is found to have RUQ tenderness. Her blood gloves whenever she leaves the house. What is to teratogenic effects in the child?
results show mild anemia, low platelets, the most probable dx? a. Folic acid
elevated liver enzymes and hemolysis. What is the a. Kawasaki disease b. Vit B12
most likely dx? b. Takayasu arteritis c. Thiamine
a. Acute fatty liver of pregnancy c. Buerger’s disease d. Riboflavine.
b. Acute pyelonephritis d. Embolism e. Pyridoxine
c. Cholecystitis e. Raynaud’s phenomenon Ans. The key is A. Folic acid. [Frequently
d. HELLP syndrome associated with neural tube defect].
e. Acute hepatitis Ans. The key is E. Raynaud’s phenomenon. 53. A 23yo woman has been having pain at the
Ans. The key is D. HELLP syndrome. [The main [Intermittent nature points towards some triggers base of her thumb, the pain is reproduced when
treatment is to deliver the baby as soon as and wearing of gloves during going out indicates lifting her 3 month old baby or changing diapers
possible [as early as after 34 weeks if multisystem cold weather. Also female sex makes the and also with forceful abduction of the thumb
disease is present]. diagnosis of Raynaud’s phenomenon more likely]. against resistance. What is the likely cause?
a. Avascular necrosis of scaphoid
46. A woman comes with an ulcerated lesion 3 cm 50. A 22yo lady has been unwell for some time. b. Trigger finger.
in the labia majorum. What is the lymphatic She came to the hospital with complaints of fever c. De Quervain’s tenosynovitis
drainage of this area? and painful vesicles in her left ear. What is the
a. External iliac most probable dx? Ans. The key is c. De Quervain’s tenosiovitis. [Can
b. Superficial inguinal LN a. Acne be diagnosed by Finkelstein’s test:
c. Para-aortic b. Herpes zoster The physician grasps the thumb and the hand is
d. Iliac c. Chicken pox ulnar deviated sharply. If sharp pain occurs along
e. Aortic d. Insect bite the distal radius (top of forearm, about an inch
Ans. Key is B. Superficial inguinal LN. e. Cellulitis below the wrist), de Quervain's syndrome is
47. A man post-cholecystectomy presented with Ans. The key is B. Herpes Zoster. [This is a case of likely].
jaundice, fever and dark urine. What is the most Herpes zoster oticus and if facial nerve is also
diagnostic inv? involved then it is called Ramsay Hunt syndrome]. 54. A 6m child presents with fever and cough. His
a. ERCP 51. A 5yo girl had earache and some yellowish mother has rushed him to the ED asking for help.
b. USG Abdomen foul smelling discharge, perforation at the attic Exam: temp=39C and the child is feeding poorly.
c. CT Scan and Dx?
d. MRCP conductive hearing loss. She has no past hx of any a. Bronchiolitis
e. MRI ear infections. What is the most appropriate b. Asthma
Ans. The key is A. ERCP [Post operative US will not dx? c. Bronchitis
give good results. We shall not go for ercp first as a. Acute OM Ans.The key is A. Bronchiolitis. [Management: 1.
it has complications like injury and pancreatitis. b. OM with effusion Oxygen inhalation 2. Nasogastric feeding. DON’T
Acceptable options are CT, MRI and MRCP among c. Acquired cholesteatoma USE: i) bronchodilator ii) steroid iii) antibiotics
which most easiest and less time consuming but d. Congenital cholesteatoma routinely. [OHCS, 9th edition, page-160]
with very good test result is CT scan. So CT is most e. Otitis externa
appropriate! But as the Question wants most 55. A 75yo man collapsed while walking in his
diagnostic it is ERCP (though not practical)!!! The Ans. The key is c. Acquired cholesteatoma. garden. He recovered fully within 30 mins with BP
diagnosis here is choledocolithiasis with [Acquired cholesteatomas develop as a result of 110/80 mmHg and regular pulse of 70bpm. He has
cholangitis]. chronic middle ear infection and are usually a systolic murmur on examination. His
48. A 79yo stumbled and sustained a minor head associated with perforation of the tympanic activities have been reduced lately which he
injury 2 weeks ago. He has become increasingly membrane at the attic (mass is seen in attick with attributes to old age. What is the definitive
confused, drowsy and unsteady. He has a GCS of perforation at pars flaccida- in contrast to medial diagnostic inv that will assist you with his
13. He takes warfarin for Afib. What is the most to tympanic membrane which is in congenital). condition?
likely dx? Clinical presentation usually consists of a. ECG
a. Extradural hemorrhage conductive hearing loss, often with purulent b. Echo
b. Cerebellar hemorrhage discharge from the ear].  c. 24h ECG monitoring
c. Epidural hemorrhage d. 24h BP monitoring
d. Subdural hemorrhage In congenital: e. Prv CIN
e. Subarachnoid hemorrhage • mass medial to the tympanic membrane Ans. B. Echo. [2 mainD/D. Aortic stenosis and
Ans. 1. The key is D. Subdural hematoma. [In • normal tympanic membrane hypertrophic cardiomyopathy. Aortic stenosis –
elderly head injury usually leads to subdural • no previous history of ear discharge, perforation more likely in elderly. And hypertrophic
hematoma even if head injury is minor or trivial or ear surgery. cardiomyopathy – less likely in this age].
and extradural hematoma in elderly is extremely
uncommon even in more severe head injury.
56. A 35yo man with a hx of schizophrenia is of trunk up to the umbilicus. Joint position sense e. Herpes zoster
brought to the ER by his friends due to is impaired at her left great toe but is normal Ans. 1. Key is b. Trigeminal neuralgia. [Electric
drowsiness. On elsewhere. She has a definite left extensor plantar pains in her facethat starts with the jaw and
examination he is generally rigid. A dx of response and the right plantar response is moves upwards [this description indicates
neuroleptic malignant syndrome except: equivocal. Where is the lesion? neurological pain of trigeminal nerve. In
a. Renal failure a. Left cervical cord trigeminal neuralgia transient loss of corneal
b. Pyrexia b. Midline mid-thoracic cord reflexs are seen just after attacks but in between
c. Elevated creatinine kinase c. Right mid-thoracic cord attacks corneal reflexes are quite normal].
d. Usually occurs after prolonged tx d. Left mid-thoracic cord
e. Tachycardia e. Left lumbo-sacral plexus 63. A 32yo man presented with slow progressive
dysphagia. There is past hx of retro-sternal
Ans. The key is D. Usually occurs after prolonged Q. 1. What is the key? discomfort and he has been treated with
tx. [Malignant neuroleptic syndrome usually Q. 2. What is the name of this condition? prokinetics and H2 blockers. What is the probably
occurs SOON after start or in increasing dose of dx?
antipsychotic drugs! All others mentioned are Ans. 1. The key is d. Left mid-thoracic cord. a. Foreign body
seen in this syndrome like renal failure, pyrexia, Ans. 2. Brown-sequard syndrome. [In brown- b. Plummer vinson syndrome
elevated creatinine kinase, tachycardia etc]. sequard syndrome paralysis and loss of c. Pharyngeal puch
proprioception on the same (or ipsilateral) side as d. Peptic stricture
57. A 33yo drug addict wants to quit. She says she the injury or lesion, and loss of pain and e. Esophageal Ca
is ready to stop the drug abuse. She is supported temperature sensation on the opposite (or Ans. 1. The key is D. Peptic stricture. [Progressive
by her friends and family. What drug tx would you contralateral) side as the lesion]. dysphagia to mostly solid is suggestive of peptic
give her? stricture which is supported here by the use of
a. Benzodiazepines 60. A 26yo man presents to ED with increasing prokinetic drugs and H2 blockers which are used
b. Diazipoxide SOB on left side and chest pain. He has been a for reflux oesophagitis].
c. Lithium heavy
d. Methadone smoker for the past 4 years. He doesn’t have any 64. A 56yo man comes with hx of right sided
e. Disulfiram past med hx. What is the likely dx? weakness & left sided visual loss. Where is the
a. Pulmonary embolism occlusion?
Q.1. What is the key. b. MI a. Ant meningeal artery
Q.2. What drugs should you use in i) tobacco c. Asthma b. Mid meningeal artery
abuse and in ii) alcohol abuse? d. Pleural effusion c. Mid cerebral artery
e. Pneumothorax d. Carotid artery
Ans. 1. Key is d. Methadone. (used in opiate Ans. The key is e. Pneumothorax. [Increased e. Ant cerebral artery
abuse) shortness of breath and chest pain with no past f. Ant communicating artery
Ans. 2. i) tobacco abuse: a) bupropion ii) alcohol: medical history favours the dx of pneumothorax.
a) acamprosate b) disulfirum Heavy smoking or tobacco is a risk factor for Q. 1. What is the key?
spontaneous pneumothorax]. Q. 2. How will you differentiate between middle
58. A 16m child presents with drooling, sore cerebral artery occlusion from anterior cerebral
throat and loss of voice. He has fever with a temp 61. A pt with hepatocellular ca has raised levels of artery occlusion?
of ferritin. What is the most probable cause?
38.2C. What is your next step towards a. Hemochromatosis Ans. 1. The key is d. Carotid artery. [Carotid artery
management? b. A1 antitrypsin def divides to internal and external carotid of which
a. Direct pharynoscopy c. Cystic fibrosis internal continues as middle cerebral ultimately.
b. Call ENT surgeon Ans. The key is A. Haemochromatosis. But just before it becomes middle cerebral
c. Call anesthesiologist [Hemochromatosis itself is a cause of internal carotid gives rise to ophthalmic branch.
d. IV fuilds hepatocellular carcinoma and associated with So middle cerebral occlusion may give partial
e. Start antibiotics raised level of ferritin. Serum ferritin levels visual loss but not complete mono-ocular
elevated higher than 200 mcg/L in premenopausal blindness. For complete mono-ocular blindness
Q. 1. What is the key? women and 300 mcg/L in men and occlusion should be proximal to ophthalmic artery
Q. 2. What is the diagnosis? postmenopausal women indicate primary iron i.e. either in internal carotid or more proximally to
Q. What is the urgent management? overload due to hemochromatosis, especially carotid artery].
when associated with high transferrin saturation Ans. 2.
Ans. 1.The key is C. Call anesthesiologist. [As an and evidence of liver disease. Ferritin i) Middle cerebral artery occlusion:
expert to intubate]. concentration higher than 1000 mcg/L suggests paralysis or weakness of contralateral
Ans. 2. Diagnosis is Acute epiglottitis. liver damage with fibrosis or cirrhosis]. face and arm (faciobracheal). Sensory
Ans. 3. In given case urgent intubation is needed loss of the contralatera face and arm.
to secure airway to prevent closure of airway. 62. A woman has electric pains in her face that ii) Anterior cerebral artery occlusion:
start with the jaw and move upwards. Her corneal paralysis or weakness of the
59. A 62yo woman complains of unsteadiness reflexes are normal. What is the most likely dx? contralateral foot and leg. Sensory loss
when walking. On examination she has pyramidal a. Atypical face pain at the contralateral foot and leg.
weakness of her left lower limb and reduced pain b. Trigeminal neuralgia 65.A young college student is found in his dorm
and temp sensation on right leg and right side c. Tempero-mandibular joint dysfunction unconscious. He has tachyarrhythmia and high
d. GCA
fever. He also seems to be bleeding from his nose, e. Liver transplantation
which on examination shows a perforated 68. An 18yo man is rushed into the ER by his
nasal septum. What is the most likely dx? friends who left him immediately before they Q. 1. What is the key?
a. Marijuana OD could be Q. 2. What are the indications of this
b. Cocaine OD interviewed by staff. He is semiconscious, management?
c. Heroin OD RR=8/min, BP=120/70mmHg, pulse=60bpm. He is
d. Alcohol OD noted to have needle track marks on his arms and Ans. 1. The key is E. Liver transplantation.
e. CO poisoning his pupils are small. What is the single best Ans. 2. King's College Hospital criteria for liver
initial tx? transplantation in paracetamol-induced acute
Q. 1. What is the key? a. Insulin liver failure.
Q. 2. What are the points that favours the b. Naloxone arterial pH <7.3 or arterial lactate >3.0 mmol/L
diagnosis in given question? c. Methadone after adequate fluid resuscitation, OR
Q. 3. What are other important findings? d. Gastric lavage if all three of the following occur in a 24-hour
period:
Ans. 1. Key is B. Cocaine overdose. Q. 1. What is the key? Creatinine >300 μmol/L.
Ans. 2. Points in favour: i) Tachyrhythmia ii) High Q. 2. What is the diagnosis? PT >100 seconds (INR >6.5).
fever iii) perforated nasal septum iv) Q. 3. What are the points in favour of the Grade III/IV encephalopathy.
unconsciousness diagnosis?
Ans. 3. Other findings of cocaine toxicity: i) 71. A 75yo alcoholic presents with a mass up to
Psychiatric: anxiety, paranoia ii) Tachypnoea iii) Ans.1. The key is B. Naloxone. umbilicus, urinary dribbling, incontinence, and
Increased energy and talking rapidly iv) Dilated Ans. 2. The diagnosis is opiate overdose. clothes smelling of ammonia. What is the next
pupils. Also: [rhabdomyolysis, metabolic acidosis, Ans. 3. Points in favour are: i) reduced step in management?
convulsion]. consciousness ii) RR 8/min (12<) iii) hypotension a. Urethral catheter
(here lower normal) iv) miosis v) needle track b. Suprapubic catheter
66. A 56yo pt whose pain was relieved by oral marks on his arms. c. Antibiotics
Morphine, now presents with progressively d. Condom catheter
worsening pain relieved by increasing the dose of 69. A 30yo man and wife present to the e. Nephrostomy
oral morphine. However, the pt complains reproductive endocrine clinic because of
that the increased morphine makes him drowsy infertility. The man Q. 1. What is the key?
and his is unable to carry out his daily activities. is tall, has bilateral gynecomastia. Examination of Q. 2. What is the cause of this retention?
What is the next step in his management? the testes reveals bilateral small, firm testes.
a. Oral oxycodone Which of the following inv is most helpful in dx? Ans. 1. The key is A. Urethral catheter.
b. Oral tramadol a. CT of pituitary Ans. 2. Alcohol consumption [Alcoholism can
c. PCA b. Chromosomal analysis cause urinary retention but it is a less common
d. IV Fentanyl c. Measure of serum gonadotropins cause of retention].
e. Diamorphine d. Measure of serum testosterone
72. In CRF, main cause of Vit D deficiency is the
Ans. The key is oral oxycodon. Q. 1. What is the key? failure of:
Q. 2. What is the diagnosis? a. Vit D absorption in intestines
67. A 30yo man presents with a 5cm neck mass Q. 3. What are the points in favour of your b. 25 alpha hydroxylation of Vit D
anterior to the sternocleido-mastoid muscle on diagnosis? c. Excess Vit D loss in urine
the d. 1 alpha hydroxylation of Vit D
left side in its upper third. He states that the Ans. 1. The key is B. Chromosomal analysis. e. Availability of Vit D precursors
swelling has been treated with antibiotics for Ans. 2. The diagnosis is Klinefelter’s syndrome.
infection in the past. What’s the most likely (xxy) Ans. The key is D. 1 alpha hydroxylation of Vit D.
cause? Ans. 3. The points in favour are: i) Infertility ii) Tall [There are 3 steps in the synthesis of vitamin D. i)
a. Branchial cyst stature iii) Bilateral gynaecomastia iv) Bilateral Cholecalciferol in the skin from 7-
b. Parotitis small firm testes. dehydrocholesterol under the action of ultraviolet
c. Pharyngeal pouch light. ii) Hydroxylation in the liver of
d. Thyroglossal cyst 70. An 18yo female just received her A-Level cholecalciferol to 25-hydroxycholecalciferol. iii)
e. Thyroid swelling results and she didn’t get into the university of Hydroxylation in the kidneys (1 alpha
her hydroxylation by enzyme 1-alphahydroxylase) of
Q. 1. What is the key? choice. She was brought into the ED after 25-hydroxycholecalciferol to calcitriol (1,25-
Q. 2. Justify your answer. ingestion of 24 paracetamol tablets. Exam: dihydroxyvitamin D3)].
confused
Ans. 1. The key is A. Branchial cyst. and tired. Initial management has been done. Inv 73. Pt with puffiness of face and rash showing
Ans. 2. i) Branchial cyst is anterior triangular lump. after 24h: normal CBC, ABG = pH7.1, PT=17s, cotton wool spots on fundoscopy. What’s the dx?
[parotid is also anterior triangular lump but Bilirubin=4umol/L, creatinine=83umol/L. What is a. Macular degeneration
parotitis regresses with appropriate treatment i.e. the next step in management? b. Hypertensive retinopathy
becomes normal in size]. ii) pharyngeal pouch is a. Observation for another 24h c. Diabetic background
posterior triangular lump. iii) Thyroglossal is b. Refer to psychologist d. Proliferative diabetic retinopathy
midline lump. iv) thyroid swelling moves with c. Give N-Acetylcysteine e. SLE
swallowing]. d. Discharge with psychiatry referral
Q. 1. What is the key? Ans. 1. The key is E. Abdominal US. Q. 3. What are the signs of deficiency may be
Q. 2. Why there is puffyness of face? Ans. 2. The diagnosis is Abdominal aortic present?
Q. 3. Why there is cotton wool spots on aneurism. Ans. 1. The key is C. Malabsorption.
fundoscopy? What is the most common ocular Ans. 3. Points in favour of AAA are i) hypertension Ans. 2. Diarrhoea, IDA and folic acid deficiency.
manifestation of SLE? ii) abdominal pain iii) back pain iv) expansile Ans. 3.
abdominal mass lateral and superior to the  Iron-deficiency anaemia.
Ans. 1. The key is SLE. umbilicus.  Folate deficiency or vitamin B12
Ans. 2. Puffiness is due to lupus nephritis. deficiency.
Ans. 3. SLE, can involve the retina. The classic 77. A 55yo man has had severe pain in the right  Bleeding, resulting from low vitamin K.
lesion of SLE is a white fluffy appearing lesion hypochondrium for 24h. The pain comes in waves  Oedema, which occurs in
within the retina known as a cotton wool spot. and is accompanied by nausea. Nothing seems to protein/calorie malnutrition.
The most common ocular manifestation in SLE is relieve the pain. He feels hot and sweaty but 80. A 35yo male is bitterly annoyed with people
Keratoconjunctivits sicca. has normal temp. What is the most appropriate around him. He thinks that people are putting
next inv? ideas
74. A 35yo man presents with progressive a. US Abdomen into his head. What is the single most likely dx?
breathlessness. He gave a hx of polyarthralgia b. ERCP a. Thought block
with c. MRCP b. Thought insertion
painful lesions on the shin. CXR: bilateral hilar d. Serum amylase c. Thought broadcasting
lymphadenopathy. What’s the most likely dx? e. UGI endoscopy d. Thought withdrawal
a. Bronchial asthma e. Reference
b. Cystic fibrosis Q. 1. What is the key?
c. Sarcoidosis Q. 2. What is the diagnosis? Q. 1. What is the key?
d. Bronchiectasis Q. 3. What are the points in favour of your Q. 2. In which disease you will find this feature?
e. Pneumonia diagnosis? Ans. 1. The key is B. Thought insertion.
Ans. 2. It is seen in schizophrenia.
Q. 1. What is the key? Ans. 1. The key is A. US abdomen.
Q.2 . What is the specific name of this condition? Ans. 2. The diagnosis is biliary colic. 81. A 10yo girl presents with hoarseness of the
What is the triad? Ans. 3. Points in favour- i) severe right voice. She is a known case of bronchial asthma
hypochondrial pain. ii) colicky nature of the pain and
Ans. 1. The key is C. Sarcoidosis. (comes in waves) iii) nausea iv) absence of fever has been on oral steroids for a while. What is the
Ans. 2. Lofgren syndrome. The triad is i) Erythema iv) absence of jaundice. [OHCM, 9th edition, page- most likely cause of hoarseness?
nodosum ii) Bilateral hilar lymphadenopathy iii) 637]. a. Laryngeal candidiasis
Arthralgia. b. Infective tonsillitis
78. A 67yo man has deteriorating vision in his left c. Laryngeal edema
75. A child presents with clean wound, but he has eye. He has longstanding COPD and is on multiple d. Allergic drug reaction
never been immunized as his parents were drug therapy. What single medication is likely to e. Ludwigs angina
worried about it. There is no contraindication to cause this visual deterioration?
immunization, what is the best management? a. B2 agonist Q. 1. What is the key?
a. Full course of DTP b. Corticosteroid Q. 2. What is the reason for this condition?
b. 1 single injection DT c. Diuretic
c. 1 single injection DTP d. Theophylline Ans. 1. The key is A. Laryngeal candidiasis.
d. Only Ig Ans. 2. Steroids predisposes to fungal infection.
e. Antibiotic Q. 1. What is the key?
Q. 2. What is the cause of deteriorating vision? 82. A lady with breast cancer has undergone
Ans. The key is A. Full course of DTP. axillary LN clearance. She develops arm swelling
Ans. 1. The key is B. Corticosteroid. after
76. A 65yo HTN man presents with lower Ans. 2. Prolonged corticostiroids [also topical i.e. being stung by a bee. What is the most likely
abdominal pain and back pain. An expansive eye drop] can cause cataract. mechanism responsible for the swelling?
abdominal a. Lymphedema
mass is palpated lateral and superior to the 79. A woman who returned from abroad after 3 b. Cellulitis
umbilicus. What is the single most discriminating weeks of holiday complains of severe diarrhea of c. Hypersensitivity reaction
inv? 3 d. DVT
a. Laparascopy weeks. She also developed IDA and folic acid def. e. Fluid retention
b. KUB XR What condition best describes her situation? Q. 1. What is the key?
c. Pelvic US a. Jejunal villous atrophy Q. 2. What is the reason for this condition?
d. Rectal exam b. Chronic diarrhea secretions
e. Abdominal US c. Malabsorption Ans. 1. The key is A. Lymphoedema.
d. Increased catabolism Ans. 2. Reason is compromised lymphatic
Q. 1. What is the key? e. Increased secretions of acid drainage of arm due to axillary LN clearance. So if
Q. 2. What is the diagnosis? there is any inflammation or selling, lymph
Q. 3. What are the points given here in favour of Q. 1. What is the key? drainage is compromised further giving rise to
your diagnosis? Q. 2. What are the points in favour? swelling of limb of that operated side.
83. A 34yo pt presents with 50% partial thickness 38.4C. What is the most appropriate empirical b. IUCD
burns. What should be the most appropriate antibiotic to be started? c. POP
management? a. Ceftriaxone d. COCP
a. IV fluids calculated from the time of hospital b. Penicillin Ans. None of them! Before it was thought that
arrival c. Gentamicin hepatic enzyme inhibitor drugs may affect COCP
b. IV fluids calculated from the time of burn d. Tazobactam but later it was established that actually there is
c. No IVF e. Meropenem no such significant effect. Only drugs like
d. IV dextrose stat refumpicin, rifabutin etc. can cause this. [For
e. Burns ointment Ans. The key is A. Ceftriaxone. [Cfftriaxone is the exam purpose if you have to choose one please
drug of choice in hospital management. Probable choose D. COCP].
Q. 1. What is the key? dx is meningitis. [In OHCM-Cefotaxime <55yrs and
Q. 2. How the calculation of fluid is made? Cefotaxime + Ampicillin if age >55yrs (pre- 90. An 11yo boy is being checked by the diabetic
hospital)]. specialist nurse. His HbA1c was high and he has
Ans. 1. The key is B. IV fluids calculated from the been skipping meals recently. He has been
time of burn. 87. A man with prosthetic heart valve underwent unhappy at school. Which single member of the
Ans. 2. Resuscitation fluids required in the first 24 hemicolectomy and after some days complains of clinical team would you refer him to next?
hours from the time of injury. left hypochondriac pain, fever and has a systolic a. GP
For adults: 3 ml (in partial thickness burn) of murmur. What is the next inv to ascertain the b. Pediatrician
Hartmann’s solution/kg body weight/% total cause of HF? c. Dietician
Body surface area. a. CT d. Clinical psychologist
Half of this calculated volume is given in the first 8 b. Blood culture
hours and the other half is given over the c. ECG Ans. The key is D. Clinical psychologist. [Skipping
following 16 hours. d. MRI meals, unhappy at school these are psychological
e. Radioactive thyroid scan issues. So he should be referred to clinical
84. A 54yo man has recently been dx with psychologist].
moderate depression. He has hx of MI and is Q. 1. What is the key?
suffering Q. 2. What is the diagnosis? 91. A 35yo man who has served in the army
from insomnia. What is the drug of choice for Q. 3. Why have you made this diagnosis? presents with lack of interest in enjoyable
him? Q. 4. What are the important risk factors for this activities and
a. Citalopram condition? feeling low. He doesn’t feel like reading the news
b. Lofepramine or watching movies as he believes there is
c. ECT Ans. 1. The key is B. Blood culture. violence everywhere. What is the most
d. Haloperidol Ans. 2. The diagnosis is infective endocarditis. appropriate first line therapy?
e. Diazepam Ans. 3. Fever + new murmur = endocarditis until a. Citalopram
proven otherwise. b. Lofepramine
Ans. Key is A. Citalopram. [Sertraline is the drug of Ans. 4. Important risk factors: dermatitis, IV c. CBT
choice in post MI as citalopram may be a risk injections, renal failure, organ transplantation, d. Chlordiazepoxide
factor for precipitating torsades-de-pointes. But if DM, post operative wond. Risk factors for e. Desensitization
sertraline is not in option second choice is abnormal valves: aortic or mitral valve disease,
citalopram and where SSRIs are not suitable next tricuspid valve in IV drug users, prosthetic valves. Q. 1. What is the key?
choice is mirtazapine]. Q. 2. What is the diagnosis?
88. A 45yo man with posterior gastric ulcer Q. 3. What is the first line treatment?
85. A man presented with cellulitis and swelling. presented with severe excruciating pain which
He was started on flucloxacillin. What other subsided Ans. 1. The key is C. CBT
medication do you want to add? after conservative treatment. 10 days later he Ans. 2. The diagnosis is mild depressive illness.
a. Vancomycin developed swinging pyrexia. US shows a Ans. 3. In mild depressive illness CBT is preferred
b. Penicillin collection in the peritoneum. What will be the option.
c. Metronidazole most likely location of the collection?
d. Ceftriaxone a. Hepatorenal puch 92. A man has reducible bulge below the pubic
e. Amoxicillin b. Left paracolic gutter tubercle, and on occlusion of the deep inguinal
c. Subphrenic ring,
Q. 1. What is the key? d. Pelvic cavity cough impulse is present. What is the most likely
Q. 2. Please mention why? e. Lesser sac dx?
a. Direct inguinal
Ans. 1. The key is B. Penicillin. Ans. The key is E. Lesser sac. b. Indirect inguinal
Ans. 2. cellulitis is usually caused by c. Femoral
staphylococcus and streptococcus. To cover both 89. A 23yo lady was prescribed with azithromycin d. Spigelian
Flucloxacillin (for staphylococcus) and Penicillin 1gm for her chlamydial pelvic infection. She has e. Lumbar
(to cover streptococci) should be prescribed. got a new boyfriend for the last 2 months. She has
recently started contraception to avoid Q. 1. What is the key?
conception. Which of the following contraception Q. 2. What are the points in favour of your
86. A 24yo college student presents with nausea, method will be affected by azithromycin? answer?
vomiting, headache, neck stiffness and a fever of a. Barrier
Ans. 1. The key is C. Femoral hernia. bright light since the last 4 days. She feels b. US Abdomen
Ans. 2. It is just below the pubic tubercle that is nauseated, but doesn’t vomit. What is the most c. Cystoscopy
just below the inguinal ligament. On occlusion of likely d. Mid-stream urine for culture
deep ring cough impulse is present (means visible) dx? e. Transrectal US
as femoral hernia doesn’t come through deep ring a. SAH
but indirect inguinal hernia. b. Brain tumor Q. 1. What is the key?
c. Migraine Q. 2. What is the diagnosis?
93. A 48yo woman is admitted to ED with a d. Cluster headache Q. 3. What are the reasons for this diagnosis?
productive cough and moderate fever. She has e. Subdural headache Q. 4. If there is painless haematuria in young (say
often 25-30yrs) what diagnosis will come first?
central chest pain and regurgitation of undigested Q. 1. What is the key?
food most times but doesn’t suffer from acid Q. 2. What is the type of the given case? Ans.1. Key is C. Cystoscopy.
reflux. These symptoms have been present for the Q. 3. What are the points in favour of mentioned Ans. 2. Bladder cancer.
last 3.5 months which affects both food and type? Ans. 3. Age 65, asymptomatic haematuria.
drink. A CXR shows an air-fluid level behind a Ans. 4. ADPKD [at the beginning there is very few
normal sized heart. What is the most likely dx? Ans. 1. The key is C. Migraine. or no symptoms].
a. Pharyngeal pouch Ans. 2. It is migraine without aura.
b. Hiatus hernia Ans. 3. Criteria of migraine without aura: ≥5 99. A 30yo woman had a gradual decrease of
c. Bulbar palsy headaches lasting 4-72 hours + nausea/vomiting visual acuity since the last 3 years. Now she has a
d. Achalasia (or photo/phono-phobia) + any 2 of: i) unilateral disability due to very low vision. What’s the dx?
e. TB ii) pulsating iii) worsen by routine activity [OHCM, a. Glaucoma
9th edition, page-462]. b. Cataract
Q. 1. What is the key? c. Macular degeneration
Q. 2. What are the points in favour? 96. A 35yo man presented with hematuria, d. Retinitis pigmentosa
abdominal swelling and has a BP of 190/140. e. Keratitis
Ans. 1. The key is D. Achalasia. What is the
Ans. 2. Points in favour: Aspiration pneumonia most diagnostic inv? Q. 1. What is the key?
due to retained food and fluid in oesophagus. In a. Cystoscopy Q. 2. Why it is not the other given D/D s?
achalasia usually there is no acid reflux. Dysphagia b. USG
for both food and drink. Air-fluid level behind c. CT Ans. 1. The key is D. Retinitis pigmentosa.
heart. d. Renal biopsy Ans. 2. i) It is not angle closure glaucoma as angle
Why it is not hiatus hernia? Ans. Differentiating e. Urine analysis closure glaucoma occurs usually after the age of
point:-i) In hiatus hernia usually you will get 50; In open angle glaucoma visual loss is not
associated GORD ii) Also in hiatus hernia there Q. 1. What is the key? evenly gradual rather occurs a bit suddenly at its
may be nausea or vomiting. Q. 2. What is the diagnosis? later part. It is not cataract as cataract occurs
Why it is not pharyngeal pouch? Ans. In Q. 3. What will be the USG findings to establish usually in elderly. In macular degeneration near
pharyngeal pouch there will be halitosis. diagnosis in given case? blindness does not occur rather causes inability to
identify face or cannot read small prints;
94. A 64yo man has been waking up in the middle Ans. 1. The key is B. USG. otherwise peripheral vision is not that depressed.
of the night to go to the bathroom. He also had Ans. 2. The diagnosis is ADPKD. In keratitis will be pain, redness, photophobia and
difficulty in initiating micturition and complains of Ans. 3. In given case patients age is 35. So the USG vision is ok.
dribbling. A dx of BPH was made after a diagnostic criteria is: Age 18 – 39 yrs>3 unilateral
transrectal US guided biopsy and the pt was or, bilateral cysts, 40 – 59 yrs >2 cysts in each 100. A 27yo lady has had an uncomplicated
prepared for a TURP. What electrolyte kidney, >60 yrs >4 cysts in each kidney. [OHCM, pregnancy so far. She came to the hospital 2h ago
abnormality 9th edition, page- 312]. after
is highly likely due to this surgery? her water broke. The midwife is looking at her
a. Hypokalemia 97. A young man is brought to the ED after an now. She has regular contractions. P.V exam
b. Hypocalcemia RTA. His GCS on initial evaluation is 6. What is the revealed 2cm dilated cervix. Vital signs are
c. Hyperkalemia most normal. What stage of labour is she in?
d. Hyponatremia appropriate next step? a. Second stage
e. Hypernatremia a. CT b. First stage
b. MRI c. Latent stage
Q. 1. What is the key? c. IV fluids d. Third stage
Q. 2. Why this happens? d. Skull XR e. Active phase
e. Secure airway
Ans. 1. The key is D. Hyponatremia. Ans. The key is B. First stage. First stage starts
Ans. 2. Absorption of fluid used for bladder Ans. The key is E. Secure airway. with softening of cervix with start of opening of
irrigation to flush out blood clots and IV fluids all cervix and ends when cevix is fully dilated (i.e. 10
may lead to hypervolaemia and dilutional 98. A 65yo man presented with frank hematuria. cm dilated).
hyponatremia. He has no other urinary symptoms. What is the
most appropriate next step that will lead to the 1o1. A 2yo boy fell off his tricycle and hurt his
95. A 56yo lady has developed severe right sided dx? arm. He got up to start crying, but before there
headache which worsens whenever she comes to a. IVU was
any sound, he went pale, unconscious and rigid. year. His conjunctiva is pale. Choose the single cell 109. A 33yo male involved in a street fight
He recovered after 1-2 mins but remained pale. type you will find on the blood film. presents with bruises and deformity in the upper
After an hour he was back to normal. His mother a. Granulocyte without blast cells part of
says she was afraid he was going to die, and b. Myelofibroblasts his leg. XR shows fx of the neck of fibula. What is
that he had a similar episode 3 months prior after c. Plasma cells the single most associated nerve injury?
falling down some steps. What single inv is d. Mature lymphocytes a. Sciatic nerve
indicated? Q. 1. What is the key? b. Gluteal nerve
a. CT head Q. 2. What is the diagnosis? c. Musculocutaneous nerve
b. EEG Q. 3. What are the points in favour of your d. Lateral peroneal nerve
c. CBC diagnosis? e. Tibial nerve
d. None Ans. 1. The key is D. Mature lymphocytes. f. Femoral nerve
e. Skeletal survey Ans. 2. The diagnosis is CLL. Ans. The key is D. Lateral peroneal nerve. [Lateral
Q. 1. What is the key? Ans. 3. It is CLL because of his age (73 yrs), cervical peroneal nerve is other name of superficial
Q. 2. What is the diagnosis? lymphadenpathy, recurrent infections (mature peroneal nerve].
Ans. 1. The key is D. None. but functionally defective lymphocytes), and pale 110. A 35yo man presents with hx of dyspepsia.
Ans. 2. Diagnosis is breath holding spell. conjunctiva (anaemia). H.Pylori antibodies are negative. No improvement
106. A 45yo lady has 10m hx of SOB. She is found is
102. A 29yo woman had just delivered a still born to have irregularly irregular pulse and loud P2 seen after 1m of tx. What is the next step?
vaginally, following a major placental abruption. with a. Urea breath test
Choose the single most likely predisposing factor fixed splitting and ejection systolic murmur in left b. Gastroscopy
for developing PPH in this lady? 2nd ICS. What is the probable dx? c. CT
a. Retained product a. TOF d. MRI
b. DIC b. ASD Q. 1. What is the key?
c. Fibroid uterus c. VSD Q. 2. What may be the D/D here?
d. Uterine infection d. PDA Q. 3. At this age what are the indications of this
e. Large placental site e. CoA procedure?
Q. 1. What is the key? Q. 1. What is the key? Ans. 1. Gastroscopy.
Q. 2. What are the causes of this condition here? Q. 2. What is the diagnosis? Ans. 2. Not responding to treatment D/D is: i)
Ans. 1. The key is B. DIC. Ans. 1. The key is B. Atrial septal defect. Jollinger Elison syndrome ii) Ca stomach
Ans. 2. Pregnancy itself is a risk factor for DIC. Ans. 2. Diagnosis is ASD with atrial fibrillation. [i) Ans. 3. Indications of gastroscopy in a 35 yo man
Placental abruption is a more common cause of atrial fibrillation = irregularly irregular pulse. ii) (for man of age <50): i) Acute symptoms with H/O
DIC. ASD = SOB, fixed splitting with loud P2, ESM in previous episode (PUD) ii) Alarm features [weight
Other causes of pregnancy related DIC are: pulmonary area]. This picture is typical. One loss, anaemia, vomiting, hematemesis and
eclampsia, retention of a dead fetus, amniotic should not misdiagnose SOB, ESM in pulmonary melaena, dysphagia, palpable abdominal mass],
fluid embolism, retained placenta or bacterial area and loud P2 as pulmonary hypertension fear of cancer, evidence of organic disease.
sepsis. (though in elderly this can develop with ASD).
103. A 28yo woman has delivered with rotational 107. A 5m baby present with recurrent vomiting. 111. A 15yo male has bilateral ankle edema. His
forceps after an 8h labor and 3h second stage. Mother noticed some of the vomitus is blood BP=110/70mmHg and urinalysis shows protein+++
Choose the single most likely predisposing factor stained. Choose the single most likely inv? +.
for PPH for this pt? a. Upper GI endoscopy What is the most likely dx?
a. Atonic uterus b. Barium meal a. HUS
b. Cervical/vaginal trauma c. US b. IgA nephropathy
c. Retained product d. Colonoscopy c. Membranous GN
d. Preterm labor e. CT abdomen d. Minimal change GN
e. Uterine infection Ans. The key is A. upper GI endoscopy. e. Nephrotic syndrome
Ans. The key is B. Cervical/vaginal trauma. 108. A 76yo is treated with HTN. He suffers from Q. 1. What is the key?
[complication of forceps delivery]. pain and redness at the MTP joint of his right big Q. 2. What are the points in favour of your
104. A 50yo man has had anterior resection of the toe. diagnosis?
rectum for carcinoma. He expressed concerns Which of the following anti-HTN cause this Q. What is the treatment?
about control of post-op pain in discussions with symptoms? Ans. 1. The key is D. Minimal change disease.
the anaesthetist before surgery. What is the a. Losartan Ans. 2. Points in favour: i) Age 15 ii) Ankle oedema
best management strategy? b. Bendroflumethiazide iii) Normotension iv) Heavy proteinuria.
a. Oral diclofenac c. Ramipril Ans. 3. Treatment of choice is steroid
b. Oral codeine d. Bisoprolol (prednisolon). Failure of steroid or frequent
c. IM morphine e. Verapamil relapse (>3) cyclophosphamide.
d. IM dihydrocodeine Q. 1. What is the key?
e. Ondansetron oral Q. 2. What is the diagnosis? 112. A 28yo man has developed a red, raised rash
Ans. The key is C. IM morphine. [Some post Q. 3. What is the cause of the disease? on trunk after playing football. His PMH shows he
operative pain is severe pain which needs strong Ans. 1. The key is B. Bendroflumethiazide had childhood asthma. The rash is becoming
opioid analgesic]. Ans. 2. Diagnosis is acute gout. increasingly itchy. What is the most appropriate
105. A 73yo male presents with enlarged cervical Ans. 3. Thiazide diuretics may cause tx?
nodes. He has had recurrent infections over the hyperuricemia and thus cause or precipitate gout. a. Oral chlorpheneraime
last b. Oral amoxicillin
c. IM adrenaline e. Stop breastfeeding e. CoA
d. Nebulized salbutamol Ans. The key is D. PDA. [Continuous mechinary
e. Histamine Q. 1. What is the key? murmur is well known feature of PDA].
Q. What is the key? Q. 2. What is breast milk jaundice?
Q. 2. What is the diagnosis? Q. 3. What type of hyperbilirubinemia occurs in 120. A 12yo child with episodes of sudden bluish
Ans. 1. The key is A. Oral chlorpheneramine. breast milk jaundice? discoloration and brief loss of consciousness.
Ans. 2. Diagnosis is Atopy (allergy). Q. 4. What is the cause of this jaundice? Exam:
Ans. 1. The key is A. Continue breast feeding. clubbing, central cyanosis, systolic thrill with
113. A 72yo man has been advised to have Ans. 2. If jaundice lasts past the first week of life systolic ejection murmur in 2nd left ICS. What is
antibiotic prophylaxis for some years now before in a breastfed baby who is otherwise healthy, the the
dental tx. condition may be called "breast milk jaundice." most probable dx?
He has never experienced chest pain. Three Ans. 3. Unconjugated hyperbilirubinaemia. a. TOF
weeks ago, he noticed breathlessness on exertion Ans. 4. Cause of breast milk jaundice: factors in a b. ASD
and for one week he had orthopnea. His pulse is mother's milk that help a baby absorb bilirubin c. VSD
normal. What is the most probable dx? from the intestine. d. PDA
a. Aortic regurgitation e. CoA
b. Ischemic mitral regurgitation 117. A 12yo girl when playing in the garden Ans. The key A. TOF. [TOF usually does not
c. Mitral valve prolapse accidentally stepped on a hive and was bitten become symptomatic at birth or early infancy and
d. Pulmonary stenosis several given features (central cyanosis and clubbing with
e. Mitral valve stenosis times. She has numerous wheals on her body and murmur of right ventricular outflow obstruction
Ans. The kay is E. Mitral valve stenosis. [Mitral complains of severe itching. What is the single i.e. ejection systolic murmur in 2nd left ICS) are
stenosis is the most common valvular most appropriate management? well known features of TOF]. [Tetralogy of fallot =
complication of rheumatic fever]. a. Oral antihistamine 1. VSD + 2. Overriding of the aorta + Right
114. A 37yo woman presents with fatigue. Exam: b. IV antihistamine ventricular outflow tract obstruction + Right
angular stomatitis, no koilonychea. Choose the c. IM adrenaline ventricular hypertrophy].
single cell type you will find on the blood film. d. Oral ciprofloxacin
a. Macrocytes e. Reassurance 121. An 8yo child who is tall for his age and has a
b. Microcytes Ans. The given key is C. IM adrenaline which is a refractory error for which he wears glasses has
c. Granulocytes wthout blast cells wrong key. The correct answer is A. Oral presented with severe crushing chest pain. What
d. Blast cells antihistamine. [Intravenous antihistamines are is the most likely dx?
Q. 1. What is the key? used as an adjunct to epinephrine in the a. Fragile X syndrome
Q. 2. What is the cause here? emergency treatment of anaphylaxis and b. Prader-willi syndrome
Q. 3. What are the points in favour of mentioned angioedema. Parenteral antihistamine is not c. DiGeorge syndrome
cause? recommended in less severe allergic reaction d. Marfans syndrome
Ans. 1. The given key is A. Macrocytes. (other than anaphylaxis)]. Q. 1. What is the key?
Ans. 2. The cause here is VIT. B12 or folate Followings are the indications of adrenaline in Q. 2. What is the cause of this severe crushing
deficiency. anaphylaxis: chest pain?
Ans. 3. Points in favour of Vit. B12 or folate 1. Horseness of voice Q. 3. What are the most common cardiac
deficiency: i) fatigue (anaemia) ii) angular 2. Wheeze abnormalitis found in this disease?
stomatitis (can be seen in Vit. B12 or folate 3. Shortness of breath Ans. 1. The key is D. Marfans syndrome.
deficiency) iii) absence of koilonychea is against 4. Shock Ans. 2. Cause of severe crushing chest pain may
IDA. 5. Stridor be aortic dissection.
6. Swelling of the tongue and cheek Ans. 3. Most common cardiac abnormalities in
115. A 4yo boy with a febrile convulsion lasting 7. Facial swelling Marfans syndrome are: dilatation of the aorta and
eight minutes has been given IV lorazepam to mitral regurgitation.
control 118. A term baby born to a 30yo woman of blood 122. A 4yo child presents with pain of
them. What is the single most likely serious side group A-ve develops severe jaundice within the spontaneous onset in his knee of 2 days duration.
effect? first 24h of birth. What is the most likely dx? He has
a. Amnesia a. Hereditary spherocytosis developed mild fever in the 2nd day. He can walk
b. Anaphylactic shock b. G6PD but has a limp. Exam: painful restriction in the
c. Apnea c. ABO incompatibility right hip. What is the most probable dx?
d. Bronchospasm d. Rh incompatibility a. Osteosarcoma
e. Cardiac arrhythmia e. Physiological jaundice b. Septic arthritis
Ans. The key is C. Apnoea. [Respiratory depression Ans. The key is D. Rh incompatibility. c. TB arthritis
may occur following lorazepam administration]. d. Exostosis
119. A 4yo girl is found to have bounding pulse e. Osteomyelitis
116. A 4wk girl has been dx of having breast milk and continuous machinery murmur. What is the Q. 1. What is the key?
jaundice. She is otherwise well. What is the single most Q. 2. What are the points in favour of your
most appropriate management? probable dx? diagnosis?
a. Continue breastfeeding a. TOF Ans. Given key is E. Osteomyelitis which is a
b. Exchange transfusion b. ASD wrong key. The correct answer is B. Septic
c. Increase fluid intake c. VSD arthritis.
d. Phototherapy d. PDA
Ans. Points in favour of diagnosis: i) Pain in joints Q. 4. What is the management? Ans. 3. Points in favour of PTSD: i) H/O stressor
(knee and hip). In osteeomyelitis there is no joint Ans. 1. The key is A. X-ray abdomen. (house fire a year ago) ii) Nightmares of the
pain but pain in other parts of bone like shaft. ii) Ans. 2. The diagnosis is paralytic ileas. stressor iii) Hyper arousal (very anxious and
Fever iii) Painful restricted movement of joint. Ans. 3. Causes of paralytic ileus: i) electrolyte inability to relax (leading to irritability) iv)
NB This controversial question was debated and imbalance ii) gastroenteritis iii) appendicitis iv) associated depression (poor sleep, tearful).
ultimately settled as septic arthritis by old pancreatitis v) surgical complications and vi) Note: Fluoxetin and peroxetin are the drugs of
plabbers. certain drugs. choice in PTSD. CBT is the non-pharmacological
123. A man with anterior resection and end to Ans. 4. Management of paralytic ileus: i) nil by treatment.
end anastomosis done complains of severe pain in mouth ii) nasogastric suction to alleviate the 130. A 22yo woman with longstanding
the distension and remove the obstruction. constipation has severe ano-rectal pain on
chest and abdominal distension. What is the most 127. A 30yo man complains of hoarseness of defecation. Rectal exam: impossible due to pain
appropriate inv likely to review the cause this voice. Exam: unilateral immobile vocal cord. What and spasm. What is the most probable dx?
deterioration? is the a. Anal hematoma
a. XR abdomen most probable dx? b. Anal fissure
b. Exploratory laparoscopy a. Graves disease c. Anal abscess
c. CT b. Hematoma d. Protalgia fugax
d. US c. Unilateral recurrent laryngeal nerve injury e. Hemorrhoids
e. Laparotomy d. External laryngeal nerve injury Ans. The key is B. Anal fissure.
Ans. The key is E. Laparotomy. e. Tracheomalacia
124. Pt with hx of alcoholism, ataxic gait, Ans. The key is C. unilateral recurrent laryngeal 131. A 20yo student attends the OPD with
hallucinations and loss of memory. He is given nerve injury. complaint of breathlessness on and off, cough and
acamprosate. 128. A 38yo woman has delivered after an sputum.
What other drug can you give with this? induced labor which lasted 26h. choose the single His sleep is disturbed and skin is very dry in
a. Chlordiazepoxide most flexural areas of the body. Exam: tachypnea,
b. Thiamine likely predisposing factor for postpartum hyperresonant percussion and wheezing on
c. Diazepam hemorrhage? auscultation. What is the most likely dx?
d. Disulfiram a. Atonic uterus a. Extrinsic allergic alveolitis
e. Haloperido b. Cervical/vaginal trauma b. Asthma
Ans. The key is B. Thiamine. [These are features of c. Rupture uterus c. Wegener’s granulomatosis
delirium tremens]. d. Fibroid uterus d. COPD
e. Age of mother e. Cystic fibrosis
125. A 35yo male builder presented with sudden Ans. The key is A. Atonic uterus. [Prolonged Q. What is the key?
onset of severe abdominal pain. He was labour is a risk factor for PPH secondary to atonic Q. What are the diagnostic criteria?
previously uterus]. Ans. The key is B. Asthma.
fit and well other than taking ibuprofen for a long 129. A 32yo woman in tears describing constant Ans. 2. Diagnostic criteria of asthma: i) Airway
term knee injury. On examination he is in irritability with her 2 small children and inability hyper-responsiveness to certain stimuli ii)
severe pain, pulse=110bpm, BP=110/70mmHg to Recurrent variable airflow limitation usually
and has a rigid abdomen. What is the most likely relax. She describes herself as easily startled with reversible iii) presents as wheezing,
dx? poor sleep and disturbed nightmares following breathlessness, chest tightness and cough.
a. Biliary peritonitis a house fire a year ago, while the family slept. 132. A pt with thought disorder washes hands 6x
b. Ischemic colon What is the single best tx? each time he uses the toilet. What is the best
c. Pancreatic necrosis a. Rassurance management?
d. Perforated diverticulum b. Relaxation therapy a. Psychodynamic therapy
e. Perforated peptic ulcer c. Quetiapine b. CBT
Ans. The key is E. Perforated peptic ulcer. [NSAIDs d. Lofepramine c. Antipsychotics
induced perforation. Points in favour- 1. e. Fluoxetine d. Refer to dermatology
Prolonged use of NSAIDs, 2. Sudden onset of Q. 1. What is the key e. Reassure
severe abdominal pain, 3. Rigid abdomen]. Q. 2. What is the diagnosis?
126. A woman 5 days post-op for bilateral Q. 3. What are the points in favour of your Q. 1. What is the key?
salphingo-oopherectomy and abdominal diagnosis? Q. 2. What is the diagnosis?
hysterectomy has developed abdominal pain and Ans. 1 The key is E. Fluoxetine. [NICE guidelines Ans. 1. The key is B. CBT. [For OCD CBT is the 1st
vomiting a/w abdominal distension and can’t pass suggest that trauma-focussed psychological treatment of choice and if fails comes drugs].
gas. No bowel sounds heard, although well therapies (CBT or EMDR) should be offered before Ans. 2. The diagnosis is obsessive compulsive
hydrated. What is the most appropriate next medication, wherever possible. As these are not disorder.
step? in option the best answer here is Fluoxetine
a. XR abdomen (SSRI). At present, there is evidence that EMDR, 133. A 25yo woman presented to her GP on a
b. Exploratory laparoscopy psychotherapy, behaviour therapy and routine check up. Upon vaginal exam, she was
c. CT antidepressants are all effective. There is not fine except for finding of cervical ectropion which
d. USG enough information for us to say that one of was painless but mild contact bleeding on touch.
e. Barium enema these treatments is better than another]. What is the next management?
Q. 1. What is the key? Ans. 2. The diagnosis is post traumatic stress a. Endometrial ablation
Q. 2. What is the diagnosis? disorder. b. Cervical smear (2nd line)
Q. 3. What are the causes of it? c. Colposcopy
d. Antibiotics (1st line) pressure normal; Respiratory rate 14- influenza. For AFB there should be low grade
e. Vaginal US 20; urine output greater than 30 evening fever with night sweats, weight loss,
f. Pack with gauze and leave to dry ml/hour. anorexia etc].
Q. 1. What is the key? 2. Class 2 15%-30% blood volume lost: 140. A 67yo man had successful thrombolysis for
Q. 2. Points in favour correct of key. pulse 100-120; systolic blood pressure an inf MI 1 month ago and was discharged after
Ans. The key is D. Antibiotics. This is a wrong key. normal; pulse pressure decreased; 5days. He is now readmitted with pulmonary
The correct key is B. Cervical smear. [Before respiratory rate 20-30; urine output 20- edema. What is the most probable dx?
antibiotic we have to take swab]. 30 ml/hour. a. Aortic regurgitation
Ans. 2. Cervical smear is a screaning service which 3. Class 3 30%-40% blood volume lost: b. Ischemic mitral regurgitation
follows its own schedule and can not be done pulse 120-140; systolic BP decreased; c. Mitral valve prolapse
unless it is due or overdue. As the Lady is of 25 pulse pressure decreased, respiratory d. Pulmonary stenosis
years and has ectropion and contact bleeding rate 30-40; urine output 5-15 ml/hr e. Rheumatic mitral valve stenosis
smear is the best option here as in UK 1st smear is 4. Class 4, blood loss of greater than 40%: Ans. The key is B. Ischaemic mitral regurgitation.
offered at 25 yrs. pulse rate >140; systolic BP decreased; [ischaemic mitral regurgitation > raised
pulse pressure decreased’ respiratory pulmonary capillary pressure > pulmonary
134. A 32yo had a normal vaginal delivery 10 days rate >35; urine output negligible. oedema]. [ Inferior myocardial infarction causes
ago. Her uterus has involuted normally. Choose left ventricular remodeling, which displaces
the single most likely predisposing factor for PPH? 137. A 67yo man presents with palpitations. ECG posterior papillary muscle away from its normal
a. Retained product shows an irregular rhythm and HR=140bpm. He is position, leading to ischemic mitral regurgitation].
b. DIC otherwise stable, BP=124/80mmHg. What is the
c. Uterine infection most appropriate management? 141. A 60yo lady who had stroke 3 years ago now
d. Von Willebrand disease a. Bisoprolol reports having increased dyspnea on exertion and
e. Primary PPH b. ACEi atrial fibrillation. CXR: straight left border on the
c. Ramipril cardiac silhouette. What is the most probable
Q. 1. What is the key? d. Digoxin dx?
Q. 2. What type of PPH it would be? Ans. The key is A. Bisoprolol. [In acute AF (<48h) if a. Aortic regurgitation
Ans. 1. The key is C. uterine infection. stable rate control by verapamil b. Ischemic mitral regurgitation
Ans. 2. Secondary PPH 40-120mg/8hourly po or bisoprolol 2.5-5mg/d po. c. Mitral valve prolapse
In chronic AF (>48h) rate control with beta- d. Pulmonary stenosis
135. A 37yo man slipped while he was walking blocker or rate limiting CCB; OHCM, 9th edition, e. Rheumatic mitral valve stenosis
home and fell on his out stretched hand. He page-124]. Q. 1. What is the key?
complains 138. A 78yo man is depressed after his wife’s Q. 2. What are the points in favour of your
of pain in the right arm. XR showed fx of the head death. He has been neglecting himself. His son answer?
of radius. What is the single most associated found him in a miserable state when he went to Ans. 1. The key is E. Rheumatic mitral valve
nerve injury? visit. The son can’t deal with his father. What is stenosis.
a. Radial nerve the appropriate management? Ans. 2. Points in favour: i) Dyspnoea on exertion ii)
b. Musculocutaneous nerve a. Voluntary admission to psychiatry ward Straight left border of the cardiac silhouette. To
c. Median nerve b. Hand over to social worker straighten the left heart border it requires
d. Ulnar nerve c. Request son to move in with father underfilling of the left ventricle and aorta which
Q. 1. What is the key? d. Send pt to care home occurs in mitral stenosis. Iii) Atrial fibrillation is a
Q. 2. What is the root value? Ans. The key is A. Voluntary admission to common association.
Ans. 1. The key is A. Radial nerve. [At wrist, radial psychiatry ward. 142. A 60yo diabetic complains of pain in thigh
nerve injury cause finger drop with a normal wrist and gluteal region on walking up the stairs for the
and intact sensation]. 139. An old alcoholic presents with cough, fever, last
Ans. 2. Root value of radial nerve: C5,6,7,8 and T1. bilateral cavitating consolidation. What is the 6 months. She is a heavy smoker and has ischemic
most heart disease. What is the most appropriate
136. A butcher stabbed accidently his groin. He probable cause? dx?
bled so much that the towel was soaked in blood a. Gram +ve diplococcic a. Thromboangitis Obliterans
and b. Coagulase +ve cocci b. Sciatica
BP=80/50mmHg, pulse=130bpm. What % of c. Gram –ve cocci c. DVT
circulatory blood did he lose? d. AFB d. Atherosclerosis
a. <15% e. Coagulase –ve cocci e. Embolus
b. 15-30% Q. 1. What is the key?
c. 30-40% Q. 2. What is the organism? Q. 1. What is the key?
d. 40-50% Ans. 1. The key is B. Coagulase +ve cocci. [The Q. 2. What are the points in favour?
e. >50% picture is of pneumonia and bilateral cavitating Ans. 1. The key is D. Atherosclerosis.
Q. 1. What is the key? consolidation favours staphylococcus as the Ans. 2. i) It is not sciatica as sciatica pain is worse
Q. 2. What is the classification of blood loss causative agent]. when sitting. There may be weakness, numbness,
according to vita sign? Ans. 2. Name of organism is Staphylococcus difficulty moving the leg or foot. A constant pain
Ans. 1. The key is C. 30-40% aureus. [Both coagulase positive and coagulase on one side of the rear. A shooting pain that
Ans. 2. Classification: negative cocci are staphylococci. Gram positive makes it difficult to stand up. ii) It is not DVT as no
1. Class 1 up to 15% of blood volume lost: diplococcic is pneumococcus and gram negative swelling, warmth or redness of skin are there iii) It
pulse <100; systolic BP normal; pulse nisseria, moraxella catarrhalis and hemophilus is not thromboangitis obliterans as pulses are ok,
no colour change or reduced hair growth, no Ans. 2. Diagnosis is multiple sclerosis. b. 0.25ml PO adrenaline
ulceration or gangrene iv) not embolism as no 146. A 55yo male presents with malaise and c. 0.25ml IV adrenaline
pain (rest pain), no numbness, no redness or tiredness. Exam: spleen approaching RIF, no d. IV chlorphearamine
itching or rash, no ulceration of skin. lymphadenopathy. Choose the single cell type? Ans. The key is A. 0.25 ml IM adrenaline [Presence
143. A 3yo child who looks wasted on a. Helmet shaped cell of stridor and wheeze are suggestive of
examination has a hx of diarrhea on and off. The b. Sickle cell anaphilaxis and treatment option is IM
mother c. Granulocyte without blast cells adrenaline].
describes the stool as bulky, frothy and difficult to d. Blast cells 150. A 5yo boy is referred to the hospital and seen
flush. What is the single inv most likely to Q. 1. What is the key? with his father who is worried that he has been
lead to dx? Q. 2. What is the diagnosis? listless. He is not sure why his GP suggested he
a. Sweat chloride test Q. 3. What are the diagnostic features? should come to the ED and is keen to get some
b. Anti-endomysial antibodies Ans. 1. The key is C. Granulocyte without blast tablets and go home. Exam: tired and irritable,
c. LFT cells. swelling around eyes. Renal biopsy: remarkable
d. US abdomen Ans. 2. The diagnosis is CML. for podocyte fusion on EM. What is the most
e. TFT Ans. 3. Diagnostic features are i) increased probable dx?
Q. 1. What is the key? number of mature granulocytes ii) huge a. NAI
Q. 2. What is the diagnosis? splenomegaly. b. Myelodysplastic disease
Ans. 1. The key is B. Anti-endomysial antibody [* Helmet shaped cells (or shistocytes) = c. HSP
Ans. 2. The diagnosis is celiac disease. [It is not hemolytic or Microangiopathic hemolytic anemia d. Membranous GN
cystic fibrosis as lung problem is most commonly * Sickle cell = sickle cell anemia e. Minimal change GN
seen in cystic fibrosis along with GI problem like * Blast cells (immature cells) = in acute leukemia]. Ans. The key is E. Minimal change
indigestion]. 147. A 6yo pt comes with easy bruising in glomerulonephritis. [Podocyte fusion on electron
144. A 45yo woman has had severe epigastric and different places when she falls. CBC: WBC=25, microscopy].
right hypochondrial pain for a few hours. She has Hgb=10.9, 151. A 6yo boy is brought to the hospital for a 3rd
a Plt=45. Her paul bunnel test +ve. What is the most episode of sore throat in 1 month. He is found
normal CBC, serum ALP is raised, normal likely dx? bleeding from gums and nose and has pale
transaminase. 3 months ago she had a a. Glandular fever conjunctiva. What’s the single cell type?
cholecystectomy done. What is the most b. ITP a. Clumped platelets
appropriate inv? c. Trauma b. Microcytes
a. US abdomen d. NAI c. Granulocyte without blast cells
b. ERCP e. Septicemia d. Blast cells
c. MRCP Q. 1. What is the key? e. Mature lymphocytes
d. CT abdomen Q. 2. What are the lab. Values that suggests the Q. 1. What is the key?
e. Upper GI endoscopy diagnosis here? Q. 2. What is the diagnosis?
Q. 1. What is the key? Ans. 1. The key is A. Glandular fever. Q. 3. What are the points that favour diagnosis?
Q. 2. What is the diagnosis? Ans. 2. Suggestive lab. Values: WBC=25 Ans. 1. The key is D. Blast cells.
Ans. 1. The key is B. ERCP. It is probably a wrong (leucocytosis), Hgb=10.9 (usually patient is not Ans. 2. The diagnosis is ALL
key and correct key should be C. MRCP. [Post anaemic), Plt=45 (thrombocytopenia-leading to Ans. 3. Points in favour: i) Age-6yrs ii) recurrent
operative US of abdomen does not give good easy bruising), Positive paul bunnel test. infection (sorethroat) due to neutrpenia and
result for hepatobiliary system. ERCP is invasive 148. A 41yo woman who has completed her abnormal lymphoblasts which cannot protect
procedure and it has its considerable family, has suffered from extremely heavy periods from infection iii) thrombocytopenia causing gum
complications like cholangitis, injury, pancreatitis for and nose bleeding. Iii) anaemia (pale conjunctiva)
etc. Among given options MRCP is most many years. No medical tx has worked. She due to reduced red cell production from marrow
appropriate. We shall go for ERCP after making admits that she would rather avoid open surgery. occupation by blast cells. [Here debate came why
the dx confirm. For this MRCP is preferred. If the After discussion, you collectively decide on a it is not aplastic anaemia? There is no risk factor
question asks which is the “DEFINITIVE” or the procedure that wouldn’t require open surgery or mentioned for this patient for aplastic anaemia.
“MOST DIAGNOSTIC” then the option will be GA. Select the most appropriate management for There may be congenital aplastic anaemia but
ERCP]. this case. again it would present earlier in life. So it goes
Ans. 2. Diagnosis is choledocolithiasis. a. Endometrial ablation more with leukaemia but it cannot be confirmed
145. A 53yo woman presented with pain in the b. Hysterectomy unless we do bone marrow aspiration.]
eye, blurry vision and clumsiness for 3 months. c. Fibroid resection 152. A 23yo man has been stabbed in the back
She d. Myomectomy and has SOB. The trachea is not deviated, he has
has a hx of difficulty in swallowing and weakness e. Uterine artery embolization engorged neck veins and absent breath sounds on
in her right upper limb 2y ago. What is the inv Ans. The key is E. Uterine artery embolization. the right. What is the most appropriate dx?
of choice? [Done by interventional radiologist expert in a. Tension pneumothorax
a. CSF analysis arterial embolization technique. Particles are b. Cardiac tamponade
b. EEG placed in uterine artery to block circulation to c. Simple pneumothorax
c. EMG uterine body. No operation or GA is required]. d. Hemothorax
d. MRI brain 149. A girl with hx of allergies visited a friend’s e. Pleural effusion
e. Visual evoked response test farm. She got stridor, wheeze and erythematous Q. 1. What is the key?
Q. What is the key? rash. Q. 2. What are the point in favour of your answer?
Q. What is the diagnosis? What is the most appropriate tx? Ans. 1. The key is A. Tension pneumothorax.
Ans. 1. The key is D. MRI brain. a. 0.25ml IM adrenaline
Ans. 2. Points in favour: i) Stab wound in the back e. Reassurance Choose the most appropriate C-Section
ii) SOB iii) Engorged neck vein iv) Absent breath complication in this case?
sound. Q. 1. What is the key? a. Retained POC
These features are common for both hemothorax Q. 2. What is the diagnosis and why? b. Aspiration pneumonitis
and tension pneumothorax and tracheal deviation c. Endometritis
is common to both! But chance of tension Ans. 1. The key is D. Fluid replacement. d. Uterine rupture
pneumothorax is more in stab wond and no Ans. 2. Diagnosis is severe dehydration. Points in e. DIC
tracheal deviation is controversial. This is favour: i) low urine output ii) dry mucous
probability of bad recall!! membrane and iii) low skin turgor. Q. 1. What is the key?
153. A 44yo pt comes with right hemiparesis. Q. 2. What are the points in favour?
Exam: left sided ptosis and left dilated pupil. 157. A 60yo smoker presents with cramp-like pain
Where is in the calves relived by rest and non-healing Ans. 1. The key is C. Endometritis.
the lesion? ulcers. Ans. 2. More handling of tissue like manual
a. Cerebral infarct Exam: cold extremities with lack of hair around removal of placenta, intermittent vaginal bleeding
b. Cerebellar infarct the ankles, absent distal pulses. What is the and raised temperature points toward infective
c. Medulla oblongata most probable dx? process like endometritis.
d. Pons a. Intermittent claudication
e. Midbrain b. Chronic ischemia of the limbs 160. A 30yo woman has brief episodes of severe
Q. 1. What is the key? c. Buerger’s disease shooting pain in the rectum. Rectal examination
Q. 2. What is the name of this condition? d. DVT and
Ans. 1. The key is E. Midbrain. e. DM flexible sigmoidoscopy are normal. What is the
Ans. 2. Weber syndrome [presence of ipsilateral most probable dx?
oculomotor nerve palsy and contralateral Q. 1. What is the key? a. Anal hematoma
hemiparesis or hemiplagia]. Q. 2. Points that support your diagnosis. b. Anal fissure
154. A 50yo man has a stab wound to his left c. Rectal carcinoma
anterior chest at the level of the 4th ICS. He has a Ans. 1. The key is B. Chronic ischaemia of the limb. d. Proctalgia fugax
BP Ans. 2. Intermittent claudication is a symptom not e. Piles
80mmHg, pulse=130bpm. His neck veins are diagnosis. It is not buerger’s disease as buerger
dilated and his heart sounds are faint. His trachea occur in more younger heavy smoker (before the Ans. is D. Proctagia fugax [normal rectal
is age of 50yrs), It is not DVT as dvt pain or examination and flexible sigmoidoscopy excludes
central. What is the most appropriate dx? tenderness is not of an intermittent claudication other options].
a. Cardiac tamponade pattern. Again in DM there is no intermittent
b. Diaphragmatic rupture claudication. 161. A 78yo male, DM and HTN, had a fall and
c. Fractured ribs since then is unable to walk. He presents with
d. Tension pneumothorax 158. An otherwise healthy 13yo boy presents with deformity and tenderness over the right hip area.
e. Traumatic rupture of aorta recurrent episodes of facial and tongue swelling XR=fx of femur neck. What is the single most
Q. 1. What is the Key? and abdominal pain. His father has had similar associated nerve injury?
Q. What are the points in favour of your answer? episodes. What is the most likely dx? a. Sciatic nerve
Ans. 1. Theakey is Cardiac tempoade. a. C1 esterase deficiency b. Gluteal nerve
Ans. 2. Points in favour: i) Systolic BP 80 mmHg ii) b. HIV c. Lateral peroneal nerve
Pulse 130 bpm iii) Engorged neck vein iv) Faint c. Mumps d. Tibial nerve
heart sounds v) Trachea is central. d. Sarcoidosis e. Femoral nerve
155. A 15yo boy has a soft painless swelling in the e. Sjogren’s syndrome
left scrotum, blue in color and can be compressed. Ans. The key is A. Sciatic nerve. [Sciatic nerve
What is the most appropriate next step? Q. 1. What is the key? injury though may occur but may not be very
a. Analgesia Q. 2. What is the name of this condition? common!]
b. Antibiotic Q. 3. Why it is not acquired?
c. Biopsy 162. A 20yo man has a head on collision in a car.
d. Immediate surgery Ans. 1. The key is A. C1 esterase inhibitor On presentation he is breathless, has chest pain
e. Reassurance deficiency. and
Q. 1. What is the key? Ans. 2. Hereditary angioedema. fx of 5-7th rib. CXR confirms this. What is the most
Q.2. What is the name of this condition? Ans. 3. Acquired angeoedema usually manifest appropriate initial action in this pt?
Ans. 1. The key is E. Reassurance. after the age of 40 yrs. a. Antibiotics
Ans. 2. Name of the condition is Varicocele. b. Analgesia
156. A 12yo pt presents with copious diarrhea. 159. A 25yo had an LSCS 24h ago for fetal distress. c. O2 by mask
Exam: urine output=low, mucous membrane=dry, She now complains of intermittent vaginal d. Physiotherapy
skin bleeding. Observations: O2 sat=98% in air, e. Refer to surgeon
turgor=low. What is the most appropriate initial BP=124/82mmHg, pulse=84bpm, temp=37.8C.
management? The Ans. The key is C. O2 by mask. [There was debate
a. Antibiotic midwife tells you that she had a retained in this forum that pain relief should be given first
b. Antimotility placenta, which required manual removal in the which will automatically relieve breathing
c. Anti-emetic OT. problem. But others told O2 first]. O2 first is the
d. Fluid replacement correct answer!
166. A 75yo lady who had mitral valve Q. 1. What is the key?
replacement 13 yrs ago has developed recurrent Q. 2. Why the patient’s death was unnoticed?
breathlessness. Her husband has noticed Ans. 1. The key is A. MI.
prominent pulsation in her neck. She complains of Ans. 2. In diabetics MI become painless when the
abdominal pain and ankle swelling. What is the patient develop autonomic neuropathy (till there
most probable dx? is no autonomic neuropathy diabetic patients will
a. Aortic regurgitation feel MI pain). In this case the disease was
b. Mitral regurgitation unnoticed as it was a painless attack.
c. Mitral stenosis 170. A 38yo pt presented with tingling, numbness,
d. Tricuspid regurgitation paraesthesia, resp stridor and involuntary spasm
[http://www.urmc.rochester.edu/encyclopedia/ e. Pulmonary stenosis of the upper extremities. She has undergone
content.aspx? surgery for thyroid carcinoma a week ago. What is
contenttypeid=22&contentid=flailchest] Ans. The key is D. Tricuspid regurgitation. [Points the most likely dx?
in favour: i) recurrent breathlessness – if the a. Thyroid storm
163. A 28yo man with complains of headache and cause is LV dysfunction, ii) prominent pulsation in b. Hyperparathyroidism
nose bleeds also has pain in the lower limbs on the neck – giant v waves, iii) abdominal pain – c. Unilateral recurrent laryngeal nerve injury
exertion. Exam: radio-femoral delay, cold legs pain in liver on exertion, ankle swelling; These are d. External laryngeal nerve injury
with weak pulse and mild systolic murmur with features of tricuspid regurgitation. Reference:- e. Hypocalcemia
normal S1S2. What is the most probable dx? OHCM, 9th edition, page- 142]. Q. 1. What is the key?
a. TOF Q. 2. What is the cause of this condition?
b. ASD 167. A 45yo T1DM had an annual check up. Q. 3. Why there is respiratory stridor?
c. VSD Ophthalmoscopy showed dot and blot Ans. 1. The key is E. Hypocalcaemia.
d. PDA hemorrhage + Ans. 2. Hypocalcaemia may be due to accidental
e. CoA hard exudate and multiple cotton wool spots. parathyroid gland removal during thyroidectomy.
What is the next step in management? Ans. 3. Laryngospasm is a feature in
Ans. The key is coarctation of aorta. [headache a. Reassurance and annual screening only hypocalcaemia which may cause stridor.
and nosebleeds - >hypertension, pain in lower b. Urgent referral to ophthalmologist
limb on exertion -> as reduced blood supply to leg c. Laser therapy 171. A 50yo chronic smoker came to OPD with
due to coarctation, radio-femoral delay, cold legs d. Non-urgent referral to ophthalmologist complaint of chronic productive cough, SOB and
with week pulse, mid-systolic murmur are all e. Nothing can be done wheeze. Labs: CBC=increase in PCV. CXR >6ribs
features of coarctation of aorta]. seen above the diaphragm in midclavicular line.
Ans. The key is D. Non-urgent referral to ABG=pO2 decreased. What is the most likely dx?
164. A 23yo male has a tonic clonic seizure whilst ophthalmologist. [It is pre-proliferative a. Interstitial lung disease
at college. His GCS is 12, BP=120/77mmHg, retinopathy so non-urgent referral; If proliferative b. Wegener’s granulomatosis
HR=99bpm. What is the most appropriate inv for (with neovascularization) urgent referral]. c. Ca bronchi
his condition? d. COPD
a. CT 168. A 2m baby who has ambiguous genitalia e. Amyloidosis
b. MRI presents to the ED with vomiting. Labs: Q. 1. What is the key?
c. Serum blood glucose Na+=125mmol/L, K+=6mmol/L. What is the most Q. 2. What are the points in favour?
d. Serum drug levels likely dx? Ans. 1. The key is D. COPD.
a. Fragile X syndrome Ans. 2. Points in favour: i) Age 50 yrs ii) Chronic
Ans. The key is C. Serum blood glucose [it is also b. Turners syndrome smoker iii) Chronic productive cough, SOB and
possible that he may have taken drug, even c. Noonan syndrome Wheeze iv) Raised PCV secondary to chronic
though first we have to do serum glucose as low d. Congenital adrenal hyperplasia hypoxaemia v) Low set diaphragm and widened
blood glucose can be very easily managed and it Q. 1. What is the key? horizontal ribs vi) Hypoxaemia on ABG.
needs urgent management to save life. If it is Q. 2. What are the points in favour? 172. A 44yo pt has sudden onset of
excluded then we can look for other causes which Ans. 1. The key is D. Congenital adrenal breathlessness and stridor few minutes after
may be not fatal in short time as hypoglycaemia]. hyperplasia extubation for
Ans. 2. Points in favour: i) ambiguous genitalia ii) thyroidectomy. The pat had longstanding goiter
165. A 20yo man complains of recent onset of salt wasting manifested as hyponatremia and for which he had the surgery. What is the most
itching which followed a viral infection. There are hyperkalemia (In mild forms of salt-wasting likely dx?
numerous wheals of all sizes on his skin adrenal hyperplasia, salt wasting may not become a. Thyroid storm
particularly after he has scratched it. These can apparent until an illness stresses the child). [here b. Hematoma
last up hyperkalaemia inspite of vomiting is indicating the c. Unilateral recurrent laryngeal nerve injury
to an hour. What is the most probable dx? disease]. d. External laryngeal nerve injury
a. Uremia 169. A 40yo man collapsed at home and died. The e. Tracheomalacia
b. Urticaria GPs report says he suffered from T2DM and Ans. The key is tracheomalacia. [Prolonged
c. Psychogenic itching BMI=35. What is the most likely cause of death? pressure over trachea by goiter is a cause of
d. Atopic eczema a. MI tracheomalacia following thryroidectomy].
e. Primary biliary cirrhosis b. DM 173. A 15yo boy presents with generalized edema.
c. HF His urinalysis reveals protein +++, eGFR =110.
Ans. The key is B. Urticaria. d. PE What is the most likely dx?
e. Renal failure a. IgA nephropathy
b. Membranous nephropathy this has happened 3-4x before. What is the most vocal cords by activating the cricothyroid muscle,
c. Minimal change disease likely dx? increasing pitch].
d. PSGN a. Febrile convulsion 181. A 28yo woman at 39wk gestation is in labor.
e. Lupus nephritis b. Absence seizures She develops abdominal pain and HR=125bpm,
Ans. The key is C. Minimal change disease. [Points c. Epilepsy BP=100/42mmHg, temp=37.2C and
in favour: i) Age 15 yrs ii) Generalized oedema iii) d. Partial complex seizure saturation=99%. Exam: lower abdomen is
Protein in urine +++ vi) Normal eGFR of 110 Ans: The key is C. Epilepsy. It is wrong key. Correct exquisitely tender.
(Normal range- 90 to 120 mL/min)]. key should be A. Febrile convulsion. [Points in CTG=prv normal, now showing reduced variability
174. A 72yo man is receiving chemotherapy for favour: seizers in presence of high fever. Same and late deceleration develops with slow
SCLC. He has his 4th tx 8 days ago. He has a cough previous illness indicate seizers during fever which recovery. She has had 1 prv LSCS for a breech
with some green sputum but feels well. goes against epilepsy. Though usually febrile baby. Choose the most appropriate CS
Temp=37.6C. Chest exam = few coarse convulsion dosen’t tend to recur but it can recur complication for this lady?
crepitations in as well. Epilepsy has no relation to fever but is a. Endometritis
the right base. HR=92bpm. CBC: Hgb=12.5g/dL, due to pathology in brain]. b. UTI
WBC=1.1, Neutrophils=0.6, Plt=89. Sputum, 178. A middle aged Asian presents with episodes c. Urinary tract injury
urine and blood culture sent to microbiology. of fever with rigors and chills for last 1y. Blood d. Pleurisy
What is the most appropriate management? film: e. Uterine rupture
a. Broad spectrum antibiotics IV ring form plasmodium with schaffners dots in Ans. The key is E. Uterine rupture. [Features of
b. Broad spectrum antibiotics PO RBCs. What is the drug to eradicate this infection? shock, exquisitely tender lower abdomen with
c. GCSF a. Doxycycline abdominal pain, previously normal CTG now
d. Postpone tx until bacteriology results available b. Mefloquine becoming non-reassuring and previous history of
e. Reassure and send home c. Proguanil LSCS makes the diagnosis of uterine rupture].
Q. 1. What is the key? d. Quinine 182. An 8m infant presented with FTT and
Q. 2. What is the Diagnosis? e. Artesonate constipation. Exam: large tongue and fam hx of
Q. 3. What is the treatment of low WBC count? Q. 1. What is the key? prolonged
Ans. 1. The key is A. Broad spectrum antibiotics IV Q. 2. What does Shuffner’s dot in RBC indicate? neonatal jaundice. What is the most likely dx?
Ans. 2. The diagnosis is lower respiratory tract Ans. 1. The key is B. Mefloquine. Probably wrong a. Downs syndrome
infection. key as mefloquine can not eradicate hepatic cycle! b. Fragile X syndrome
Ans. 3. GCSF subcutaneously. [it is the treatment Primaquine is the drug that can eradicate hepatic c. Praderwilli syndrome
of chemotherapy induced leucopenia] cycle. d. DiGeorge syndrome
175. A 25yo woman with T1DM has delivered a Ans. 2. Shuffners dot indicates, it is plasmodium e. Congenital hypothyroidism
baby weighing 4.5kg. Her uterus is well ovale or plasmodium vivex infestation. Q. 1. What is the key?
contracted. 179. A 35yo woman had an uneventful lap chole Q. 2. What are the points in favour?
Choose the single most likely predisposing factor 18h ago. She has a pulse=108bpm, temp 37.8C. Ans.1. The key is E. Congenital hypothyroidism.
for PPH from the options? There are signs of reduced air entry at the right Ans. 2. Points in fevour:i) FTT (failure to thrive) ii)
a. Atonic uterus base but the CXR doesn’t show an obvious constipation iii) macroglossia iv) prolonged
b. Cervical/vaginal trauma abnormality. What is the most appropriate neonatal jaundice.
c. Retained POC management strategy? 183. A 3m infant has presented with recurrent
d. Large placental site a. Cefuroxime PO infections. He has abnormal facies and CXR shows
e. Rupture uterus b. Ceftriaxone IV absent thymic shadow. What is the most likely
Q. 1. What is the key? c. Chlorpheniramine PO dx?
Q. Reason for your answer. d. Chest physiotherapy a. Downs syndrome
Ans. 1. The key is B. Cervical/vaginal trauma e. Reassure b. Fragile X syndrome
Ans. 2. The baby is a big baby. If patient’s uterus Q. 1. What is the key? c. DiGeorge syndrome
was not well contracted we would fear of atonic Q. 2. What is the diagnosis? d. Marfans syndrome
uterus! But as uterus is well contracted it is not Ans. 1. The key is D. Chest physiotherapy. Q. 1. What is the key?
atonic uterus. Rather most likely cause is trauma Ans. 2. Atelactasis. [Atelectasis can be seen in Q. 2. What are the points in favour?
dring delivery of this big baby. chest x-ray but sometimes it becomes Ans. 1. The key is C. DiGeorge syndrome.
176. A 23yo lady presents with headache. Exam: undetectable in chest x-ray as we are suspecting Ans. 2. Points in favour: i) Early age of onset ii)
photophobia and generalized rash that doesn’t in this case]. abnormal facies iii) absent thymic shadow on
blanch on pressure. What must be done 180. A 20yo pop star singer complains of inability Chest X-ray iii) history of recurrent infection
immediately? to raise the pitch of her voice. She attributes this [Infections are common in children due to
a. IV benzylpenicillin to problems with the immune system's T-cell-
b. Isolate pt the thyroid surgery she underwent a few months mediated response that in some patients is due to
c. Gown and mask back. What is the most likely dx? an absent or hypoplastic thymus]. [in newborne
d. Blood culture a. Thyroid storm can be recognized by convulsions from
b. Bilateral recurrent laryngeal nerve injury hypocalcaemia due to malfunctioning parathyroid
Ans. The key is A. IV benzylpenicillin. c. Unilateral recurrent laryngeal nerve injury glands and low level of parathyroid hormones].
d. External laryngeal nerve injury 184. A 30yo man presents with deep penetrating
177. A 4yo baby has generalized tonic-clonic e. Thyroid cyst knife wound. He said he had TT when he left
seizure and fever of 39C. his mother informs you Ans. The key is D. External laryngeal nerve injury. school.
that [External laryngeal nerve functions to tense the What will you do for him now?
a. Human Ig only
b. Human Ig and TT pulse=88bpm, BP=110/70mmHg. He seems to be 193. A 35yo lady who has been using IUCD for one
c. Full course of tetanus vaccine only sweating and muttering some year now complains of pelvic pain and heavy
d. Human Ig and full course of tetanus vaccine incomprehensible words. What is the most imp painful periods. Select the most likely cause
e. Antibiotic next step? leading to her symptoms?
Ans. The key is B. Human Ig and TT. It is a wrong a. CT a. PID
key!! According to UK greenbook correct key is A. b. MRI b. Endometriosis
Human Ig only. c. Blood sugar c. Adenomyosis
185. A 32yo previously healthy woman has d. Body temp d. Fibroids
developed pain and swelling of both knees and e. IV fluids e. Asherman syndrome
ankles Ans. The key is C. Blood sugar. [Normal vital signs, Q. 1. What is the key?
with nodular rash over her shins. As part of the sweating and collapse during exercise gives Q. 2. What points favour your diagnosis?
inv a CXR has been performed. What is the single suspicion of hypoglycemia which is very easy to Ans. 1. The key is A. PID.
most likely CXR appearance? treat but fatal if left unrecognized. So most Ans. 2. The given picture may have D/D of PID or
a. Apical granuloma important next step is to check the blood sugar]. fibroid. As IUCD is a risk factor for PID, it is the
b. Bilateral hilar lymphadenopathy 190. A 45yo waitress complains of pelvic pain most likely diagnosis of given picture.
c. Lobar consolidation which worsens pre-menstrually and on standing Menorrhagia for fibroids are usually painless].
d. Pleural effusion and 194. The dx cells of Hodgkin disease are:
e. Reticular shadowing in the bases walking. She also complains of post-coital ache. a. T-cells
Q. 1. What is the key? Select the most likely cause leading to her b. R-S cells
Q. 2. What is the name of this condition? What symptoms? c. B-cells
are the points in favour? a. PID d. Macrophages
Ans. 1. The key is B. bilateral hilar b. Endometritiosis e. Auer rods
lymphadenopathy. c. Pelvic congestion syndrome Ans. The key is B. R-S cells. [Diagnostic cell in
Ans. 2. The name is Lofgren’s syndrome. It is the d. Adenomyosis Hodgkins disease is Reed-Sternberg cells].
triad of i) erythema nodosum ii) bilateral hilar e. Premature ovarian failure 195. A 16yo girl is admitted after taking a
lymphadenopathy and iii) arthralgia. Ans. The key is C. Pelvic congestion syndrome. paracetamol OD 4 h ago. She has consumed large
186. A neonate’s CXR shows double bubble sign. [Women with pelvic congestion syndrome amounts
Exam: low set ears, flat occiput. What is the most experience a constant pain that may be dull and of alcohol. Her plasma paracetamol conc is just
likely dx? aching, but is occasionally more acute. The pain is below the conc that would suggest tx. What
a. Downs syndrome worse at the end of the day and after long periods should be the tx option for her?
b. Fragile X syndrome of standing, and sufferers get relief when they lie a. Refer to psychiatry ward
c. Turner’s syndrome down. The pain is worse during or after sexual b. Refer to medical ward
d. DiGeorge syndrome intercourse, and can be worse just before the c. N-acetylcystine
Q. 1. What is the key? onset of the menstrual period. Cause is attributed d. Serum plasma paracetamol
Q. 2. What double bubble sign indicate? to female hormone estrogen induced vascular e. No further investigation
Ans. 1. The key is A. Down’s syndrome. (venous) changes. So it is usually seen in females]. Ans. The key is A. Refer to psychiatry ward. [Short
Ans. 2. Double bubble sign indicate duodenal 191. A 37yo female had a fall with outstretched term acute alcohol consumption causes enzyme
atresia. hand, presented with dinner fork deformity and inhibition as in present case and even then
187. A 19yo boy complains of itching on the site tenderness over the right arm. What is the single paracetamol level is under tx level. So the patients
of insect bite. What is the single most appropriate most associated nerve injury? drug level is in safe side but simultaneous drug
management? a. Axillary nerve overdose and alcohol consumption needs
a. Penicillin oral b. Radial nerve psychiatric evaluation and hence the option here
b. Doxycycline oral c. Musculocutaneous nerve is A].
c. Oral antihistamine d. Median nerve 196. A 64yo woman has been on HRT for 9yrs. She
d. Oral ciprofloxacin e. Ulnar nerve had regular withdrawal bleeds until 3 yrs ago and
e. Reassurance Ans. The key is D. Median nerve. [Median nerve is since then has been taking a no bleed prep.
Ans. The key is C. Oral antihistamine. the nerve injured in Colle’s fracture]. Recently she noticed a brown vaginal discharge.
188. A man presents with scrotal swelling, the 192. A mother comes with her 15m child. Which Choose the single most appropriate initial inv?
swelling is cystic and is non-tender. It is located in of the following will bother you? a. Cervical smear
the a. Shies away from strangers b. High vaginal swab
upper pole of the posterior part of the testis. b. Can walk but not run c. TFT
What is the most likely dx? c. Vocabulary consists of only 2 meaningless d. Transvaginal US
a. Epididymal cyst words Q. 1. What is the key?
b. Testicular ca d. She can’t make a sentence Q. 2. Why this test will be done?
c. Hydrocele e. None Ans. 1. The key is D. Transvaginal US.
d. Teratoma Q. 1. What is the key? Ans. 2. To determine the endometrial thickness!
e. Testicular torsion Q. 2. How many words is told clearly by a 15 In a postmenopausal woman with vaginal
Ans. The key is A. Epididymal cyst. [the location of month old child? bleeding, the risk of cancer is approximately 7.3%
upper pole of the posterior part of testis is the Ans. 1. The key is C. Vocabulary consists of only 2 if her endometrium is thick (> 5 mm) and < 0.07%
common site for epididymal pathology]. meaningless words. if her endometrium is thin (≤ 5 mm).
189. A young footballer has collapsed during a Ans. 2. At 15 months the child can clearly say 5 In postmenopausal women without vaginal
game. During initial evaluation: RR=14/min, words and his first meaningful clear word he says bleeding, the risk of cancer is approximately 6.7%
at 12 months.
if the endometrium is thick (> 11 mm) and 0.002% 201. A 29yo teacher is involved in a tragic RTA. e. Steroids
if the endometrium is thin (≤ 11 mm). After that incident, he has been suffering from Ans. The key is A. ACEI. [renal impairment is
197. A young girl complains of episodic headaches nightmares and avoided driving on the motorway. delayed by ACEI].
preceded by fortification spectra. Each episode He has been dx with PTSD. What is the most 206. A 23 yo girl presented with perioral
last for 2-3 days. During headache pt prefers appropriate management? paresthesia and carpopedal spasm 20 mins after a
quiet, dark room. What is the tx of choice for a. CBT huge
acute b. Diazepam argument with her boyfriend. What is the next
stage? c. Citalopram step for this pt?
a. Paracetamol d. Dosalepin a. SSRI
b. Aspirin e. Olanzepin b. Diazepam
c. Sumatriptan Ans. The key is A. CBT.[CBT is the treatment of c. Rebreath into a paper bag
d. Gabapentin choice in PTSD]. d. Propranolol
e. Cafergot 202. A 5yo child presents with fever. He looks e. Alprazolam
Ans. The key is B. Aspirin. This is a wrong key! pale. His parents say he always feels tired. On Q. 1. What is the key?
Correct key is C. Sumatryptan. [Aspirin though 1st exam: Q. 2. What is the likely diagnosis?
line can cause Reye’s syndrome in young girl and orchidomegaly & splenomegaly. Labs: WBC=1.7, Ans. 1. The key is C. Rebreathing in paper bag.
that is why Sumatryptan is the better option Hgb=7.1, Plt=44. What is the dx? [hyperventilation causes CO2 washout and
here]. a. ALL respiratory alkalosis. If you continue breathing
b. CLL and rebreathing in paper bag it will allow CO2
198. A 60yo pt recovering from a surgery for toxic c. AML concentration to rise in paper bag and as you
goiter is found to be hypotensive, cyanosed in the d. CML rebreath this again and again you will regain some
the RR. Exam: tense neck. There is blood oozing e. Hodgkins washed out CO2 and thus relief to this alkalosis].
from the drain. What is the most likely dx? Ans. The key is A. ALL. [normally in ALL CBC shows Ans. 2. The girl may have anxiety disorder when it
a. Thyroid storm raised WBC, low RBC and low platelet; but it is precipitates leads to hyperventilation syndrome. X
b. Reactionary hemorrhage also possible to all cell lines to be depressed, as is 207. A 25 yo woman has been feeling anxious and
c. Secondary hemorrhage the presented case]. nervous for the last few months. She also
d. Primary hemorrhage 203. A 6wk child is brought in with vomiting, complains of palpitations and tremors. Her
e. Tracheomalacia constipation and decreased serum K+. What is the symptoms last for a few minutes and are very
dx? hard
Ans. The key is B. Reactionary haemorrhage. [in a. Pyloric stenosis to control. She tells you that taking alcohol
the recovery room, cyanosis, hypotension, tense b. Duodenal atresia initially helped her relieve her symptoms but now
neck, woozing of blood from drain; all these goes c. Hirschsprung disease this effect is wearing off and she has her
in favour of reactionary haemorrhage]. d. Achalasia cardia symptoms even after drinking alcohol. What is the
e. Tracheo-esophageal fistula dx?
199. A 33yo man is hit by a car. He loses Ans. The key is A. Pyloric stenosis. [why not a. Panic disorder
consciousness but is found to be fine by the duodenal atresia? Pyloric stenosis is much more b. Depression
paramedics. commoner than duodenal atresia; in duodenal c. OCD
When awaiting doctors review in the ED he atresia the vomitus should contain bile, which is d. Alcohol addiction
suddenly becomes comatose. What is the most not the case in pyloric stenosis]. e. GAD
likely 204. A 17 yo girl had an episode of seizure.
dx? Contraction of muscles started from around the Ans. The key is A. Panic disorder.
a. SAH interphalangeal joints, which spread to the 208. A 2yo child is very naughty. His teacher
b. Subdural hemorrhage muscles of wrist and elbow. Choose possible type complains that he is easily distracted. His parents
c. Intracerebral hemorrhage of say
d. Extradural hemorrhage seizure? that he can’t do a particular task for a long time.
a. Grand mal He sometimes hurts himself and breaks many
Ans. The key is D. Extradural haemorrhage. [Age b. Tonic clonic things. This causes many troubles at home. What
33 (younger age), considerable head trauma, and c. Myoclonic is the dx?
lucid interval (present in both extradural and d. Absent a. ASD
subdural) are the points in favour]. Ans. The key is C. Myoclonic. [seizers associated b. Dyslexia
200. A 77yo male presents with hx of enuresis and with contraction of specific muscle group is seen c. ADHD
change in behavior. Exam: waddling gait. What is in myoclonic seizers]. d. Antisocial personality disorder
the most likely dx? 205. 46yo man, known case of chronic GN e. Oppositional defiant
a. Subdural hemorrhage presents to OPD. He feels well. BP = Ans. The key is C. ADHD (Attention deficit
b. Brain tumor 140/90mmHg. Urine hyperreactive disorder).
c. Normal pressure hydrocephalus dipstick: protein ++, blood ++ and serum 209. A 79 yo lady who is otherwise well recently
d. Psychotic depression creatinine=106mmol/L. Which medication can started abdominal pain. She is afebrile and
Ans. The key is C. Normal pressure hydrocephalus. prevent complains that she passed air bubbles during
[age (usually occurs in 60s or 70s), loss of bladder the progression of this dx? urination. A urethral catheter showed fecal
control (enuresis), waddling gait and behavior a. ACEi leakage in the urinary bag. What is the likely
change are all features of normal pressure b. Diuretics pathology?
hydrocephalus]. c. Cytotoxic meds a. Diuretics
d. Longterm antibiotics b. CD
c. Rectosigmoid tumor Ans. 2. i) as symptomatic only reassurence is not the best inv?
d. Large bowel perforation appropriate ii) as big stone ursodyoxycholic acid is a. MRI cerebellum
e. UC less effective iii) less invasiv is preferred so b. CT cerebellum
laparoscopic rather than laparotomy. c. Skull XR
Ans. The key is B. CD. [debate came that Crohn’s 214. In a man who is neglected and alcohol d. LP
disease cannot occur in 79 yrs but this is not the dependent, whith high suicidal risk, which factor e. Blood culture
case! “Crohn’s disease can occur at any age, but is can Ans. The key is A. MRI cerebellum. [Balance
most frequently diagnosed in people ages 15 - 35. increase this risk further? difficulties, vomitin and nausea suggests
About 10% of patients are children under age 18”. a. Alcohol dependence cerebellar lesion. In posterior fossa lesion MRI is
[http://www.nytimes.com/health/guides/disease/ b. SSRI preferred].
crohns-disease/risk-factors.html]. So I think it can c. S]moking 219. A 2 yo pt presents with colicky pain which
occur in this age also and the feature like fistula is d. Agoraphobia radiates from loin to groin. He complains of
a common association of CD]. e. Court involvement similar
210. A 2 month child with diarrhea and vomiting Ans. The key is A. Alcohol dependence. This is episodes in the past. Inv has been done and 7mm
for 6 days is brought in looking lethargic. What is wrong key!! Correct key should be B. SSRI. [He is stone was found in the ureter. What is the
the already alcohol dependent. So further (additional) most appropriate management?
appropriate initial inv? risk factor is SSRI (SSRI has well known risk of a. Percutaneous nephrolithiotomy
a. BUE developing suicidal ideation]. b. Open surgery
b. Random blood sugar 215. A 71 yo man presents with coarse tremor. He c. Ureteroscopy or laser
c. CBC is on some meds. Which one can be the reason d. Conservative tx
d. CXR for e. ESWL
e. AXR the tremor?
Ans. The key is A. BUE. [Electrolyte imbalance a. Lithium Q. 1. What is the key?
should be checked in this baby presenting with b. Diazepam Q. 2. What treatments are recommended for
diarrhea and vomiting for considerable time]. c. Fluoxetine different sized stones in Adults?
211. A 72 yo man fell while shopping and hurt his d. Imipramine Ans. 1. The key is E. ESWL. Probably a wrong key!
knee. His vitals are fine. He speaks in a low voice e. Haloperidol The correct key is C. Ureteroscopy or laser. [For 6-
and is very slow to give answers. What is the most Ans. The key is A. Lithium. Actually in therapeutic 16 mm stone in pediatric group ureteroscopy is
probable dx? dose lithium causes fine tremor but in toxic dose the treatment of choice. So answer here is C.
a. Alzheimers it causes coarse tremor. So the probable answer is Ureteroscopy or laser].
b. Vascular demetia lithium.
c. TIA 216. A young woman complains of diarrhea,
d. Pseudo-dementia abdominal cramps and mouth ulcers. AXR shows
e. Picks dementia distended transverse colon with globet cell
depletion on rectal biopsy. What is the most
Ans. The key is A. Alzheimers probable dx?
212. A 47 yo man met with a RTA. He has multiple a. CD
injuries. Pelvic fx is confirmed. He has not passed b. UC
urine in the last 4 hrs. What is the next c. Bowel Ca
appropriate management for this pt? d. Bowel obstruction
a. Urethral catheter e. IBS Ans. 2. Stones < 5mm: pass spontaneously,
b. Suprapubic catheter Q. 1. What is the key? Increase fluid intake.
c. IV fluids Q. 2. What are points in favour? Stones 5mm-10mm /pain not resolving: medical
d. IV furosemide Ans. 1. The key is B. UC. expulsive therapy---> Nifedipine or
e. Insulin Ans. 2. In UC there is goblet cell depletion and Tamsulosin(and/or prednisolone).
Q. 1. What is the key? less mucous production in contrast with CD where Stones 10mm-2cm: ESWL or Ureteroscopy using
Q. 2. What is the reason of this management? there may be goblet cell hyperplasia and mucous dormia basket.
Ans. 1. The key is B. Suprapubic catheter. secretion is not reduced. Please note aphthous Stones > 2cm/large/multiple/complex:
Ans. 2. In pelvic fracture there is chance of ulcer can develop in both CD and UC. Percutaneous nephrolithotomy.
urethral rupture and hence displacement of 217. After eating a cookie at a garden party, a
urethral catheter while try to place it. child began to cough and went blue. The mother 220. A footballer has been struck in the groin by a
213. A 49 yo pt presents with right hypochondriac also kick and a presents with severe pain and mild
pain. Inv show a big gallstone. What is the most noticed that there were swollen patches on the swelling in the scrotum. What is the most
appropriate management? skin. What is the dx? appropriate next step?
a. Lap Cholecystectomy a. Allergic reaction a. USG
b. Reassure b. Aspiration of food b. Doppler
c. Low fat diet c. Cyanotic heart disease c. Exploratory surgery
d. Ursodeoxycholic acid d. Trachea-esophageal fistula d. IV fluids
e. Emergency laparotomy e. Achalasia cardia e. Antibiotics
Q. 1. What is the key? Ans. The key is A. Allergic reaction.
Q. 2. Points in favour? 218. A 70 yo man presents with balance Ans. The key is C. Exploratory surgery. [To exclude
Ans. 1. The key is A. Lap Cholecystectomy. difficulties, vomiting and nausea. Which of the torsion].
following is
221. A 47 yo ex-soldier suffers from low mood and bark like cough is diagnostic of croup! So the to diagnose basilar migraine there should at least
anxiety. He can’t forget the images he faces correct key should be a. Corticosteroids. [high history of two migraine attacks with aura. As here
before and has always had flashbacks. He is not fever, bark-like cough in a child suggest croup diagnostic criteria of basilar migraine is not
able to watch the news because there are which is treated with corticosteroids. Drooling is fulfilled we can not discharge the patient without
usually some reports about war. What is he sign of epiglottritis but some authority say it neuroimaging like CT or MRI].
suffering from? occasionally can be present in croup though not
a. Depression usual]. 228. A 25 yo woman was brought to the ED by her
b. PTSD boyfriend. She has many superficial lacerations on
c. Panic attack 225. A 78yo lady on warfarin for atrial fibrillation her forearm. She is so distressed and constantly
d. Agoraphobia lives in a care home. She presents with hx of says her boyfriend is going to end the
e. GAD progressive confusion for three days. She was also relationship. She denies trying to end her life.
noticed to have bruises on her arms. INR = 7. What is the most likely dx?
Ans. The key is B. PTSD. [repeated flashbacks and What is the most probable dx? a. Acute psychosis
tendency to avoid the thoughts of stressor is a. Alzheimers b. Severe depression
diagnostic of PTSD]. b. Delirium c. Psychotic depression
c. Subdural hemorrhage d. Borderline personality disorder
222. A 36 yo woman has recently spent a lot of d. Vascular dementia e. Schizophrenia
money on buying clothes. She goes out almost e. Pick’s dementia
every Ans. The key is D. Borderline personality disorder.
night with her friends. She believes that she Ans. The key is C. Subdural haemorrhage. [Age 78 [ Borderline personality disorder: Act impulsively
knows better than her friends, so she should yrs, living in a care home where unnoticed trivial and develop intense but short-lived emotional
choose injury is possible (like fall), warfarine and high INR attachment to others. They are usually attention
the restaurant for eating out. She gave hx of is potential risk factor of subdural haemorrhage seekers but not suicidal].
having low mood at 12 yo. What’s the dx? suggested by bruises on arms also].
a. Mania 229. A young woman was brought to the hospital.
b. Depression 226. A 28 yo drug user presents to the ED with On exam she has low temperature and tremor.
c. Bipolar affective disorder collapse and anuria. His serum K+ = 7.5mmol/L. She says when she closes her eyes, she can see
d. Borderline personality disorder CXR = colors. What drug has been used?
e. Dysthymia early pulmonary edema. What is the next a. Amphetamines
appropriate management? b. LSD
Ans. The key is C. Bipolar affective disorder. [Initial a. Urgent hemodialysis c. Cocaine
depressive episode (may be befor a long) followed b. IV calcium gluconate d. Heroine
by mania is bipolar affective disorder]. c. IV insulin + dextrose e. Ecstasy
d. Furosemide
223. A 28 yo female presents with a 3m hx of e. IV NS 0.9% Ans. The key is B. LSD.
diarrhea. She complains of abdominal discomfort
and Ans. The key is B. IV calcium gluconate. [IV 230. A lady comes in severe liver disease and
passing stool 20x/day. Exam=febrile. Barium calcium gluconate is given toprotectthe heart hematemesis. Her INR is >10. What should she be
enema shows cobblestone mucosa. What is the from lethal arrhythmia or cardiac arrest from high given?
most likely dx? potassium level and used to buy time while a. FFP
a. Ameoba definitive management is being taken. Actually b. Steroids
b. Colon Ca calcium gluconate don’t lower the potassium level c. Whole blood
c. GE that much but protect the heart from d. IV fluids
d. CD arrhythmia]. e. Vit K
e. UC
227. A 32 yo woman suffers an episode of severe Ans. The key is A. FFP.
Ans. The key is D. CD. [Hx of diarrhea, abdominal occipital headache with vomiting and loss of
discomfort, and patient being febrile indicate gut consciousness. She is brought to the hospital 231. After eating a cookie at a garden party, a
inflammation and cobblestone appearance on where she is found to be conscious and child began to cough and went blue. The mother
barium enema is suggestive of CD]. completely also
alert. Exam: normal pulse & BP. No abnormal noticed that there were swollen patches on the
224. A child is brought in with high grade fever, neurological signs. What is the next step in skin. What is the initial management?
runny nose and bark-like cough. He is also management? a. OTC antihistamine
drooling. a. Admission for observation b. Oxygen
What is the most appropriate tx for this child? b. CT brain c. Bronchodilators
a. Corticosteroids c. MRI head d. Epinephrine IM
b. Paracetamol d. Reassurance and discharge e. Nebulized epinephrine
c. Adrenaline nebulizer e. XR skull
d. IV antibiotics Ans. The key is D. Epinephrine IM [anaphylaxis
e. Intubation under GA Ans. The key is B. CT brain. [basilar migraine can with partially blocked airway].
cause severe headache and LOC. But there occurs
Ans. The key is E. Intubation under GA. This is a no neurological deficit and on recovering from
wrong key! In epiglottitis there is no cough and unconsciousness becomes completely alert. But
232. A 63 yo female is noted to have left pupil unsuccessful relationships. He has 2 boys but
irresponsive to light and is dilated. What is the doesn’t contact them. What is the most probable 239. A 27 yo woman has hit her neck in an RTA
most dx? without complains of tingling or motor loss. What
.probably dx? a. Borderline personality disorder is
a. Pontine hemorrhage b. Schizophrenia the next most appropriate inv?
b. Subdural hemorrhage c. Avoidant personality disorder a. MRI
c. Cerebellar hemorrhage d. Histrionic personality disorder b. XR
d. Extradural hemorrhage e. Antisocial behavior disorder c. CT cervical
e. Subarachnoid hemorrhage d. Diagonal XR
Ans. The key is D. Extradural hemorrhage. This is a Ans. Ans. The key is E. Antisocial behavior
wrong key! Correct key is B. Subdural disorder. [Antisocial personality disorder is a Ans. The key is B. X-ray. [As there is no
hemorrhage. particularly challenging type of personality neurological deficit we can exclude any fracture
disorder, characterised by impulsive, irresponsible by x-ray first]. [Diagonal x ray means ,oblique view
233. A 28yo business executive presents at the and often criminal behavior]. of cervical spine. By this view we can assess facet
GP asking for some help because she has been joint arthopathy. This doesn't related to RTA].
arguing 236. A 60 yo man has a pathological rib fx. He also
with her boyfriend frequently. She is worried complains of recurrent infection. BMA is done. 240. A young female who has many superficial
about her weight, and she thinks she may be fat. Labs: Ca2+ = 3.9mmol/L and ALP = 127u/L. what lacerations was brought into the ED by her
She has been on a diet and lost 7 kgs in the last 2 type of cell would be found in abdundance in boyfriend
months on purpose. She is eating less. She the marrow smear? for superficially lashing her upper arm. She is
used to do a lot of exercise. Now she says she’s a. Plasma cell adamant and screaming that she is not suicidal
feeling down, has some insomnia and feels tired b. Myeloid cell but
and without energy. She has not showed up at c. Bence-jones protein scared her boyfriend wants to leave her. What is
work. She is worried because recently she got a d. Megakaryocytes the dx?
loan to buy a luxury car. She can’t be fired. She e. Reticulocytes a. Acute psychosis
complains about her low mood. She thinks this is b. Severe depression
weird because she used to be extremely Q. 1. What is the key. c. Obsessive
productive. She used to work showing an Q. 2. What is the diagnosis? d. Bipolar
excellent Q. What are the points in favour of diagnosis? e. Borderline personality
performance at the office. She even received f. Schizophrenia
compliments from her boss. How, she says her Ans. 1. The key is A. Plasma cell.
boyfriend is angry because her apartment is a Ans. 2. The diagnosis of multiple myeloma. Ans. The key is acute psychosis. Probably this is
chaos. Usually she spends a lot of time cleaning it, Ans. 3. Points in favour: i) age 60 yrs ii) wrong key! Correct key should be E. Borderline
even upto 3 AM. She liked it to be perfect, but not pathological rib fracture (from metastases) iii) personality disorder. [Borderline personality
it’s a mess. On exam: BMI=23, no other signs. recurrent infection (due to  B cell dysfunction disorder (BPD), also known as emotionally
What is the most probably dx? (manifested as hypogammaglobulinemia), unstable personality disorder, is a long term
a. Anorexia nervosa numerical and functional abnormalities of T cells, pattern of abnormal behavior characterized by
b. Bipolar disease and dysfunction of natural killer cells), iv) raised unstable relationships with other people, unstable
c. Binge eating disorder calcium level. sense of self, and unstable emotions.[3][4] There
d. Hyperthyroidism is often an extreme fear of abandonment,
e. Schizophrenia 237. A child presents with blue marks on the frequent dangerous behavior, a feeling of
sclera, short stature and heart murmur. What is emptiness, and self-harm].
Ans. The key is B. Bipolar disorder. [Now she is the dx?
depressed but before hypomanic which makes a. Osteogenesis imperfect 241. A 22yo woman was brought by her boyfriend
the likely dx of bipolar disorder]. b. Hypopituitarism with multiple superficial lacerations. There are
c. VSD scars of old cuts on her forearms. She is distressed
234. A woman brought her husband saying she d. Achondrogenesis because he wants to end the relationship.
wants the ‘thing’ on his forehead removed. The e. Dwarfism She denies suicide. What is the most likely dx?
husband is refusing tx saying it improves his a. Acute psychosis
thinking. What is the most appropriate next Ans. The key is A. Osteogenesis imperfecta. b. Borderline personality disorder
step? c. Severe depression
a. Assess his mental capacity to refuse tx 238. A 5month child can’t speak but makes d. Schizoid personality
b. Remove lesion sounds. She can hold things with palm, not e. Psychotic depression
c. Refer to ED fingers. Can’t
d. Mini-mental state exam sit independently but can hold her hand and sit Ans. The key is B. Borderline personality.
e. Refuse surgery and send pt back when propped up against pillows. How’s the [Borderline personality disorder (BPD), also
childs development? known as emotionally unstable personality
Ans. The key is A. Assess his mental capacity to a. Normal disorder, is a long term pattern of abnormal
refuse treatment. b. Delayed speech behavior characterized by unstable relationships
c. Delayed sitting with other people, unstable sense of self, and
235. A 37 yo man who has many convictions and d. Delayed motor development unstable emotions.[3][4] There is often an
has been imprisoned many times has a hx of many extreme fear of abandonment, frequent
Ans. The key is A. normal
dangerous behavior, a feeling of emptiness, and c. Outer ear fatigue, headache, and heart racing. Labs:
self-harm]. d. Brain MCV=108fL, Hgb=8.9g/dL. What is the most likely
dx?
242. A 31yo single man lives with his mother. He Ans. The key is A. CSF. [probable fracture base of a. Vit B12 def
usually drives to work. He always thinks when the skull]. b. Iron def
traffic lights change, his mother is calling him, so c. Folate def
he drives back home. What is the dx? 247. A 40 yo manic depressive is noted to have d. Hemolytic anemia
a. OCD high serum levels of lithium and profound e. Anemia of chronic disease
b. GAD hypokalemia. His GP had started him on anti-
c. Schizophrenia HTNs. Choose the single most likely cause? Q. 1. What is the key?
d. Bipolar a. Verapamil Q. 2. What are the points in favour?
e. Cyclothymia b. Amiodarone
c. Ranitidine Ans. 1. The key is Vit. B12 deficiency.
Ans. The key is C. Schizophrenia. [ delusion of d. Lithium Ans. 2. Vit B12 is absorbed mostly in ileum.[As
reference - he thinks that the changing traffic e. Thiazide ileal resection is done B12 is not absorbed leading
lights are giving message to him]. to megaloblastic anaemia].
Ans. The key is E. Thiazide. [Thiazide was
243. A 56yo woman is known case of pernicious prescribed for Hpt and when lithium was 251. A 7yo is brought by his mother who says that
anemia. She refuses to take hydroxycobalamin IM prescribed its level increased due to thiazide and he was well at birth but has been suffering from
as thiazide also caused hypokalemia resulting the repeated chest and GI infections since then. She
she is needle shy. She asks for oral medication. given picture]. also says that he is not growing well for this age.
Why will oral meds be not effective? What is the likely condition of this child?
a. Intrinsic factor def 248. A 74yo man presents with weakness in his a. CF
b. Malabsorption arm and leg from which he recovered within a few b. SCID
c. Irritated gastric mucosa days and short term memory loss. He has an c. Primary Tcell immunodeficiency
d. Lack of gastric acidity extensor plantar response. He has similar d. Primary Bcell immunodeficiency
episodes 2 e. Malabsorption
Ans. The key is A. Intrinsic factor def. [Vitamin B12 years ago and became unable to identify objects
cannot be absorbed without intrinsic factor]. and to make proper judgment. What is the Q. 1. What is the key?
most appropriate dx? Q. 2. What are the points in favour?
244. An 11m baby had an apnea event. The a. Alcoholic dementia
parents are worried that if something like this b. Pick’s dementia Ans. 1. The key is A. cystic fibrosis.
happens in c. Huntington’s disease Ans. 2. CF involved in production of sweat,
the future, how they are to deal. Advise them d. Alzheimer’s disease respiratory mucous, digestive fluid and mucous.
about infant CPR. e. Vascular dementia These secretion becomes thick than normal
a. Index and middle finger compression predisposing to lung and GI infections since birth.
b. Compression with palm of one hand Ans. The key is E. Vascular dementia.
c. Compression with palm of two hands [hemiparesis, memory impairment, extensor 252. A 3yo child has a high temp for 4 days and he
d. Compression with rescue breaths 30:2 planter reflex, inability to identify objects, poor had not seen a doctor. Then mother notices
e. Compression with rescue breaths 15:2 judgment are features of strokes and thus rashes
vascular dementia]. on buccal mucosa and some around the mouth.
Ans. The key is A. Index and middle finger What is the most appropriate dx?
compression. 249. A nurse comes to you saying that she has a. Measles
recently developed the habit of washing her b. Roseola infectiosum
245. A teacher brings in a child who says she fell hands c. Rubella
down after hitting a table. On probing further, you after every 15-20 mins. She is unable to conc on d. Chicken pox
decide that it was most probably an absence her work and takes longer than before to finish e. Impetigo
seizure. What led you to this dx? tasks as she must constantly wash her hands.
a. The child had not eaten since morning What is the most appropriate management? Ans. The key is B. Roseola infectiosum. It is a
b. The child suddenly went blank and there was a. CBT wrong key! The correct key should be A. Measles!
up-rolling of eyes b. SSRI [As the rash developed after 4 days fever the dx is
c. The child started moving his fingers c. ECT measles!].
uncontrollably before he fell d. Antipsychotics
d. The child’s body became rigid and then started e. Desensitization
to jerk
Q. 1. What is the key?
Ans. The key is B. The child suddenly went blank Q. 2. What is the diagnosis?
and there was up-rolling of eyes.
Ans. 1. The key is A. CBT.
246. A man has discharge from his left ear after a Ans. 2. The diagnosis is OCD.
fight. Where is the discharge coming from?
a. CSF 250. A 61yo man underwent a surgery in which
b. Inner ear ileal resection had been done. He complains of
Ans. The key is B. Hypochondriac. [Worry about Ans. 1. The key is E. Hypomania.
having a serious illness]. Ans. 2. i) elevated mood ii) more energy than
before iii) getting more work done at the office
256. A 6wk child presents with progressive (loss of inhibition). These features are common
cyanosis, poor feeding, tachypnea over the first 2 for both mania and hypomania!! Then why it is
wks of not mania? It is not mania as in mania you will get
life and holosystolic murmur. What is the most psychotic symptoms like i) delusion of grandeur ii)
appropriate condition? auditory hallucinations, which are absent here!
a. ASD
b. VSD 259. A 35yo female attempts suicide 10x. There is
c. Tricuspid atresia no hx of psychiatric problems and all neurological
d. PDA exams are normal. What is the best tx?
e. TOF a. Problem focused tx
b. CBT
Q. 1. What is the key? c. Antipsychotic
Q. 2. What are the points in favour? d. Antidepressant
e. ECT

Ans. 1. The key is E. Tetralogy of Fallot. It is a Ans. The key is A. Problem focused tx. [patient is
wrong key!! Correct answer is C. tricuspid atresia. not psychotic and with normal neurology! So she
Ans. 2. Points in favour: i) tachypnoea over first 2 may getting some problem in family life, finance,
wks of life ii)progressive cyanosis iii) poor feeding job or somewhere like this which she is not able
iv) holosystolic murmur of VSD. to cope with and that is leading to her suicidal
253. A 70yo lady presents with fever for 3d and
thoughts].
confusion. There is no significant PMH. What is
the
257. A 29yo woman who was dx to have migraine 260. A 57yo man presents with weight loss,
most probable dx?
presents with severe onset of occipital headache. tiredness, fever and abdominal discomfort. Exam:
a. Delirium
She lost her consciousness. CT=normal. spleen
b. Hypoglycemia
Neurological exam=normal. What is the most palpable up to the umbilicus. Labs: WBC=127,
c. Alzheimers
appropriate management? Hgb=8.7, Plt=138. What is the most likely dx?
d. DKA
a. Repeat CT a. CML
b. MRI b. AML
Ans. The key is A. Delirium. Delirium is an acute
c. LP c. CLL
confusional state and declined cognitive function
d. XR d. AML
which involves changes in arousal (hyperactive,
e. No inv required e. Polycythemia
hypoactive or mixed), perceptual deficits, altered
sleep-wake cycle, and psychotic features such as
Q. 1. What is the key? Q. 1. What is the key?
hallucinations and delusions.
Q. 2. What is the diagnosis? Q. 2. Why it is so diagnosed?
Q. 3. What are the points in favour? Ans. 1. The key is A. CML.
254. An obese mother suffers from OSAS. Which
Ans. 2. Points in favour of CML: i) Age 57 years ii)
of the following inv is best for her?
Ans. 1. The key is E. No investigation required. weight loss iii) abdominal discomfort iv) anaemia
a. ABG
Ans. 2. The diagnosis is basilar migraine. v) fever vi) marked splenomegaly.
b. Overnight pulse-oximetry
Ans. 3. Points in favour i) history of migraine ii)
c. Polysomnography
severe occipital headache iii) LOC iv) CT normal v) 261. A baby born at 34 weeks with a heart
d. EEG
neurological examination is normal. murmur is kept in the incubator for almost 4
weeks. There
Ans. The key is B. Overnight pulse-oxymetry. [It is
258. A 19yo man has been happier and more is no murmur at discharge. What is the likely
already a diagnosed case of OSAS. So no need for
positive than usual, with more energy than he has cause of this murmur?
reconfirmation with polysomnography. If like to
ever a. PDA
know the current status or monitor overnight
felt before for no particular reason. He has been b. TOF
pulse oxymetry is good].
getting more work done at the office today and c. Aneurysm of sinus of Valsalva
has been socializing with his friends as usual. d. Aorto-pulmonary septal defect
255. A 28yo business man came to the sexual
What is the most likely dx? e. AVM
clinic. He was worried that he has HIV infection. 3
a. Atypical depression
HIV
b. Marked depression Q. 1. What is the key?
tests were done and all the results are negative.
c. Bipolar syndrome Q. 2. Why it was present in this baby?
After a few months, he comes back again and
d. Psychosis Q. If it is present after birth what is the
claims that he has HIV. What is the dx?
e. Hypomania management?
a. Somatization
b. Hypochondriac
Q. 1. What is the key? Ans. 1. The key is A. PDA.
c. Mancheusens
Q. 2. What are the points in favour of this Ans. 2. As it is more common in premature baby!
d. OCD
diagnosis? Ans. 3. Management:
e. Schizophrenia
 indomethacin closes the connection in the Ans. 3. Points in favour: i) onset 3 weeks after Ans. The key is A. POP. Probably wrong key!
majority of cases childbirth ii) Depressive symptoms (very low Correct key should be B. Barrier method!
 if associated with another congenital heart mood, insomnia) iii) thought of harming her little [spermatozoa itself acts as a carcinogen!!! So
defect amenable to surgery then prostaglandin E1 baby but no attempt or action of harming. barrier method is the best protection from the
is useful to keep the duct open until after surgical given option!!].
repair. 265. A 65yo woman presents with headache. She
also complains of dizziness and tinnitus. She has 269. A 66yo man, an hour after hemicolectomy
262. A 6yo girl who has previously been well Recently realized she has visual problems. There is has an urine output of 40ml. However, an hour
presented with a hx of tonic-clonic seizures lasting hx of burning sensation in fingers and toes. after
4mins. Her mother brought her to the hospital On exam: splenomegaly, itchy after hot bath. that, no urine seemed to be draining from the
and she appeared well. She is afebrile and didn’t Labs: RBC=87, Hgb=31.9, Plt=796. What is the dx? catheter. What is the most appropriate next step?
lose consciousness during the episode of seizure. a. CML a. IV fluids
She has no neurologic deficit. What is the most b. CLL b. Blood transfusion
appropriate inv for her? c. Polycythemia vera c. Dialysis
a. ABG d. Myelofibrosis d. IV furosemide
b. Serum electrolytes e. NHL e. Check catheter
c. ECG
d. Blood glucose Q. 1. What is the key? Ans. The key is E. Check catheter.
Q. 2. What are the points in favour?
Q. 1. What is the key? 270. A 24yo pt presented with anaphylactic shock.
Q. 2. What are the points in favour? Ans. 1. The key is C. Polycythemia vera. What would be the dose of adrenaline?
Ans. 2. Points in favour: i) hyperviscosity a. 0.5ml of 1:1000
Ans. 1. The key is B. Serum electrolyte. symptoms (headache, dizziness, tinnitus, visual b. 0.5ml of 1:10000
Ans. 2. In epilepsy patient becomes unconscious. problem) ii) pruritus, typically after a hot bath, c. 1ml of 1:500
The child has no previous illness and she was iii)splenomegaly iv) RBC=87, Hb=31.9, Plt=796. d. 5ml of 1:1000
conscious during the episode of seizure. So e. 0.05ml of 1:100
electrolyte imbalance may be the cause. 266. A 29yo male brought to ED in unconscious
state. There is no significant past hx. Which of the Ans the key is A. o.5 ml of 1:1000. [in cardiac
263. A 60yo woman was found by her son. She following should be done as the initial inv? arrest 1 ml of 1:1000 iv].
was confused and had urinary incontinence. She a. CT
has b. Blood glucose 271. A 44yo woman complains of heavy bleeding
recovered fully after 6h with no neurological c. ABG per vagina. Transvaginal US was done and normal.
complaints. What is the most likely dx? d. MRI Which of the following would be the most
a. Stroke e. CBC appropriate inv for her?
b. Vestibular insufficiency a. Hysterectomy
c. TIA Ans. The key is B. Blood glucose. b. Endometrial biopsy
d. Intracranial hemorrhage c. CBC
267. A 45yo woman comes with red, swollen and d. High vaginal swab
Ans. The key is C. TIA. exudating ulcer on the nipple and areola of right e. Coagulation profile
breast with palpable lump under the ulcer. What Ans. The key is E. Coagulation profile.
264. A 34yo woman presents 3 weeks after do you think is causing this skin condition? [Transvaginal US is normal i.e. no endometrial
childbirth. She has had very low mood and has a. Inflammatory cells releasing cytokines hyperplasia, no fibroid or obvious cause for heavy
been b. Infiltration of the lymphatics by the bleeding was found. So now most appropriate
suffering from lack of sleep. She also has thought carcinomatous cells investigation should be coagfulation profile].
of harming her little baby. What is the most c. Infiltration of the malignant skin cells to the
appropriate management for this pt? breast tissue 272. A 60yo woman presented to OPD with
a. ECT dysphagia. No hx of weight loss or heartburn. No
b. CBT Ans. The key is B. Infiltration of the lymphatics by change
c. IV haloperidol the carcinomatous cells. in bowel habits. While doing endoscopy there is
d. Paroxethine some difficulty passing through the LES, but no
e. Amitryptiline 268. A 20yo young lady comes to the GP for other abnormality is noted. What is the single
advice regarding cervical ca. she is worried as her most useful inv?
Q. 1. What is the key? mother a. CXR
Q. 2. What is the diagnosis? past away because of this. She would like to know b. MRI
Q. 3. What are the points in favour of your what is the best method of contraception in c. Esophageal biopsy
diagnosis? her case? d. Esophageal manometry
a. POP e. Abdominal XR
Ans. 1. The key is A. ECT. This is a wrong key. b. Barrier method
Correct key is B. CBT. c. IUCD Q. 1. What is the key?
Ans. 2. The diagnosis is post purtum depression d. COCP Q. 2. What is the diagnosis?
(thinking of harming baby is not psychosis but e. IUS Q. 3. What is the treatment.
depression and attempt or action to harm baby is Ans. 1. The key is D. Oesophageal manometry.
psychosis). Ans. 2. Achalasia cardia
Ans. 3. i) Oral medication: Nitrates or CCB ii) a. Refer to OP psychiatry Ans. The key is D. Nihilism. [nihilism (medical term
Balloon dilatation of the spincter iii) b. Refer to OP ENT is nihilistic delusion): parts of the body do not
Oesophagomyotomy. c. CT brain exist or are dead]
d. Dipstick for B-hCG Guilt: an emotion that occurs when a person feels
273. A 24yo woman presents with deep e. MRI brain that they have violated a moral standard.
dyspareunia and severe pain in every cycle. What Q. 1. What is the key? Hypochondriasis:  worry about having a serious
is the Q. 2. What is the likely diagnosis? illness. 
initial inv? Ans. 1. The key is D. Dipstick for B-hCG. Munchausen’s: a psychiatric factitious
a. Laparoscopy Ans. 2. Likely diagnosis is pregnancy. [Features disorder wherein those affected feign disease,
b. Pelvic US like vomiting, nausea and dizziness are consistent illness, or psychological trauma to
c. Hysteroscopy with early pregnancy supported by delayed draw attention,sympathy, or reassurance to
d. Vaginal Swab menstruation]. themselves. 
277. A 16yo girl came to the sexual clinic. She Capgras syndrome: a delusion that a friend,
Q. 1. What is the key? complains of painful and heavy bleeding. She says spouse, parent, or other close family member (or
Q. 2. What is the likely diagnosis? she pet) has been replaced by an identical-
Q. 3 What is the treatment? doesn’t a regular cycle. What is the most looking impostor.
Ans. 1. The key is B. Pelvic US. appropriate management? 282. A 31yo woman who is 32weeks pregnant
Ans. 2. The likely diagnosis is endometriosis. a. Mini pill attends the antenatal clinic. Labs: Hgb=10.7,
Ans. 3. Treatment: There is no cure for b. Combined pill MCV=91.
endometriosis, but a number of treatments may c. IUS What is the most appropriate management for
improve symptoms. This may include pain d. Anti-prostoglandins this pt?
medication [NSAIDs such as naproxen], hormonal e. Anti-fibrinolytics a. Folate supplement
treatments [COCP, or mirena], or surgery [Surgical Ans. The key is B. Combined pill. b. Ferrous sulphate 200mg/d PO
removal of endometriosis when other measures 278. A 36yo man walks into a bank and demands c. Iron dextran
fail]. money claiming he owns the bank. On being d. No tx req
274. A 38yo woman, 10d postpartum presents to denied, Ans. The key is D. No tx required. [According to
the GP with hx of passing blood clots per vagina he goes to the police station to report this. What NICE, cut offs for iron supplements:
since yesterday. Exam: BP=90/40mmhg, kind of delusions is he suffering from? at booking (8-10 weeks)- if less than 11 
pulse=110bpm, temp=38C, uterus tender on a. Delusion of reference at 28 weeks and further- if less than 10.5
palpation b. Delusion of control if less than these values=> give iron].
and fundus 2cm above umbilicus, blood clots +++. c. Delusion of guilt 283. A 47yo man who is a chronic alcoholic with
Choose the single most likely dx/ d. Delusion of persecution established liver damage, has been brought to the
a. Abruption of placenta 2nd to pre-eclampsia e. Delusion of grandeur hospital after an episode of heavy drinking. His is
b. Concealed hemorrhage Ans. The key is E. Delusion of grandeur. not able to walk straight and is complaining of
c. Primary PPH 279. Which method of contraception can cause double vision and is shouting obscenities and
d. Secondary PPH the risk of ectopic pregnancy? expletives. What is the most likely dx?
e. Retained placenta a. COCP a. Korsakoff psychosis
f. Scabies b. IUCD b. Delirium tremens
Q. 1. What is the key? c. Mirena c. Wernickes encephalopathy
Q. 2. How the condition is defined? d. POP d. Tourettes syndrome
Ans. 1. The key is D. Secondary PPH. Ans. The key is B. IUCD. e. Alcohol dependence
Ans. 2. Secondary PPH: Secondary PPH is defined 280. A woman has pernicious anemia. She has Ans. The key is C. Wernicke’s encephalopathy.
as abnormal or excessive bleeding from the birth been prescribed parenteral vitamin B12 tx but she [triad of i) ophthalmoplegia, ii) ataxia iii)
canal between 24 hours and 12 weeks postnatally. is confusion].
[www.rcog.org.uk/en/guidelines-research- needle phobic. Why is oral tx not preferred for 284. A 32yo woman of 39wks gestation attends
services/guidelines/gtg52/]. this pt? the antenatal day unit feeling very unwell with
275. A 32yo female with 3 prv 1st trimester a. IM B12 is absorbed more sudden onset of epigastric pain a/w nausea and
miscarriages is dx with antiphospholipid b. Intrinsic factor deficiency affects oral B12 vomiting. Temp 36.7C. Exam: RUQ tenderness.
syndrome. Anticardiolipin antibodies +ve. She is utilization Bloods: mild anemia, low plts, elevated LFT and
now 18wks pregnant. What would be the most c. IM B12 acts faster hemolysis. What is the most likely dx?
appropriate d. IM B12 needs lower dosage a. Acute fatty liver of pregnancy
management? e. Pernicious anemia has swallowing difficulties b. Acute pyelonephritis
a. Aspirin Ans. The key is B. Intrinsic factor deficiency affects c. Cholecystitis
b. Aspirin & warfarin oral B12 utilization. d. HELLP syndrome
c. Aspirin & heparin e. Acute hepatitis
d. Heparin only 281. An old man comes to the doctor complaining Ans. The key is D. HELLP syndrome. [H=hemolysis,
e. Warfarin only that a part of this body is rotten and he wants it EL=elevated liver enzyme, LP=low platelet count].
Ans. The key is C. Aspirin & heparin. removed. What is the most likely dx? 285. A 57yo woman presents with dysuria,
276. A 23yo presents with vomiting, nausea and a. Guilt frequency and urinary incontinence. She
dizziness. She says her menstrual period has been b. Hypochondriasis complains of
delayed 4 weeks as she was stressed recently. c. Munchausen’s dyspareunia. Urine culture has been done and is
There are no symptoms present. What is the next d. Nihilism sterile. What is the most appropriate step?
appropriate management? e. Capgras syndrome a. Oral antibiotics
b. Topical antibiotics 290. A 32yo female who has had 3 prv d. Metaplasia
c. Topical estrogen miscarriages in the 1st trimester now comes with e. Hyperplasia
d. Oral estrogen vaginal Ans. The key is A. Squamous to columner
e. Oral antibiotics and topical estrogen bleeding at 8wks. US reveals a viable fetus. What epithelium.
Ans. The key is C. Topical estrogen. [There may be would be the most appropriate definitive 295. A 34yo male presents with hx of headache
UTI like symptoms and dyspareunia in atrophic management? presents with ataxia, nystagmus and vertigo.
vaginitis for which topical oestrogen can be used]. a. Admit Where
286. A pt came to the ED with severe lower b. Aspirin is the site of the lesion?
abdominal pain. Vitals: BP=125/85mmHg, c. Bed rest 2 weeks a. Auditory canal
Temp=38.9C. d. Cervical cerclage b. 8th CN
Exam: abdomen rigid, very uncomfortable during e. No tx c. Cerebellum
par vaginal. She gave a past hx of PID 3 years Ans. The key is B. Aspirin. [Early miscarriage is d. Cerebral hemisphere
ago which was successfully treated with more common in antiphospholipid syndrome and e. Brain stem
antibiotics. What is the appropriate inv? treated with heparin or aspirin when become Ans. The key is C. Cerebellum. [Features described
a. US pregnant]. are consistent with cerebellar lesion].
b. Abdomen XR 291. A 6yo girl started wetting herself up to 296. A 24yo girl comes to the woman sexual clinic
c. CT 6x/day. What is the most appropriate tx? and seeks advice for contraception. She is on
d. High vaginal a. Sleep alarms sodium valproate.
e. Endocervical swab b. Desmopressin a. She can’t use COCP
Ans. The key is A. US. [Patient had previous PID. c. Reassure b. She can use COCP with extra precaution
Current symptoms of severe cervical motion d. Behavior training c. She can use COCP if anticonvulsant is changed
tenderness with significant rise of temperature is e. Imipramine to carbamezapin.
very much suggestive of pelvic abscess]. Ans. The given key is B. Desmopressin.This is d. She can use COCP with estrogen 50ug and
287. A pregnant woman with longterm hx of wrong key! Correct key is D. Behavior training. progesterone higher dose
osteoarthritis came to the antenatal clinic with [behavior training seems to be more appropriate. e. She can use COCP
complaints of restricted joint movement and Desmopressin is given for short term relief Ans. The key is E. She can use COCP. [sodium
severe pain in her affected joints. What is the generally and after alarm bells fail to control valproate has no effect on COCP].
choice of drug? symptoms, it is used in children above 7 yrs 297. A 27yo lady came to the ED 10 days ago with
a. Paracetamol whereas given case is of a child of 6 yrs]. fever, suprapubic tenderness and vaginal
b. Steroid 292. A 27yo 34wk pregnant lady presents with discharge.
c. NSAID headache, epigastric pain and vomiting. Exam: PID was dx. She has been on the antibiotics for
d. Paracetamol+dihydrocoiene pulse=115, BP=145/95mmHg, proteinuria ++. She the last 10days. She presents again with lower
e. Pethadine complains of visual disturbance. What is the abdominal pain. Temp=39.5C. what is the most
Ans. The key is A. Paracetamol. best medication for the tx of the BP? appropriate next management?
288. A 24yo 18wk pregnant lady presents with a. 4g MgSO4 in 100ml 0.9%NS in 5mins a. Vaginal swab
pain in her lower abdomen for the last 24h. She b. 2g MgSO4 IV bolus b. Endocervical swab
had c. 5mg hydralazine IV c. US
painless vaginal bleeding. Exam: abdomen is d. Methyldopa 500mg/8h PO d. Abdominal XR
tender, os is closed. What is the most probable e. No tx e. Laparoscopy
dx? Ans. The given key is A. 4g MgSO4 in 100ml Ans. The key is C. US. [Initial presentation was of
a. Threatened miscarriage 0.9%NS in 5mins. It is a wrong key. Correct key is PID. But recurrance of symptoms suggests
b. Inevitable miscarriage E. No tx. [Here, question specifically asked for tx resistant condition like abscess formation].
c. Incomplete miscarriage of BP. In case of BP of 145/95 mmHg no 298. An 18yo man complains of fatigue and
d. Missed miscarriage treatment for BP is needed. Ref: NICE guideline]. dyspnea, he has left parasternal heave and
e. Spontaneous miscarriage 293. A 24yo lady who is 37wk pregnant was systolic thrill
Ans. The key is A. Threatened miscarriage. brought to the ED. Her husband says a few hours with a harsh pan-systolic murmur at left
[gestational age 18 weeks, lower abdominal pain, ago she parasternal edge. What is the most probable dx?
tender abdomen, closed os and painless vaginal complained of headache, visual disturbance and a. TOF
bleeding indicates threatened abortion]. abdominal pain. On arrival at the ED she has a b. ASD
289. A 2yo child playing in the garden had a clean fit. What is the next appropriate management for c. VSD
cut. She didn’t have any vaccinations. Also, there this pt? d. PDA
is a. 4g MgSO4 in 100ml 0.9%NS in 5mins e. TGA
no contraindication to vaccinations. Parents were b. 2g MgSO4 IV bolus Ans. The key is C. VSD.
worried about the vaccine side effects. What c. 2g MgSO4 in 500ml NS in 1h 299. A young girl presenting with fever, headache,
will you give? d. 4g MgSO4 IV bolus vomiting, neck stiffness and photophobia. She has
a. Clean the wound and dress it e. 10mg diazepam in 500ml 0.9%NS in 1h no rashes. What is the most appropriate test to
b. Give TT only Ans. The key is A. 4g MgSO4 in 100ml 0.9%NS in confirm dx?
c. Give DPT only 5mins [NICE]. [Dx is eclumpsia]. a. Blood culture
d. Give DPT and tetanus Ig 294. What is the pathological change in Barret’s b. Blood glucose
e. Give complete DPT vaccine course esophagitis? c. LP
Ans. The key is E. Give complete DPT vaccine a. Squamous to columnar epithelium d. CXR
course. b. Columnar to squamous epithelium e. CT
c. Dysplasia
Ans. The key is C. LP. [case of meningitis. LP will Ans. 2.  Hypercalcemia in sarcoidosis is due to the [For repeated UTI prophylactic antibiotic should
confirm the diagnosis]. uncontrolled synthesis of 1,25-dihydroxyvitamin be given].
300. A 65yo HTN man wakes up in the morning D3 by macrophages. 1,25-dihydroxyvitamin D3 309. A pt presents with complete anuria following
with slurred speech, weakness of the left half of leads to an increased absorption of calcium in the prolonged hypotension and shock in a pt who
his intestine and to an increased resorption of bled
body and drooling. Which part of the brain is calcium in the bone.  profusely from a placental abruption. What is the
affected? 304. Which of the following is NOT a physiological most probable dx?
a. Left parietal lobe change during pregnancy? a. Post viral infection
b. Right internal capsule a. Tidal volume 500ml b. Acute papillary necrosis
c. Right midbrain b. RBC vol 1.64L c. Acute cortical necrosis
d. Left frontal lobe c. Cardiac output 6.5L/min d. HUS
Ans. The key is B. Right internal capsule. [As d. Uterus weight 1.1kg e. Renal vein thrombosis
symptoms are on left side lesion is on right side of e. ESR up by 4x Q. 1. What is the key?
the brain. So answer should be either b) right Ans. The key is A. Tidal volume 500 ml. Q. 2. What is the reason for this?
internal capsule or c) right midbrain. If it was 305. A 10yo boy presents with nose bleed. What Ans. 1. The key is C. Acute cortical necrosis.
midbraine there would have cranial nerve measure should be taken to stop the bleeding? Ans. 2. There are 2 reasons for this acute cortical
involvement. On the other hand given picture is a. Press base of the nose necrosis. i) significant diminished arterial
very much consistent with lacunar infarction of b. Ice packs perfusion of the kidneys due to spasm of the
internal capsule!] c. Press soft parts of the nose feeding artery secondary to profuse bleeding
301. A 27yo presents with abdominal pain, d. Start tranexemic acid from placental abruption ii) DIC secondary to
bleeding, vomiting and diarrhea. Her LMP was e. IV fluids placental abruption.
7wks ago. Ans. The key is C. Press soft parts of the nose. 310. An alcoholic 56yo man had ascetic fluid
Exam: abdominal tenderness, BP=90/60mmHg. 306. An MI pt who is already on aspirin no longer analysis done which was found to be yellow color.
What is the next appropriate management? smokes and his cholesterol, ECG, echo and BP are What
a. Immediate laparotomy normal. Choose the best option for him: is the most appropriate cause?
b. Laparoscopy a. Give statin a. Alcoholic hepatitis
c. Salpingotomy b. Give statin+warfarin b. Decompensated cirrhosis
d. Salpingectomy c. Low cholesterol diet c. TB peritonitis
e. MTX d. Statin+ACEi d. Pyogenic peritonitis
Q. 1. What is the key? Ans. The key is D. Statin + ACEi. e. Neoplasm
Q. 2. What is the diagnosis? [Offer all people who have had an acute MI Q. 1. What is the key?
Q. 3. Justify the key. treatment with the following drugs: Q. 2. How this diagnosis is made?
Ans. 1. The key is A. Immediate laparotomy.  ACE (angiotensin-converting enzyme) Ans. 1. The key is B. Decompansated cirrhosis.
Ans. 2. The diagnosis is ruptured ectopic inhibitor Ans. 2. If the patient experiences serious
pregnancy .  dual antiplatelet therapy (aspirin plus a problems described below his disease has
Ans. 3. In ruptured ectopic pregnancy if there is second antiplatelet agent) progressed from compensated cirrhosis to
shock we should go for immediate laparotomy.  beta-blocker decompensated cirrhosis:
302. A woman presents with complains of i) Bleeding varices (internal bleeding)
 statin. [2007, amended 2013] [NICE
abdominal pain, unsteadiness, numbness of lower ii) Ascites (fluid in the belly)
guideline].
limb iii) Encephalopathy (confusion)
307. A 46yo man is being treated for a pleural
and palpitations. All inv are normal. What is the iv) Jaundice (yellowing of eyes and
effusion. A chest drain has been sited just below
dx? skin).
the
a. Manchausen 311. A 15yo boy presents with testicular pain for
4th rib in the mid-axillary line on his right side.
b. Somatization 2days. There is no hx of trauma. Exam:
What single structure is at particular risk of injury?
c. Hypochondriac temp=38.5C,right hemi-scrotum tenderness.
a. Arzygos vein
d. Bipolar What is the single most appropriate
b. Diaphragm
Ans. The key is B. Somatization. [This is multiple, management?
c. Intercostal artery
recurrent, medically unexplained symptoms a. Give antibiotics
d. Internal thoracic artery
usually starting early in life. Usually patient b. Give analgesia
e. Liver
presents with one symptom at a time. c. Reassure
Ans. The key is C. Intercostal artery. [Most
Investigations are normal]. d. US scrotum
vulnerable structure is intercostal nerve, then
303. A 34yo African-caribbean man with a hx of e. Exploratory surgery
intercostal artery then intercostals vein. As
sarcoidosis has presented with bilateral kidney
intercostal nerve is not in option intercostal artery
stones. What is the most likely cause for this pt’s Q. 1. What is the key?
is the answer here].
stones? Q. 2. What is the diagnosis?
308. What advice would you give for the parents
a. Hypercalcemia Q. 3. What are the points in favour?
of a child with repeated UTI?
b. Hyperuricemia Ans. 1. The key is A. Give antibiotics.
a. Surgery
c. Diet Ans. 2. The diagnosis is epididymo-orchitis.
b. Prophylactic antibiotics
d. Recurrent UTIs Ans. 3. Points in favour: i) No history of trauma ii)
c. Increase fluids
e. Hyperparathyroidism testicular pain with fever points towards
d. Toilet training
Q. 1. What is the key? epididymo-orchitis.
e. Laxatives
Q. 2. Why this occurs?
Ans. The given key is A. Surgery. This is a wrong
Ans. 1. The key is A. Hypercalcemia.
key. Correct option is B. Prophylactic antibiotic.
312. A 58yo lady presented with urinary psychosis starts within 2 wks (occasionally later) Q. 2. Please justify the key.
incontinence. She looks anxious for her condition. of delivery and it can take 6 -12 months or more Ans. 1. The key is B. Fat necrosis.
Urine to recover from postpartum psychosis]. Ans. 2. Fat necrosis usually occurs following
culture is sterile. Her urodynamic study is normal. 317. A 44yo man presents with periorbital and trauma or surgery. Given case is a fat necrosis of
What is the next step? pedal edema. 24h urine shows 8g of protein/d breast as there is no discharge and there is a
a. Antibiotics and bruise indicating prior trauma.
b. Topical estrogen serum cholesterol=7mmol/L. Renal biopsy results
c. Systemic estrogen are awaited. What would be the most likely dx? 321. A 67yo female who had undergone a radical
d. Duloxetine a. Minimal change disease mastectomy now comes with the complaint of
e. Pelvic floor exercise b. Glomerulonephropathy swelling and redness in her right upper limb.
Ans. The key is E. Pelvic floor exercise. c. Membranous glomerulonephropathy Involvement of which of the following structures
313. A 45yo lady came to family planning clinic for d. FSGS explain these symptoms?
contraception advice. She is not keen to be e. IgA nephropathy a. Epitrochlear LN
pregnant for the next 3yrs. Her recent US showed f. Mesangiocapillary b. Cephalic vein
multiple small submucosal fibroid. What is the Ans. The given key is C. Membranous c. Subclavian artery
best method of contraception for her? glomerulonephritis. [Some authority claims FSGS d. Axillary group of LN
a. Etonogestrol as more common cause of nephrotic syndrome]. e. Long thoracic nerve
b. COCP 318. A 53yo man presents complaining of weight Ans. The key is D. Axillary group of LN. [Axillary
c. IUS loss, lethargy, increasing abdominal discomfort clearance compromise lymphatic flow and may
d. POP and results in swelling of upper limb].
e. IUCD gout for the past yr. Exam: spleen palpated 5cm 322. A 50yo smoker and heavy drinker presents
Ans. The key is C. IUS. [IUS gives 3-5 yrs long below left costal margin, no fluid wave. CBC: with complaints of racing heart. A 24h ECG comes
contraception. It also helps to shrink the fibroid]. Hgb=10.5g/dL, WBC=200 – 85% neutrophils, out normal. What is your next step in
314. A child presents with eczema. She was given plts=100, Na+=140mmol/L, K+ 4mmol/L, management?
two creams by the GP – emollient and steroid. create=151umol/L, urea=7mmol/L. Serum B12 a. ECHO
What increased. Philadelphia chromosome +ve. What b. Reassure
advice would you give her regarding application of is the most likely dx? c. Stress test
the cream? a. CML Ans. The key is B. Reassure. [Smoking and alcohol
a. Sparingly use both the cream b. CLL excess can cause palpitation without any
b. First use emollient, then steroid c. AML recognizable arrhythmia and for this no treatment
c. Apply steroid then emollient d. ALL is required].
d. Mix emollient & steroid before use e. Lymphoma 323. A 47yo man comes to the GP with a swelling
e. Emollient at night with steroid Q. 1. What is the key? in his left groin which disappears on lying down.
Ans. The key is B. First use emollient, then steroid. Q. 2. What are points in favour of this diagnosis? The swelling was bluish in color and felt like a bag
[emmolient 30 minutes before steroid]. Ans. 1. The key is A. CML. of worms. He also complains of a mass in the
315. All the following drugs do not cause Ans. 2. Points in favour: i) wt loss ii) lethargy iii) left loin along with hematuria occasionally. What
bronchoconstriction except? abdominal discomfort iv) splenomegaly v) gout could be the possible dx?
a. Atenolol [Elevated uric acid and vitamin B12 levels are a. Left sided RCC
b. Salbutamol found in 25% of patients of CML]. A blood picture b. Varicosity 2nd to liver disease
c. Salmetrol is suggestive and +ve Philadelphia chromosome is c. Testicular tumor
d. Ipratropium bromide diagnostic. d. UTI
e. Cocaine 319. In a group of cancer pts, 10 died that wasn’t e. IVC obstruction
Ans. The key is A. Atenolol. treated while 5 died in the tx group. Which Q. 1. What is the key?
316. A 28 yo female who delivered 6 weeks ago statement is correct? Q. 2. What is the condition described?
feels sad and has no interest to feeding the baby. a. Absolute risk =10 Q. 3. What is the link between these two
She has been eating poorly and having difficulty b. Relative risk =10 conditions?
sleeping. She feels weak throughout the day and c. Relative risk =5 Ans. 1. The key is A. Left sided Renal cell
has stopped taking the baby out of the house. She d. Absolute risk=5 carcinoma.
also says that the baby has evil eyes. What is the e. Relative risk=2 Ans. 2. Left sided varicocele.
most likely diagnosis? Ans. Given key is E. relative risk = 2. [RR= Number Ans. 3. Most common secondary cause of left
a. Postpartum blues of death in not treated group/number of death in sided varicocele is RCC. Newly diagnosed
b. Postpaetum depression treated group]. varicocele over the age of 40yrs are very much
c. Postpurtum psychosis 320. A 67yo woman has presented with hard, suggestive of RCC. Varicocele is common on left
d. Schizophrenia irregular, poorly defined 5cm lump in her right side as left testicular veins drain to the left renal
e. Psychotic depression breast. vein, while the right testicular vein drain directly
Q. 1. What is the key? She has a bruise on the surface and there is no into IVC.
Q. 2. What are the points in favour? discharge. What is the most likely dx? 324. A man presents with muffled hearing and
Ans. 1. The key is C. Postpartum psychosis. a. Fibroadenosis feeling of pressure in ear with tinnitus and
Ans. 2. Points in favour: i) features of depression: b. Fat necrosis vertigo. Healso complains of double vision when
feels sad, poor eating, difficulty sleep, feeling c. Fibroadenoma looking to the right. What is the most appropriate
weak ii) delusional ideas: thinks baby has evil eyes d. Duct ectasia dx?
and not taking the baby out of the house. These e. Ca breast a. Meniere’s disease
points to postpartum psychosis. [Postpartum Q. 1. What is the key? b. Acoustic neuroma
c. Acute labyrinthytis 328. A 6yo fell on outstretched hand while ST elevation. CXR: loss of margin at costo-
d. Meningioma playing. He feels tender at the elbow but vertebral angle. What is the single most likely
e. Otosclerosis otherwise well. cause?
Q. 1. What is the key? What is the most likely dx? a. Cardiac tamponade
Q. 2. Justify the key. a. Spiral fx b. Mitral regurge
Ans. 1. The key is B. Acoustic neuroma. b. Green stick fx c. Dressler’s syndrome
Ans. 2. Hearing loss, feeling of pressure in the ear c. Compound fx d. Atrial fib
with tinnitus, vertigo and involvement of cranial d. Supracondylar fx e. Emboli
nerve i.e. right abducent nerve are suggestive of e. Pulled elbow
acoustic neuroma. Ans. The key is B. Green stick fracture. Q. 1. What is the key?
325. In 85% of the population this artery is 329. A man has a BP of 160/90mmHg, Q. 2. Why it is not reinfarction as there is ST
dominant. What is the single most appropriate proteinuria++. KUB US are equally reduced in size elevation?
option? with smooth
a. Left ant descending artery borders and normal pelvic calyceal system. What Ans. 1. The key is C. Dressler’s syndrome.
b. Coronary sinus is the cause of HTN in the pt? Ans. 2. There is pericardial rub there is pericarditis
c. Circumflex artery a. Chronic glomerulonephritis and in pericarditis there is widespread ST
d. Left main stem, post descending artery b. Chronic pyelonephritis elevation. So the condition is not new MI but
e. Right coronary artery c. Bilateral renal artery stenosis Dressler’s syndrome.
Q. 1. What is the key? d. Essential HTN
Q. 2. Justify the key. e. Polycystic kidney 334. A 12yo girl presented with tics, LOC, no
Ans. 1. The key is E. Right coronary artery. Ans. The key is bilateral renal artery stenosis. This residual sign and no post-ictal phase. EEG
Ans. 2. If the posterior discending artery is is probably a wrong key. The correct key should abnormality in
supplied by the circumflex artery then it is left be A. Chronic glomerulonephritis. [In bilateral temporal lobe. The girl had a rapid recovery.
dominant and if posterior descending artery is renal artery stenosis BP is very high (much more What is the most probably dx?
supplied by the right coronary artery then it is than 160/90 mmHg]. a. Generalized tonic-clonic
right dominant. As in 85% of population posterior 330. A lady presents with abdominal pain, dysuria, b. Myoclonic
descending artery is supplied by right coronary dyspareunia and vaginal discharge. What si your c. Partialgeneralized seizure
artery it is called the dominant that is right next step? d. Atonic seizure
coronary artery is dominant. a. Laparoscopy e. Febrile convulsion
326. A 54 yo lady presents with sudden, severe b. High vaginal swab
pain in the left half of her skull. She also c. Hysteroscopy Ans. The key is C. Partial –> generalized seizure
complains of d. Laparotomy
pain around her jaw. What is the next likely step? e. US 335. A 48yo woman who has been taking
a. CT Ans. The key is B. High vaginal swab. [Probable medications for asthma for a long time has now
b. MRI diagnosis is PID]. presented
c. Fundoscopy 331. An old lady 72yo staying at a nursing home with decreasing vision. What is the most probable
d. ESR for a few years, a known HTN on reg tx presented cause for her decrease in vision?
e. Temporal artery biopsy with sudden dysphagia while eating with drooling a. Inhaled salbutamol
Q. 1. What is the key? of saliva and req urgent inv. What would be b. Inhaled steroids
Q. 2. What is the diagnosis? your next step? c. Aminophylline
Q. 3. What are the points in favour of your a. Ba swallow d. Beta-blockers
diagnosis? b. Chest CT e. Oral steroids
Ans. 1. The key is ESR. c. Endoscopy
Ans. 2. The diagnosis is Giant cell arteritis or d. Laryngoscopy Q. 1. What is the key?
temporal arteritis. e. CXR Q. 2. Justify the key.
Ans. 3. Points in favour: i) Age >50yrs ii) Female f. Endoscopy with biopsy
sex iii) Severe pain in the left half of skull iv) Pain  Ans. The key is C. Endoscopy. [Probable Ans. 1. The key is E. Oral steroid.
around the jaw (jaw claudication). impacted food bolus (usually meat)which can be Ans. 2. Prolonged steroid use leads to cataract
327. A teenage girl who was ‘fine’ until her visualized and removed with the aid of formation.
boyfriend said he didn’t want the relationship endoscopy].
anymore. 332. A man presents with outward deviation of 336. A 34yo man after a car crash is in the ED and
She took 10 tablets of paracetamol in front of his his right eye and diplopia. Which nerve is deteriorating. His GCS has fallen from 13 to 7.
mother after taking alcohol. What should you affected? What is the most appropriate next step?
do? a. Left trochlear a. CT
a. Refer to psychiatry b. Left oculomotor b. Burr hole
b. Counselling c. Right trochlear c. MRI
c. GP to sort out family issues d. Right abducens d. Intubation
d. Return to work to relieve her anger e. Right oculomotor e. IV fluids
Ans. The key is A. Refer to psychiatry. [1o tablets Ans. The given key is B. Left oculomotor! It is
of paracetamol is not a life threatening toxic dose wrong key! Answer should be D. Right abducens. Ans. The key is D. Intubation. [ABC protocol].
and simultaneous drug overdose and alcohol 333. A 60yo pt who has had a MI a week back
consumption needs psychiatric evaluation]. presents with dyspnea and pericardial rub. ECG 337. A pt with alternating swings or episodes from
shows elation and depression had underwent tx and
gotten better. What medication needed to be 341. A 54yo woman has presented with episodes of bleeding and treatment is given with IV
continued so he can stay well? of abdominal ache, vomiting and postural antibiotics].
a. Anxiolytics hypotension. She also has a dark pigmentation of
b. Mood stabilizers her skin. A dx of Addison’s disease was made. 346. A child was admitted following a RTA with
c. Antidepressants What is the most likely electrolyte abnormality initial GCS=15. Then during the night the noticed
d. Antipsychotics expected in this pt? GCS
a. High Na+, Low K+ reduced to 13. What is the management?
Ans. The key is B. Mood stabilizers [bipolar b. Low Na+, High K+ a. Refer to neuro-surgeon
disorder treated with mood stabilizers]. c. Low Na+, Low K+ b. IV fluids
d. High Na+, High K+ c. Oxygen
338. A 40yo male with pre-existing e. Low Na+, Normal K+ d. CT brain
glumerulonephritis having proteinuria and Ans. The key is B. Low Na+, High K+. [ with e. Skull XR
hematuria suddenly Addison disease, the sodium, chloride, and carbon
deteriorates and presents with oliguria and serum dioxide levels are often low, while the potassium Ans. The key is D. CT brain. [probable intracranial
K+=7.8mmol/L, urea=13mmol/L, level is high]. haemorrhage].
creat=342mmol/L, GFR=19mL/h. The best 342. An 8yo returned from Spain with severe pain
management would be? in one ear. Exam: pus in auditory canal, tympanic 347. A 57yo woman who is suffering from HTN,
a. Calcium supplement membrane looks normal. What is the tx option? presented to the hospital with complaints of
b. Calcium resonate enema 30g a. Gentamicin topical recurrent falls when trying to get out of bed or
c. 10units insulin with 50% dextrose b. Amoxicillin PO getting up from sitting. She is on some anti-HTN
d. Nebulized salbutamol c. Analgesia therapy with no other med prbs. What is the
e. 10ml of 10% calcium gluconate d. Amoxicillin IV cause of her fall?
f. Hemodialysis urgent Q. 1. What is the key? a. CCB
Q. 2. What is the diagnosis? b. Vertibrobasiliar insufficiency
Q. 1. What is the key? Ans. 1. The key is A. Gentamycin topical. c. Thiazide
Q. 2. Justify the key. Ans. 2. Diagnosis is otitis externa. d. Hypoglycemia
343. A 6wk child is very sick-looking. Bloods: e. Infection
Ans. 1. The key is E. 10 ml of 10% calcium Na+=124, K+=2.8. Dehydrated. What would you
gluconate. choose Ans. The key is C. Thiazide. [It causes postural
to resuscitate? hypotension by volume depletion].
Ans. 2. To prevent cardiac arrhythmia. [Actually a. 0.18% NS + 4% dextrose + 20mmol KCl
calcium gluconate neither shifts K+ to cells nor b. 0.9% NS 348. A 56yo woman with MS presents with
reduces serum K+ levelthat much. It just prevents c. 0.45% NS drooping of the left side of her lips. She also has
cardiac arrest or life threatening cardiac d. 0.45% NS + 5% dextrose loss of
arrhythmia and buys time till definitive measures e. 0.45% NS + 5% dextrose + 20 mmol KCl sensation over her face, hearing impairment and
are taken]. Ans. The given key is E. But it is wrong key! The some in-coordination of her movements. What
correct key is B. 0.9% NS. Explanation: is the most likely anatomical site affected?
339. 34yo man was brought to the ED after a RTA. Rsuscitation is mostlydone with 0.9% NS or ringers a. Cerebellum
BP=50/0mmHg and chest wall not moving lactate, or hartmans solution. Here is b. Cerebrum
symmetrically, RR=34bpm. What would be initial hypokalemia. To treat hypokalaemia the cut off c. Spinal cord
action? value is below 2.5 mmol/L and absence of anuria d. Brain stem
a. IV fluid infusion during resuscitation. Maintenance is with fluid E. e. Optic nerve
b. Intubation and ventilation 344. A 68yo man gets repeated attacks of LOC and
c. CT chest TIA. What is the most likely cause for this? Ans. The key is D. Brain stem. Features of 5, 7, 8 th
d. Transfer to ITU a. Atrial fib cranial nerve and cerebellum involvement
b. Mitral stenosis suggestive of brainstem lesion.
Ans. The key is B. Intubation and ventilation [ABC c. Aortic stenosis
protocol]. d. HOCM 349. A 68yo male presented with swelling in the
e. Carotid artery stenosis lower pole of the parotid gland for the last 10yrs.
340. A pt complains of SOB, wheeze, cough and Exam: firm in consistency. What’s the most
nocturnal waking. He has dry scaly shin with Ans. The key is E. Carotid artery stenosis. probable dx?
rashes a. Pleomorphic adenoma
that are itchy. What is the single most likely dx? 345. Pt presented with hemoptysis 7d post- b. Adenolymphoma
a. Scabies tonsillectomy. What is the next step? c. Mikulicz’s disease
b. Eczema a. Packing d. Parotiditis
c. Rheumatism b. Oral antibiotics + discharge e. Frey’s syndrome
d. Dermatitis c. Admit + IV antibiotics
e. Psoriasis d. Return to theatre and explore Ans. The key is A. Pleomorphic adenoma.
e. Ice cream and cold fluids [Pleomorphic adenoma (most common) - also
Ans. The key is B. Eczema. [Asthma may be called benign mixed tumour: is the most common
associated with atopy]. Ans. The key is C. Admit + IV antibiotic. [infection tumour of the parotid gland and causes over a
is a common cause of secondary haemorrhage. third of submandibular tumours. They are slow-
Patient should be admitted to observe the course
growing and asymptomatic, having a malignant areas frequently exposed to the sun, such as the Q. 3. What is the diagnosis?
potentiality]. rim of the ear, lower lip, face, balding scalp, neck, Ans. 1. The key is D. Synchrnized DC
hands, arms and legs. SCCs often look like scaly cardioversion.
350. A 28yo shipyard worker was admitted for red patches, open sores, elevated growths with a Ans. As the patient is in probable hemodynamic
pain in calf while at work which has been central depression, or warts; they may crust or instability (suggested by cool peripheries due to
increasing bleed. A tissue sample (biopsy) will be examined low BP or inadequate CO) so we should go for DC
over the last 3m. There is no hx of HTN or DM but under a microscope to arrive at a cardioversion.
he is a smoker. Exam: loss of posterior tibial diagnosis. Squamous cell carcinomas detected at Ans. 3. Probable diagnosis is SVT.
and dorsalis pedis pulsation along with a non- an early stage and removed promptly are almost 357. A 7yo child presented with chronic cough
healing ulcer at the base of the right 1st MTP always curable and cause minimal damage].  and is also found to be jaundiced on examination.
joint. 353. A 47yo man with hx of IHD complains of What is the most likely dx?
What is the most probably dx? chest pain with SOB on exertion over the past few a. Congenital diaphragmatic hernia
a. Thromboangitis obliterans days. b. Congenital cystic adenematoid malformation
b. Sciatica ECG normal, Echo= increased EF and decreased c. Bronchiolitis
c. DVT septal wall thickness. What is the most likely dx? d. RDS
d. Baker’s cyst a. Dilated CM e. Alpha 1 antitrypsin deficiency
e. Embolus b. Constrictive pericarditis Q. 1. What is the key?
c. Amyloidosis Q. 2. Justify the key.
Q. 1. What is the key? d. Subacute endocarditis Ans. 1. The key is E. Alpha 1 antitrypsin deficiency.
Q. 2. What are the points in favour? Ans. The key is A. Dilated CM. [In dilated Ans. 2. Unexplained liver disease with respiratory
cardiomyopathy ejection fraction is decreased symptoms are very suggestive of AATD.
Ans. The key is A. Thromboangitis obliterans. (but here increased which goes in favour of 358. A 35yo construction worker is dx with
Ans. 2. i) young age ii) smoker iii) pain in cuff iv) constrictive pericarditis). On the other hand indirect inguinal hernia. Which statement below
loss of posterior tibial and dorsalis pedis pulsation decreased septal wall thickness favours the best
v) non-healing ulcer at the base of the right 1st diagnosis of dilated cardiomyopathy. So it seems describes it?
MTP joint all are suggestive of Buerger’s disease. to be a bad recall!!]. a. Passes through the superficial inguinal ring only
354. An elderly pt who is known to have DM b. Lies above and lateral to the pubic tubercle
351. A 35yo lady presents with painful ulcers on presents to the hospital with drowsiness, tremors c. Does not pass through the superficial inguinal
her vulva, what is the appropriate inv which will and ring
leadto the dx? confusion. What inv should be done to help in d. Passes through the deep inguinal ring
a. Anti-HSV antibodies further management? Ans. The key is D. Passess through the deep
b. Dark ground microscopy of the ulcer a. Blood sugar inguinal ring.
c. Treponema palladium antibody test b. ECG 359. A woman has numerous painful ulcers on her
d. Rapid plasma regain test c. Standing and lying BP vulva. What is the cause?
e. VDRL d. Fasting blood sugar a. Chlamydia
Ans. The key is A. Anti-HSV antibodies. [Genital e. CT b. Trichomonas
Herpes may be asymptomatic or may remain Ans. The key is A. Blood sugar. c. Gardenella
dormant for months or even years. When 355. A 28yo pregnant woman with d. HSV
symptoms occur soon after a person is infected, polyhydramnios and SOB comes for an anomaly e. EBV
they tend to be severe. They may start as multiple scan at 31 wks. Ans. The key is D. HSV.
small blisters that eventually break open and US= absence of gastric bubble. What is the most 360. A 72 yo man has been on warfarin for 2yrs
produce raw, painful sores that scab and heal likely dx? because of past TIA and stroke. What is the most
over within a few weeks. The blisters and sores a. Duodenal atresia important complication that we should be careful
may be accompanied by flu-like symptoms with b. Esophageal atresia with?
fever and swollen lymph nodes. c. Gastrochiasis a. Headache
There are three major drugs commonly used to d. Exomphalos b. Osteoporosis
treat genital e. Diaphragmatic hernia c. Ear infection
herpessymptoms: acyclovir (Zovirax), famciclovir ( Ans. The key is B. Oesophageal atresia. d. Limb ischemia
Famvir), and valacyclovir(Valtrex). These are all 356. A 1m boy has been brought to the ED, e. Diarrhea
taken in pill form. Severe cases may be treated conscious but with cool peripheries and has Ans. The given key is E. Diarrhoea which is
with the intravenous (IV) drug acyclovir]. HR=222bpm. considered as a wrong key and A. Headache is the
352. A 53yo man presents with a longstanding hx He has been irritable and feeding poorly for 24h. correct key. [Headache is the warning sign of
of a 1cm lesion on his arm. It has started bleeding CXR=borderline enlarged heart with clear lung hemorrhagic stroke].
on touch. What is the most likely dx? fields. ECG=regular narrow complex tachycardia, 361. A 55yo man has been admitted for elective
a. Basal cell carcinoma with difficulty identifying p wave. What is the herniorraphy. Which among the following can be
b. Kaposi’s sarcoma single most appropriate immediate tx? thereason to delay his surgery?
c. Malignant melanoma a. Administer fluid bolus a. Controlled asthma
d. Squamous cell carcinoma b. Administer oxygen b. Controlled atrial fib
e. Kerathoacanthoma c. Oral beta-blockers c. DVT 2yrs ago
Ans. The key is D. Squamous cell carcinoma. [SSCs d. Synchronized DC cardio-version d. Diastolic BP 90mmHg
Arises in squamous cells. SCCs may occur on all e. Unilateral carotid sinus massage e. MI 2 months ago
areas of the body including the mucous Q. 1. What is the key?
membranes and genitals, but are most common in Q. 2. Justify the key.
Ans. The key is E. MI 2 months ago [better go for d. Stop warfarin and add heparin radiotherapy x) Pioglitazone for more than one
surgery 6 months post MI as surgery before this e. Stop warfarin year and certain herb xi) Arsenic in drinking water
has higher mortality rate]. Ans. The key is D. Stop warfarin and add heparin. xii) Low fluid consumption.
362. A 65yo known case of liver ca and metastasis 366. A 65yo known alcoholic is brought into
presents with gastric reflux and bloatedness. On hospital with confusion, aggressiveness and 369. A 34yo woman is referred to the endocrine
bone exam there is osteoporosis. He also has ophthalmoplegia. He is treated with diazepoxide. clinic with a hx of thyrotoxicosis. At her 1st
basal consolidation in the left lung. What is the What other drug would you like to prescribe? appointment she is found to have a smooth
next appropriate step? a. Antibiotics goiter, lid lag and bilateral exophthalmos with
a. PPI IV b. Glucose puffy
b. Alendronate c. IV fluids eyelids and conjunctival injection. She wants to
c. IV antibiotics d. Disulfiram discuss the tx of her thyroid prb as she is keen to
d. Analgesic e. Vit B complex become pregnant. What is the most likely tx you
e. PPI PO would advise?
Ans. Here is two key C. IV antibiotics and E. PPI Ans. The key is E. Vitamin B complex. a. 18m of carbimazole alone
PO. Correct key is C. IV antibiotics. [Pneumonia b. 18m of PTU alone
should be treated first]. 367. A 32yo woman has severe right sided c. A combo od anti-thyroid drug an0d thyroxine
363. A 66yo man has the following ECG. What is abdominal pain radiating into the groin which has d. Radioactive iodine
the most appropriate next step in management? lasted for 3h. She is writhering in pain. She has no e. Thyroidectomy
abdominal signs. What is the most likely cause of
her abdominal pain? Q. 1. What is the key?
a. Appendicitis Q. 2. What is the reason?
b. Ruptured ectopic pregnancy
c. Salpingitis Ans. 1. The key is B. 18m of PTU alone.
d. Ureteric colic
e. Strangulated hernia Ans. 2. Other drug option i.e Carbimazole is
teratogenic [can cause i) spina bifida
Q. 1. What is the key? ii)cardiovascular malformations, hypospadius etc]
Q. 2. Abdominal pain radiating to groin, at which if become pregnant. PTU is on the other hand
a. Metoprolol level of stone does it indicate? relatively safe in pregnancy.
b. Digoxin
c. Carotid sinus massage Ans. 1. The key is D. Ureteric colic. 370. A child living with his stepfather is brought by
d. Adenosine Ans. 2. It indicate stone at lower ureter. [i) Pain the mother with multiple bruises, fever and
e. Amiodarone. from upper ureteral stones tends to radiate to the fractures. What do you suspect?
Ans. The key is A. Metoprolol. [P waves are flank and lumbar areas. ii) Midureteral calculi a. NAI
replaced by fibrillatory f-waves. Irregular R-R cause pain that radiates anteriorly and caudally. b. Malnutrition
intervals. Dx atrial fibrillation]. This midureteral pain in particular can easily c. Thrombocytopenia
364. A 22yo sexually active male came with 2d hx mimic appendicitis on the right or acute d. HIV
of fever with pain in scrotal area. Exam: scrotal diverticulitis on the left. iii) Distal ureteral stones
skin cause pain that tends to radiate into the groin or Ans. The key is A. NAI. [H/O living with stepfather,
is red and tender. What is the most appropriate testicle in the male or labia majora in the female. multiple bruises, fever and fractures are
dx? suggestive of NAI].
a. Torsion of testis 368. A 39yo coal miner who smokes, drinks and
b. Orchitis has a fam hx of bladder cancer is suffering from 371. A young man who was held by the police was
c. Inguinal hernia BPH. The most important risk factor for his punched while in custody. He is now cyanosed
d. Epididymo-orchitis bladder carcinoma is? and unresponsive. What is the 1st thing you
a. Fam hx would do?
Q. 1. What is the key? b. Smoking a. IV fluids
Q. 2. How will you differentiate torsion from c. Exposure to coal mine b. Clear airway
epididymo-orchitis? d. BPH c. Turn pt and put in recovery position
Ans. 1. The key is D. Epididymo-orchitis. d. Give 100% oxygen
Ans 2. In epididymo-orchitis there should be Q. 1. What is the key? e. Intubate and ventilate
fever, elevation of testes reduces pain (positive Q. 2. What are the risk factors for bladder cancer?
prehn sign), In torsion testis lies at a higher level. Ans. The key is B. Clear airway. [ABC protocol].
In torsion urinalysis negative but in epididymo- Ans. 1. The key is B. smoking.
orchitis it is positive. Epididymo-orchitis usually Ans. 2. Risk factors of bladder cancer: i) Smoking 372. A HTN male loses vision in his left eye. The
occurs in sexually active man. ii) Exposure to chemicals used in dye industry iii) eye shows hand movement and a light shined in
365. A man on warfarin posted for Whites are more likely to develop bladder cancer the eye is seen as a faint light. Fundus exam:
hemicolectomy. As the pt is about to undergo iv) Risk increases with age v) More common in flame shaped hemorrhages. The right eye is
surgery. What men vi) Chronic bladder irritation and infections normal. What is the cause of this pts unilateral
option is the best for him? (urinary infections, kidney and bladder stones, blindness?
a. Continue with warfarin bladder catheter left in place a long time.) vii) a. HTN retinopathy
b. Continue with warfarin and add heparin Personal history of bladder or other urothelial b. CRA thrombosis
c. Stop warfarin and add aspirin cancer viii) Family history ix) Chemotherapy or c. CRV thrombosis
d. Background retinopathy d. Oral morphine d. DM
e. Retinal detachmen e. Laxatives
Q. 1. What is the key? Q. 1. What is the key?
Ans. The key is A. HTN retinopathy [it is a wrong Q. 2. Why we should go for this option? Q. 2. Why glycosuria occurred?
key. Correct key is C. central retinal vein Ans. 1. The key is C. Palliative colostomy.
thrombosis]. Ans. 2. Cancer or chemotherapy induced Ans. 1. The key is B. Normal finding.
obstructions are unlikely to respond to Ans. 2. Stress during operation can cause
373. A mentally retarded child puts a green pea in conservative management [NBM, IV fluid, transient hyperglycemia causing glycosuria
his ear while eating. The carer confirms this. nasogastric suction] and hence only analgesia will secondary to stress induced rise of cortisole which
Otoscopy shows a green colored object in the ear not relieve it. So in such cases we have to go for becomes normal after some time.
canal. What is the most appropriate single palliative colostomy.
best approach to remove this object? 377. A 70yo man admits to asbestos exposure 380. A pt presented with hx of swelling in the
a. By magnet 20yrs ago and has attempted to quit smoking. He region of the sub-mandibular region, which
b. Syringing has became
c. Under GA noted weight loss and hoarseness of voice. more prominent and painful on chewing. He also
d. By hook Choose the single most likely type of cancer a.w gave hx of sour taste in the mouth, the area is
e. By instilling olive oil risk tender on palpation. Choose the most probable
factors present. dx?
Ans. The key is Under GA. [Pea is not a magnetic a. Basal cell carcinoma a. Chronic recurrent sialadenitis
material and hence it cannot be removed by b. Bronchial carcinoma b. Adenolymphoma
magnet, it will swell up if syringing is attempted, c. Esophageal carcinoma c. Mikulicz’s disease
as hook placement is likely with risk of pushing d. Nasopharyngeal carcinoma d. Adenoid cystic carcinoma
the pea deeper it is not also suitable in a mentally e. Oral carcinoma e. Sub-mandibular abscess
retarded child, and olive oil is not of help in case
of pea. So to avoid injury it is better to remove Q. 1. What is the key? Ans. The key is A. Chronic recurrent sialadenitis.
under GA]. Q. 2. What are the conditions related to asbestos [pain, swelling, more pain on chewing,
exposure? tenderness, and submandibular region suggests
374. A pt presents with longstanding gastric Ans. 1. The key is B. Bronchial carcinoma. diagnosis of submandibular chronic recurrent
reflux, dysphagia and chest pain. On barium [Asbestos exposure is a risk factor for lung cancer sialadenitis, usually secondary to sialolithiasis or
enema, dilation of esophagus with tapering end is and also has a synergistic effect with cigarette stricture].
noted. He was found with Barrett’s esophagus. He smoke. Horseness can be from involvement of
had progressive dysphagia to solids and then recurrent laryngeal nerve]. 381. ECG of an 80yo pt of ICH shows saw-tooth
liquids. What is the single most appropriate dx? Ans. 2. Conditions related to asbestos exposure: i) like waves, QRS complex of 80ms duration,
a. Achalasia Pleural plaques (after a latent period of 20-40 yrs) ventricular rate=150/min and regular R-R interval.
b. Esophageal spasm ii) Pleural thickening iii) Asbestosis (latent period What is the most porbable dx?
c. GERD is typically 15-30 yrs) iv) Mesothelioma (prognosis a. Atrial fib
d. Barrett’s esophagus is very poor) v) Lung cancer. b. Atrial flutter
e. Esophageal carcinoma 378. A 32yo woman had progressive decrease in c. SVT
Ans. The key is E. Oesophageal carcinoma. [there vision over 3yrs. She is now dx as almost blind. d. Mobitz type1 second degree heart block
is dilatation in oesophagus which is seen both in What e. Sinus tachycardia
achalasia and carcinoma. Dysphagia to solid would be the mechanism?
initaially is very much suggestive of carcinoma and a. Cataract Ans. The key is B. Atrial flutter. [Saw-tooth like
also barrett’s change is a clue to carcinoma] b. Glaucoma waves, normal QRS complex of 80 ms (normal
c. Retinopathy range 70-100 ms), ventricular rate of 150/min and
375. A 48yo lady presents with itching, d. Uveitis regular R-R interval is diagnostic of atrial flutter].
excoriations, redness, bloody discharge and e. Keratitis
ulceration around her nipple. What is the most Q. 1. What is the key ? 382. A 50 yo woman who was treated for breast
likely dx? Q. 2. Why you made this diagnosis? cancer 3 yrs ago now presents with increase thirst
a. Paget’s disease of the breast Ans. 1. The key is B. Glaucoma. This is wrong key! and confusion. She has become drowsy now.
b. Fibrocystic dysplasia Correct option is retinopathy. What is the most likely metabolic abnormality?
c. Breast abscess Ans. 2. Cataract is unlikely at this age. Uveitis and a. Hypercalcemia
d. Duct papilloma iritis doesn’t have such degree of vision loss and b. Hyperkalemia
e. Eczema iritis and anterior uveitis have pain, redness and c. Hypoglycemia
photophobia. Open angle glaucoma mostly occurs d. Hyperglycemia
Ans. The key is A. Paget’s disease of the breast. after the age of 50yrs. Answer should be e. Hypocalcemia
retinopathy (example retinitis pigmentosa).
376. Pt with widespread ovarian carcinoma has 379. A child during operation and immediately Q. 1. What is the key?
bowel obstruction and severe colic for 2h and was after showed glycosuria, but later his urine sugar Q. 2. Justify the key
normal in between severe pain for a few hours. was
What is the most appropriate management? normal. Choose the most probable dx. Ans. 1. The key is A. Hypercalcemia.
a. PCA (morphine) a. Pre-diabetic state Ans. 2. Increased thirst, confusion, drowsiness
b. Spasmolytics b. Normal finding these are features of hypercalcemia. Any solid
c. Palliative colostomy c. Low renal tubular threshold organ tumour can produce hypercalcemia. Here
treated Ca breast is the probable cause of e. Nebulized salbutamol strongly indicative of the presence of cancer in
hypercalcemia. Q. 1. What is the key? the abdomen, specifically gastric cancer].
Q. 2. What is the diagnosis? 390. A 64yo man has presented to the ED with a
383. A 29yo woman presents to her GP with a hx Q. 3. What is the significant of tall tented T waves stroke. CT shows no hemorrhage. ECG shows
of weight loss, heat intolerance, poor conc and and wide QRS complex? atrial
palpitations. Which of the following is most likely Ans. 1. The key is B. IV calcium chloride (both IV fib. He has been thrombolysed and he’s awaiting
to be a/w dx of thyroiditis a/w viral infection? calcium gluconate or IV calcium chloride can be discharge. What prophylactic regimen is best
a. Bilateral exophthalmos used when there is ECG changes). for him?
b. Diffuse, smooth goiter Ans. 2. The ECG changes are suggestive of a. Warfarin
c. Reduced uptake on thyroid isotope scan Hyperkalemia. b. Heparin
d. Positive thyroid peroxidase antibodies Ans. 3. At potassium level of >5.5mEq/L occurs tall c. Aspirin
e. Pretibial myxedema tented T waves and at potassium level >7mEq/L d. Statins
occurs wide QRS complex with bizarre QRS e. Beta blockers
Q. 1. What is the key? morphology. Ans. The key is A. Warfarine. [Atrial fibrillation:
Q. 2. What is the diagnosis? 387. A 54yo pt 7 days after a total hip post stroke- following a stroke or TIA warfarine
Q. 3. Whats are the points in favour? replacement presents with acute onset should be given as the anticoagulant of choice.
breathlessness and NICE guideline].
Ans. 1. The key is C. Reduced uptake on thyroid raised JVP. Which of the following inv will be most
isotope scan. helpful in leading to a dx? 391. A 54yo man after a CVA presents with ataxia,
Ans. 2. The diagnosis is De Quervain’s or subacute a. CXR intention tremors and slurred speech. Which part
thyroiditis. b. CTPA of the brain has been affected by the stroke?
Ans. 3. Viral or subacute thyroiditis: diagnostic c. V/Q scan a. Inner ear
criteria: i) Features of hyperthyroidism present. ii) d. D-Dimer b. Brain stem
Pain thyroid, not mentioned. iii) Investigations: e. Doppler US of legs c. Diencephalon
high esr (60-100) not mentioned, Reduced uptake Q. 1. What is the key? d. Cerebrum
of radioactive iodine by the gland. Q. 2. Justify the key. e. Cerebellum
Ans. 1. The key is B. CTPA. Q. 1. What is the key?
384. A lady, post-colostomy closure after 4days Ans. 2. The patient has a +ve two level PE Wells Q. 2. What are the features of the condition?
comes with fluctuating small swelling in the score (if it was negative we should do D-Dimer) Ans. 1. The key is E. Cerebellum.
stoma. and there is no renal impairment or history Ans. 2. i) Ataxia ii) slurred speech or dysarthria iii)
What is the management option for her? suggestive of allergy to contrast media (if these dysdiodokokinesis iv) intention tremor v)
a. Local exploration present we should have go for VQ scan) the nystagmus.
b. Exploratory laparotomy investigation of choice is PTCA. NICE guideline. 392. A 57yo man with blood group A complains of
c. Open laparotomy 388. A 7yo girl has been treated with penicillin symptoms of vomiting, tiredness, weight loss and
d. Reassure after sore throat, fever and cough. Then she palpitations. Exam: hepatomegaly, ascites,
develops palpable left supraclavicular mass. What is the
Ans. The key is A. Local exploration. skin rash and itching. What is the most probable most
dx? likely dx?
385. A 65yo female pt was given tamoxifen, which a. Erythema nodosum a. Gastric carcinoma
of the following side effect caused by it will b. Erythema multiforme b. Colorectal carcinoma
concern you? c. SJS c. Peptic ulcer disease
a. Fluid retention d. Erythema marginatum d. Atrophic gastritic
b. Vaginal bleeding e. Erythema gangernosum e. Krukenburg tumor
c. Loss of apetite Q. What is the key? Ans. The key is A. Gastric carcinoma. [i) blood
d. Headache and dizziness Q. What common drugs causes this to occur? group A is associated with gastric cancer ii)
e. Anorgasm Ans. 1. The key is B. Erythema multiforme. vomiting, tiredness, weight loss are general
Ans. 2. Common drugs causing erythma features of gastric cancer iii) palpitation from
Q. 1. What is the key? multiforme are: antibiotics anemia of cancer iv) hepatomegaly (metastasis)
Q. 2. What is the reason to select this key? (including, sulphonamides, penicillin), and ascites are late features of gastric cancer. v)
anticonvulsants (phenytoin,barbiturates), aspirin, palpable left supraclavicular mass- is Vershow’s
Ans. 1. The key is B. Vaginal bleeding. antituberculoids, and allopurinol. gland,  has long been regarded as strongly
Ans. 2. Tamoxifen can promote development of 389. A 60yo man presented with a lump in the left indicative of gastric cancer].
endometrial carcinoma. So vaginal bleeding will supraclavicular region. His appetite is decreased 393. A 21yo girl looking unkempt, agitated,
be of concern for us. and he has lost 5kg recently. What is the most malnourished and nervous came to the hospital
probably dx? asking
386. A 39yo man with acute renal failure presents a. Thyroid carcinoma for painkillers for her abdominal pain. She is
with palpitations. His ECG shows tall tented T b. Stomach carcinoma sweating, shivering and complains of joint pain.
waves and wide QRS complex. What is the next c. Bronchial carcinoma What can be the substance misuse here?
best step? d. Mesothelioma a. Alcohol
a. Dialysis e. Laryngeal carcinoma b. Heroin
b. IV calcium chloride Ans. The key is B. Stomach carcinoma. c. Cocaine
c. IV insulin w/ dextrose [Mentioned lump in the left supraclavicular region d. LSD
d. Calcium resonium is Vershow’s gland,  has long been regarded as e. Ecstasy
Ans. The key is B. Heroin. [agitation, nervousness, Q. 1. What is the key? a. Cardiac enzymes
abdominal cramp, sweating, shivering and Q. 2. What are the causes of sudden painless loss b. CXR
piloerection, arthralgia these are features of of vision? c. ECG
heroin withdrawal]. Ans. 1. The Key is D. Retinal detachment. d. Echo
394. A child presents with increasing jaundice and Ans. 2. Causes of sudden painless loss of vision: e. 24h ECG
pale stools. Choose the most appropriate test? 1. Retinal detachment Ans. The key is E. 24h ECG.
a. US abdomen 2. Vitreous haemorrhage Indications of 24 h ambulatory holter monitoring:
b. Sweat test 3. Retinal vein occlusion  To evaluate chest pain not reproduced
c. TFT 4. Retinal artery occlusion with exercise testing
d. LFT 5. Optic neuritis  To evaluate other signs and
e. Endomyseal antibodies 6. Cerebrovascular accident symptoms that may be heart-related,
Ans. The key is A. US abdomen. [This is a picture such as fatigue, shortness of breath,
suggestive of obstructive jaundice. LFT can give 399. A child was woken up from sleep with severe dizziness, or fainting
clue like much raised bilirubin, AST and ALT not pain in the testis. Exam: tenderness on palpation  To identify arrhythmias or palpitations
that high and raised alkaline phosphatase but still and only one testis was normal in size and
 To assess risk for future heart-related
USG is diagnostic in case of obstructive jaundice]. position. What would be your next step?
events in certain conditions, such as
395. A 32yo man presents with hearing loss. a. Analgesia
idiopathic hypertrophic
AC>BC in the right ear after Rhine test. He also b. Antibiotics
cardiomyopathy, post-heart attack
complains of tinnitus, vertigo and numbness on c. Refer urgently to a surgeon
with weakness of the left side of the
same half of his face. What is the most d. Reassurance
heart, or Wolff-Parkinson-White
appropriate inv for his condition? e. Discharge with analgesics
syndrome
a. Audiometry Ans. The key is A. Analgesia. [According to some
 To assess the function of an implanted
b. CT US sites it is analgesia but no UK site support
pacemaker
c. MRI this!!! So for Plab exam the more acceptable
d. Tympanometry option is C. Refer urgently to a surgeon].  To determine the effectiveness of
e. Weber’s test 400. A child suffering from asthma presents with therapy for complex arrhythmias
Ans. The key is C. MRI. [features are suggestive of Temp 39C, drooling saliva on to the mother’s lap, 403. A woman dx with Ca Breast presents now
acaustic neuroma, so MRI is the preferred option]. and taking oxygen by mask. What sign will with urinary freq. which part of the brain is the
396. A 56 yo lady with lung cancer presents with indicate that he is deteriorating? metastasis spread to?
urinary retention, postural hypotension, a. Intercostal recession a. Brain stem
diminished b. Diffuse wheeze b. Pons
reflexes and sluggish pupillary reaction. What is c. Drowsiness c. Medulla
the most likely explanation for her symptoms? Ans. The key is A. Intercostal recession. This is d. Diencephalon
a. Paraneoplastic syndrome wrong key. Correct key is C. Drowsiness. e. Cerebral cortex
b. Progression of lung cancer [Intercostal recession is a sign of severe asthma Ans. The key is D. Diencephalon. [diencephalon is
c. Brain metastasis but it can be seen at a lesser degree as well. So made up of four distinct components: i) the
d. Hyponatremia drowsiness is more appropriate answer]. thalamus ii) the subthalamus iii) the
e. Spinal cord compression 401. A 12yo boy presents with painful swollen hypothalamus and iv) the epithalamus. Among
Ans. The key is A. Paraneoplastic syndrome. knew after a sudden fall. Which bursa is most these the hypothalamus has crucial role in causing
[Features given are well known features of likely to urinary frequency].
autonomic neuropathy which can be a result of be affected? 404. A man is very depressed and miserable after
paraneoplastic syndrome]. a. Semimembranous bursa his wife’s death. He sees no point in living now
397. An old woman having decreased vision can’t b. Prepatellar bursa that
see properly at night. She has changed her glasses c. Pretibial bursa his wife is not around and apologises for his
quite a few times but to no effect. She has normal d. Suprapatetaller bursa existence. He refuses any help offered. His son
pupil and cornea. What is the most likely dx? Ans. The key is B. Prepatellar bursa. [A fall onto has
a. Cataract the knee can damage the prepatellar bursa. This brought him to the ED. The son can’t deal with
b. Glaucoma usually causes bleeding into the bursa sac causing the father any more. What is the most
c. Retinal detachment swellen painful knee. Prepatellar bursitis that is appropriate next step?
d. Iritis caused by an injury will usually go away on its a. Voluntary admission to psychiatry ward
e. GCA own. The body will absorb the blood in the bursa b. Compulsory admission under MHA
Ans. The key is B. Glaucoma. It is a wrong key. over several weeks, and the bursa should return c. Refer to social services
Correct key should be A. Cataract. [Age and to normal. If swelling in the bursa is causing a d. Alternate housing
normal pupil and cornea are suggestive of slow recovery, a needle may beinserted to drain e. ECT
cataract. If it was glaucoma pupil would be a bit the blood and speed up the process. There is a Ans. The key is B. Compulsory admission under
dilated and/or oval in shape]. slight risk of infection in putting a needle into the MHA. [This patient is refusing any help offered!
398. A pt comes with sudden loss of vision. On bursa]. And his son cannot deal with him anymore! In this
fundoscopy the optic disc is normal. What is the 402. A 61yo man has been referred to the OPD situation voluntary admission to psychiatry ward
underlying pathology? with frequent episodes of breathlessness and is not possible and the option of choice is
a. Iritis chest “compulsory admission under MHA”. The point
b. Glaucoma pain a/w palpitations. He has a regular pulse here is the man has felt himself in danger by self
c. Vitreous chamber rate=60bpm. ECG=sinus rhythm. What is the most neglect].
d. Retinal detachment appropriate inv to be done?
405. A 31yo man has epistaxis 10 days following 410. A T2DM is undergoing a gastric surgery. The pain of acute pericarditis may radiate to your
polypectomy. What is the most likely dx? What is the most appropriate pre-op left shoulder and neck. It often intensifies when
a. Nasal infection management? you cough, lie down or inhale deeply. Sitting up
b. Coagulation disorder a. Start him in IV insulin and glucose and K+ just and leaning forward can often ease the pain. 
c. Carcinoma before surgery 413. A 6m boy has been brought to ED following
Ans. The key is A. Nasal infection. [Infection is one b. Stop his oral hypoglycemic on the day of the an apneic episode at home. He is now completely
of the most important cause of secondary procedure well but his parents are anxious as his cousin died
hemorrhage]. c. Continue regular oral hypoglycemic of SIDS (Sudden Infant Death Syndrome) at a
406. A woman had an MI. She was breathless and d. Stop oral hypoglycemic the prv night and start similar age. The parents ask forguidance on BLS
is put on oxygen mask and GTN, her chest pain IV insulin with glucose and K+ before for a baby of his age. What is the single most
has surgery recommended technique for
improved. Her HR=40bpm. ECG shows ST e. Change to short acting oral hypoglycemic cardiac compressions?
elevation in leads I, II, III. What is your next step? Ans. The key is D. Stop oral hypoglycemic the prv a. All fingers of both hands
a. LMWH night and start IV insulin with glucose and K+ b. All fingers of one hand
b. Streptokinase before c. Heel of one hand
c. Angiography Surgery. d. Heel of both hand
d. Continue current management e. Index and middle fingertips of one hand
e. None 411. A 19yo boy is brought by his mother with Ans. The key is E. Index and middle fingertips of
Ans. The key is B. Streptokinase.This is wrong key. complaint of lack of interest and no social one hand.
Correct key is C. Angiography. interactions. 414. A 70yo man had a right hemicolectomy for
407. A 67yo male presents with polyuria and He has no friends, he doesn’t talk much, his only ceacal carcinoma 6days ago. He now has
nocturia. His BMI=33, urine culture = negative for interest is in collecting cars/vehicles having abdominal
nitrates. What is the next dx inv? around 2000 toy cars. What is the most distension and recurrent vomiting. He has not
a. PSA appropriate dx? opened his bowels since surgery. There are no
b. Urea, creat and electrolytes a. Borderline personality disorder bowel sounds. WBC=9, Temp=37.3C. What is the
c. MSU culture and sensitivity b. Depression single most appropriate next management?
d. Acid fast urine test c. Schizoaffective disorder a. Antibiotic therapy IV
e. Blood sugar d. Autistic spectrum disorder b. Glycerine suppository
Ans. The key is E. Blood sugar. [Age at c. Laparotomy
presentation and class1 obesity favours the Ans. The key is D. Autistic spectrum disorder. d. NG tube suction and IV fluids
diagnosis of type2 DM]. Autism spectrum disorders affect three different e. TPN
408. A pt from Africa comes with nodular patch areas of a child's life: Ans. The key is D. NG tube suction and IV fluids.
on the shin which is reddish brown. What is the  Social interaction [The patient has developed paralytic ileus which
most  Communication -- both verbal and should be treated conservatively].
probable dx? nonverbal 415. A 60yo man with a 4y hx of thirst, urinary
a. Lupus vulgaris  Behaviors and interests freq and weight loss presents with a deep painless
b. Erythema nodosum In some children, a loss of language is the major ulcer on the heel. What is the most appropriate
c. Pyoderma gangrenosum impairment. In others, unusual behaviors (like inv?
d. Erythema marginatum spending hours lining up toys) seem to be the a. Ateriography
e. Solar keratosis dominant factors. b. Venography
Ans. The key is B. Erythema nodosum. [Causes of c. Blood sugar
erythema nodosum: MOST COMMON CAUSES- i) 412. A 45yo man who is diabetic and HTN but d. Biopsy for malignant melanoma
streptococcal infection ii) sarcoidosis. Other poorly compliant has chronic SOB, develops e. Biopsy for pyoderma
causes- tuberculosis, mycoplasma pneumonia, severe Ans. The key is C. Blood sugar. [The patient
infectious mononucleosis, drugs- sulfa related SOB and chest pain. Pain is sharp, increased by probably developed diabetic foot].
drug, OCP, oestrogen; Behcet’s disease, CD, UC; breathing and relieved by sitting forward. What 416. A 16yo boy presents with rash on his
lymphoma, leukemia and some others]. is the single most appropriate dx? buttocks and extensor surface following a sore
409. A 29yo lady came to the ED with complaints a. MI throat.
of palpitations that have been there for the past 4 b. Pericarditis What is the most probable dx?
days and also feeling warmer than usual. Exam: c. Lung cancer a. Measles
HR=154bpm, irregular rhythm. What is the tx for d. Good pastures syndrome b. Bullous-pemphigoig
her condition? e. Progressive massive fibrosis c. Rubella
a. Amiadarone d. ITP
b. Beta blockers Ans. The key is B. Pericarditis. [Nature of pain i.e. e. HSP
c. Adenosine sharp pain increased by breathing and relieved by Ans. The key is D. ITP. It’s probably a wrong key!
d. Verapamil sitting forward is suggestive of pericarditis]. The correct key should be E. HSP. [In HSP rash
e. Flecainide Nature of pericardial pain: the most common typically found in buttocks, legs and feets and may
Ans. The key is B. Beta blockers [the probable symptom is sharp, stabbing chest pain behind the also appear on the arms, face and trunk. But in
arrhythymia is AF secondary to thyrotoxicosis. So sternum or in the left side of your chest. ITP it mostly occurs in lower legs. HSP usually
to rapid control the symptoms of thyrotoxicosis However, some people with acute pericarditis follow a sorethroat and ITP follow viral infection
Beta blocker should be used which will improve describe their chest pain as dull, achy or pressure- like flue or URTI. HSP is a vasculitis while ITP is
the arrythmia]. like instead, and of varying intensity. deficiency of platelets from more destruction in
spleen which is immune mediated].
417. A 34yo man with a white patch on the 421. A 54yo man with alcohol dependence has preferred. Antidepressants can take two to four
margin of the mid-third of the tongue. Which is tremor and sweating 3days into a hosp admission weeks before becoming effective].
the single for
most appropriate LN involved? a fx femur. He is apprehensive and fearful. What 424. A 28yo man presents with rapid pounding in
a. External iliac LN is the single most appropriate tx? the chest. He is completely conscious throughout.
b. Pre-aortic LN a. Acamprossate The ECG was taken (SVT). What is the 1st med to
c. Aortic LN b. Chlordiazepoxide be used to manage this condition?
d. Inguinal LN c. Lorazepam a. Amiodarone
e. Iliac LN d. Lofexidine b. Adenosine
f. Submental LN e. Procyclidine c. Lidocaine
g. Submandibular LN Ans. The key is B. Chlordiazepoxide. C. Lorazepam d. Verapamil
h. Deep cervical LN is also correct key! [Dx alcohol withdrawal e. Metoprolol
symptom. According to NICE 1st line treatment is Ans. The key is B. Adenosine. [Management of
Ans. The key is G. Submandibular LN. oral lorazepam and if symptom persists or oral SVT: i) vagal manoeuvres (carotid sinus message,
418. A 50yo lady presents to ED with sudden medication is declined, give parenteral lorazepam, valsalva manoeuvre) transiently increase AV-
severe chest pain radiating to both shoulder and haloperidol or olanzapine. According to OHCM 1st block, and unmask the underlying atrial rhythm. If
accompanying SOB. Exam: cold peripheries and line treatment is chlordiazepoxide. unsuccessful then the first medicine used in SVT is
paraparesis. What is the single most appropriate Chlordiazepoxide should only be used at the adenosine, which causes transient AV block and
dx? lowest possible dose and for a maximum of up to works by i) transiently slowing ventricles to show
a. MI four weeks. This will reduce the risks of the underlying atrial rhythm ii) cardioverting a
b. Aortic dissection developing tolerance, dependence and junctional tachycardia to sinus rhythm. OHCM].
c. Pulmonary embolism withdrawal].
d. Good pastures syndrome 425. A 56yo woman who is depressed after her
e. Motor neuron disease 422. A 5yo child complains of sore throat and husband died of cancer 3m ago was given
Ans. The key is B. Aortic dissection. [Cold earache. He is pyrexial. Exam: tonsils enlarged and amitryptaline. Her sleep has improved and she
peripheries due to reduced blood flow to dista hyperemic, exudes pus when pressed upon. What now wants to stop medication but she still
parts of dissection and reduced perfusion of is the single most relevant dx? speaks about her husband. How would you
nerves resulted in paraparesis. Usual a. IM manage her?
management for type A dissection is surgery and b. Acute follicular tonsillitis a. CBT
for type B is conservative]. c. Scarlet fever b. Continue amitryptaline
419. A 54yo myopic develops flashes of light and d. Agranulocytosis c. Psychoanalysis
then sudden loss of vision. That is the single most e. Acute OM d. Bereavement counselling
appropriate tx? Ans. The key is B. Acute follicular tonsillitis. e. Antipsychotic
a. Pan retinal photo coagulation [Tonsillitis is usually caused by a viral infection Ans. The key is B. Continue amitriptyline.
b. Peripheral iridectomy or, less commonly, a bacterial infection. The [depression is important feature of bereavement.
c. Scleral buckling given case is a bacterial tonsillitis (probably Patient may pass sleepless nights. As this patients
d. Spectacles caused by group A streptococcus). There are four sleep has improved it indicate she has good
e. Surgical extraction of lens main signs that tonsillitis is caused by a bacterial response to antidepressant and as she still speaks
Ans. The key is C. Scleral buckling. [It is a case of infection rather than a viral infection. They are: about her husband there is chance to
retinal detachment with treatment option of  a high temperature deterioration of her depression if antidepressant
scleral buckling].  white pus-filled spots on the tonsils is stopped. For depressive episodes
420. A 40yo chronic alcoholic who lives alone,  no cough antidepressants should be continued for at least
brought in the ED having been found confused at 6-9 months].
 swollen and tender lymph nodes
home after a fall. He complains of a headache and
(glands).
gradually worsening confusion. What is the 426. A 64yo man presents with a hx of left sided
most likely dx? hemiparesis and slurred speech. He was
423. A man with a fam hx of panic disorder is
a. Head injury absolutely
brought to the hosp with palpitations, tremors,
b. Hypoglycemia fine 6h after the episode. What is the most
sweating and muscles tightness on 3 occasions in
c. Extradural hematoma appropriate prophylactic regimen?
the last 6 wks. He doesn’t complain of
d. Subdural hematoma a. Aspirin 300mg for 2 weeks followed by aspirin
headache and his BP is WNL. What is the single
e. Delirium 75mg
most appropriate long-term tx for him?
Ans. The key is D. Subdural hematoma. [subdural b. Aspirin 300mg for 2 weeks followed by aspirin
a. Diazepam
hematoma may be acute or chronic. In chronic 75mg and dipyridamole 200mg
b. Olanzapine
symptoms may not be apparent for several days c. Clopidogrel 75mg
c. Haloperidol
or weeks. Symptoms of subdural hematomas are: d. Dipyridamole 200mg
d. Fluoxetine
fluctuating level of consciousness, ± insidious e. Aspirin 300mg for 2 weeks
e. Alprazolam
physical or intellectual slowing, sleepiness, Ans. The key is B. Aspirin 300mg for 2 weeks
Ans. The key is D. Fluoxetine. [Recommended
headache, personality change and unsteadiness. followed by aspirin 75mg and dipyridamole
treatment for panic disorder is i) CBT ii)
Tx. Irrigation/evacuation e.g. via barr twist drill 200mg. It is wrong key! Current recommended
Medication (SSRIs or TCA). NICE recommends a
and barr hole craniostomy 1st line. Craniotomy if secondary prophylaxis is C. Clopidogrel 75mg.
total of seven to 14 hours of CBT to be completed
the clot organized 2nd line].
within a four month period. Treatment will usually
involve having a weekly one to two hour session.
When drug is prescribed usually a SSRI is
427. A 63yo lady with a BMI=32 comes to the ED formation in the ribs. Exam: bruises on childs there a bleed from a berry aneurysm from the
with complaints of pigmentation on her legs. back. What is the most appropriate next step? circle of Willis. In which space did the bleeding
Exam: a. Check child protection register occur?
dilated veins could be seen on the lateral side of b. Coagulation profile a. Subarachnoid
her ankle. Which of the following is involved? c. Skeletal survey b. Subdural
a. Short saphenous vein d. Serum calcium c. Extradural
b. Long saphenous vein e. DEXA scan d. Subparietal
c. Deep venous system Ans. The key is C. Skeletal survey. [Skeletal survey e. Brain ventricles
d. Popliteal veins is a series of x-ray which is usually used in NAI]. Ans. The key is A. Subarachnoid.
e. Saphano-femoral junction [after survey should think of childs protection].
Ans. The key is A. Short saphenous vein. [short 436. A schizophrenic pt hears people only when
saphenous vein travels lateral aspect of ankle 432. A 35yo woman has had bruising and he is about to fall asleep. What is the most likely
while great or long saphenous vein travels medial petechiae for a week. She has also had recent dx?
aspect of ankle]. menorrhagia a. Hypnopompic hallucinations
but is otherwise well. Blood: Hgb=11.1, WBC=6.3, b. Hyponogogic hallucinations
428. A 55yo man presents with hx of weight loss Plt=14. What is the single most likely dx? c. Hippocampal hallucinations
and tenesmus. He is dx with rectal carcinoma. a. Acute leukemia d. Delirious hallucinations
Which risk factors help to develop rectal b. Aplastic anemia e. Auditory hallucinations
carcinoma except following? c. HIV infection Ans. The key is B. Hypnogogic hallucinations.
a. Smoking d. ITP
b. Family hx e. SLE 437. A pt who came from India presents with
c. Polyp Ans. The key is D. ITP. [As the patient is otherwise cough, fever and enlarged cervical LN. Exam:
d. Prv carcinoma well acute leukemia, HIV and SLE is unlikely. caseating
e. High fat diet Normal wbc count excludes aplastic anemia. So granulomata found in LN. What is the most
f. High fibre diet likely diagnosis is ITP]. appropriate dx?
Ans. The key is F. High fibre diet. [except high a. Lymphoma
fiber diet all others are risk factors to develop 433. A 30yo man complains of episodes of hearing b. TB adenitis
rectal carcinoma]. music and sometimes threatening voices within a c. Thyroid carcinoma
couple of hours of heavy drinking. What is the d. Goiter
429. A pt presents with a painful, sticky red eye most likely dx? e. Thyroid cyst
with a congested conjunctiva. What is the most a. Delirium tremens Ans. The key is B. TB adenitis. [caseating
suitable tx? b. Wernicke’s encephalopathy granulomata are diagnostic of TB].
a. Antibiotic PO c. Korsakoff’s psychosis
b. Antihistamine PO d. Alcohol hallucinosis 438. A 44yo man comes with hx of early morning
c. Antibiotic drops e. Temporal lobe dysfunction headaches and vomiting. CT brain shows ring
d. Steroid drops Ans. The key is D. Alcoholic hallucinosis. [Alcohol enhancing lesions. What is the single most
e. IBS hallucinosis can occur during acute intoxication or appropriate option?
Ans. The key is C. Antibiotic drops. [bacterial withdrawal.  It involves auditory and visual a. CMV
conjunctivitis is treated with antibiotic drops]. hallucinations, most commonly accusatory or b. Streptococcus
threatening voices. Source: Wikipedia]. c. Toxoplasmosis
430. A 45yo woman complains of pain in her 434. A pt had TIA which he recovered from. He d. NHL
hands precipitated by exposure to the cold has a hx of stroke and exam shows HR in sinus e. Pneumocystis jerovii
weather. She rhythm. Ans. The key is C. Toxoplasmosis.
has breathlessness on walking. When she is He is already on aspirin 75mg and anti-HTN drugs.
eating, she can feel food suddenly sticking to the What other action should be taken? 439. A 72yo man is found to be not breathing in
gullet. It seems to be in the middle of the a. Add clopidogrel only the CCU with the following rhythm. What is the
esophagus but she can’t localize exactly where it b. Increase dose of aspirin to 300mg most
sticks. It is usually relieved with a drink of water. c. Add warfarin likely dx?
Choose the single most likely cause of d. Add clopidogrel and statin
dysphagia from the options? e. Add statin only
a. Esophageal carcinoma Ans. The key is D. Add clopidogrel and statin. This
b. Systemic sclerosis is wrong key! Correct key should be E. Add statin
c. SLE only. [He who is already on aspirin need no
d. Pharyngeal carcinoma change in aspirin dose. Clopidogrel is now drug of
e. Globus hystericus choice for secondary prevention in TIA but if
Ans. The key is B. Systemic sclerosis. [Raynods someone is already on aspirin he should continue
phenomena, pulmonary involvement, it as same dose and should be considered to shift a. SVT
oesophageal dysmotility are suggestive of to clopidogrel in next few visits.So for given case b. VT
systemic sclerosis]. correct option is add statin only]. c. VF
d. Atrial fib
435. A 40yo woman suddenly collapsed and died. e. Atrial flutter
431. A 3yo child brought to the ED with a swelling At the post-mortem autopsy, it was found that Ans. The key is C. VF.
over the left arm. XR shows multiple callus
440. A 65yo man with difficulty in swallowing b. Pilocarpine eye drops Ans. The key is D. Goiter.
presents with an aspiration pneumonia. He has a c. Corticosteroids
bovine cough and fasciculating tongue. d. Peripheral iridectomy 448. A 76yo woman has become tired and
Sometimes as he swallows food it comes back e. Surgical extraction of lens confused following an influenza like illness. She is
through Ans. The key is C. Corticosteroid. [Sudden loss of also
his nose. Choose the single most likely cause of vision, remission and relapse of optic neuritis and breathless with signs of consolidation of the left
dysphagia from the given option? focal neurological symptoms and exaggerated lung base. What is the most likely dx?
a. Bulbar palsy reflexes all points towards multiple sclerosis. a. Drug toxicity
b. Esophageal carcinoma Treatment option is corticosteroids]. b. Delirium tremens
c. Pharyngeal pouch c. Infection toxicity
d. Pseudobulbar palsy 444. A 15yo boy presents with a limp and pain in d. Hypoglycemia
e. Systemic sclerosis the knee. Exam: leg is externally rotated and 2cm e. Electrolyte imbalance
Ans. The key is A. Bulbar palsy. [Dysphagia, nasal shorter. There is limitation of flexion, abduction A. The key is C. Infection toxicity. [Infection
regurgitation, fasciculating tongue these are and medial rotation. As the hip is flexed toxicity or toxic shock syndrome is likely here as
features of bulbar palsi due to lower motor external rotation is increased. Choose the most preceding flue like illness points towards toxin
neuronal lesion of IX-XII nerves]. likely dx? (enterotoxin type B) from Staphylococcus aureus].
a. Juvenile rheumatoid arthritis
441. A 16yo teenager was brought to the ED after b. Osgood-schlatter disease 449. A young pt is complaining of vertigo
being stabbed on the upper right side of his back. c. Reactive arthritis whenever she moves sideways on the bed while
Erect CXR revealed homogenous opacity on the d. Slipped femoral epiphysis lying
lower right lung, trachea was centrally placed. e. Transient synovitis of the hip supine. What would be the most appropriate next
What is the most probable explanation for the XR Ans. The key is D. Slipped femoral epiphysis. [The step?
findings? given presentation is classic for slipped femoral a. Head roll test
a. Pneumothorax epiphysis]. b. Reassure
b. Hemothorax c. Advice on posture
c. Pneumonia 445. A 64yo woman has difficulty moving her right d. Carotid Doppler
d. Tension pneumothorax shoulder on recovering from surgery of the e. CT
e. Empyema posterior triangle of her neck. What is the single Ans. The key is A. Head roll test. [this is a case of
Ans. The key is B. Hemothorax. [In blunt trauma most appropriate option? “benign paroxysmal positional vertigo” for which
there may be hemo-pneumothorax but in sharp a. Accessory nerve the diagnosis is made by head roll test].
wound like stabbing there may occur only b. Glossopharyngeal nerve
hemothorax]. c. Hypoglossal nerve 450. A 32yo man has OCD. What is the best tx?
d. Vagus nerve a. CBT
442. A 55yo woman complains of retrosternal e. Vestibule-cochlear nerve b. SSRI
chest pain and dysphagia which is intermittent Ans. The key is A. Accessory nerve. [Accessory c. TCA
and nerve lesion causes weakness of the trapezius d. MAO inhibitors
unpredictable. The food suddenly sticks in the muscle and can produce a drooping shoulder, e. Reassure
middle of the chest, but she can clear it with a winged scapula, and a weakness of forward Ans. The key is B. SSRI. [It is wrong key! There is a
drink of water and then finish the meal without elevation of the shoulder]. GMC sample question with most appropriate
any further problem. A barium meal shows a management being CBT. So the correct key is
‘corkscrew esophagus’. What is the single most 446. A 37yo man with an ulcer on the medial CBT].
likely dysphagia? malleolus. Which of the following LN is involved?
a. Esophageal candidiasis a. External iliac LN 451. A 65yo woman says she died 3m ago and is
b. Esophageal carcinoma b. Pre-aortic LN very distressed that nobody has buried her. When
c. Esophageal spasm c. Aortic LN she is outdoors, she hears people say that she is
d. Pharyngeal pouch d. Inguinal LN evil and needs to be punished. What is the
e. Plummer-vinson syndrome e. Iliac LN most likely explanation for her symptoms?
Ans. The key is C. Esophageal spasm. [chest pain, f. Submental LN a. Schizophrenia
unpredictable intermittent dysphagia and food g. Submandibular LN b. Mania
suddenly sticks in the middle of the chest which h. Deep cervical LN c. Psychotic depression
can be cleared with a drink of water indicates Ans. The key is D. Inguinal LN. d. Hysteria
uncoordinated irregular esophageal peristalsis e. Toxic confusional state
which is characteristic of esophageal spasm! Also 447. A pt presents with weight loss of 5kgs Ans. The key is C. Psychotic depression. [Psychotic
“cork-screw esophagus” seen in barium swallow is despite good appetite. He also complains of depression, also known as depressive psychosis, is
diagnostic of esophageal spasm]. palpitations, a major depressive episode that is accompanied
sweating and diarrhea. He has a lump in front of by psychotic symptoms (hallucinations,
443. A 38yo female presents with sudden loss of his neck which moves on swallowing. What is delusions). In this patient nihilistic delusion
vision but fundoscopy is normal. She a similar the most appropriate dx? favours the diagnosis of psychotic depression. It
episode about 1 y ago which resolved completely a. Lymphoma can occur in the context of bipolar disorder or
within 3m. Exam: mild weakness of right b. TB adenitis majordepressive disorder].
upper limb and exaggerated reflexes. What is the c. Thyroid Ca
single most appropriate tx? d. Goiter
a. Pan retinal photo coagulation e. Thyroid cyst
452. A 50yo woman presents following a fall. She c. Cryptogenic organizing e. Progressive massive fibrosis
reports pain and weakness in her hands for d. Extrinsic allergic alveolitis f. Spinal cord compression
several e. Progressive massive fibrosis Ans. The key is D. Motor neuron disease.
months, stiff legs, swallowing difficulties, and has [involvement of respiratory muscles in MND is
bilateral wasting of the small muscles of her Ans. The key is D. Extrinsic allergic alveolitis. associated with poor respiration causing sleep
hands. Reflexes in the upper limbs are absent. apnoea].
Tongue fasciculations are present and both legs 456. A 35yo lady is admitted with pyrexia, weight
show increased tone, pyramidal weakness and loss, diarrhea and her skin is lemon yellow in 460. A 55yo man presents with mild headache. He
hyper-reflexia with extensor plantars. Pain and color. has changed his spectacles thrice in 1 yr. there is
temp sensation are impaired in the upper limbs. CBC = high MCV. What is the most probably dx? mild cupping present in the disc and sickle shaped
What is the most likely dx? a. Aplastic anemia scotoma present in both eyes. What is the
a. MS b. Pernicious anemia single most appropriate tx?
b. MND c. Leukemia a. Pan retinal photo coagulation
c. Syringobulbia d. ITP b. Pilocarpine eye drops
d. Syringomyelia e. Lymphoma c. Corticosteroids
e. Myasthenia gravis Ans. The key is B. Pernicious anemia. [It may be d. Scleral buckling
Ans. The key is C. Syringobulbia. [In MS there are graves with pernicious anemia. Lemon yellow e. Analgesics alone
characteristic relapse and remission which is pallor occurs in pernicious anemia. Ans. The key is B. Pilocarpine. [This is a case of
absent here; In MND there is no sensory deficit; Hyperthyroidism may cause persistently raised open angle glaucoma, treatment is with
Syringomyelia doesn’t cause cranial nerve lesion body temperature. Both are autoimmune disease pilocarpine].
and in myasthenia there is muscular weakness which favours this association].
without atrophy. Here the features described well 461. A 55yo woman was found collapsed at home,
fits with syringobulbia]. 457. A 72yo woman who had a repair of paramedics revived her but in the ambulance she
strangulated femoral hernia 2 days ago becomes had a cardiac arrest and couldn’t be saved. The
453. Which of the following formulas is used for noisy, paramedic’s report tells that the woman was
calculating fluids for burn pts? aggressive and confused. She is febrile, CBC immobile lately due to hip pain and that they
a. 4 x weight(lbs) x area of burn = ml of fluids normal apart from raised MCV. What is the most found ulcers on the medial side of ankle. She had
b. 4 x weight(kgs) x area of burn = L of fluids likely dx? DM and was on anti-diabetics. What is the cause
c. 4 x weight(kgs) x area of burn = ml of fluids a. Electrolyte imbalance of her death?
d. 4 x weight(lbs) x area of burn = L of fluids b. Delirium tremens a. Acute MI
e. 4.5 x weight(kgs) x area of burn = dL of fluids c. Wernicke’s encephalopathy b. DKA
d. Infection toxicity c. Pulmonary embolism
Ans. The key is C. 4 x weight(kgs) x area of burn = e. Hypoglycemia d. Acute pericarditis
ml of fluids. Ans. The key is B. Delirium tremens. [Electrolyte e. Cardiac tamponade
imbalance may cause confusion but not Ans. The key is C. Pulmonary embolism.
454. A 65yo male presents with dyspnea and aggressiveness; infection toxicity will cause high [Immobilization due to hip pain may resulted in
palpitations. Exam: pulse=170bpm, fever, low BP, rash etc which are absent here DVT and later pulmonary embolism].
BP=120/80mmHg. (fever here is hyperthermia of delirium tremens).
Carotid massage has been done as first instance. Abstinance from alcohol in the hospital caused 462. An 18yo previously well student is in his 1st
What is the next step of the management? delirium tremens (chronic alcoholism is supported year at uni. He has been brought to the ED in an
a. Adenosine by high MCV) here]. agitated, deluded and disoriented state. What is
b. Amlodipine the most probable reason for his condition?
c. DC cardioversion 458. An old lady had UTI and was treated with a. Drug toxicity
d. Lidocaine antibiotics. She then developed diarrhea. What is b. Delirium tremens
e. Beta blocker the c. Infection toxicity
Ans. The key is A. Adenosine. [The likely diagnosis single most likely tx? d. Electrolyte imbalance
is SVT. 1st vagal manoeuvres, if fails iv adenosine. a. Co-amoxiclav e. Head injury
• Vagal manoeuvres (carotid sinus massage, b. Piperacillin + tazobactam Ans. The key is A. Drug toxicity. [Young age and 1st
Valsalva manoeuvre) transiently increase c. Ceftriaxone yr in university is likely to point towards drug
AV block, and may unmask an underlying atrial d. Vancomycin toxicity].
rhythm. Ans. The key is D. Vancomycin.
• If unsuccessful, give adenosine, which causes [Pseudomembranous colitis is treated with 463. A young adult presents to the ED after a
transient AV block; OHCM, 9th edition]. metronidazole or vancomycin]. motorcycle crash. The pt has bruises around the
left
455. A 48yo farmer presented with fever, malaise, 459. A 56yo man has symptoms of sleep apnea orbital area. GCS=13, examination notes alcoholic
cough and SOB. Exam: tachypnea, coarse and daytime headaches and somnolence. breath. Shortly afterwards, his GCS drops to 7.
endinspiratory crackles and wheeze throughout, Spirometry What is the single most important initial
cyanosis. Also complaint severe weight loss. His shows a decreased tidal volume and vital capacity. assessment test?
CXR shows fluffy nodular shadowing and there is What is the single most appropriate dx? a. MRI brain
PMN leukocytosis. What is the single most a. Ankylosing spondylitis b. CT brain
appropriate dx? b. Churg-strauss syndrome c. CXR
a. Ankylosing spondylitis c. Good pasture syndrome d. CT angio brain
b. Churg-strauss syndrome d. Motor neuron disease e. Head XR
Ans. The key is B. CT brain. [Likely cause is the most likely cause thouh splenomegaly is a cavitating bronchopneumonia. What is the single
epidural hematoma]. relatively uncommon feature of it!!! This most likely causative organism?
combination does not fit in other options!] a. Mycoplasma
464. A 30yo female attends OPD with a fever and b. Staphylococcus
dry cough. She says that she had headache, 468. A 62yo man who was admitted for surgery c. Chlamydia pneumonia
myalgia 3days ago suddenly becomes confused. His attn d. Pseudomonas
and joint pain like one week ago. Exam: span e. PCP
pulse=100bpm, temp=37.5C. CXR: bilateral patchy is reduced. He is restless and physically aggressive Ans. The key is B. Staphylococcus. [Among the
consolidation. What is the single most likely and picks at his bed sheets. What single given causes Staphylococcus and PCP are
causative organism? aspect of the pt’s hx recovered in his notes is recognized cause of cavitating pneumonia. This
a. Pneumococcal pneumonia most likely to aid in making the dx? case is with productive cough which goes more
b. Legionella a. Alcohol consumption with staphylococcus as PCP is not productive but
c. Mycoplasma b. Head trauma rather associated with dry cough. Drug abuse can
d. Klebsiella c. Hx of anxiety support both staphylococcus and PCP].
e. Chlamydia pneumonia d. Prescribed med
Ans. The key is C. Mycoplasma. e. Obvious cognitive impairment 472. A 71yo woman looks disheveled, unkempt
Ans. The key is A. Alcohol consumption. and sad with poor eye contact. She has recently
465. A 46yo man is being investigated for [abstinence from alcohol in the hospital lead to lost
indigestion. Jejunal biopsy shows deposition of delirium tremens]. her husband. Which of the following describes her
macrophages containing PAS (Periodic acid-schiff) condition?
+ve granules. What is the most likely dx? 469. A 10yo girl presents with pallor and features a. Anxiety
a. Bacterial overgrowth of renal failure. She has hematuria as well as b. Hallucination
b. Celiac disease proteinuria. The serum urea and creat are c. Mania
c. Tropical sprue elevated. These symptoms started after an d. High mood
d. Whipple’s disease episode of e. Low mood
e. Small bowel lymphoma bloody diarrhea 4days ago. What is the most Ans. The key is E. Low mood.
Ans. The key is D. Whipple’s disease. [periodic probable dx?
acid-schiff +ve granules containing macrophages a. TTP 473. A 62yo male comes to the GP complaining of
in jejunal biopsy is diagnostic of whipples b. HUS double vision while climbing downstairs. Which of
disease]. c. ITP the following nerve is most likely involved?
d. HSP a. Abducens nerve
466. A 32yo woman of 38wks gestation complains e. ARF b. Trochlear nerve
of feeling unwell with fever, rigors and abdominal Ans. The key is B. HUS. [Most cases of hemolytic c. Oculomotor nerve
pains. The pain was initially located in the uremic syndrome develop in children after two to d. Optic nerve
abdomen and was a/w urinary freq and dysuria. 14 days of diarrhea often bloody, due to infection e. Trigeminal nerve
The with a certain strain of E. coli. Features may be i) Ans. The key is B. Trochlear nerve. [oculomotor
pain has now become more generalized abdominal pain, ii) pale skin, iii) hematuria and may cause palsy of inferior rectus, medial rectus
specifically radiating to the right loin. She says proteinuria, iv) features of renal failure like- and superior rectus causing double vision in
that she nausea/vomiting, swelling of face, hand, feet or multiple gaze! But trochlear involving superior
has felt occasional uterine tightening. CTG is entire body etc. v) elevated urea and creatinine oblique only causes diplopia in downgaze only. So
reassuring. Select the most likely dx? etc.]. the answer is Trochlear nerve].
a. Acute fatty liver of pregnancy
b. Acute pyelonephritis 470. A 40yo woman has had intermittent tension, 474. L1 level, what is the most appropriate
c. Roung ligament stretching dizziness and anxiety for 4months. Each episode landmark?
d. Cholecystitis usually resolves after a few hours. She said she a. Mcburney’s point
e. UTI takes alcohol to make her calm. She is in a loving b. Stellate ganglion
Ans. The key is B. Acute pyelonephritis. [Fever, relationship and has no probs at work or home. c. Deep inguinal ring
rigor, abdominal pain a/w frequency, dysurea and What is the next step in her management? d. Termination of the spinal cord
radiation to the rt loin suggests rt sided a. Collateral info e. Transpyloric plane
pyelonephritis]. b. CT brain Ans. The given key is D. Which is a wrong key. The
c. CBC correct key is E. Transpyloric plane. [
467. A 32yo pt presents with cervical d. LFT The termination of the spinal cord is between L1
lymphadenopathy and splenomegaly. What is the e. TFT and L2 (variable between people).  L1 landmark-
single most Ans. The key is A. Collateral info. [Likely diagnosis duodenum first part, superior mesenteric artery,
appropriate option? is panic disorder. Collateral info from family, hila of both kidneys, upper border of pancreas,
a. Hemophilus friends and other peers should be asked to find splenic artery, pylorus and fundus of gall bladder].
b. Streptococcus out the cause for her anxiety].
c. Toxoplasmosis 475. A 32yo woman presents to the ED with
d. NHL 471. A 45yo IV drug abuser is brought into the ED headache and vomiting. She was decorating her
e. Pneumocystis jerovcii with complaint of fever, shivering, malaise, SOB ceiling
Ans. The key is D. NHL. [ Here only two points are and productive cough. Exam: temp=39C, that morning when the headache began, felt
mentioned- cervical lymphadenopathy and pulse=110bpm, BP=100/70mmHg. Inv: mainly occipital with neck pain. Some 2hs later
splenomegaly! This combination makes NHL as CXR=bilateral she
felt nauseated, vomited and was unable to walk. sputum, night sweats and weight loss are classic postchemotherapy neutropenia. Even febrile
She also noticed that her voice had altered. She features of tuberculosis]. neutropenia can be seen in patients with cancer
takes no reg meds and has no significant PMH. per-se!].
Exam: acuity, field and fundi are normal. She has 478. A 20yo pregnant 32wks by date presents to
upbeat nystagmus in all directions of gaze with the antenatal clinic with hx of painless vaginal 482. A 25yo woman presents with urinary freq,
normal facial muscles and tongue movements. bleeding after intercourse. Exam: P/A – soft and dysuria and fever. Urine microscopy shows 20-50
Her uvulas deviated to the right and her speech is relaxed, uterus=dates, CTG=reactive. Choose RBC
slurred. Limb exam: left arm past-pointing and the single most likely dx? and 10-20 WBC in each field. What is the most
dysdiadochokinesis with reduced pin prick a. Abruption of placenta 2nd to pre-eclampsia probable dx?
sensation in her right arm and leg. Although b. Antepartum hemorrhage a. Schistosmiasis
power is c. Placenta previa b. Kidney trauma
normal, she can’t walk as she feels too unsteady. d. Preterm labor c. Ureteric calculus
Where is the most likely site of lesion? e. Placenta percreta d. Bladder calculi
a. Right medial medulla Ans. The key is C. Placenta previa. e. Cystitis
b. Left medial pons Ans. The key is E. Cystitis. [Hematuria and
c. Left cerebellar hemisphere 479. A 30yo man presents to the ED with difficulty significant WBC in urine (>10 per HPF) makes
d. Right lateral medulla breathing. He has returned from India. Exam: cystitis the most likely diagnosis].
e. Left lateral medulla throat reveals grey membranes on the tonsils and
Ans. The key is Left lateral medulla. [ There is a uvula. He has mild pyrexia. What is the single 483. A 65yo presents with dyspareunia after sex.
loss of pain and temperature sensation on the most relevant dx? She in menopause. She complains of bleeding
contralateral (opposite) side of the body a. Diphtheria after
and ipsilateral (same) side of the face. There is b. IM sex. What is the most probably dx?
associated cerebellar symptoms and other cranial c. Acute follicular tonsillitis a. Cervical ca
nerve involvement. The condition is known as d. Scarlet fever b. Endometrial ca
Lateral Medullary Syndrome]. e. Agranulocytosis c. Ovarian ca
Ans. The key is A. Diphtheria. [history of travel to d. Breast ca
476. A 28yo female presents with 1 wk hx of india, grey membrane in tonsil and uvula, low e. Vaginal ca
jaundice and 2d hx of altered sleep pattern and grade fever, and dyspnoea support the diagnosis Ans. The key is B. Endometrial carcinoma. [Any
moods. of diphtheria]. post menopausal bleeding if associated with pain
She was dx with hypothyroidism for which she is lower abdomen (may be even in the form of
receiving thyroxine. TFT showed increased TSH. 480. A 23yo man comes to the ED with a hx of dyspareunia) is endometrial carcinoma unless
PT=70s. What is the most probable dx? drug misuse. He recognizes that he has a prb and proven otherwise. (even painless vaginal bleeding
a. Acute on chronic liver failure is in post menopausal women is highly suspicious!)].
b. Hyper-acute liver failure willing to see a psychiatrist. Which of the
c. Autoimmune hepatitis following terms best describes this situation? 484. A 45yo man underwent an emergency
d. Acute liver failure a. Judgement splenectomy following a fall from his bicycle. He
e. Drug induced hepatitis b. Thought insertion smokes
Ans. The key is C. Autoimmune hepatitis. c. Thought block 5 cigarettes/day. Post-op, despite mobile, he
[Autoimmune hepatitis may present as acute d. Mood develops swinging pyrexia and a swollen painful
hepatitis, chronic hepatitis, or well-established e. Insight left calf. His CXR shows lung atelectasis and
cirrhosis. Autoimmune hepatitis rarely presents Ans. The key is E. Insight. abdominal U demonstrates a small sub-[phrenic
as fulminant hepatic failure. One third may [in psychiatry, the patient's awareness and unders collection. What is the single most likely risk
present as acute hepatitis marked by fever, tanding of the origins and meaning of his factor for DVT in this pt?
hepatic tenderness and jaundice. Non specific attitudes, feelings, and behavior and of his disturb a. Immobility
features are anorexia, weight loss and behavioural ing symptoms (self-understanding) is known as b. Intraperitoneal hemorrhage
change (here altered sleep pattern and moods). insight]. c. Smoking
There may be coagulopathy (here PT=70s.) d. Splenectomy
leading to epistaxis, gum bleeding etc. Presence of 481. A pt with hodgkins lymphoma who is under e. Sub-phrenic collection
other autoimmune disease like hypothyroidism tx develops high fever. His blood results show Ans. The key is D. Splenectomy. [Splenectomized
supports the diagnosis of autoimmune hepatitis]. WBC patients have an increased risk of developing
<2800 and has a chest infection. Choose the most deep vein thrombosis and pulmonary embolism].
477. A 55yo man has a chronic cough and sputum, likely tx?
night sweats and weight loss. What is the single a. Co-amoxiclav 485. A 6m baby had LOC after which he had jerky
most likely causative organism? b. Piperacillin+tazobactam movement of hands and feet. What is the most
a. Coagulase +ve cocci in sputum c. Erythromycin probable dx?
b. Gram -ve diplococci in sputum d. Piperacillin+Co-amoxiclav a. Infantile spasm
c. Gram +ve diplococci in sputum e. Penicillin+tazobactam b. Absence
d. Pneumocystis carinii in sputum Ans. The key is B. Piperacillin+tazobactam. [Here c. Partial simple seizure
e. Sputum staining for mycobacterium patients WBC is <2800, i.e. patient has leucopenia d. Atonic seizure
tuberculosis (probable neutropenia). Piperacillin/Tazobactam e. Partial complex
Ans. The key is E. Sputum staining for may be used in the management of neutropenic Ans. The given key is A. Infantile spasm which is a
mycobacterium tuberculosis. [Chronic cough and patients with fever suspected to be due to a wrong key! Correct key is E. Partial complex. [In
bacterial infection as in patient with partial simple seizer there is no LOC (loss of
consciousness). Infantile spasm though may be d. Biliary colic difficulties and is constantly eating despite
associated with occasional LOC but its nature is e. Cholecystitis measures by his parents to hide food out of reach.
more generalized rather than the focal nature Ans. The key is cholecystitis. [Fat, female, fare, What is the most probable dx?
described here. So the likely option is E. Partial forty and fertile are the pneumonic for a. Cushing’s syndrome
complex seizure]. cholecystitis! Here the presentation of severe b. Congenital hypothyroidism
upper abdominal pain with fever along with the c. Prader Willi syndrome
486. A 24yo primigravida who is 30wk pregnant pneumonic features points towards the diagnosis d. Lawrence moon biedel syndrome
presents to the labor ward with a hx of constant of cholecystitis]. e. Down’s syndrome
abdominal pain for the last few hours. She also Ans. The key is C. Prader Willi syndrome. [Prader
gives a hx of having lost a cupful of fresh blood 490. A child has just recovered from meningitis. Willi syndrome rare congenital disorder
per vagina before the pain started. Abdominal What inv will you do before discharge? characterized by learning difficulties, growth
exam: irritable uterus, CTG=reactive. Choose the a. CT scan abnormalities, and obsessive eating, caused
single most likely dx? b. EEG especially by the absence of certain genes
a. Abruption of placenta 2nd to pre-eclampsia c. Blood culture normally present on the copy of chromosome 15
b. Antepartum hemorrhage d. Repeat LP inherited from the father].
c. Placenta previa e. Hearing test
d. Vasa previa Ans. The key is E. Hearing test. [deafness is a 494. A 20yo lady is suffering from fever and loss of
e. Revealed hemorrhage common complication of meningitis, so hearing appetite. She has been dx with toxoplasmosis.
Ans. The key is B. Antepartum hemorrhage. test is suggested before discharge]. What is the tx?
[Though presentation indicates abruption a. Pyrimethamine
placenta but abrutio placenta 2nd to pre-eclumpsia 491. A primiparous woman with no prv infection b. Pyrimethamine + sulfadiazine
can’t be the option as no features are suggestive with herpes zoster is 18wk pregnant. She had c. Clindamycin
of pre-eclumpsia here. So better option is B. recentcontact with a young 21yo pt having d. Spiramycin
Antepartum hemorrhage which also includes widespread chicken pox. What is the most e. Trimethoprim + sulfamethoxazole
abruption placenta. Only bleeding cannot indicate suitable Ans. The key is B. Pyrimethamine + sulfadiazine.
whether it is concealed, mixed or revealed]. management for the pregnant lady? [If the eye is involved, or if immunocompromized,
a. Acyclovir PO tx option is pyrimethamine + sulfadiazine OHCM
487. A 62yo lady presents with right sided b. Acyclovir IV +IVIG 9th edition, page 404].
headache and loss of vision. What is the single c. Acyclovir IV
most inv? d. Reassure 495. A 68yo woman has a sudden onset of pain
a. ESR e. IVIG and loss of hearing in her left ear and
b. BUE Ans. The key is E. IVIg. [If the pregnant woman is unsteadiness
c. CT head not immune to VZV and she has had a significant when walking. There are small lesions visible on
d. XR orbit exposure, she should be offered varicella-zoster her palate and left external auditory meatus.
e. IOP immunoglobulin (VZIG) as soon as possible. VZIG What is the single most likely dx?
Ans. The key is A. ESR. [Elderly (age 62), rt sided is effective when given up to 10 days after contact a. Acute mastoiditis
headache and loss of vision are suggestive of (in the case of continuous exposures, this is b. Cholesteatoma
temporal arteritis where elevated ESR is highly defined as 10 days from the appearance of the c. Herpes zoster infection
suggestive of this diagnosis]. rash in the index case). RCOG guideline]. d. Oropharyngeal malignancy
e. OM with infusion
488. A 24yo man asks his GP for a sick note from 492. A 40yo woman presents to the GP with low Ans. The key is C. Herpes zoster infection. [Herpes
work. He says that feels down, is lethargic and has mood. Of note, she has an increased appetite and zoster oticus (Ramsay Hunt syndrome) occurs
stopped enjoying playing the piccolo (his main has gone up 2 dress sizes. She also complains that when latent varicalla zoster virus reactivates in
hobby). He was admitted to the psychiatry ward she can’t get out of bed until the afternoon. the geniculate ganglion of the 7th cranial nerve.
last year following an episode of overspending, What is the most likely dx? Symptoms: Painful vesicular rash on the auditory
promiscuity and distractibility. What is the most a. Pseudo depression canal ± on drum, pinna, tongue, palate or iris with
probable dx? b. Moderate depression ipsilateral facial palsy, loss of taste, vertigo,
a. Psychosis c. Severe depression tinnitus, deafness, dry mouth and eyes. OHCM 9th
b. Cyclothymia d. Dysthymia edition, page 505].
c. Bipolar affective disorder e. Atypical depression
d. Seasonal affective disorder Ans. The key is E. Atypical depression. [Atypical 496. A 45yo woman has been dx with GCA and is
Ans. The key is C. Bipolar affective disorder. depression is a subtype of major being treated with steroids. What is the other
[presently patient has depression and previous depression or dysthymic disorder that involves drug
features of mania makes the diagnosis of bipolar several specific symptoms, including increased that can be added to this?
affective disorder likely]. appetite or weight gain, sleepiness or a. ACEi
excessive sleep, marked fatigue or weakness, b. Beta blockers
489. A 42yo female who is obese comes with moods that are strongly reactive to environmental c. Aspirin
severe upper abdominal pain with a temp=37.8C. circumstances, and feeling extremely sensitive to d. Interferons
She rejection]. e. IVIG
has 5 children. What is the most probable dx? Ans. The key is C. Aspirin [Low dose aspirin is
a. Ectopic pregnancy 493. An 8yo boy is clinically obese. As a baby he increasingly being recommended for people with
b. Ovarian torsion was floppy and difficult to feed. He now has a history of giant cell arteritis. It has been found
c. Hepatitis learning to be effective in preventing complications of
giant cell arteritis, such as heart attacks or stroke a. Cushing’s syndrome d. Costochondritis
(nhs.uk)]. b. Congenital hypothyroidism e. Pneumothorax
c. Down’s syndrome Ans. The key is A. MI. [In pericarditis pain is
497. A 17yo man has acute pain and earache on d. Lawrence moon biedel syndrome aggravated by inspiration or lying flat and relieved
the right side of his face. Temp=38.4C and has e. Primary obesity by leaning forward. Pericardial rub may present
extensive pre-auricular swelling on the right, Ans. The key is E. Primary obesity. and there may be fever. In pneumothorax pain is
tender on palpation bilaterally. What is the single not central but pleuritic. Pulmonary
most likely dx? 501. A 20yo boy is brought by his parents embolism=dyspnoea and pleuritic chest pain. In
a. Acute mastoiditis suspecting that he has taken some drug. He is costrochondritis localized pain/tenderness at the
b. Acute otitis externa agitated, costochondral junction enhanced by motion,
c. Acute OM irritated and can’t sleep. Exam: perforated nasal coughing, or sneezing. The given picture of
d. Mumps septum. Which of the following is the most central chest pain for 45 minutes (more than 30
e. OM with effusion likely to be responsible for his symptoms? minutes), sweating and dyspnoea with major risk
Ans. The key is D. Mumps. [C/F: prodromal a. Heroine factor of DM and Htn suggest the diagnosis of MI].
malaise, increased temperature, painful parotid b. Cocaine
swelling, becoming bilateral in 70%. OHCS 9th c. Ecstasy/MDMA/amphetamine 505. A man was brought to the ED from a
edition, page 142]. d. Alcohol shopping mall after collapsing there. He is
e. Opioids conscious and
498. An ECG of an elderly lady who collapsed in Ans. The key is B. Cocaine. [drug abuse with answering questions now. His ECG shows irregular
the ED shows rapid ventricular rate of 220bpm, perforated nasal septum indicates cocaine abuse]. rhythm. Your choice of inv:
QRS=140ms. What is the most probable dx? a. CT
a. Atrial fibrillation 502. For a pt presenting with Parkinson’s disease b. MRI
b. VT which of the following drugs is most useful in the c. 24h ECG
c. SVT management of the tremor? d. Echo
d. Mobitz type1 2nd degree heart block a. Apomorphine Ans. The key is D. Echo. [Echo may show clot in
e. Sinus tachycardia b. Cabergoline atrial appendage responsible for this attack of TIA
Ans. The key is B. Ventricular tachycardia. [Dx: i) c. Selegiline secondary to atrial fibrillation].
history (if IHD/MI likelihood of a ventricular d. Amantadine
arrhythmia is > 95%), ii) 12 lead ECG, and iii) lack e. Benzhexol 506. A 10yo boy is clinically obese and the
of response to IV adenosine). ECG findings in Ans. The key is E. Benzhexol. [Benzhexol (an shortest in his class. He had a renal transplant last
favour of VT: antiparkinsonian agent of anticholinergic class) is year
 Positive QRS concordance in chest leads the drug of choice in parkinson’s disease induced and his mother is worried that he is being bullied.
 Marked left axis deviation tremor)]. What is the most probable dx?
 AV dissociation (occurs in 25%) or 2:1 or a. Cushing’s syndrome
3:1 AV block 503. A 26yo woman has become aware of b. Congenital hypothyroidism
 Fusion beats or capture beats increasing right sided hearing deficiency since her c. Pseudocushing’s syndrome
OHCM 9th edition, page 816]. [In the given case recent d. Lawrence moon biedel syndrome
collapse, ventricular rate of 220 and broad QRS of pregnancy. Her eardrums are normal. Her hearing e. Down’s syndrome
140ms points towards VT]. tests show: BC-normal. Weber test lateralizes Ans. The key is A. Cushing’s syndrome. [Renal
to the right ear. What is the single most likely dx? transplant--> immune suppression is needed->
499. A pt presents with purple papular lesions on a. Encephalopathy exogenous steroid--> cushing syndrome. short
his face and upper trunk measuring 1-2 cm across. b. Functional hearing loss stature--> if steroids are used in early age then
They aren’t painful or itchy. What is the single c. Tympano-sclerosis they cause premature fusion of growth
most likely dx? d. Otosclerosis plate/calcification].
a. Kaposi’s sarcoma e. Sensorineural deafness
b. Hairy leukoplakia Ans. The key is D. Otosclerosis. [There are no 507. A 45yo man had cancer of head of pancreas
c. Cryptosporidium features of encephalopathy. As Weber test is which has been removed. He has a hx of
d. CMV infection lateralized it is unlikely to be functional hearing longstanding heartburn. He now comes with rigid
e. Cryptococcal infection loss. In tympanosclerosis ear drum becomes abdomen which is tender, temp 37.5C,
Ans. The key is A. Kaposis sarcoma. [It is a spindle- chalky white. So as the ear drum is normal it is not BP=90/70mmHg, pulse=120bpm. What is the next
cell tumour derived from capillary endothelial tympanosclerosis. Weber test is lateralized to step of the inv?
cells or from fibrous tissue, caused by human right and deafness is also on the right. So it not a. CT abdomen
herpes virus. It presents as purple papules (½ to 1 sensorineural deafness but conductive deafness b. XR abdomen
cm) or plaques on skin and mucosa (any organ). It which makes otosclerosis as the most likely c. MRI abdomen
metastasizes to nodes. OHCM 9th edition, page diagnosis]. d. US abdomen
716]. e. Endoscopy
504. A 58yo T1DM on anti-HTN therapy for 13yrs The answer is B. X-ray abdomen. [X-ray abdomen
500. A 6yo boy is clinically obese, his BMI >95th developed central chest pain for 45 mins while will help diagnosing perforation by showing gas
centile. He has no other medical prbs, driving a/w cold sweating and dyspnea. What is under diaphragm. This is a case of perforated
examination is the single most appropriate dx? peptic ulcer with the features of shock, abdominal
unremarkable. His mother says that she has tried a. MI rigidity and raised temperature. Stress from
everything to help him lose weight. What is b. Pericarditis serious disease and operation causes the body to
the most probable dx? c. Pulmonary embolism produce higher amounts of acid, which can
irritate preexisting ulcers leading to easy 512. A pt presents with dysphagia and pain on c. Whiplash injury
perforation]. swallowing. He has sore mouth and soreness in d. MI
508. A 50yo man presents to the ED with acute the e. Pancreatitis
back pain radiating down to his legs. Pain which is corners of the mouth. What is the single most Ans. The key is B. Myofacial pain. [Myofascial pain
usually relieved by lying down and exacerbated by likely dx/ syndrome is a chronic pain disorder. In myofascial
long walks and prolong sitting. What inv a. Kaposi’s sarcoma pain syndrome, pressure on sensitive points in
would be the best option? b. Molluscum contagiosum your muscles (trigger points) causes pain in
a. MRI c. CMV infection seemingly unrelated parts of your body. This is
b. CT spine d. Candida infection called referred pain. Myofascial pain
c. XR spine e. Toxoplasma abscess syndrome typically occurs after a muscle has been
d. Dual energy XR abruptiometry Ans. The key is D. Candida infection. [Candida is contracted repetitively].
e. Serum paraprotein electrophoresis more common than CMV].
Ans. The key is A. MRI. [Back pain radiating to leg, 513. A 30yo lady has epistaxis for 30mins. Her Hgb 518. A 70yo woman presents with recurrent
pain releaved by lying down and exacerbated by is normal, MCV normal, WBC normal, episodes of parotid swelling. She complains of
long walk and prolonged sitting are characteristic PT/APTT/Bleeding time are normal. Where is the difficulty
of lumber (intervertebral) disc disease]. defect? in talking and speaking and her eyes feel gritty on
a. Plts waking in the morning. What is the single
509. What is the most appropriate antibiotic to b. Coagulation factor most likely dx?
treat uncomplicated chlamydial infection in a c. Sepsis a. C1 esterase deficiency
21yo d. Anatomical b. Crohns disease
female who isn’t pregnant? e. RBC c. Mumps
a. Erythromycin Ans. The key is D. Anatomical. [bleeding time, d. Sarcoidosis
b. Ciprofloxacin coagulation profile, Hb%, cell count and e. Sjogrens syndrome
c. Metronidazole parameters are normal. So the cause of bleeding Ans. The key is E. Sjogrens syndrome. [parotid
d. Cefixime here is anatomical defect]. swelling, difficulty talking and speaking (due to
e. Doxycycline 514. Midpoint between the suprasternal notch dryness or less salive), eyes feeling gritty on
Ans. The key is E. Doxycicline. [Doxycycline 100 and pubic symphysis. What is the single most waking in the morning due to dryness of eye are
mg twice-daily for seven days or a single dose of 1 appropriate landmark? suggestive of Sjogrens syndrome].
g of azithromycin or Erythromyin 500 mg twice a. Fundus of the gallbladder 519. A 39yo woman has not had her period for
daily for 14 days or four times daily for seven days b. b. Mcburney’s point 10months. She feels well but is anxious as her
or Ofloxacin 200 mg twice-daily or 400 mg once- c. c. Stellate ganglion mother
daily for 7 days. In pregnant Azithromycine 1g d. d. Deep inguinal ring had an early menopause. Choose the single most
single dose is recommended then erythromycin e. e. Transpyloric plane appropriate initial inv?
500 mg twice daily for fourteen days or four a. Serum estradiol conc.
times daily for seven days. Then amoxicillin 500 Ans. The key is E. Transpyloric plane. b. Serum FSH/LH
mg three times daily for 7 days.]. 515. Tip of the 9th costal cartilage. What is the c. Serum progesterone conc.
510. A 45yo manual worker presented with a 2h single most appropriate landmark? d. None
hx of chest pain radiating to his left arm. His ECG a. Fundus of the gallbladder e. Transvaginal US
is b. Deep inguinal ring Ans. The key is B. Serum FSH/LH [here serum
normal. What is the single most appropriate inv? c. Termination of the spinal cord oestrogen is also important as i) low oestrogen +
a. Cardiac enzymes d. Transpyloric plane low FSH + low LH suggest hypothalamic
b. CXR e. Vena cava opening in the diaphragm amenorrhoea and i) low oestrogen + high FSH +
c. CT Ans. The key is A. Fundus of the gallbladder. high LH suggest premature ovarian failure! So the
d. ECG 516. A child complains of RIF pain and diarrhea. main determinant is serum FSH/LH. Likely cause
e. V/Q scan On colonoscopy, granular transmural ulcers are here is premature ovarian failure].
Ans. A. Cardiac enzymes. seen 520. A 50yo man with DM suddenly develops
511. A 26yo woman had bipolar disorder for 10yrs near the ileo-cecal junction. What should be the persistent crushing central chest pain radiating to
and is on Lithium for it. She is symptom free for management? the
the past 4 years. She is now planning her a. Sulfasalazine neck. What is the single most appropriate dx?
pregnancy and wants to know whether she should b. Paracetamol a. Angina
continue taking lithium. What is the single most c. Ibuprofen b. Costochondritis (tietz’s disease)
appropriate advice? d. Metronidazole c. Dissecting aneurysm
a. Continue lithium at the same dose and stop Ans. The key is A. Sulfasalazine. [Pain in RIF, d. MI
when pregnancy is confirmed diarrhea, granular transmural ulcers near the ileo- e. Pulmonary embolism
b. Continue lithium during pregnancy and stop cecal junction points towards the diagnosis of Ans. The key is C. Dissecting aortic aneurism.
when breast feeding Crohn’s diseas (predominantly ileo-cecal type)]. Probably wrong key. Correct key should be D. MI.
c. Reduce lithium dosage but continue throughout 517. A 60yo woman presents with acute onset of [The features described is insufficient and can be
pregnancy bone and back pain following a rough journey in a seen in both aortic dissection and MI. However
d. Reduce lithium gradually and stop when car. Exam: tenderness at mid-thoracic vertebra dissection pain is described as tearing and
pregnancy is confirmed with spasm, she feels better once she bends crushing pain is often used for mi pain. Both
e. Switch to sodium valproate forward. What is the single most probable dx? dissection and mi can have pain radiation to neck.
Ans. The key is D. Reduce lithium gradually and a. Osteoporotic fx verterbra History of diabetes goes with mi as it is a
stop when pregnancy is confirmed. b. Myofacial pain recognized risk factor for mi. Some may argue in
DM mi will be painless! But it is not always the e. Transpyloric plane Ans. The key is B. Bronchiectasis. [Persistent
case. MI is only painless when autonomic Ans. The key is D. Deep inguinal ring. cough with copious purulent sputum and finger
neuropathy becomes well established]. 525. 5th ICS in the ant axillary line. What is the clubbing points towards the diagnosis of
single most appropriate landmark? bronchiectasis. Severe lung infections such as
521 A 22yo man has rushed into the ED asking for a. Apex beat tuberculosis (TB), whooping cough, pneumonia or
help. He describes recurrent episodes of b. Chest drain insertion measles can damage the airways at the time of
fearfulness, palpitations, faintness, c. Stellate ganglion infection. Bronchiectasis may then develop
hyperventilation, dryness of the mouth with peri- d. Transpyloric plane (WHO)].
oral e. Vena cava opening into the diaphragm
tingling and cramping of the hands. His symptoms Ans. B. Chest drain insertion. 530. A 68yo man has had malaise for 5 days and
last 5-10 mins and have worsened since their 526. A 34yo man with MS has taken an OD of 100 fever for 2 days. He has cough and there is
onset 3months ago. He is worried he may be tablets of paracetamol with intent to end his life. dullness
having a heart attack. An ECG shows sinus He has been brought to the ED for tx but is to percussion at the left lung base. What is the
tachycardia. What is the single most appropriate refusing all intervention. single most appropriate inv?
immediate intervention? a. Assessment a. Bronchoscopy
a. High flow oxygen b. Evaluate pt’s capacity to refuse tx b. CXR
b. IV sedation c. Establish if pt has a prv mental illness c. CT
c. Rebreathe into a paper bag Ans. The key is B. Evaluate patients capacity to d. MRI
d. Refer for anxiety management course refuse treatment. e. V/Q scan
e. Refer for urgent cardiology opinion 527. A 23yo woman with painless vaginal bleeding
Ans. The key is C. Rebreathing into paper bag. at 36wks pregnancy otherwise seems to be Ans. The key is B. CXR. [Given presentation is
[Patient has anxiety disorder (panic) which causes normal. What should be done next step? suggestive of pneumonia for which investigation
hyperventilation and CO2 washout leading to a. Vaginal US of choice is CXR].
respiratory alkalosis. Symptoms will improve by b. Abdominal US
rebreathing into paper bag as it will cause gradual c. Vaginal exam 531. A 5yo child was admitted with hx of feeling
increase of CO2 in paper bag and decrease the d. Reassurance tired and lethargic all the time, bleeding gums and
severity of respiratory alkalosis]. Ans. The key is B. Abdominal US. This is a wrong sore throat since the last 3months. Exam:
522. An 8yo boy has longstanding asthma. He has key. The correct key is A. Vaginal US.[Painless hepatosplenomegaly. What is the most probable
admitted with a severe episode and is tired and vaginal bleeding at 36 weeks indicates the dx?
drowsy. He has not improved on oxygen, inhaled diagnosis of placenta previa, which can be better a. ALL
B2 agonist and IV hydrocortisone. CXR shows evaluated by vaginal US]. b. AML
bilateral hyperinflation. He is too breathless to c. CML
use a peakflow meter and is O2 sat <90%. What d. CLL
is the single most appropriate inv? e. Lymphoma
a. CBG Ans. The key is A. ALL. [Commonest leukemia in
b. CXR children is ALL. Bleeding gums (low platelet),
c. CT chest feeling tired and lethargic, sorethroat,
d. Pulse oximetry hepatosplenomegally all are well known features
e. Spirometry of ALL].
Ans. The key is A. CBG. [It will point towards 532. A 65yo man presents with back pain. Exam:
acidosis and indicate whether assisted ventilation splenomegaly and anemia. Blood: WBC=22,
is needed or not]. 528. A 29yo lady admitted with hx of repeated UTI Hgb=10.9, Plt=100, ESR=25. He has been found to
523. A man was operated for colorectal ca. His now developed hematuria with loin pain. What is have Philadelphia chromosome. What is the
pain is relieved with morphine 60mg bd PO but the most probable dx? single most likely dx?
now a. Acute pyelonephritis a. ALL
he can’t swallow medications. What will be the b. Chronic pyelonephritis b. AML
next regimen of analgesic administration? c. UTI c. CML
a. Oxycodone d. Bladder stone d. CLL
b. Fentanyl patch Ans. The key is A. Acute pyelonephritis. [In a e. Lymphoma
c. Morphine 60mg IV/d patient having hematuria and loin pain with Ans. The key is C. CML. [anaemia, raised WBC
d. Morphine 240mg IV/d history of repeated UTI suggest acute count, low platelet (platelet may be variable) are
Ans. The key is B. Fentanyl patch. [Here S/C pyelonephritis]. known features of CML, splenomegaly
morphine 1/2 the dose of oral can be given (not 529. A 45yo chronic smoker attends the OPD with (particularly if massive) is very suggestive of CML
present in option) or I/V morphine 1/3rd the oral complaints of persistent cough and copious and Philadelphia chromosome is characteristic of
dose can be given. Here I/V doses are not amount of purulent sputum. He had hx of measles CML].
appropriate so we should go for B. Fentanyl patch in the past. Exam: finger clubbing and 533. A 24yo woman has 8wk amenorrhea, right
as required morphine dose is known]. inspiratory crepitations on auscultation. What is sided pelvic pain and vaginal bleeding. She is
524. Just above the mid-inguinal point. What is the single most likely dx/ apyrexial. Peritonism is elicited in the RIF. Vaginal
the single most appropriate landmark? a. Interstitial lung disease exam reveals right sided cervical excitation.
a. Femoral artery pulse felt b. Bronchiectasis What is the most probable dx?
b. Mcburney’s point c. Asthma a. Ectopic pregnancy
c. Stellate ganglion d. COPD b. Salpingitis
d. Deep inguinal ring e. Sarcoidosis c. Endometriosis
d. Ovarian torsion 537. A woman presented with blurred vision and wasted and weak hands with diminished felexes.
e. Ovarian tumor intermittent clumsiness for 3m. Reflexes are brisk She also has weak spastic legs and dissociated
Ans. The key is A. [Salpingitis, Endometriosis, in her arm and optic disc is pale. What is the sensory loss. What is the dx?
overian torsions do not associated with single most appropriate test to confirm dx? a. MS
amenorrhoea. In ovarian tumour three main a. CSF analysis b. Syringomyelia
features are i) increased abdominal size and b. CT c. MND
persistent bloating (not bloating that comes and c. MRI d. Guillian-barre
goes) ii) persistent pelvic and abdominal pain iii) d. EEG e. Freidriech’s ataxia
difficulty eating and feeling full quickly, or feeling e. EMG Ans. The key is B. Syringomyelia. [weak limbs,
nauseous. Patient with pelvic pain and vaginal Ans. The key is C. MRI. [Features are suggestive of burn mark on fingertip (as pain and temperature
bleeding, peritonism and cervical exitation multiple sclerosis. Investigation of choice is sensation are lost due to spinothalamic tract
obviously points towards Ectopic pregnancy]. gadolinium enhanced MRI]. damage), wasted and weak hands with diminished
538. A 63yo man presents after having a seizure. reflexes, weak spastic legs with dissociated
534. A 64 yo woman has been treated for breast Exam: alert, orientated, inattention on the left sensory loss are features suggestive of
cancer with tamoxifen. What other drug should side Syringomyelia].
be and hyperreflexia of the arm. What is the most Common features are given below:
added to her tx regime? probable dx? Sensory features: 1) loss of pain and temperature
a. Bisphosphonates a. Cerebral tumor sensation 2) sensory loss is experienced over the
b. Calcium b. Pituitary adenoma arms, shoulders and upper body 3) light touch,
c. Vit D c. Cerebellar abscess vibration and position senses in the feet are
d. Calcitonin d. Huntingtons chorea affected as the syrinx enlarges into the dorsal
e. Phosphate binders e. Parkinsonism column.
Ans. The key is A. Bisphosphonates. Ans. The key is A. Cerebral tumour. Motor features: (when lower motor neurons of
[ bisohosphonates reduce the risk of bone 539. A 40yo man with a 25y hx of smoking the anterior horn cells are affected) 1) muscle
metastasis in cancers and is normally taken as presents with progressive hoarseness of voice, wasting and weakness begins in the hands and
adjuvant therapy in many types of tumours difficulty then affects the forearms and shoulders. 2)
including breast cancer. Plus it prevents bone swallowing and episodes of hemoptysis. He tendon reflexes are lost. Autonomic involvement
resorption]. mentioned that he used to be a regular cannabis like bladder and bowel can occur. [patient.co.uk]
535. A 26yo woman with regular menses and her user. What is the single most likely dx? 542. A 23yo woman is being followed up 6wks
28yo partner comes to the GP surgery a. Nasopharyngeal cancer after a surgical procedure to evacuate the uterus
complaining b. Pharyngeal carcinoma following a miscarriage. The histology has shown
of primary infertility for 2yrs. What would be the c. Sinus squamous cell carcinoma changes consistent with a hydatidiform mole.
single best investigation to see whether she is d. Squamous cell laryngeal cancer What is the single most appropriate inv in this
ovulating or not? e. Hypopharyngeal tumor case?
a. Basal body temp estimation Ans. The key is D. Squamous cell laryngeal cancer. a. Abdominal US
b. Cervical smear  Chronic hoarseness is the most common b. Maternal karyotype
c. Day2 LH and FSH early symptom. c. Paternal blood group
d. Day21 progesterone  Other symptoms of laryngeal cancer d. Serum B-HCG
e. Endometrial biopsy include pain, dysphagia, a lump in the e. Transvaginal US
Ans. The key is D. Day 21 progesterone. [Mid- neck, sore throat, earache or a Ans. The key is D. Serum β-HCG. [When you are
luteal progesterone level to assess ovulation: If persistent cough. first diagnosed with a hydatidiform mole, your
low, it may need repeating, as ovulation does not  Patients may also describe hCG level will be raised. When the hydatidiform
occur every month. The blood test is taken seven breathlessness, aspiration, haemoptysis, mole is treated (removed), the hCG level will
days before the anticipated period, that is on day fatigue and weakness, or weight loss. usually return to a normal, non-pregnant amount
21 of a 28-day cycle. However, this day will need (Patient.co.uk) and should remain so. If you develop GTN, the
to be adjusted for different lengths of cycle. Ref: 540. A 30yo lady complains of intermittent hCG level can remain elevated or continue to rise
patient.co.uk]. diarrhea, chronic abdominal and pelvic pain and further. So, this blood test is a good way to check
536. A 10yo boy who takes regular high dose tenesmus. Sometimes she notices blood in her for the possible development of GTN (Gestational
inhaled steroids for his longstanding asthma has stool. Select the most likely cause leading to her trophoblastic neoplasia).
been symptoms? 543. A 67yo man with hx of weight loss complains
advised to use bronchodilators to control his a. Inflammatory bowel disease of hoarseness of voice. CT reveals opacity in the
acute attacks. His parents are unsure when should b. Diverticulosis right upper mediastinum. He denied any hx of
he use his bronchodilator. What is the single most c. Irritable bowel disease difficulty breathing. What is the single most
appropriate inv? d. Adenomyosis appropriate inv?
a. CXR e. UTI a. Laryngoscopy
b. None Ans. The key is A. Inflammatory bowel disease. b. Bronchoscopy
c. Peak flow rate diary [Tenesmus excludes diverticulitis, occasional c. LN biopsy
d. Pulse oximetry blood in stool excludesirritable bowel disease. d. Bronchoalevolar lavage
Ans. The key is C. Peak flow rate diary. [Peak flow Features are not consistent with adenomyosis or e. Barium swallow
rate diary shows diurnal variation. This diary UTI but suggestive of inflammatory bowel Ans. The key is C. Lymph node biopsy. [There is
shows when the bronchoconstriction remains disease]. weight loss and there is an opacity in right upper
worse and guides to use bronchodilators prior to 541. A 50yo lady with weak limbs when examined mediastinum. May indicate enlarged lymph node
that times]. was found to have burn marks on finger tips, or lymphoma causing pressure on right recurrent
laryngeal nerve resulting in horseness. As CT  speech problems, such as slurred is more suggestive of panic disorder. Hence here
didn't reveal any bronchial lesion and no speech the diagnosis is B. Panic disorder].
breathing difficulty it is unlikely to be a bronchial  problems with your vision, such 550. A 55yo woman with a persistent cough and
pathology. So CT guided lymph node biopsy can as double vision hx of smoking develops left sided chest pain
reveal the diagnosis].  paralysis (loss of movement) on exacerbated by deep breathing with fever and
544. A 52yo man whose voice became hoarse one side of the body localized crackles. What is the single most
following thyroid surgery 1 wk ago shows no  problems walking and appropriate dx?
improvement. Which anatomical site is most likely frequent falls a. Dissecting aneurysm
affected?  seizures (fits) b. Pericarditis
a. Bilateral recurrent laryngeal nerve  loss of consciousness c. Pneumonia
b. Unilateral recurrent laryngeal nerve [Reference: nhs.uk] d. Pneumothorax
c. Unilateral external laryngeal nerve 547. A 50yo woman returned by air to the UK e. Pulmonary embolism
d. Bilateral external laryngeal nerve from Australia. 3days later she presented with Ans. The key is C. Pneumonia. [chest pain
e. Vocal cords sharp exacerbated with deep breathing, fever and
Ans. The key is B. Unilateral recurrent laryngeal chest pain and breathlessness. Her CXR and ECG localized crackles are highly suggestive of
nerve. are normal. What is the single most pneumonia].
545. A 73yo male presents with a 12m hx of falls. appropriate inv? 551. A 40yo woman complains of dysphagia for
His relatives have also noticed rather strange a. Bronchoscopy both solids and liquids. She sometimes suffers
behavior of late and more recently he has had b. Cardiac enzymes from
episodes of enuresis. Exam: disorientation to c. CT severe retrosternal chest pain. Barium swallow
time and place, broad-based, clumsy gait. What is d. MRI reveals a dilated esophagus which tapers to a
the most probable dx? e. Pulse oximetry fine distal end. What is the best management
a. Dementia f. V/Q scan strategy?
b. Pituitary adenoma g. CTPA a. Reassurance
c. CVD Ans. The key is G. CTPA. [Prolonged plane journey b. Antispasmodics
d. Syringomyelia is a recognized risk factor for thromboembolism c. Dilatation of the LES
e. Normal pressure hydrocephalus and hence pulmonary embolism also! Sharp chest d. Endoscopic diverticulectomy
Ans. The key is E. Normal pressure hydrocephalus. pain and breathlessness after 3 days of plane e. Barium swallow
[hx of falls and broad based clumsy gait (balance journey is highly suggestive of pulmonary Ans. The key is C. Dilatation of LES. [Dysphagia for
and gait disturbance), strange behavior and embolism the investigation of choice for which is both solids and liquids suggest neuromuscular
disorientation to time and place (due to CTPA]. dysphagia while dysphagia only for solid suggests
dementia), episodes of enuresis (urinary 548. A tall thin young man has sudden pain in the mechanical obstruction. Here features are
incontinence) points towards normal pressure chest and becomes breathless while crying. What consistent with achalasia for which lower
hydrocephalus. Classic triad of normal pressure is oesophageal sphincter dilation (balloon dilatation)
hydrocephalus: i) gait abnormality ii) urinary the single most appropriate inv? is a treatment modality].
incontinence and iii) dementia]. a. Cardiac enzymes 552. A 38yo female G4 at 32wks of pregnancy
546. A 75yo nursing home resident complains of b. CXR presented with thick white marks on the inside of
headache, confusion and impaired vision for c. CT her
4days. d. ECG mouth for 3wks. Her mouth including her tongue
She has multiple bruises on her head. What is the e. V/Q scan appeared inflamed on examination. She
most likely cause of confusion in this pt/ Ans. The key is B. CXR. [tall thin young men are smokes 20 cigarettes/day despite advice to quit.
a. Alcohol intoxication particularly prone to develop pneumothorax. She attends her ANC regularly. What is the
b. Infection Sudden pain and breathlessness in this young man most probable dx?
c. Subdural hematoma are highly suggestive of pneumothorax. So a. Lichen planus
d. Hypoglycemia investigation of choice is CXR]. b. Aphthous ulcer
e. Hyponatremia 549. A 21yo woman has had several sudden onset c. Smoking
Ans. The key is C. Subdural hematoma. [elderly episodes of palpitations, sweating, nausea and d. Candidiasis
patient with multiple bruises on her head suggest overwhelming fear. On one occasion she was e. Leukoplakia
head injury which can lead to subdural woken from sleep and feared she was going Ans. The key is D. Candidiasis. [lichen planus may
hematoma. Headache, confusion and impaired insane. have lace like appearance and not thick white
vision for 4 days indicate subacute subdural There is no prv psychiatric disorder. What is the mark. Aphthous ulcer has yellowish floor and
hematoma (three pahges i) acute ii) subacute 3 – most probable dx? surrounded by erythematous halo. Smoking may
7 days and iii) chronic 2 – 3 weeks]. Symptoms of a. Pheochromocytoma cause tongue coating but not like thick white
subdural hematoma: b. Panic disorder mark on the inside of mouth. Leukoplakia is with
 a headache that keeps getting c. GAD raised edges/Bright white patches and sharply
worse d. Phobia defined and cannot be rubbed out like candida
 feeling and being sick e. Acute stress disorder patch; here also inflamed tongue points towards
 confusion Ans. The key is B. Panic disorder. [here closest d/d infection. So candidiasis is the most probable
 personality changes, such as being to panic disorder is pheochromocytoma. But in option].
unusually aggressive or having pheochromocytoma the most important feature is 553. A 69yo woman has had a stroke. Her left
rapid mood swings resistant hypertension and other important upper and lower limbs are paralyzed and she is
 feeling drowsy and finding it features are headache and abdominal pain which having
difficult to keep your eyes open all are absent here. Moreover overwhelming fear
difficulty in speaking. Which anatomical site is 557. A 28yo woman has been admitted at 38wks Ans. The key is A. Bulimia nervosa. [18 year thin
most likely affected? gestation. Her BP=190/120mmHg and proteinuria girl, bilateral parotid swelling and thickened
a. Hippocampus +++. Immediately following admission she has a calluses on the knuckles from self induced
b. Cerebellum grand-mal seizure. What is the single most vomiting are suggestive of Bulimia nervosa.
c. Internal capsule appropriate initial management? Bulimia often is associated with bilaterak parotid
d. Thalamus a. Diazepam IV swelling (parotid hypertrophy)].
e. Brain stem b. Fetal CTG
Ans. The key is C. internal capsule. [Hippocampal c. Hydralazine IV 561. A 48yo presents with severe chest pain since
lesion causes mainly memory impairment. d. Immediate delivery the last 40mins. In the ED he is given oxygen, GTN,
Cerebellum has its diagnostic features that is not e. Magnesium sulphate IV morphine. ECG=ST elevation. Bloods=increased
present here. Thalamic lesion can lead to Ans. The key is E. Magnesium sulphate IV. [patient troponin levels. What is the next step of
impairment of arousal, orientation, learning and has established eclampsia. So she should be management?
memory, facial paresis, language deficit, treated with Magnesium sulphate as with 4g a. Beta blockers
hemispatial neglect, hemisensory loss, magnesium sulfate in 100mL 0.9% saline IVI over b. Percutaneous angiography
hemiparesis, hemiataxia and visual field defect. 5min + maintenance IVI of 1g/h for 24h. Beware c. Anticoagulant & heparin
Brainstem stroke causes impaired consciousness, depressed respiration. If recurrent seizure give 2g d. Clopidogrel
disorder of blood pressure, and breathing IVI magnesium sulfate over 5 min. Check tendon e. Aspirin
abnormality. Given picture is typical of lesion in reflexes and respiratory rate every 15min. Stop
internal capsule]. magnesium sulfate IVI if respiratory rate <14/min Ans. The key is B. Percutaneous angiography.This
554. A 72yo man brought to the ED with onset of or tendon reflex loss, or urine output <20mL/h]. is a wrong key! Correct key is E. Aspirin. [In any
paraplegia following a trivial fall. He was treated [OHCS, 9th edition, page 49]. case of IHD, most important initial management is
for aspirin].
prostatic malignancy in the past. What is the 558. A 27yo woman had pre-eclampsia and was
single most probable dx? delivered by C-section. She is now complaining of 562. A 34yo female presents with a foul smelling
a. Paget’s disease RUQ pain different from wound pain. What inv discharge. What set of organisms are we looking
b. Osteoporotic fx of vertebre will you do immediately? for
c. Secondary a. Coagulation profile to be treated here?
d. Multiple myeloma b. LFT a. Chlamydia, gonorrhea
e. Spondylosis c. Liver US b. Chlamydia, gardenella
d. MRCP c. Chlamydia, gonorrhea, gardenella
Ans. The key is C. Secondary. [In male e. None d. Gonorrhea, gardenella
osteoporotic fracture is less common. As patient e. Gardenella only
had prostatic malignancy pathological fracture Ans. The key is B. LFT. [In a post ceasarian patient
from secondary metastasis to bone (here vertebra having pre-eclampsia RUQ pain different from Ans. The key is E. Gardenella only. [Here foul
leading to paraplegia) is more common]. wound pain points towards the diagnosis of HELLP smelling discharge is caused by gardenella. So
syndrome. So immediate investigation to be done most acceptable answer is E. Gardenella only].
555. A 14yo girl has developed an itchy, scaly is LFT]. [OHCS, 9th edition, page 26].
patch on her scalp. She had a similar patch that 563. A 6wk formula fed baby boy is found at the
cleared 559. A 10yo girl has been referred for assessment child health surveillance to be deeply jaundiced.
spontaneously 2yrs ago. Her aunt has a similar of hearing as she is finding difficulty in hearing her His
undiagnosed rash on the extensor aspects of her teacher in the class. Her hearing tests show: BC weight gain is poor and his stools are pale. What
elbows and knees. What is the single most likely normal, symmetrical AC threshold reduced is the most likely dx?
dx? bilaterally, weber test shows no lateralization. a. Galactosemia
a. Eczema What is the single most likely dx? b. Biliary atresia
b. Fungal infection a. Chronic perforation of tympanic membrane c. G6PD deficiency
c. Impetigo b. Chronic secretory OM with effusion d. Rh incompatibility
d. Lichen planus c. Congenital sensorineural deficit e. Congenital viral infection
e. Psoriasis d. Otosclerosis
Ans. The key is E. Psoriasis. [itchy, scaly patch on e. Presbycusis Ans. The key is B. Biliary atresia. [Deep jaundice at
scalp are classic presentation of scalp psoriasis. 6th week with pale stools suggests obstructive
Her aunts presentations (similar rash on extensor Ans. The key is B. Chronic secretory OM with jaundice. So most likely diagnosis here is biliary
aspects of her elbow and knees) are suggestive of effusion. atresia].
psoriasis. It is thought to be an immunological
disease]. 560. A thin 18yo girl has bilateral parotid swelling 564. A 45yo man with colon cancer now develops
556. A pt after transurethral prostatic biopsy. with thickened calluses on the dorsum of her increased thirst, increased frequency in urination
What electrolyte imbalance can he develop? hand. and weight loss. His fasting blood
a. Hyperkalemia What is the single most likely dx? glucose=9mmol/L. what is the most appropriate
b. Hyponatremia a. Bulimia nervosa management?
c. Hypocalcemia b. C1 esterase deficiency a. Oral hypoglycemic
d. Hypernatremia c. Crohn’s disease b. Insulin long acting
e. Hypercalcemia d. Mumps c. Short acting insulin before meal
Ans. The key is B. Hyponatremia. [Use of fluid for e. Sarcoidosis d. IV insulin
bladder irrigation may lead to hyponatremia]. e. Subcutaneous insulin
most likely dx? e. Continue IV ceftriaxone as mono-therapy
Ans. The key is A. Oral hypoglycemic. [Increased a. OM with effusion
thirst and increased frequency in urination along b. Otitis externa Ans. The key is B. Change to IV amoxicillin +
with weight loss is suggestive of DM supported by c. Cholesteatoma gentamycin. [From the given option B is the most
fasting blood glucose of 9 mmol/L. At the age of d. CSOM acceptable. However Ampicillin + gentamycin is
45 most likely type of diabetes is NIDDM or type 2 e. Tonsillitis the drug combination of choice].
DM which is treated by oral hypoglycemic agents].
Ans. The key is A. OM with effusion. This is a 572. A pt presents with fever, dry cough and
565. A 34yo man from Zimbabwe is admitted with wrong key. Correct key is B. Otitis externa. [The breathlessness. He is tachypneic but chest is clear.
abdominal pain to the ED. An AXR reveals bladder childs preschool hearing loss and increasing the tv Oxygen saturation is normal at rest but drops on
calcification. What is the most likely cause? volume suggests that he has OM with effusion but exercise. What is the single most likely dx?
a. Schistosoma mansoni present earache and fever points towards the a. CMV infection
b. Sarcoidosis diagnosis of otitis externa]. b. Candida infection
c. Leishmaniasis c. Pneumocystis carinii infection
d. TB 569. A pt presents with gradual onset of d. Cryptococcal infection
e. Schistosoma hematobium headache, neck stiffness, photophobia and e. Toxoplasma abscess
fluctuating LOC.
Ans. The key is E. Schistosoma hematobium. CSF shows lymphocytosis but no organism on Ans. The key is C. Pneumocystis carinii infection.
[Bladder involvement is caused by Schistosoma gram stain. CT brain is normal. What is the single [Fever, dry cough, breathlessness, tachypnoea
hematobium while Schistosoma mansoni causes most likely dx? with clear chest is seen in pneumocystis carinii
intestinal disease]. a. Hairy leukoplakia pneumonia. Normal oxygen saturation which
b. TB drops on exercise is characteristic of
566. A 6yo came with full thickness burn. He is c. CMV infection pneumocystis carinii pneumonia].
crying continuously. What is the next step of d. Candida infection
management? e. Cryptococcal infection 573. A 14yo boy fell and hit his head in the
a. Refer to burn unit playground school. He didn’t lose consciousness.
b. IV fluid stat Ans. The key is B. TB. [Fungal meningitis can also He has
c. Antibiotic present like this but it is much more rare. swelling and tenderness of the right cheek with a
d. Analgesia Moreover negative gram stain excludes fungal subconjuctival hemorrhage on his right eye.
e. Dressing cause here. Hence TB meningitis is more What is the most appropriate initial inv?
acceptable answer]. a. CT brain
Ans. The key is analgesia. This is a wrong key. b. EEG
Correct key should be B. IV fluid stat. [Here 570. An 18m boy has been brought to the ED c. MRI
already mentioned full thickness burn which is because he has been refusing to move his left arm d. Skull XR
painless. Child often cry from anxiety for and e. Facial XR
hypoxaemia and hypovolaemia rather than pain. crying more than usual for the past 24h. He has
The patient then responds better to oxygen or recently been looked after by his mother’s new Ans. The key is E. Facial X-ray. This is a wrong key
increased fluid administration rather than to bf while she attended college. Assessment shows Correct key is A. CT brain. [With the risk of basal
narcotic analgesics. Ref: patient.info]. multiple bruises and a fx of the left humerus fracture we should do CT scan to diagnose this. In
which is put in plaster. What is the single most present case CT is better than MRI. Skull X-rays
567. A 78yo nursing home resident is revived due appropriate next step? are no longer recommended as first line
to the development of an intensely itchy rash. a. Admit under care of pediatrician investigation].
Exam: white linear lesions are seen on the wrists b. Discharge with painkillers
and elbows and red papules are present on the c. Follow up in fx clinic 574. A 15m child is due for his MMR vaccine.
penis. What is the most appropriate d. Follow up in pediatric OPD There is a fam hx of egg allergy. He is febrile with
management? e. Follow up with GP acute
a. Topical permethrin OM. What is the single most appropriate action?
b. Referral to GUM clinic Ans. The key is A. Admit under care of a. Defer immunization for 2wks
c. Topical betnovate pediatrician. [This is NAI. So the child cannot be b. Don’t give vaccine
d. Topical ketoconazole handover to the risk again and should be admitted c. Give half dose of vaccine
e. Topical selenium sulphidehyosine to protect him from further injury done by d. Give paracetamol with future doses of the
mothers boyfriend while serial x-rays and relevant same vaccine
Ans. The key is A. Topical permathrine. [The investigations done and asked for child protection e. Proceed with standard immunization schedule
intensely itchy rash, scratch marks and burrows unit’s help].
on wrist and elbow red papules on penis are Ans. The key is A. Defer immunization for 2 wks.
suggestive of scabies. Topical permethrine are 571. A 74yo female presents with headache and
used to treat it]. neck stiffness to the ED. Following a LP the pt was 575. A 33yo lady with Hodgkin’s lymphoma
started on IV ceftriaxone. CSF culture = listeria presents with temp=40C, left sided abdominal
568. A 4yo has earache and fever. Has taken monocytogenes. What is the appropriate tx? pain and
paracetamol several times. Now it’s noticed that a. Add IV amoxicillin lymphadenitis. Blood was taken for test. What will
he b. Change to IV amoxicillin + gentamicin you do next?
increases the TV volume. His preschool hearing c. Add IV ciprofloxacin a. Wait for blood test
test shows symmetric loss of 40db. What is the d. Add IV co-amoxiclav b. Start broad spectrum IV antibiotics
c. Oral antibiotics 580. An 83yo woman who is a resident in a d. Acyclovir
d. CBC nursing home is admitted to hospital with a 4d hx e. Dexamethasone
e. Monitor pyrexia of
diarrhea. She has had no weight loss or change in Ans. The key is B. Cefotaxime. [The patient is
Ans. The key is B. Start broad spectrum IV appetite. She has been on analgesics for 3wks getting probable meningococcal meningitis.
antibiotics. [The patient is immunocompromized for her back pain. She is in obvious discomfort. On Before confirming the diagnosis suggested
with signs of infection (temp=40◦C, left sided rectal exam: fecal impaction. What is the treatment is, where
abdominal pain and lymphadenitis) broad single most appropriate immediate management? the organism is unknown:
spectrum IV antibiotic should be started a. Codeine phosphate for pain relief • <55yrs: cefotaxime 2g/6h slow IV.
empirically while waiting for blood reports]. b. High fiber diet • >55yrs: cefotaxime as above + ampicillin 2g
c. Oral laxative IV/4h (for Listeria). So in given case Cefotaxime is
576. A 40yo man with marked weight loss over d. Phosphate enema the option. Ref: OHCM, 9th edition, page 832].
the preceding 6m has bilateral white, vertically e. Urinary catheterization
corrugated lesion on the lateral surfaces of the 5084. A 15yo girl was admitted with anemia, chest
tongue. What is the single most likely dx? Ans. The key is D. Phosphate enema. [In feacal infection and thrombocytopenia. She was treated
a. C1 esterase deficiency impaction oral laxative is not the choice but and her symptoms had regressed. She was
b. Crohns disease phosphate enema is the best option here]. brought again with fever and the same symptoms
c. HIV disease a
d. Sarcoidosis 581. A 26yo woman being treated for a carcinoma few days later. She also seems to have features of
e. Sjogren’s syndrome of the bronchus with steroids presents with meningitis. What is the most likely dx?
vomiting, abdominal pain and sudden falls in the a. AML
Ans. The key is C. HIV disease. [The lesion morning. What is the most specific cause for b. ALL
described is leukoplakia which is likely association her symptoms? c. Aplastic anemia
of HIV disease]. a. Steroid side effects d. CML
b. Postural hypotension e. CLL
577. A 3m baby was miserable and cried for 2h c. Adrenal insufficiency
following his 1st routine immunization with DTP, d. Conn’s disease Ans. The key is B. ALL. [The age supports the
HiB e. Cushing’s disease diagnosis of ALL along with the given picture.
and meningitis. What is the single most Same picture can happen in aplastic anaemia but
appropriate action? Ans. The key is C. Adrenal insufficiency. [Patients there is not a single factor mentioned in favour of
a. Defer immunization for 2wks on steroid develop suppression of hypothalamic it. So ALL can be taken as best option in the given
b. Don’t give vaccine pituitary adrenal axis and frequently may lead to scenario].
c. Give half dose of vaccine adrenal insufficiency (here vomiting, abdominal
d. Give paracetamol with future doses of the pain and sudden falls in the morning secondary to 585. A pt was admitted to the ED after a head
same vaccine postural hypotension on getting up from bed injury. When examined on arrival his GCS=15 and
e. Proceed with standard immunization schedule points towards the diagnosis of adrenal then
insufficiency)]. at night his GCS deteriorated to 12. What
Ans. The key is E. Proceed with standard investigation should be done?
immunization schedule. 582. A 78yo woman presents with unilateral a. CT head
headache and pain on chewing. ESR=70mm/hr. b. XR skull
578. A 65yo man with HTN develops gingival She is on c. IV mannitol
hyperplasia. What is the single most likely dx? oral steroids. What is the appropriate additional d. Drill a burr hole
a. ACEi tx? e. Shift to OR
b. Beta blockers a. Bisphosphonates
c. Crohns disease b. HRT Ans. The key is A. CT head. [Initial GCS 15 followed
d. Nifedipine c. ACEi by later GCS 12 are suggestive of intracranial
e. Sarcoidosis d. IFN haemorrhage. So the best investigation to be
e. IV steroids done is CT head].
Ans. The key is D. Nifedipine. [Gingival hyperplasia
is a recognized side effect of calcium channel Ans. The key is A. Bisphosphonates. [The elderly 586. A 4yo boy who prv had normal hearing, has a
blockers]. lady with giant cell arteritis is getting high dose mild earache relieved by paracetamol. He has
steroid which can lead to demineralization and been noticed to turn up the vol on the TV. He has
579. A 65yo woman is undergoing coronary osteopenia or osteoporosis. So to prevent this bilateral dull tympanic membranes. His preschool
angiography. What measure will protect her bisphosphonates are given]. hearing test shows symmetrical loss of 40dB.
kidneys What is the single most likely dx?
from contrast? 583. A 30yo man is suffering from fever, rash and a. Acute otitis externa
a. Furosemide photophobia. Doctors are suspecting he is b. Acute OM
b. Dextrose suffering c. Ear wax
c. 0.45% saline from meningitis. Which is the best medication for d. Foreign body
d. 0.9% saline this condition? e. OM with effusion
a. Ampicilling
Ans. The key is D. 0.9% saline. b. Cefotaxime Ans. The key is OM with effusion.
c. Tetracycline
587. An 18yo man presents to his GP with thirst Ans. The key is D. OSAS. [Day time somnolence Ans. The key is A. Hypercalcemia. [Hypercalcemia
and polyuria. Some 6m ago he had a significant and obesity (high BMI of 36) points the diagnosis is common in squamous cell carcinoma].
head of OSAS].
injury as the result of a RTA. He is referred to the 594. A 72yo man presents with intermittent
local endocrine clinic. Which of the following 590. A 35yo pregnant woman has been having difficulty in swallowing with regurgitation of stale
results would be the most useful in confirming the tingling and numbness of her thumb, index and food
dx of diabetes insipidus after a water middle fingers for a while. She has been treated materials. Sometimes he wakes up at night with a
deprivation test (without additional with local steroids but it hasn’t helped her feeling of suffocation. Choose the single most
desmopressin)? much and now she has planned to undergo a likely cause of dysphagia?
a. Plasma sodium of 126mmol/l surgical procedure. Which of the following a. Benign structure
b. Plasma sodium of 150mmol/l structures will be incised? b. Esophageal carcinoma
c. Plasma osmolality of 335mosmol/kg and urine a. Flexor digitorum profundus c. Esophageal spasm
osmolality of 700mosmol/kg b. Transverse carpal ligament d. Pharyngeal pouch
d. Plasma osmolality of 280mosmol/kg and urine c. Palmar aponeurosis e. Systemic sclerosis
osmolality of 700mosmol/kg d. Extensor retinaculum
e. Plasma osmolality of 335mosmol/kg and urine Ans. The key is D. Pharyngeal pouch. [In benign
osmolality of 200mosmol/kg Ans. The key is B. Transverse carpal ligament. stricture, esophageal carcinoma and systemic
[This is a case of carpal tunnel syndrome. So sclerosis there is persistent dysphagia not
Ans. The key is E. Plasma osmolality of 335 transverse carpal ligament is the structure which intermittent. In oesophageal spasm there may be
mosmol/kg and urine osmolality of 200 will be incised]. intermittent dysphagea but there will be no
mosmol/kg. [In DI plasma osmolality will be high regurgitation of stale food material and no
due to excess fluid loss with urine and urine 591. A 58yo pt presents with altered bowel habits nocturnal regurgitation in recumbency. The
osmolality will be low due to polyuria. Hence E. is and bleeding per rectum. Exam and clinical picture described well fits with pharyngeal
the best option here(normal plasma osmolality sigmoidoscopyshowed an ulcer. What is the single pouch].
275-295 mosmol/kg and normal urine osmolality most likely dx?
is 300-900 mosmol/kg)]. a. Colorectal carcinoma 595. A 9m child is brought to the ED with an
b. Celiac disease irreducible firm swelling which descended into the
588. A 75yo man has left-sided earache and c. Crohns disease left
discomfort when he swallows. There is ulceration d. UC groin when the child has been crying. Exam: both
at the e. IBS testicles are palpable in the scrotum. What is
back of his tongue and he has a palpable non- the most appropriate management strategy?
tender cervical mass. What is the single most Ans. The key is A. Colorectal carcinoma. [Age, a. Reassurance
likely altered bowel habits, bleeding per rectum and b. Emergency herniotomy
dx? isolated ulcer on sigmoidoscopy suggest c. Elective herniotomy
a. Acute mastoiditis colorectal cancer]. d. Emergency herniotomy + orchidopexy
b. Dental abscess e. Elective herniotomy + orchidopexy
c. Herpes zoster infection 592. A mother is concerned that her 18m son has
d. Oropharyngeal malignancy a vocabulary of ten words but can’t form a Ans. The key is C. Elective herniotmy. [If there was
e. Tonsillitis sentence. What is the best management strategy? features of strangulation we would go for
a. Arrange hearing test emergency herniotomy. But as only irreducible we
Ans. The key is D. Oropharyngeal malignancy. b. Assess developmental milestones shall proceed to elective herniotomy].
[Pain ear and discomfort during swallowing, c. Reassurance
ulceration at the back of the tongue and palpable d. Refer to speech therapist 596. A 37yo woman was admitted for femur fx
non tender cervical lymphnode is suggestive of e. MRI brain repair after a RTA. On the 4th post-op day she
oropharyngeal malignancy. Acute mastoiditis and became
tonsillitis will not cause tongue ulcer. Similarly Ans. The key is C. Reassurance. [Two words confused and starts picking on her bed sheets and
dental abscess will not cause tongue ulcer. In joining can be done in 2yrs and inability to form a complains of seeing spiders all over. What is
herpes zoster pain and vesicle will be in the sentence in 18 months is quite normal. So the the most likely dx?
affected nerve distribution]. option is reassurance]. a. Delirium tremens
b. Wernickes encephalopathy
589. A 42yo man has been tired and sleepy for the 593. A 55yo man has weight loss, dyspnea and c. Korsakoffs psychosis
last few weeks in the morning. His work has syncope. He smokes 20 cigarettes/day. Inv d. Psychotic depression
started confirms
getting affected as he feels sleepy in the meetings. squamous cell carcinoma in the left bronchus. Ans. The key is A. Delerium tremens. [withdrawal
His BMI=36. What is the single most likely dx? What is the single most likely biochemical of alcohol due to hospital admission lead to
a. Idiopathic hypersomnia abnormality to be a/w the condition? delirium tremens. Warnicke’s encephalopathy has
b. Narcolepsy a. Hypercalcemia triad of i) mental confusion ii) abnormal eye
c. Chest hyperventilation syndrome b. Hyperkalemia movements & iii) unsteady gait. Korsakoff's
d. OSAS c. Hypernatremia syndrome cannot be diagnosed until the person
e. REM-related sleep disorder d. Hypocalcemia has stopped drinking alcohol for several weeks, to
e. Hypomagnesium enable the immediate symptoms of alcohol
intoxication and withdrawal to subside. Features
of Korsakoffs psychosis i) dementia, loss of short
term memory ii) difficulty in acquiring new infancy. So she needs to assess developmental in the third person, for example "he is an evil
information or learning new skills iii) personality milestones]. person".].
change iv) confabulation].
602. A pt presents with progressive visual 606. A 65yo woman had an excision of colonic
597. A 36yo pt came with diarrhea, bleeding, deterioration. Exam: large, multiple cotton wool tumor 3yrs ago. Now she is losing weight and
weight loss and fistula. What is the single most spots in feels
likely dx? both eyes. What is the single most likely dx? lethargic. Exam: pale but no abdominal findings.
a. Celiac disease a. Kaposi’s sarcoma What is the most appropriate inv?
b. Crohns disease b. Cryptosporidium a. CA 125
c. UC c. CMV infection b. CA 153
d. IBS d. Pneumocystis carinii infection c. CA 199
e. Cryptococcal infection d. CEA
Ans. The key is B. Crohns disease. e. AFP
Ans. The key is C. CMV infection. [Large multiple
598. A 72yo woman who is taking loop diuretics is cotton wool spots are seen in early stage of CMV Ans. The key is D. CEA. [CA 125 = ovarian cancer;
suffering from palpitations and muscle weakness. retinitis]. CA 153 = cancer breast; CA 199 = pancreatic
What is the electrolyte imbalance found? cancer; CEA = colorectal carcinoma; AFP =
a. Na+ 130mmol/l, K+ 2.5mmol/l 603. A 53yo had a dental extraction after which he hepatocellular carcinoma].
b. Na+ 130mmol/l, K+ 5.5mmol/l recently had a mitral valve prolapse, high temp of
c. Na+ 140mmol/l, K+ 4.5mmol/l 39C, cardiac failure and new cardiac murmur. 607. A 46yo African-Caribbean man is found to
d. Na+ 150mmol/l, K+ 3.5mmol/l What is the single most likely dx? have BP=160/90mmHg on 3 separate occasions.
e. None a. Atheroma What
b. Congenital is the best initial tx?
Ans. The key is A. Na+ 130mmol/l, K+ 2.5mmol/l. c. Regeneration a. ACEi
[Loop diuretic causes hypokalemia and d. Infection b. Beta-blockers
hyponatremia]. e. Neoplastic c. ARBs
d. None
599. A 60yo diabetic pt on anti-diabetic Ans. The key is D. Infection. [Infective e. CCB
medication developed diarrhea. What is the most endocarditis].
likely cause Ans. The key is E. CCB. [If age less than 55 years
for his diarrhea? 604. A 12yo boy with a hx of fall on an but Afro-Caribbean origin then CCB].
a. Autonomic neuropathy outstretched hand was brought to the ED with
b. Infective swelling and 608. A 39yo woman will undergo tubal
c. Celiac disease pain around the elbow. His radial nerve was sterilization and she wants to know the failure
d. Crohns disease affected. What is the type of fx? rate of this
a. Angulated fx type of sterilization.
Ans. The key is A. Autonomic neuropathy. b. Epiphyseal fx a. 1:50
c. Compound fx b. 1:200
600. Which artery runs in the anterior inter- d. Spiral fx c. 1:500
ventricular groove? d. 1:1000
a. Acute marginal branch Ans. The key is D. Spiral fracture. It is wrong key. e. 1:5000
b. Left ant descending artery The correct option should be A. Angulated
c. Septal branches (supracondylar fracture). [Around 50% Ans. The key is B. 1:200.
d. Circumflex artery interosseous nerve lesions occur in supracondylar
e. Right coronary artery fracture whereas 25% shows radial nerve damage. 609. Which of the following reflexes and
If the fracture is spiral fracture of lower third of innervating spinal nerves are correctly paired?
Ans. B. Left anterior descending artery. humerus it causes nerve damage in 18% almost all a. Anal reflex – S1
of which are radial nerve lesion. However as the b. Ankle jerk – L5
601. A mother presents with her 12m daughter. fracture is around the elbow (not above) it is c. Biceps jerk – C7 & C8
The child has no meaningful words, is unable to sit supracondylar fracture]. d. Knee jerk – L3 & L4
unaided and can’t play with her toys. She doesn’t 605. A 32yo lady complains that she hears e. Triceps jerk – T1
laugh and has poor interaction with her everyone saying that she is an evil person. What
siblings. What is the best management strategy? type of Ans. The key is D. Knee jerk – L3 & L4. [Anal reflex
a. Arrange hearing test hallucinations is she suffering from? – S2-4; Ankle jerk – S1-2; Biceps jerk – C5-6; Knee
b. Assess developmental milestones a. 2nd person auditory hallucinations jerk – L3-4; Triceps jerk – C7].
c. Reassure b. 3rd person auditory hallucinations
d. Refer to speech therapist c. Echo de la pense 610. A 62yo man with rheumatoid arthritis struck
e. MRI brain d. Gedankenlautwerden his hand against a door. He subsequently found
that although he could extend the interphalangeal
Ans. The key is B. Assess developmental Ans. The key is B. 3rd person auditory joint of his right thumb, the MCP joint of the
milestones. [At 12 month one word should be said hallucinations. [Third person hallucinations are thumb remained flex. What is the single most
clearly, in 8 months child can sit independently, auditory hallucinations in which patients hear likely tendon to have been damaged?
smiles at 2 months and plays with toys since early voices talking about themselves, referring to them a. Extensor carpi ulnaris
b. Extensor digitorum episodes responded quickly to bronchodilator, Ans. The key is D. PCOS. [It is not cushing’s as
c. Extensor indicis she has no symptoms or abnormal physical signs. insulin levels in cushing’s are not usually raised!
d. Extensor pollicis brevis She has slight eczema and her mother has Here obesity, primary amenorrhea , acne and
e. Extensor pollicis longus asthma. What is the single most appropriate inv? particularly high level of insulin makes the likely
a. CXR diagnosis to PCOS].
Ans. The key is D. Extensor pollicis brevis. [Action b. Peak flow rate diary
of extensor pollicis brevis = extension of thumb at c. Pulse oximetry 618. An 18yo girl with primary amenorrhea
metacarpophalangeal joint. Extensor pollicis d. Spirometry complains of severe abdominal pain every 4-
longus = extends the terminal phalanx of the e. Sweat test 8weeks
thumb]. which is now getting worse. Exam: lower
Ans. The key is D. Spirometry. [spirometry is the abdominal mass is felt. What is the most probable
611. A 68yo lady complains of falls to the ground preferred initial test (if available) to assess the dx?
without any warning, maintains consciousness presence and severity of airflow obstruction less a. Ectopic pregnancy
and effort dependent and more repeatable though b. Ovarian carcinoma
no confusion. She says this has occurred at less applicable in acute severe asthma]. c. Hematometrium
number of times. What is the dx? d. Biliary colic
a. Stokes Adams attack 615. A 45yo man had recently started taking anti- e. Renal carcinoma
b. Hypoglycemia HTN therapy. 6m later his RBS=14mmol/l. Which
c. Vasovagal syncope single drug is most likely to have caused this? Ans. The key is C. Hematometrium. [Primary
d. Drop attacks a. Amlodipine amenorrhea and periodic pain indicate
e. Epilepsy b. Bendroflumethiazide hematometrium either secondary to imperforated
c. Doxazosin hymen or vaginal septum].
Ans. The key is D. Drop attacks. [Drop attacks are d. Lorsartan
sudden spontaneous falls while standing or e. Ramipril 619. A 14yo boy with asthma suddenly developed
walking, with complete recovery in seconds or chest pain and increasing breathlessness during a
minutes. There is usually no recognized loss of Ans. The key is B. Bendroflumethiazide. [High game of football. When seen in the ED he was not
consciousness, and the event is remembered]. blood sugar is a well known side effect of cyanosed. He has reduced breath sounds on
bendroflumethiazide]. the right side. His oxygen saturation is 94% on air.
612. A 50yo man complains of being pursued by What is the single most appropriate inv?
the police for a crime he denies committing. He 616. A 27yo waitress has pelvic pain, a. Capillary blood gases
has dysmenorrhea and increasingly heavy periods. b. CXR
poor concentration and impaired short-term She also c. CT chest
memory. He admits to drinking large amounts of complains of dyspareunia. There is generalized d. Exercise challenge
alcohol for the last 20yrs. What is the most pelvic tenderness without peritonism. Pelvic US e. MRI chest
probable dx? is normal. What is the most likely dx?
a. Dementia a. Endometriosis Ans. The key is B. CXR. [Asthma is a predisposing
b. Hallucination b. Uterine fibroid factor for spontaneous pneumothorax. The
c. Wernicke’s encephalopathy c. Pelvic congestion syndrome presentation indicates pneumothorax for which
d. Schizophrenia d. PID most appropriate investigation is CXR].
e. Korsakoff psychosis e. Tubal pregnancy
620. A 36yo woman was recently admitted to a
Ans. The key is E. Korsakoff psychosis. [Dementia, Ans. The key is C. Pelvic congestion syndrome. [In psychiatric ward. She believes that the staff and
i.e. short term memory loss is seen in korsakoff pelvic congestion syndrome there develops other pts know exactly what she is thinking all the
psychosis]. varicose veins in the lower abdomen from time. What is the most likely symptom this pt
prolonged standing (as occurred here in a is suffering from?
613. A pt with prv hx of HTN, the membranes waitress who remains standing for long) with a. Thought insertion
have ruptured and the cervix is 3cm dilated. 4h some pain syndromes like pelvic pain, b. Thought withdrawal
later on dysmenorrea, dyspareunia and generalized pelvic c. Thought block
examination showed that the cervix was still 3cm tenderness without peritonism. Also there may be d. Though broadcasting
dilated. What is the single most appropriate associated menorrhagia]. e. Hallucination
management for her labor?
a. Repeat vaginal examination in 4h 617. A 14yo girl is clinically obese. She has not Ans. The key is D. Thought broadcasting.
b. CTG started her periods yet and has severe acne.
c. C-section Among 621. A 60yo woman is admitted to the hospital
d. External rotation her inv, a high insulin level is found. What is the after a fall. She is noted to have poor eye contact.
e. IV syntocin drip most probable dx? When asked how she is feeling, she admits to
a. Cushing’s syndrome feeling low in mood and losing enjoyment in all
Ans. The key is E. IV syntocinon drip. [There is no b. Grave’s disease her usual hobbies. She has also found it difficult to
progress of labour in 4 hours. Hence syntocinon c. Acquired hypothyroidism concentrate, feels that she is not good at
drip should be given]. d. PCOS anything, feels guilty over minor issues and feels
e. Addison’s disease very negative about the future. What is the
614. A 6yo girl has had 2 short episodes of cough most likely dx?
and wheeze over the last 12m. These 2 acute a. Mild depression
b. Moderate depression On jejunal biopsy, there is shortening of the villi
c. Severe depression and lymphocytosis. What is the most likely dx? Ans. The key is C. Allergic reaction. [The probable
d. Psychosis a. Celiac disease diagnosis is scabies in which there is itching due
e. Seasonal depression b. Whipple’s disease to allergic reaction to mites (Sarcoptes scabii)
c. Crohn’s disease waste products].
Ans. The key is A. Mild depression. [Mild d. Tropical sprue
depression: i)Low mood ii) Anhedonia iii) Guilt iv) e. Giardiasis 629. A teacher had a respiratory infection for
Hopelessness v) Worthlessness vi) Inability to f. Cystic fibrosis which she was prescribed antibiotics. After the
concentrate]. antibiotic course when she rejoined school, she
Ans. The key is A. Celiac disease. [diarrhea, lost her voice completely. What is the single
622. A 70yo woman lives in a nursing home abdominal bloating, blistering skin rash over most appropriate dx?
following a stroke has developed reddish scaly elbow (Dermatitis herpetiformis), low serum a. Recurrent laryngeal nerve palsy
rash on albumin, calcium and folate conc. Supported by b. Angioedema
her trunk. She has many scratch marks on her shortening of villi and lymphocytosis on jejuna c. Laryngeal obstruction by medication
limbs and trunk with scaling lesions on her hands biopsy is classic presentation of celiac disease]. d. Laryngitis
and feet. What is the single most appropriate e. Functional dysphonia/vocal cords
initial tx? 626. A 19yo man presents for the 1st time with a
a. Aqueous cream firm and unshakable belief that he is being Ans. The key is E. Functional dysphonia/vocal
b. Chlorphenaramine followed cords. [Functional dysphonia is poor voice quality
c. Coal tar by terrorists who are plotting against him. What is without any obvious anatomical, neurological or
d. 1% hydrocortisone ointment the single best term for this man’s condition? other organic difficulties affecting the larynx or
e. Permethrin a. Delusion of persecution voice box. It is often secondary to viral infection].
b. Delusion of grandeur
Ans. The key is E. Permethrine. [This is a case of c. Delusion of control 630. A 43yo lady is admitted with pyrexia,
scabies and scaly rash denotes the infection of d. Delusion of reference arthropathy, breathlessness and syncope. She was
most severe type the crusted or Norwegian e. Delusion of nihilism recently
scabies. Should be treated with permethrine]. dx with pulmonary emboli. There is an early
Ans. The key is A. Delusion of persecution. diastolic sound and a mid-diastolic rumble. Her
623. A 16yo boy following a RTA was brought to [Delusions of persecution refer to false beliefs or JVP
the ED with a swelling and deformity in his right perceptions in which a person believes that they is elevated with prominent a-waves. What is the
thigh. Exam: airway is patent and is found to have are being treated with malicious intent, hostility, most likely cause?
a pulseless leg. Which structure is involved in or harassment – despite significant evidence to a. Mitral regurgitation
this fx? suggest otherwise]. b. Ventricular ectopics
a. Femoral artery c. Pulmonary regurgitation
b. Posterior tibial artery 627. A 19yo female is brought in by her parents. d. Atrial myxoma
c. Common peroneal nerve They are concerned about her BMI which is 12. e. Complete heart block
d. Dorsalis pedis She
is satisfied with it. What is the next step? Ans. The key is D. [Pyrexia, arthropathy,
Ans. The key is A. Femoral artery. a. Psychiatric referral for admission breathlessness, syncope and early diastolic sound
b. Family counselling and a mid diastolic rumble are known features of
624. A man sat cross-legged for about 30mins. c. Social service atrial myxoma].
After this he was unable to dorsiflex his left foot d. Start antidepo
and e. Medical admission 631. A 28yo man presents with a maculopapular
had loss of sensation in the web space between rash over his trunk and palms. He also has
the big toe and the 2nd toe. He also has sensory Ans. The key is E. Medical admission. [The numerous mouth ulcers. He had a penile ulcer
loss on the same side of the foot after 2h. Which diagnosis is anorexia nervosa. At this critical low which healed 2wks ago. What will you do to
of the following was affected? BMI medical admission is indicated to improve confirm the dx?
a. Femoral nerve her deficiency states and proper nutrition. ((BMI a. PCR for treponemal and non-treponemal
b. Sural nerve <15kg/m2, rapid weight loss + evidence of system antibiodies
c. Peroneal nerve failure) requires urgent referral to eating disorder b. Dark ground microscopy from mouth ulcer
d. Sciatic nerve unit (EDU), medical unit (MU) or paediatric c. Blood culture for treponema
medical wards]. d. Dengue fever
Ans. The key is C. Peroneal nerve.[Common
peroneal nerve winds round the fibular neck at 628. A lady who works at a nursing home presents Ans. The key is A. PCR for treponomal and non-
knee joint and when a man sits cross legged for a with itching. Exam: linear tracks on the wrist. She treponemal antibodies. [Non treponemal
considerable time pressure exerted on the nerve says that 2d ago she had come in contact with a antibody test if positive indicate that there may
may cause nerve palsi]. nursing home inmate with similar symptoms. be syphilis and it is not confirmatory alone. That is
What is the mechanism of itching? why treponemal antibody test should also be
625. A 25yo woman is presenting with diarrhea a. Infection done to confirm it. On the other hand resolved
and abdominal bloating over the last 4m. Exam: b. Destruction of keratinocytes disease may show negative treponemal test which
she c. Allergic reaction is confirmed by positive non-treponemal test].
has blistering rash over her elbows. Biochemistry: d. Immunosuppression
low serum albumin, calcium and folate conc. e. None
632. A 34yo man complains of arthralgia, e. Scleritis gets excited. What would be the most appropriate
abdominal pain and vomiting, a facial rash that is management for this child?
worse in Ans. The key is B. Acute dacrocystitis. a. Desmopressin
the summer and hematuria. Urea and creatinine b. Oxybutanin
are slightly elevated with urinalysis 636. A 60yo lady has severe chest pain. ECG c. Behavioural therapy
demonstrating red cell casts. PMH is remarkable shows changes of inferior wall MI. ECG also shows d. Tamsulosin
for childhood eczema. Which inv is most likely progressive prolongation of PR interval until a QRS e. Restrict fluid intake
to lead to a dx? complex is dropped. What is the most
a. US KUB probable dx? Ans. The key is C. Behavioural therapy. Probably
b. Joint aspiration a. Atrial fibrillation this is wrong option! There is no correct option for
c. Auto antibodies b. VT this question. [The question is inappropriate. Child
d. IVU c. SVT is of 4 yrs of age and before 5 yrs only
e. Renal biopsy d. Mobitz type I 2nd degree heart block reassurance, no treatment is indicated].
e. Mobitz type II 2nd degree heart block
Ans. The key is C. Auto antibodies. [Likely 641. A 34yo DM pt is undergoing contrast
diagnosis is SLE for which auto antibody (anti ds Ans. The key is D. Mobitz type I 2nd degree heart radiography. What measure should be taken to
DNA antibody) should be done]. block. [Inferior MI is frequently associated with prevent
conduction defect]. renal damage with contrast dye?
633. A 56yo woman has had severe abdominal a. Reduce contrast dye
pain for 24h radiating to her back and is 637. A 52yo woman speaks rapidly without any b. Plenty of fluids
accompanied pause and ignores interruptions. She doesn’t even c. NSAIDS
by nausea and vomiting. She appears to be pause to take enough breaths. What term best d. ACEi
tachycardic and in shock. She was found to have describes this kind of speech? e. IV dextrose
gallstones, 2yrs ago. What is the most likely inv to a. Flight of ideas
confirm dx? b. Broca’s aphasia Ans. The key is B. Plenty of fluids.
a. US abdomen c. Wernicke’s aphasia
b. LFT d. Pressure of speech 642. A 75yo woman presents to the breast clinic
c. Serum lipase e. Verbal dysphasia having noticed that she has had a blood stained
d. Angiography discharge from the left nipple, together with dry
e. CT abdomen Ans. The key is D. Pressure of speech. skin over the left areola. Exam: blood stained
discharge with dry flaky skin noted on the left
Ans. The key is C. Serum lipase. [The likely 638. A 30yo woman has been feeling low and areola. The nipple was noted to be ulcerated. Wht
diagnosis is pancreatitis hence serum lipase]. having difficulty in concentrating since her mother is the most appropriate inv?
passed away 2m ago. She feels lethargic and a. FNAC
634. A 32yo female with axillary freckles and café tends to have breathlessness and tremors from b. MRI
au lait spots wants to know the chances of her time to time. What is the most likely dx? c. Punch biopsy
child a. Adjustment disorder d. Open biopsy
also having similar condition. b. PTSD e. Stereotactic biopsy
a. 1:2 c. Panic disorder
b. 1:4 d. GAD Ans. The key is C. Punch biopsy. [As the lesion is
c. No genetic link e. Bereavement on the surface punch biopsy can be well
d. 1:16 obtained].
e. Depends on the genetic make up of the partner Ans. The key is A. Adjustment disorder. [When 2
months passed it is no more normal bereavement 643. A 50yo man presents with low mood, poor
Ans. The key is E. Depends on genetic make up of but major depression or adjustment disorder]. concentration, anhedonia and insomnia. He has
the partner. [Depends on the make up of the had
partner, ideally it's 1:2 since it's autosomal 639. A 32yo man on psychiatric medications 2 episodes of increased activity, promiscuity and
dominant with complete penetrance but if the complains of inability to ejaculate. Which drug is aggressive behavior in the past. He was arrest
patients spouse also has neurofibromatosis, it most 8m ago for trying to rob a bank claiming it as his
raises it to at least 75% with a possible 100%]. likely to cause these symptoms? own. Which drug is most likely to benefit him?
a. Lithium a. Haloperidol
635. A 40yo man has pain, redness and swelling b. Haloperidol b. Citalopram
over the nasal end of his right lower eyelid. The c. Chlorpromazine c. Desipramine
eye d. Fluoxetine d. Carbamazepine
is watery with some purulent discharge. The e. Clozapine e. Ethosuximide
redness extends on to the nasal peri-orbital area
and mucoid discharge can be expressed from the Ans. The key is D. Fluoxetine. [SSRIs are frequently Ans. The key is D. Carbamazepine. [It is a case of
lacrimal punctum. What is the single most associated with delayed ejaculation]. bipolar disorder. Mainstay of treatment is mood
appropriate clinical dx? stabilizers such as i) Lithium carbonate ii)
a. Acute conjunctivitis 640. A 4yo boy is brought by his parents with Anticonvulsant medicines iii) Antipsychotic
b. Acute dacrocystitis complains of wetting his bed at night and medicines. So from the given options
c. Acut iritis whenever he Carbamazepine is the most appropriate drug].
d. Retrobulbar neuritis
644. A 25yo woman complains of dizziness, tics like throat clearing, sniffing, grunting or
nausea, vomiting, visual disturbances and anxiety Ans. The key is B. Check serum K+ level. This is a barking and more complex like coprolalia (uttering
which wrong key! Correct key is E. IV fluid. [We shall socially inappropriate words) or echolalia
keep coming from time to time. Most of the resuscitate first with normal saline]. (repeating the word or phrase of others)].
attacks are a/w sudden change in posture. What is 649. A 55yo woman who attends the clinic has
the most likely dx? recently been dx with a depressive episode. She 653. A 52yo male presents with sudden complete
a. Panic disorder complains of unintentionally waking early in the loss of vision from right eye. He also had been
b. Carotid sinus syncope morning, a recent disinterest in sex and a loss complaining of right sided headaches which
c. BPPV of appetite, losing 5kg weight in the last month. would come up more on chewing. On fundoscopy,
d. Vertebrobasilar insufficiency She feels that her mood is worse at the the retina was pale and a cherry red spot could be
e. Postural hypotension beginning of the day. What is the most likely dx seen in the macular region. What caused this
for this pt? vision loss?
Ans. The key is C. Benign paroxysmal positional a. Mild depression a. CRAO
vertigo (BPPV). [Dizziness, nausea, vomiting and b. Moderate depression b. CRVO
nystagmus which keep coming from time to time c. Severe depression c. Branch RAO
are common features of BPPV]. d. Low mood d. Branch RVO
e. Pseudo depression e. Circumciliary vein occlusion
645. A 56yo man was recently put on anti-HTN
meds and recent biochemistry on 2 occasions Ans. The key is B. Moderate depression. [Sleep Ans. The key is A. CRAO. [Pale retina with cherry
showed: disturbance, disinterest in sex and loss of appetite red spot in macular region is seen in CRAO].
Na+=132, K+=7.6, Urea=11.3, Creat=112. Which of points towards the diagnosis of moderate
the following drugs is responsible for this depression]. 654. A 48yo woman presents with left-sided
result? severe headache. She also has a red, watering eye
a. Amlodipine 650. An employer sent his worker to the ED after and
b. Bendroflumethiazide having hit his head on a machine. Exam: normal. complains of seeing colored haloes in her vision.
c. Doxazosin What is the single most likely inv you would do? What is the most appropriate next step?
d. Atenolol a. Skull XR a. Measure IOP
e. Ramipril b. CT head b. Relieve pain with aspirin
c. MRI head c. 100% oxygen
Ans. The key is E. Ramipril. [ACEI and ARB are d. Reassure d. CT
known to raise the serum potassium level]. e. Relieve pain with sumatriptan
Ans. The key is A. Skull XR. This is wrong key!
646. A 46yo woman has offensive yellow Correct key should be B. CT head. Ans. The key is A. Measure IOP. [Probable case of
discharge from one nipple. She had a hx of breast angle closure glaucoma requiring measurement of
abscess 651. A lady with fam hx of ovarian carcinoma has IOP to establish the diagnosis].
3yrs ago. What is the possible dx? a pelvis US that fails to reveal any abnormality.
a. Duct papilloma What is the single most appropriate inv? 655. A 31yo woman presents with 7-10days
b. Duct ectasia a. Pelvic CT following childbirth, with loss of feeling for the
c. Duct fistula b. CA 125 child, loss of appetite, sleep disturbance and
d. Breast cancer c. CA 153 intrusive and unpleasant thoughts of harming the
d. Laparoscopy baby. What
Ans. The key is C. Duct fistula. e. MRI is the best tx for this pt?
a. Fluoxetine
647. A 35yo woman undergoing tx for TB presents Ans. The key is B. CA 125. [CA 125 is tumour b. Haloperidol
with malar rash, photosensitivity and hematuria. marker for ovarian tumour]. c. CBT
What is the single most likely positive antibody? d. Reassurance
a. Anti Ds DNA 652. A 10yo boy is taken to his GP by his parents e. ECT
b. Anti Sm with behavioural prbs. He attends a special school Ans. The key is A. Fluoxetine. [The diagnosis is
c. Anti Histone due to inappropriate behavior and during the post-partum depression. Treatment is fluoxetine.
d. Anti La interview with his parents the boy barks at The mode of treatment may be 1. Drugs like a)
e. Anti centromere infrequent episodes and shouts expletives. What Antidepressant b) Antypsychotic or c) Mood
is the most likely dx? stabilizers like lithium. If drug treatment fails then
Ans. The key is C. Antihistone antibody. [Anti a. Asperger syndrome 2. ECT].
histone antibody is present in 95% cases of drug b. Cotard syndrome 656. A 56yo male pt presents with intermittent
induced lupus]. c. Rett syndrome vertigo, tinnitus and hearing loss. What is the best
d. Ekbom syndrome drug tx for this pt?
648. A 6wk child with profuse projectile vomiting. e. Tourette’s syndrome a. Buccal prochlorperazine
What is the first thing you will do? b. Oral flupenphenazine
a. US Ans. The key is E. Tourette’s syndrome. [Tourete’s c. TCA
b. Check serum K+ level syndrome may have motor tics like blinking, facial d. Gentamicin patch on the round window
c. ABG grimacing, shoulder shrugging. Other complex e. No med tx available
d. NG tube motor tics may be sniffing, touching objects,
e. IV fluids hopping, jumping, bending or twisting. It has vocal
Ans. The key is A. Buccal prochlorperazine. likely explanation for the abnormal inv?
[Probable case of Menieres disease. Treated with a. Acute pericarditis 665. A 79yo woman has been dx with T2DM. Her
prochlorperazine]. b. Cardiac tamponade BMI=22. RBS are 8 and 10mmol/l. Her
c. Atrial thrombus BP=130/80mmHg. Her fasting
657. An 82yo woman has developed painful rash d. Left ventricular aneurysm cholesterol=5.7mmol/l. She is currently symptom-
on one side of her forehead and ant scalp. Lesions e. Dressler syndrome free but has
have also affected her cornea. What is the single microalbuminuria. What is the single most
most appropriate option? Ans. The key is A. Acute pericarditis. [Chest pain, appropriate drug management?
a. Accessory nerve raised temperature and ECG findings of a. ACEi and glibenclamide
b. Facial nerve widespread ST elevation with upwards concavity b. ACEi and metformin
c. Olfactory nerve is diagnostic of acute pericarditis particularly after c. Statin and ACEi
d. Optic nerve MI]. d. Statin and glibenclamide
e. Trigeminal nerve e. Statin and metformin
661. A 55yo man presents with an ulcer of the
Ans. The key is E. Trigeminal nerve. [Probable scrotum. Which of the following LN is involved? Ans. The key is C. Statin and ACEi. [Diabetic
herpes zoster opthalmicus]. a. External iliac LN patients are advocated statin irrespective of
b. Pre-aortic LN cholesterol levels and diabetic microalbuminuria
658. A 24yo woman presents with episodes of c. Aortic LN is best treated by ACEI. As initially we shall give
peri-oral tingling and carpo pedal spasms every d. Inguinal LN lifestyle advice and no medicine for diabetes even
time e. Iliac LN then we shall start with statin and ACEI].
she has to give a public talk. This also happens to f. Submental LN
her before interviews, exams and after g. Submandibular LN 666. A 68yo woman is unable to extend the IP
arguments. What is the best management h. Deep cervical LN joint of her right thumb 7wks following a fx of the
strategy for this pt? right
a. Diazepam Ans. The key is D. Inguinal LN. radius. Other finger and thumb movements are
b. Rebreathe in a paper bag normal. What is the single most likely tendon to
c. Desensitization 662. A 35yo woman has butterfly rash on her face be damaged?
d. Buspirone and she suffers symmetrical joint pains on knee a. Abductor pollicis longus
e. Propranolol and elbow, ESR is raised. What is the most b. Extensor pollicis brevis
discriminative inv for dx? c. Extensor pollicis longus
Ans. The key is B. Rebreathe in a paper bag. This is a. Anti DNA antibodies d. Flexor digitorum profundus
wrong key. More correct option is C. b. Anti Jo1 antibodies e. Flexor pollicis longus
Desensitization.[Desensitization is the treatment c. Anti nuclear antibodies
of choice in long run. For prevention proranalol d. Anti centromere antibodies Ans. The key is C. Extensor policis longus. [Full
before expected exposure and if patient presents e. Anti la antibodies extension of right thumb is achieved by extensor
with an attack then rebreathing in a paperbag to pollicis longus].
subside her acute problems]. Ans. The key is A. Anti DNA antibodies. [Anti DNA
antibodies to diagnose SLE]. 667. A mother presents her 6m son who is
659. A 32yo woman P3 of 39wks gestation reports vocalizing. She has noticed that he doesn’t
having spontaneous ROM 4days ago. She didn’t 663. Pt had a fight following which he developed respond to
attend the delivery suite as she knew that would bleeding, ringing and hearing loss from one ear. loud noises. His motor milestones are normal.
happen and had already decided on a home What is the inv of choice? What is the best management strategy?
birth. Today she feels very hot and sweaty. She a. CT a. Arrange hearing test
thought that she was starting to have labour b. XR skull b. Assess development milestones
pains but she describes the pain as more c. Otoscopy c. Reassure
constant. Exam: uterus is tender throughout. d. MRI vestibule d. Refer to speech therapist
Blood e. Coagulation study e. MRI brain
tests show raised CRP and WBC. Select the most
likely dx? Ans. The key is A. CT. This is a wrong key! Correct Ans. The key is A. Arrange hearing test. [Normal
a. Round ligament stretching key is Otoscopy. motor milestones indicate normal development
b. Chorioamnionitis but unable to respond to loud noise at 6 months
c. Uterine rupture 664. A 35yo IVDA (Intra Venous Drug Abuser) on may indicate deafness].
d. Labor penicillin and flucloxacillin for cellulitis now
e. DIC presents with jaundice, pale stools 668. A 39yo man presents to the ED with
and dark urine. What is the single most likely dx? persistent cough, sputum and dyspnea. He gave a
Ans. The key is B. Chorioamnionitis. [Prolonged a. Hep A hx of
rupture of membrane can lead to b. Cholestatic jaundice smoking 20 cigarettes/d for the last 10 years. Pt
chorioamnionitis]. c. Chronic active hepatitis was given oxygen in ambulance but he is not
d. Primary biliary cirrhosis improving. What is the next step?
660. A 63yo man continues to experience chest e. Hep B a. Prednisolone
pain and has a temp of 37.8C 2 days after an acute b. Salbutamol
MI. His ECG shows widespread ST elevation with Ans. The key is B. Cholestatic jaundice. c. Check ABG
upward concavity. What is the single most [Flucloxacillin can cause cholestatic jaundice]. d. CXR
e. ECG aortic aneurysm? 676. A lady underwent debulking surgery for
a. Atheroma ovarian carcinoma. Soon after the surgery she
Ans. The key is C. Check ABG. [The patient has b. Cystic medial necrosis presents
COPD and as no improvement with oxygen, next c. Dissecting aneurysm with signs of intestinal obstruction. What is the
step is to check ABG to give guidance for next d. Polyarteritis nodosa single most appropriate inv?
treatment plan]. e. Syphilis a. Pelvic CT
b. CA 125
669. A 66yo woman has been brought to the Ans. The key is A. Atheroma. c. Laparotomy
hospital on a number of occasions with a hx of d. Laparoscopy
loss of 673. A 33yo male came to the hospital with e. Abdominal US
memory. Her PMH is significant for an MI 6yrs complaint of occasional left sided chest pain that
ago. It is noted that she has a step wise decline of lasts Ans. The key is C. Laparotomy. [In cancer patient
her cognitive functions. What is the most likely <30mins, following exercise, which relieves upon obstruction does not respond to medical
dx? taking rest. What is the most probable dx? treatment and hence diagnostic and therapeutic
a. Alzhemiers a. Unstable angina laparotomy is the option of choice].
b. Vascular dementia b. Decubitus angina
c. Pick’s dementia c. Stable angina 677. A 45yo woman undergoing tx for RA for the
d. Huntington’s disease d. Coronary spasm last 5yrs presents with dizziness, easy fatigabiliy
e. Lewy body dementia e. MI and
lack of energy. A blood film shows MCV 106. What
Ans. The key is B. Vascular dementia. [Age 65 yrs Ans. The key is C. Stable angina. could be the most probable reason for her
(vascular dementia is rare at age before 65), anemia?
history of MI and stepwise decline of cognitive 674. A 42yo overweight smoker comes with heavy a. Steroids
function indicates vasculopathy and hence periods. A scan reveals a normal uterus. She b. Chronic disease
vascular dementia]. would like a long term tx with minimal side effects c. NSAIDs
that would offer tx for the menorrhagia and d. Methotrexate
670. A 55yo man returns for routine follow up provide contraception. She is unsure whether she e. Sulfasalazine
6wks after an MI. He gets breathless when would like more children. She is adamant that
walking she doesn’t want surgery as she is terrified of the Ans. The key is D. Methotrexate. [Methotrexate
uphill. His ECG shows ST elevation in leads V1, V2, prospect. Select the best management for her leads to folate deficiency anaemia].
V3 and V4. What is the single most likely menorrhagia?
explanation for the abnormal investigation? a. COCP 678. A 62yo man who has recently had flu-like
a. Heart block b. GrH analogues illness has woken to find his speech altered.
b. Right ventricular strain c. IU/systemic progesterone Movement of his eyelids and lips are weak on the
c. Atrial thrombus d. NSAIDs right side. Exam: normal. Which anatomical
d. Left ventricular aneurysm e. Copper containing IUCD site is most likely to be affected?
e. Dressler’s syndrome a. Facial nerve
Ans. The key is C. IU/systemic progesterone. [As b. Hypoglossal nerve
Ans. The key is D. Left ventricular aneurism. patient is smoker, COCP should be avoided. In the c. Oculomotor nerve
[Features of heart failure and persistent ST given case option C. i.e. mirena is most suitable]. d. Trigeminal nerve
elevation suggests the dx of left ventricular e. Glossopharyngeal nerve
aneurysm]. 675. A 10yo male child was brought by his mother
complaining that her child watches TV at very high Ans. The key is A. Facial nerve. [Viral infection is a
671. A 4m girl has severe FTT (Failure To Thrive) volumes, doesn’t like to play outside and instead common cause of facial nerve palsy].
and increasing jaundice which was 1st noticed at has become more sincere with reading. She
1wk of age. She has anenlarged liver and also says that her son doesn’t respond to her. 679. A 5yo girl has had an URTI for 3 days and has
scratches on her skin. Her parents have been What do you expect to see on otoscopy? been treated with paracetamol by her mother.
unable to seek medical care.What is the most a. Flamingo pink tympanic membrane For
likely dx? b. Attic perforation the last 12h she has been hot and irritable with
a. Biliary atresia c. Bluish grey tympanic membrane with air fluid severe pain in her right ear. What is the most
b. G6PD deficiency levels likely dx?
c. Hep B d. Inflamed tympanic membrane with cart wheel a. Herpes zoster infection
d. Spherocytosis appearance of vessels b. Impacted ear wax
e. Red and inflamed tympanic membrane c. Mumps
Ans. The key is A. Biliary atresia. [Increasing d. OM
jaundice at this age with failure to thrive, enlarged Ans. The key is C. Bluish grey tympanic membrane e. Perforation of eardrum
liver and scratches (itching) indicate cholestatic with air fluid levels. [In glue air there occur
jaundice likely from biliary atresia]. conductive deafness so the child watch tv with Ans. The key is D. OM.
high volume and does not respond to others for
672. A 76yo man suddenly collapsed and died. At this deafness]. 680. A 35yo man has a temp=39C, cough with
post mortem exam, a retroperitoneal hematoma purulent sputum and right sided chest pain on
due to ruptured aortic aneurysm was noted. What inspiration. He has herpes labialis. What is the
is the most likely underlying cause of the single most likely causative organism?
a. Coagulase +ve cocci in sputum likely dx? 687. A 5yo boy has cough and swelling at the knee
b. Gram -ve diplococci in sputum after falling on the ground with rashes on the
c. Gram +ve diplococci in sputum buttocks which are non-blanching. PT=13,
d. Pneumocystis carinii in sputum APTT=71, Hgb=11, WBC=8, Plt=200. Choose the
e. Serology for legionella most
likely dx?
Ans. The key is C. Gram +ve diplococcic in sputum. a. NAI
[High temperature, cough with purulent sputum, b. Hemophilia
pleuritic chest pain and herpes labialis are c. HSP
recognized feature of pneumococcal pneumonia d. Osler weber reindu syndrome
(pneumococcus=Gram +ve diplococci]. e. Von-Willebrand disease

681. A 27yo female was brought to the ED by her Ans. The key is B. Hemophilia. This is a
friend from a movie theatre. She complains of controversial key! Correct key should be E. Von-
sudden severe pain in the eye followed by Willebrand disease. [Likely D/D may be B or C! in
vomiting and also was seeing colored halos. She von willebrands disease usually there is no
gives hemarthrosis (except in type 3) and in hemophilia
a past hx of recurrent headaches which used to a. Anterior MI no nonblanching rash (there may bruising).
resolve spontaneously. Exam: fixed, dilated b. Inferior MI isolated rise in APTT is highly suggestive of
ovoid pupil seen. What is the first inv? c. Lateral MI hemophilia (given case). This is an ill defined
a. CT head d. Posterior MI question and only likely diagnosis may be type 3
b. MRI orbits e. NSTEMI von willebrands disease].
c. Blood culture and sensitivity
d. Toxicology screen Ans. The key is A. Anterior MI. [Here ST elevation 688. A 45yo woman presents with discharge from
e. Applanation tonometry in L1, aVL, v2,v3,v4 and v5. So the diagnosis is the left nipple. The discharge is brownish-green
anterior MI (as L1 and aVL and v2-v5, and foul smelling. What is the most likely dx?
Ans. The key is E. Applanation tonometry. anterolateral would be more appropriate a. Duct papilloma
[Darkness can cause dilatation of pupil (which description)]. b. Intra-ductal papilloma
occurred in the darkness of theatre room) which c. Duct ectasia
(halfway dilatation) most often precipitate acute 685. A schizophrenic says: life is unfair. I like fairs. d. Mammary duct fistula
attack of angle closure glaucoma and the test to Fairs have food. It must be good. What term e. Breast abscess
diagnose this is applanation tonometry]. describes this pt’s speech?
a. Neologism Ans. The key is C. Duct ectasia. [Women near
682. An 82yo male suddenly becomes b. Flight of ideas menopause are more affected by duct ectasia
unconscious and fell down. He recovered c. Broc1a’s aphasia characterized by brown, green or cheesy
completely within d. Wernicke’s aphasia discharge. This condition is harmless and tends to
minutes. What is the best inv you to to dx the e. Clang association clear up without treatment].
case?
a. ECG Ans. The key is E. Clang association. [The rhymic 689. A 10yo boy presents with generalized
b. EEG use of words as described is known as clanging or swelling. This has been present for 4days and
c. Blood glucose level clang association often seen in schizophrenia]. included
d. CT swollen ankles and puffiness of the face. It started
e. CXR 686. A man comes to the ED with hx of pulsatile a few days after he had a mild cold with
swelling in the abdomen, he has hx of HTN and runny nose. His only PMH was eczema. Urine
Ans. The key is A. ECG. [Here sudden exam: analysis: hematuria, proteinuria 10g/24h, creat
unconciuosness without any provocation, which pulse=120bpm, BP=70/40mmHg. He is restless 60umol/l and albumin=15g/l. What is the single
makes arrhythmia the most likely cause; hence we and in shock. What emergency management most likely dx?
should do ECG]. should be done on priority basis? a. IgA nephropathy
a. Urgent abdominal CT b. HSP
683. A child admitted with progressive muscle b. Urgent abdominal US c. Minimal change nephropathy
weakness and frequent falls. What is the most c. IV fluids 0.9% NS crystalloids to bring SBP to d. Wilson’s disease
probable dx? 90mmHg e. Cardiac failure
a. Duchenne’s MD d. IV fluids 0.9% NS crystalloids to bring SBP to
b. Becker’s MD 120mmHg Ans. The key is A. IgA nephropathy. [10 yr old boy,
c. Polymyositis e. Dopamine inj history of URTI and hematuria points towards the
d. Dermatomyositis diagnosis of IgA nephropathy. It may be present
e. Polymyalgia rheumatic Ans. The key is C. IV fluids 0.9% NS crystalloids to with proteinuria and generalized swelling. The
bring SBP to 90mmHg [Probable ruptured aortic important differentiating point from rapidly
Ans. The key is A. Duchenne’s MD. aneurism. Immediate IV normal saline to raise the progressive GN is duration. IgA nephropathy <1o
BP to 90 mmHg to keep the vital organs perfused days (usually 4/5 days history of infection but in
684. A 56yo man presents to the ED with chest till definitive measures are taken]. rapidly progressive GN history of infection for >10
pain. The following ECG was taken. What is the days].
most
690. A 28yo man complains of vertigo, nausea and She denied any urinary symptoms preoperatively.
vomiting for more than 30 mins and tinnitus, Exam: appears unwell, temp=37.5C,
hearing loss in the left ear. What is the tx for this BP=94/73mmHg, pulse=116bpm, sat=97%. Her
pt? abdomen is distended with tenderness in the left
a. Buccal prochlorperazine (2nd line) flank and suprapubic region. Bowel sounds are
b. Metachlorpromide not audible. Choose the most appropriate post
c. Cyclazine (1st line) Csection
d. Cotrimazole complication for this lady?
e. Ondansetron a. UTI
b. Urinary tract injury
Ans. The key is C. Cyclizine. More appropriate is A. c. Pleurisy
Buccal prochlorperazine (patient.info). [Both d. Acute pyelonephritis
prochlorperazine and cyclizine can be used in e. Paralytic ileus
Meniere’s disease].
Ans. The key is B. Urinary tract injury.
691. A 16yo girl has been unwell for 5days with
malaise, headache and dry cough. She has a few 695. A 58yo man has a headache and confusion of
crackles in her chest. Her CXR shows patchy 3 days after slipping and hitting his head in the
consolidation in the lower lobes. What is the garden. What is the most appropriate initial inv? a. Anterior MI
single a. XR skull b. Inferior MI
most likely causative organism? b. XR face c. Lateral MI
a. Cold agglutinins c. CT brain d. Posterior MI
b. Gram –ve diplococci in sputum d. MRI brain e. NSTEMI
c. Gram +ve diplococcic in sputum e. EEG
d. Serology for legionella Ans. The key is B. Inferior MI. [There is
e. Sputum staining for mycobacterium TB Ans. The key is C. CT brain. [The likely diagnosis is pathological Q wave and mild ST elevation in
subdural haematoma for which appropriate initial leads II, III and aVF].
Ans. The key is A. Cold agglutinins. [Mycoplasma investigation is CT scan of brain].
pneumonia, a form of atypical 698. A young man has diarrhea, loss of weight and
bacterial pneumonia related to cold agglutinin 696. A 4yo boy has a cough and arthritis followed flatulence for 3 days. What is the most imp tx?
disease]. by rash on legs which are non-blanching on glass a. Metronidazole
test. No hx of fever. PT=13, APTT=31, Hgb=12, b. Fluconazole
692. A child with increasing jaundice and pale WBC=6.5, Plt=300. What’s the most likely dx? c. Vancomycin
stools. Choose the appropriate test: a. Meningitis septicemia d. Amoxicillin
a. Endomyseal antibodies b. Hemophilia
b. Sweat test c. HSP Ans. The key is A. Metronidazole. [An incomplete
c. TFT d. ITP question!!]
d. LFT e. TTP
e. US 699. A 6yo child presented with drooling of saliva
Ans. The key is C. HSP. [Usually occurs below 10 and severe stridor. He is febrile and sick looking.
Ans. The key is E. US. [This is obstructive jaundice yrs of age. The characteristic rash and lab findings XR
where US may show dilatation of common bile matches with HSP]. Neck in extension shows a thumb sign. Choose
duct or stones]. the single most likely dx.
697. A 72yo man presents to the ED with chest a. Croup
693. A 73yo woman with RA is unable to extend pain. The following ECG was taken What is the b. URTI
the fingers of her right hand at the MCP joint and most c. Diphtheria
IP likely dx? d. Acute epiglottitis
joints following a fall. What is the single most
likely tendon to have been damaged? Ans. The key is D. Acute epiglottitis. [Drooling of
a. Extensor carpi radialis saliva and stridor along with thumb sign in neck X-
b. Extensor carpi ulnaris ray are features of acute epiglottitis].
c. Extensor digitorum
d. Extensor indicis 700. A mother presents with her 3yo son who has
e. Flexor digitorum profundum indistinct nasal speech. He snores at night and has
Ans. The key is C. Extensor digitorum. [The restless sleep. He is tired by day. What is the best
extensor digitorum extends the phalanges, then management strategy?
the wrist, and finally the elbow. It acts principally a. Arrange hearing test
on the proximal phalanges]. b. Assess development milestones
c. Refer to ENT surgeon
694. You are called to see a 20yo woman 2h post- d. Refer to speech therapist
LSCS. She has not passed urine since her e. MRI brain
operation.
Ans. The key is C. Refer to ENT surgeon. [Probable inhaler as req, but he now gets attacks with 708. A 46yo woman has weight gain, sensitivity to
enlarged adenoid]. exercise. What is the single most appropriate tx? cold, pulse=50bpm, heart is enlarged with
a. Regular salbutamol murmur.
701. A 17yo boy while playing football got a kick b. Regular salbutamol and budesonide What is the single most likely dx?
and now he is complaining of severe pain and c. Sodium cromoglycate a. Hypothyroidism
swelling of the left side of his scrotum. What inv is d. Oral steroid b. Hyperthyroidism
the most appropriate to dx? e. Inhaled steroid c. Cushing’s syndrome
a. Needle aspiration of scrotum d. Addison’s disease11
b. US scrotum Ans. The key is C. Sodium chromoglycate. This is e. Pheochromocytoma
c. MSU wrong key! Correct key should be E. Inhaled
d. Surgical exploration of scrotum steroid. [Chrommoglycate should be used in Ans. The key is A. Hypothyroidism. [The given
e. Urine test for hematuria exercise induced asthma if inhaled steroid fails]. symptoms are classic presentation of
hypothyroidism].
Ans. The key is D. Surgcal exploration of scrotum. 705. A 3yo boy has a sudden onset of fever,
[This is a case of testicular torsion which needs vomiting and bilateral face swelling. Few days 709. An alcoholic who has completely given up
urgent diagnostic and therapeutic surgical earlier the drinking hears voices. What is the most
exploration of the scrotum]. GP saw him for bilateral parotid pain and gave appropriate tx?
analgesics. What is the most appropriate next a. Olanzapine
702. A 50yo man has had hoarseness of voice and step? b. Diazepam
drooping eyelid for 2m. a mass is palpable in the a. Analgesic c. Acamprosate
right supraclavicular fossa. He smokes 20 b. Antibiotic d. Disulfiram
cigarettes/day for the last 30yrs. What is the most c. Biopsy e. Haloperidol
likely dx? d. Immediate surgery
a. Carcinoma larynx e. Reassurance Ans. The key is A. Olanzapine. This is wrong key.
b. Carcinoma thyroid Correct key is B. Diazepam. [A case of delirium
c. Carcinoma right bronchus Ans. The key is E. Reassurance. [A case of mumps. tremens. Treated with benzodiazepines like
d. Mesothelioma Self limiting condition]. chlordiazepoxide or diazepam].
e. Pancoast tumor
706. A 75yo man with adenocarcinoma of the 710. A 6yo boy has completed an induction course
Ans. The key is E. Pancoast tumour. [Hoarseness prostate which has spread outside the capsule of of chemo for ALL. He has an enlarged left
of voice is due to compression of the recurrent the scrotum.
laryngeal nerve, ptosis due to compression of the gland has ARF. What is the most appropriate next What is the most appropriate next step?
sympathetic ganglion, palpable mass in right inv? a. Herniotomy
supraclavicular fossa due to involvement of the a. MRI spine b. CT abdomen
supraclavicular lymph node. History of smoking b. Radionuclide bone scan c. Biopsy
and given picture indicates the diagnosis of c. Trans rectal US d. Immediate surgery
Pancoast tumour]. d. US pelvis e. Reassurance
e. US KUB
703. An 84yo man got surgical pain which is well Ans. The key is B. CT abdomen. This is wrong key.
controlled by oral morphine 60mg BD. However, Ans. The key is E. US KUB. [extension beyond Correct key is C. Biopsy. [Relapse may directly
now this pt is unable to swallow. What is the most capsule may cause obstruction of ureters, causing involve testis and excisional biopsy is done to
appropriate next step? loin pain, anuria, symptoms of acute kidney injury confirm recurrence of leukemia].
a. Morphine 60mg state or chronic kidney disease (here ARF)] ref: Ref: Link:
b. Morphine 60mg TDS patient.info http://ascopubs.org/doi/full/10.1200/jco.2009.23
c. Oxycodone 10mg OD .8014
d. Morphine 60mg IV 707. A 57yo male presents with sudden onset 711. A 32yo miner is rescued after being trapped
e. Fentanyl patches severe abdominal pain and rigidity against a 4d under a fallwn rock for 4h. After applying a
background of LIF pain and pyrexia. He has no bladder
Ans. The key is E. Fentanyl patches. [When dose PM/SHx of note and isn’t on any meds. What is catether, 15-20ml of reddish brown urine was
of oral morphine is known and rout should be the most likely dx? obtained. HR=120bpm, SBP=100mmHg. What
changed Phentanyl patch is adviced as the fixed a. Intussusception would be the next appropriate step?
dose is known and patch can release the required b. Ischemic colon a. Dopamine IV
dose for a given period (when we use patch we c. Sigmoid volvulus b. Fluid challenge
can not change the dose). But if the pain control is d. Perforated diverticulum c. Furosemide IV
not optimal we should follow the next step (i.e. e. Perforated Meckel’s diverticulum d. 20% Mannitol IV
parenteral morphine) as per pain ladder. But as e. Antibiotics
no correct dose of parenteral morphine is in the Ans. The key is D. Perforated diverticulum.
options (iv morphine dose is one-third of oral [Sudden onset, severe abdominal pain, rigidity, Ans. The key is B. Fluid challenge. [The diagnosis is
morphinre) we have to go for fentanyl patches]. left iliac fossa pain and fever are in favour of rhabdomyolysis. So IV fluid is the next appropriate
perforated diverticulum]. step].
704. A 19yo man has exercise induced asthma.
This has prv been controlled using a salbutamol
712. A 60yo man has had spontaneous painful 716. A 42yo woman with a PMH of severe 720. A 65yo known alcoholic is brought into the
swelling of his right knee for 3days. 5days prv he headache treated in the ED presents with signs hospital with confusion, aggressiveness and
had and ophthalmoplegia. He is treated with diazepoxide.
an inguinal hernia repaired as a day case. He takes symptoms of renal failure. She has been seen by What other drug would you like to prescribe?
bendroflumethiazide 2.5mg daily. He is her GP for HTN and abdominal pain with OP inv a. Antibiotics
apyrexial. What is the single most appropriate pending. Which inv is most likely to lead to a dx? b. Glucose
diagnostic inv? a. US KUB c. IV fluids
a. Blood culture b. CT brain d. Disulfiram
b. CRP c. IVU e. Vit B complex
c. D-dimer d. Renal artery Doppler
d. XR knee e. Renal biopsy Ans. The key is E. Vit B complex. [This is a case of
e. Serum uric acid Wernicke’s encephalopathy which is treated with
Ans. The key is A. US KUB. [Hypertension, Vit B complex].
Ans. The key is E. Serum uric acid. [Thiazide abdominal pain and features of renal failure
diuretics causes hyperuicemia which can indicates the diagnosis of ADPKD for which the 721. A pt suffering from schizophrenia laughs
precipitate acute attack of gout]. diagnosis is best made by US KUB]. while talking about his father’s death. Which term
best
713. A 27yo woman with anxiety and weight loss 717. In perforation of a post gastric ulcer, where describes his condition?
has tachycardia, tremor and mild proptosis. What will the fluid accumulate in the peritoneal cavity? a. Depression
single mechanism accounts for her weight loss? a. Left paracolic gutter b. Flat affect
a. Deficiency in thyroid hormone b. Pelvic cavity c. Emotional liability
b. Increased level of calcitonin c. Lesser sac d. Incongruent affect
c. Increased metabolic rate d. Under the diaphragm e. Clang association
d. Insulin resistance e. Right paracolic gutter
e. Reduced caloric intake Ans. The key is D. Incongruent affect.
Ans. The key is C. Lesser sac. [Incongruent affect means inappropriate
Ans. The key is C. Increased metabolic rate. [The emotional response like loughing hearing a sad
given features are of thyrotoxicosis in which 718. A 62yo male is brought to the ED by his news or crying hearing a good news].
increased metabolism causes loss of patients daughter because of his persistent lying. He is a
weight]. known 722. A 72yo man has been on warfarin for 2yrs
alcoholic and has been admitted recently with because of past TIAs and strokes. What is the
714. A man with carcinoma and multiple delirium tremens. On questioning, he denies any most
metastasis presents with intractable nausea and problem with memory. He knows his name and imp complication the pt should be careful with?
vomiting. address and states that was at the betting shop a. Headache
He has become drowsy and confused. What is the this morning, but his daughter interjects calling b. Osteoporosis
most appropriate management? him a liar explaining that he was at her home. c. Ear infection
a. Dexamethasone IM What is the most likely dx? d. Limb ischemia
b. Dexamethasone PO a. Ganser syndrome e. Diarrhea
c. Ondansetron IM b. Cotard syndrome
d. Ondansetron PO c. Wernicke’s encephalopathy Ans. The key is B. Osteoporosis. It is a wrong key.
e. Morphine oral d. Korsakoff psychosis The correct option is A. Headache. [Headache
e. Alcohol withdrawal from intracranial hemorrhage is more important
Ans. The key is C. Ondensatron IM. [For cancer or complication about which patient should be
chemotherapy induced vomiting ondensatron is Ans. The key is D. Korsakoff psychosis. [In careful with (it is more important than
the drug of choice. As here vomiting is intractable Korsakoff psychosis there is confabulation (a osteoporosis)].
IM ondensatron should be given not oral]. memory disturbance, defined as the production of
fabricated, distorted or misinterpreted memories 723. A 24yo woman is afraid to leave her house as
715. A 19yo man presents with weight loss, about oneself or the world, without the conscious whenever she goes out, she tends to have SOB
increasing thirst and increasing frequency of going intention to deceive) which may present like this]. and sweating. She has stopped going out except
to the with her husband. What is the most likely dx?
washroom. His father, grandfather and 2 sisters 719. A 70yo man presented with muscle weakness a. Social phobia
have been dx with DM. What is the most likely and inability to climb the stairs. Inv: CPK raised, b. Claustrophobia
type of DM this pt suffers from? ESR 15. What is the most likely dx? c. Depression
a. IDDM a. Polymyositis d. Panic disorder
b. NIDDM b. Polymyalgia rheumatic e. Agoraphobia
c. LADA c. Reactive arthritis
d. MODY d. RS Ans. The key is E. Agoraphobia. [Agoraphobia:
e. DKA e. Duchenne’s MD Extreme or irrational fear of open or public
Ans. The key is D. MODY. [Key features of MODY places].
are: Being diagnosed with diabetes under the age Ans. The key is A. Polymyositis. [Proximal muscle
of 25 and having a parent with diabetes, with weakness (inability to climb the stairs) and raised 724. A pt on HTN drugs develops hyperkalemia.
diabetes in two or more generations]. CPK points towards polymyositis]. Which anti-HTN is likely to cause it?
a. Ramipril
b. Lorsartan 729. A pt presents with hemoptysis 7d after Ans. The key is B. Psychotic depression. [Here
c. Thiazide tonsillectomy. What is the next step? features of atypical depression along with
d. Nifedipine a. Packing hallucination makes the likely diagnosis to be
e. Furosemide b. Oral antibiotics and discharge psychotic depression].
c. Admit and IV antibiotics
Ans. The key is A. Ramipril. [Both ramipril and d. Return to OT and explore 734. A 40yo teetotaler woman is recovering from
losartan can cause hyperkalemia]. e. Ice cream and cold fluids a hysterectomy 2days ago. At night she becomes
agitated and complains of seeing animals and
725. A young man develops itching worse at night Ans. The key is C. Admit and IV antibiotics. children walking around the ward. What is the
and following bathing. Exam: greysish white linear [Secondary hemorrhage can occur from sloughing most likely dx?
rash can be seen on the wrist and periumbilical of tissue from surgical wound following infection]. a. Delirium tremens
area. What is the dx? b. Toxic confusional state
a. Scabies 730. A 55yo man presents with HTN. He c. Hysteria
b. Polycythemia complains of headache and visual disturbances. d. Mania
c. Urticarial He also e. Drug induced personality disorder
d. Atopic eczema reports itching after a hot bath and burning
e. Lichen planus sensation in finger and toes. His face is flushed Ans. The key is B. Toxic confusional state. [This is
red. not delirium tremens as the womean is
Ans. The key is A. Scabies. PE: mild splenomegaly. Inv: Hgb=20g/dl, WBC=20, teetotaler].
plt=500, EPO normal. What is the likely dx?
726. A 40yo lady who has been a smoker since she a. Myelofibrosis 735. A woman with a hx of drug abuse and
was a teenager has the following blood result: b. Polycythemia rubra vera increased alcohol intake, now comes for help and
Hgb=19. What hormone should you check? c. Essential thrombocythemia she is
a. Aldosterone d. CML concerned about her problem. What is the most
b. Cortisol e. CLL appropriate management option?
c. Erythropoietin a. Voluntary admission
d. T4 Ans. The key is B. Polycythemia rubra vera. b. Psychiatry team
e. TSH [Raised hemglobin, raised cell counts and normal c. Mental health team
erythropoeitine along with symptoms of d. Psychiatry voluntary admission
Ans. The key is C. Erythropoetin. [Smoking causes hyperviscosity like headache and associated
raised carboxyhemoglobin level causing hypertension are diagnostic of polycythemia rubra Ans. The key is B. Psychiatry team. This is probably
hypoxemia and raised erythropoeitin which leads vera]. a wrong key. Correct option should be C. Mental
to raised hemoglobin level (secondary health team. [Drug abuse and alcohol intake has
polycythemia)]. 731. An old man having T2DM with increased skin psychological issue also and mental health team
tanning, heart failure and having high ferritin can give broader aspect of support than a
727. A 25yo man presents with hoarseness of (hemochromatosis) level is refusing tx. Where is psychiatry team. Mental Health Team: "These can
voice. He has swollen vocal cords. His BMI=32 and the first site of cancer? include psychiatrists, psychologists, community
he a. Testes psychiatric nurses, social workers, and
smokes 20-25 cigarettes/day. What would you b. Adrenal occupational therapists. They work with you to
advise him? c. Liver treat your mental health. You might get help from
a. Stop smoking d. Pancreas the team as a whole or from just one or two
b. Lose weight professionals. Your GP or primary care service has
Ans. The key is C. Liver. to refer you to the MHT]."
Ans. The key is A. Stop smoking.
732. A 60yo DM lady presents with severe peri- 736. A 28yo woman who is 32 wks pregnant in her
728. A 64yo male was admitted to the medical anal pain and swelling. What’s the cause? 3rd pregnancy is diagnosed as a case of placental
ward with complaint of diarrhea, abdominal pain a. Anal carcinoma abruption. After all the effective measures, she is
and b. Anal fissure still bleeding. What is the underlying
weight loss for few months. Exam: clubbing, c. Hemorrhoid pathology?
perianal skin tags and abdominal tenderness. d. Anal abscess a. Clotting factor problem
Colonscopy reveals transmural granulomatous b. Clauser’s syndrome
inflammation involving ileocecal junction. He was Ans. The key is D. Anal abscess. c. Platelet problem
dx with what? d. Succiturate lobe
a. CD 733. A woman is sad, fatigues and she is eating e. Villamentous insertion of placenta
b. UC more and also has sleeping disturbance and hears
c. Bowel cancer the Ans. The key is A. Clotting factor problem.
d. Gastric cancer voice of her husband who died 3yrs ago. What is
e. IBD the dx? 737. An old woman having decreased vision can’t
a. OCD see properly at night. She has changed her glasses
Ans. The key is A. CD. [The given picture is typical b. Psychotic depression quite a few times but with no effect. She has
of Crohn’s disease]. c. Grieving normal pupils and cornea. What is the most likely
d. Severe depression dx?
a. Cataract
b. Glaucoma Ans. The key is C. Lithium. [Side effect of lithium is c. He has also taken another substance apart from
c. Retinal detachment fine tremor (in therapeutic dose) but lithium methadone
d. Iritis toxicity or lithium poisoning causes coarse d. The methadone had already caused some brain
e. GCA tremor. Coarse tremor, nausea and vomiting are damage
well known feature of lithium overdose]. e. Naloxone is eliminated faster than methadone
Ans. The key is A. Cataract. [glaucoma may have
dilated oval pupil]. 742. A 23yo man feels anxious and agitated when Ans. The key is E. Naloxone is eliminated faster
faced with stress. He has an interview in 3days than methadone.
738. A 53yo man was admitted to the hospital for and
inv of hemoptysis. 3 days after admission he would like some help in relieving his symptoms. 746. A 24yo male on remand in prison for murder
developed alternating state of consciousness, What is the most appropriate management? is referred by the prison doctor. He is noted to be
ataxic gait and some visual problems. What is the a. SSRI behaving oddly whilst in prison and complains of
most appropriate management of this pt? b. CBT seeing things. He has a prv hx of IV drug abuse.
a. Acamprosate c. Propranolol On questioning he provides inappropriate but
b. Chlordiazepoxide d. Diazepam approximate answers to all questions stating that
c. Diazepam Bill Clinton is the prime minister of England. What
d. High potent vitamins Ans. The key is C. Propranolol. [Inappropriate is the prisoner suffering from?
e. Disulfiram anxiety during interview is performance phobia a. Capgras syndrome
which is better helped by propranolol when help b. Cotard syndrome
Ans. The key is D. High potent vitamins. [This is a needed for short term like here as 3 days]. c. Ganser syndrome
case of Wernicke’s encephalopathy (as hospital d. Ekbom syndrome
admission prevented him from consuming 743. An 8yo boy dx with asthma is on salbutamol e. Tourette’s syndrome
alcohol) and the treatment for this is high potent and beclomethasone. However, he wakes up at
vitamins]. night due to his symptoms. What is the next Ans. The key is C. Ganser syndrome. [Ganser
appropriate management? syndrome is characterized by nonsensical or
739. A pt underwent hip surgery. Later he a. LABA wrong answers to questions or doing things
presents with SOB and chest pain. What is the dx? b. High dose steroid incorrectlyoften with visual pseudohallucinations
a. Pulmonary embolism c. Aminophylline and a decreased state of consciousness. It is also
b. MI d. Oral prednisolone sometimes called prison psychosis, because the
c. Tension pneumothorax e. Sodium cromoglycate syndrome occurs most frequently in prison
d. Fat embolism inmates, where it may represent an attempt to
e. None Ans. The key is A. LABA. [Patient is in step2 with gain leniency from prison or court officials].
poor control. So next step is to add LABA and if
Ans. The key is A. Pulmonary embolism. still not controlled give high dose inhaled 747. A 32yo lady has recently become more
corticosteroids]. active, sleeps less and bought a house and 2 new
740. A 25yo man presents with hx of cars.
breathlessness. A transthoracic echo reveals a 744. A woman presents with a hx of poisoning 10x What is the most likely dx?
patent foramen with different substances. There are no obvious a. Bipolar disorder
ovale. What diagnostic inv would you do for a signs of depression or suicidal behavior. What is b. Mania
patent foramen ovale? the best preventive step? c. Hypomania
a. Transesophageal echo a. Open access to ED d. Schizophrenia
b. Bubble echo b. 24h help line
c. Transthoracic echo c. CBT Ans. The key is C. Hypomania. [Individuals in a
d. ECG d. Anti-depressants hypomanic state have a decreased need for sleep,
e. Insight into problem are extremely outgoing and competitive, have a
Ans. The key is B. Bubble echo. [Bubble echo is great deal of energy and are otherwise often fully
actually extension of transoesophageal echo in Ans. The key is E. Insight into problem. [Patient is functioning (unlike full mania where unlike fully
that here additional bubbles are added during not depressed and there is no suicidal behaviour. functioning there is a. obvious difficulties at work
transoesophageal echo to get better visualization Despite repeated poisoning may indicate she is or in social relationships and activities b. requires
of foramen ovale]. facing some stress and so insight into her problem admission to hospital to protect the person or
should be sought for]. others, or c. the person is suffering psychosis)].
741. A 25yo woman with a hx of several episodes
of depression is brought to the ED after she was 745. A boy was rushed to the ED unconscious 748. The body of a 65yo man who was treated for
found with several empty bottles of her meds. She after he had taken methadone belonging to the TB and bronchitis was seen at autopsy. His legs
complains of coarse tremor, nausea and sister. were swollen and his liver showed signs of a
vomiting. Which of the following drugs is likely to He was given naloxone and he regained transudate fluid. What was the cause of the
have caused her symptoms? consciousness. After a while he started getting transudate?
a. Fluoxetine drowsy a. Liver cirrhosis
b. Amitryptilline again. What is responsible for his present drop in b. Alcoholic liver disease
c. Lithium level of consciousness? c. Cardiac failure
d. Phenelzine a. Naloxone is absorbed faster than methadone d. Budd-chiari syndrome
e. Olanzapine b. Methadone is absorbed faster than naloxone e. TB
Ans. The key is C. Cardiac failure. 753. A 20yo male smoker is noted to have intense e. CT head
rubor of the feet and absent foot pulse. Exam: Ans. The key is B. 24h BP monitoring. [All these
749. A 60yo woman has tiredness. She has amputated right 2nd toe. What is the most drugs (particularly prolonged use of thiazide and
noticed that her skin looks permenantly tanned probable dx? amlodipine) are well established cause of postural
and she a. Intermittent claudication hypotension. So 24 hour BP monitoring should be
describes dizziness on standing up. What is the b. Cardiovascular syphilis done].
single most likely electrolyte pattern to be found? c. Buerger’s disease 757. A 43yo woman has been feeling lethargic and
a. Na+=120mmol/L, K+=5.9mmol/L d. Embolism tired. Her BP=160/90mmHg. Bloods:
b. Na+=125mmol/L, K+=2.9mmol/L e. Acute limb ischemia Na+=140mmol/L, K+=3.1mmol/L. What is the
c. Na+=140mmol/L, K+=4.5mmol/L most likely dx?
d. Na+=150mmol/L, K+=3.5mmol/L Ans. The key is C. Buerger’s disease. [ The a. Cushing’s syndrome
e. Na+=150mmol/L, K+=5.9mmol/L traditional diagnosis of Buerger's disease is based b. Conn’s syndrome
on 5 criteria (smoking history, onset before the c. Hyperparathyroidism
Ans. The key is A. Na+=120mmol/L, age of 50 years, infrapopliteal arterial occlusive d. Renal disease
K+=5.9mmol/L. [Tanned skin (hyperpigmentation) disease, either upper limb involvement or e. Pheochromocytoma
and postural hypotension in a tired women points phlebitis migrans, and absence of atherosclerotic Ans. The key is B. Conn’s syndrome. [Here
towards Addison’s disease where hyponatremia risk factors other than smoking). As there is no potassium is < 3.5, sodium is at its high normal
and hyperkalemia is seen]. specific diagnostic test and an absence of positive and hypertension is present!!! so no confusion
serologic markers, confident clinical diagnosis that it is conn's syndrome!! Cushing's syndrome is
750. A 20yo girl with amenorrhea and BMI of 14 should be made only when all these 5 criteria a D/D but it has typical clinical feature which is
still thinks she has to lose weight. What is the have been fulfilled]. usually described if the question is for cushing's
most 754. A young lady after a heavy bout of drinking syndrome. below is picture reference of conn's
likely dx? last night comes to the ED with dizziness, syndrome from patient.info.It can also be
a. Anorexia nervosa abdominal cushing’s syndrome! So if we like to be sure we
b. Bulimia nervosa pain, vomiting blood with cool peripheries. After have to know aldosterone and rennin level. i)
c. OCD initial resuscitation, oxygen and fluids, she still Renin low, aldosterone high- Conn’s syndrome
d. Depression continues to bleed with pulse=130bpm and (primary hyperaldosteronism) ii) Renin low,
e. Body dysmorphic disorder BP=85/58mmHg. What would be your next best aldosterone low- Cushing’s syndrome iii) Renin
management? high, aldosterone high- Renovascular disease
Ans. The key is A. Anorexia nervosa. a. Clotting screen (secondary hyperaldosteronism)].
b. US
***751. A guy who has several convictions and c. CT
has been imprisoned several times, breaks up d. Endoscopy
with his e. Omeprazole
family and doesn’t contact his children. What type Ans. The key is D. Endoscopy. [In young it is
of personality disorder is this? unlikely to get oesophageal varices rather than
a. Borderline the bleeding is from probable gastric erosion and
b. Antisocial endoscopy should be done (diagnostic and
c. Schizotypal therapeutic- like sclerotherapy].
d. Schizoid 755. A 12yo boy complains of acute development
e. Criminal of purpura on the dependent areas of his body
2wks after a URTI. The purpura doesn’t blanch on
Ans. The key is B. Antisocial. [Antisocial pressure, tests reveal: Hgb=12, plts=50,
personality disorder is characterized by a WBC=5. Bleeding time=10mins, APTT=40s, Reference: patient.info
pervasive pattern of disregard for, or violation of, PT=1.02. What is the most likely dx?
the rights of others. There may be an a. ITP 758. A 2yo child aspirated a foreign object which
impoverished moral sense or conscience and a b. TTP was removed at the hosp. the parents are now
history of crime, legal problems, and impulsive c. Von Willebrand’s disease asking how to remove it if that ever happens at
and aggressive behavior]. d. Hemophilia A home. What do you advise?
e. Hemophilia B a. Hemlich maneuver
752. A 17yo lady presents with a worm in her ear. Ans. The key is A. ITP. [Isolated thrombocytopenia b. Bring to the hospital
She is very agitated and anxious. What is the next and H/O prior URTI with development of purpura c. Turn the child on his back and give thumps
step? on the dependent areas of the body favours the d. CPR
a. Remove under GA diagnosis of ITP. (In ITP BT is prolonged which is e. Remove manually by fingers
b. Suction present here.)].
c. Alcohol drops 756. A woman presents with hx of falls, becomes Ans. The key is C. Turn the child on his back and
d. Forceps pale and clumsy. She is hypertensive and takes give thumps.
atenolol, bendroflumethiazide and amlodipine.
Ans. The key is C. Alcohol drops. [It is a living What inv is needed? 759. A 28yo pregnant lady presents with severe
object. So it should be killed first and so alcohol a. 24h ECG lower abdominal pain with excessive per vaginal
drop is the correct option]. b. 24h BP monitoring bleeding at 34wks gestation. What should be the
c. ECG initial inv of choice?
d. Echo a. Coagulation profile
b. US abdomen central vein. By contrast iron overload in 766. A 52yo man with hx of ant MI 3 wks ago
c. CT pelvis hemosiderosis causes to accumulation of iron developed sudden onset of dyspnea. Exam:
d. D-dimer granules predominantly in kupffer cells and more BP=100/60mmHg, pulse=100bpm, SaO2=88%,
e. Kleiuber test in central area rather than peripheral hepatocyte. chest=bilateral crackles. Which inv would you do
In given case there are large amount of iron to find the underlying cause?
Ans. The key is B. US abdomen. [Likely case of pigment in hepatocyte which is diagnostic of a. CXR
abruption placenta (as excessive bleeding hemochromatosis]. b. Echo
probably revealed type) for which initial c. D-dimer
investigation of choice is US abdomen. Other 763. A 27yo man presents to the ED after an RTA d. Radionucleotide lung scan
features that will favour abruptio is hard uterus where his foot was stuck under a truck for several e. Troponin
and evidence of fetal distress. It will be difficult to hours. He now has swelling on the foot. Exam:
feel the fetal parts]. foot tenderness, loss of sense in the space Ans. The key is A. CXR. It is wrong key. Correct key
between the 3rd metatarsal and big toe and his should be B. Echo. [The question does not want to
760. A 3yo child with severe diarrhea and dorsalis pedis is not felt. What is the most likely know the features of pulmonary oedema rather it
vomiting, looks lethargic, has sunken eyes and a dx? is clearly mentioned “which investigation would
feeble cry. a. Compartment syndrome you do to find out the UNDERLYING CAUSE”. As in
What is the choice of fluids? b. Arterial rupture this patient sudden papillary muscle rupture or
a. 0.9%NS c. Arterial embolus VSD may be the likely cause (ventricular aneurism
b. 0.9%NS + 5%Dextrose d. DVT may take a more slower course) so echo seems to
c. 0.45%NS + 5%Dextrose e. Fibular fx be more logical approach].
d. 0.45%NS
Ans. The key is A. Compartment syndrome. [Acute 767. A 49yo man lost his job and now is homeless.
Ans. The key is A. 0.9% NS. [Patient has features compartment syndrome occurs after a He was found wandering in the park. He is
of severe dehydration and should be resuscitated traumatic injury such as a car crash. The trauma muttering that some people are after him. Alcohol
first and for resuscitation fluid of choice is 0.9% causes a severe high pressure in was tested and it was negative. What will
NS]. the compartment which results in your next step be?
insufficient blood supply to muscles and nerves. a. Thiamine
***761. A lady with depression has a bag full of Acute compartment syndrome is a medical b. Neuropsycho analysis
meds. She now presents with coarse tremors. emergency that requires surgery to correct. If c. Mini mental state
Which untreated, the lack of blood supply leads to d. CT head
drug caused her symptoms? permanent muscle and nerve damage and can e. MRI head
a. Lithium result in the loss of function of the limb].
b. Thyroxine Ans. The key is B. Neuropsychoanalysis. [As the
c. Amitriptyline 764. You are a FY doctor in the ED when a mother patient has persecutory delusion he is likely
d. Sodium valproate brings her 2yo son to you with a 1h hx of noisy suffering from psychosis for which
e. Tetrabenazine breathing. She state that although he had mild neuropsychoanalysis is the logical approach].
coryza over the last week, he was improving and
Ans. The key is A. Lithium. It is a wrong key! Right so they had gone to a children’s picnic with 768. A pt with SNHL and loss of corneal reflex on
key is D. Sodium valproate. [Side effect of lithium nursery friends. Another parent had found him the left side. What is the most definitive inv?
in therapeutic range is fine tremor. Sodium coughing and spluttering, and ever since his a. CT of internal auditory meatus
valproate (a well known mood stabilizer) can breathing has remained noisy. Though he appears b. Nuclear imaging of brain
cause coarse tremor in therapeutic range]. well in the ED, his current observations c. MRI of internal auditory meatus
demonstrate a raised RR and sat=91% on air. d. Radio isotope scan
762. A 38yo man has had a liver biopsy as part of What is the e. XR skull
inv for abnormal LFTs. The pathologist report most likely dx?
states: a. Anaplyaxis Ans. The key is C. MRI of internal auditory meatus.
special stains demonstrate the presence of a very b. Croup [The likely cause is acaustic neuroma for which
large amount of iron pigment within the c. Foreign body aspiration appropriate investigation is MRI internal acoustic
hepatocytes. What single condition is identified by d. Epiglottitis meatus].
the pathologists report?
a. Alpha 1 antitrypsin deficiency Ans. The key is C. Foreign body aspiration. 769. A middle aged man complains of a node
b. Hemangioma which has been growing on his nose for several
c. Hemochromatosis 765. A pt taking doxycycline complains of nausea, months.
d. Hemosiderosis indigestion, abdominal pain and vomiting. What Now it’s firm with central depression. It is 0.6cm
e. Wilson’s disease will you advise? in size. What is the single most likely dx?
a. Take it after meals a. Basal cell carcinoma
Ans. The key is D. Hemosiderosis. It is a wrong b. Take it before meals b. Squamous cell carcinoma
key! The correct key should be C. c. Stop the drug c. LN
Hemochromatosis. [In hemochromatosis d. Take antiacids d. Melanoma
distribution pattern of iron in liver biopsy by e. Take antiemetic e. Kaposi’s sarcoma
pearl’s stain is more prominent iron granules in f. Keratoacanthoma
periportal hepatocyte and relative sparing of Ans. The key is A. Take it after meal. g. Molluscum contagiosum
kupffer cells and decreased intensity near the
Ans. The key is A. Basal cell carcinoma. [A pearly Ans. The key is C. Repeat XR. [Likely diagnosis is a. 5 compression: 1 breath
white umbilicated ulcer (with central depression) scaphoid fracture where X-ray may not show the b. 5 compression: 2 breaths
any where in the face with rolled edges / inverted fracture right away. In such case put her hand in a c. 15 compression: 2 breaths with nose pinched
edge is basal cell carcinoma. Any ulcer which is below elbow cast and repeat X-ray in a week or d. 15 compression: 2 breaths without nose
located above the neck is always basal cell two to see if the fracture become visible. Usually pinched
carcinoma until proven otherwise. (Samson it becomes visible as a healing fracture line]. e. 30 breaths: 2 compressions
note)].
774. A 33yo female presents with sudden severe Ans. The key is C. 15 compression:2 breaths with
770. A 45yo woman presents with rotational colicky abdominal pain in her RIF. A mobile mass nose pinched.
vertigo, nausea and vomiting, especially on is
moving her felt on examination. What is the most likely dx? 779. A 56yo man complains of increased vol of
head. She also had a similar episode 2yrs back. a. PID sputum with specks of blood and chest pain. He
These episodes typically follow an event of runny b. Appendicitis has a
nose, cold, cough and fever. What is the most c. Ovarian torsion hx of DVT. Exam: clubbing. What is the cause of
probable dx? d. Constipation blood in his sputum?
a. Acoustic neuroma a. Pulmonary thrombosis
b. Meniere’s disease Ans. The key is C. Ovarian torsion. b. Bronchial carcinoma
c. Labyrinthitis c. Bronchiectasis
d. BPPV 775. A middle aged male is feeling unwell after a d. Pulmonary TB
e. Vestibular neuronitis recent MI. The recent ECG shows prolonged QRS
complex and tented T wave. Na+=136mmol/l, Ans. The key is C. Bronhiectasis. [Increased
Ans. The key is E. Vestibular neuronitis. [Here no K+=6.2mmol/l, urea=5mmol/l. What is the most volume of blood streaked sputum and clubbing
sensoryneural hearing loss (SNHL present in appropriate management? points towards bronchiectasis].
acoustic neuroma, Meniere’s disease and a. Calcium gluconate
labyrinthitis). Runny nose, cold, cough and fever b. Calcium resonium 780. A 32yo female has a hx of SOB and fever.
are recognized trigger of vestibular neuronitis but c. Calcium with vit D supplement Pre-broncho-dilation test was done and it was
not BPPV]. d. Vit D 2/3.5
e. Calcium and post-bronchodilator was 3/3.7. The pt was dx
***771. A 65yo man with cancer of middle 1/3 of of eczema and TB. What is the possible dx?
the esophagus presents with dysphagia. What is Ans. A. Calcium gluconate. [ECG change of a. COPD
the prolonged QRS complex and tented T wave are b. Asthma
most immediate management? suggestive of hyperkalaemia which is further c. Pneumonia
a. Chemotherapy supported by K+ level of 6.2 mmol/l. So to protect d. Bronchiectasis
b. Radiotherapy the heart from arrhythmia calcium gluconate
c. Stenting should be given]. Ans. The key is B. Asthma. This is probably wrong
d. Gastrostomy key. Correct key is D. Bronchiectasis. [History of
e. TPN 776. A 68yo man with DM and HTN was noted to fever and prior TB are more suggestive of
have cholesterol level of 3.4mmol/l. he was also bronchiectasis and in bronchiectasis spirometry is
Ans. The key is C. Stenting. noted to have microalbuminuria. What is the best variable (either restrictive, obstructive or
drug to add to his regimen? normal)].
772. A 1yo child is brought to ED. He woke up in a. ACEi
the middle of the night crying severely. What b. Statin 781. A 2yo male pt was brought by his mother
initial c. Amlodipine with a swelling in the right side of his neck
measure should be taken for this child? d. Biguanides extending
a. Refer to surgeon from the angle of the mouth to the middle 1/3 of
b. Discharge with advice Ans. The key is A. ACEI. [ACEI is renoprotective the sternocleidomastoid muscle. The swelling
c. Analgesia and prevents progress in albuminuria]. was on the anterolateral side of the
d. Antibiotic sternocleidomastoid and was brilliantly
777. A child playing with toys suddenly develops transilluminant.
Ans. The key is C. Analgesia. breathlessness and stridor. Which inv will lead to What is the likely dx?
the dx? a. Lymphangioma
773. A 30yo lady was playing volleyball when her a. Laryngoscopy b. Branchial cyst
hand got injured with the ball. The right hand is b. CXR c. Thyroglossal cyst
not c. Peak flow meter d. Ranula
swollen and there is tenderness under the root of d. ABG e. Thyroid swelling
the thumb. XR is normal. What is the most
appropriate next management? Ans. The key is A. Laryngoscopy. [Breathlessness Ans. The key is B. Branchial cyst. It is a wrong key.
a. Arm sling for 1 wk and stridor in a child playing with toy is most likely Correct key is A. Lymphangioma. [Both
b. Raise hand for 2d due to aspiration of foreign body (toy or part of lymphangioma and brancheal cyst are lateral neck
c. Repeat XR toy) for which laryngoscopy should be done]. mass. 90% of lymphangioma occur in children less
d. Full arm cast than 2 yrs. Brancheal cyst usually does not
778. Anxious parents ask you for resus technique transilluminate whereas lymphangioma usually
for their 3yo. What do you tell them? transilluminate brilliantly].
Ans. The key is D. Her age. Probably wrong key. Ans. The key is A. Pulmonary embolism.
782. A 50yo newly dx with HTN complains of Correct key should be B. Tumour grading. [What [Immobilisation for hip pain is the clincher of
urinary freq and dysuria. The urinalysis reveals ever the age oestrogen receptor positive patient pulmonary embolism here].
presence will take tamoxifen as adjuvant therapy. But
of white cells and protein. Choose the single most tumour grading in some instance may influence 788. A 77yo woman suffered diarrhea 4d ago.
appropriate tx? the need of adjuvant therapy. Though the given What would be her blood gas result?
a. Imipramine key is D it is probably a wrong key and correct key a. pH =7.2, PaCO2 =8
b. Adjust diuretics should be B. Tumour grading]. b. pH =7.4, PaCO2 =1.5
c. Vaginal estrogen c. pH =7.4, PaCO2 =2.6
d. Trimethoprim d. pH =7.4, PaCO2 =2.8

Ans. The key is D. Trimethoprim. [Symptoms Ans. The key is B. pH = 7.4, PaCO2 = 1.5
(urinary frequency and dysuria) along with white
cells and protein in urine suggest UTI. Treatment 789. A man presents with abdominal pain,
is with triethoprim]. vomiting, pulse=120bpm, BP=90/60mmHg and a
rigid
783. A boy injured his ear during a rugby match. abdomen. His chest is clear. What is the
He reported it being painful. Exam: red and immediate management?
tender a. Call radiology
pinna. Tympanic membrane was normal. What b. Admit to medical ward
would be the next appropriate step? c. Urgent admission to ITU (Intensive Therapy
a. Topical gentamicin Unit)
b. Oral flucloxacillin
c. IV flucloxacillin Ans. The key is C. Urgent admission to ITU.
d. Refer to ENT specialist
e. No further intervention needed. 790. A 60yo man presents with weight loss and
Ans. The key is E. No further intervention needed. Hgb=6. Hx reveals that he has abdominal pain and
[As tympanic membrane is normal so this diarrhea for the past 6m. What is the most
transient inflammation of pinna from injury appropriate inv?
786. A 45yo man presents with hearing loss and
during rugby match is self limiting which needs no a. Barium enema
tinnitus in the right ear. Exam: weber test
further intervention]. b. Colonoscopy
lateralizes
c. Sigmoidoscopy
to the left. Audiometry: AC > BC in both ears.
784. A 2yo girl prv well presents with a hx of
What is the next best inv?
vomiting and diarrhea for 4hrs. What is the most Ans. The key is B. Colonoscopy. [Likely diagnosis is
a. CT
suitable indication for IV fluid administration? IBD. So Colonoscopy should be done].
b. MRI brain
a. Capillary refill time >4s
c. Angiogram
b. HR >90bpm ***791. A 35yo primigravida post C-section
d. Otoscopy
c. Increased RR complains of inability to void. She denies dysuria
d. Stool >10x/d but
Ans. The key is B. MRI brain. [AC>BC indicate
e. Weight of child = 10kgs complains of fullness. She was treated with an
Rinne positive; i.e. The deafness is not conductive.
epidural for analgesia. What is the single most
Again hearing loss is on right side. Weber
Ans. The key is A. Capillary refill time >4s. appropriate inv?
lateralized to left. We know weber lateralized to
[Capillary refill time >4s is sign of severe a. MSU
same side if conductive deafness and to opposite
dehydration indicating need for IV fluid b. US abdomen
if there is sensoryneural deafness. So it is now
administration. Features of severe dehydration: i) c. US KUB
obvious that the deafness of right ear is
abnormally sleepy or lethargic ii) sunken eyes iii) d. Serum calcium
sensoryneural deafness for which MRI brain is the
drinking poorly or not at all iv) reduced skin turgr
next best investigation].
> 2 seconds v) dry tongue and mucous membrane Ans. The key is B. US abdomen. [C. US KUB. Is also
vi) absent tears vii) oliguria viii) hypotension ix) correct! This is a case of POUR (post operative
787. A 74yo lady called an ambulance for an acute
tachycardia x) prolonged capillary refill time xi) urinary retention). We should do US to know the
chest pain. She has a hx of DM and HTN, and is a
depressed anterior frontanelle]. urinary volume (urine volume in bladder). If
heavy smoker. Paramedics mentioned that she
<400ml we shall observe. When volume is >600ml
was overweight and recently immobile because
*785. A 44yo woman with breast cancer had an we should do catheterization].
of a hip pain. She collapsed and died in the
extensive removal and LN clearance. She needs an
ambulance. What is the most likely cause of
adjunctive tx. Her mother had cancer when she 792. A female pt with HTN and serum
death?
was 65. Which of the following factors will be K+=2.7mmol/l. which hormone would you be
a. Pulmonary embolism
against the tx? looking for?
b. MI
a. Fam hx a. Aldosterone
c. Stroke
b. Tumor grading b. Cortisol
d. Cardiac arrhythmia
c. LN involvement c. Thyrotoxin
e. Cardiac failure
d. Her age d. Renin
e. Testosterone
Ans. The key is A. Aldosterone. [Hpt and examination and investigation points to
hypokalemia is seen in hyperaldosteronism. So we Ans. The key is 100% oxygen. Abdominal Migraine. Management is
should look for aldosterone]. reassurance].
797. A 44yo lady who has PCKD is concerned
793. A 25yo male with a hx of frequent binge because her 38yo brother has just died of an 801. A 78yo gentleman suddenly collapsed. His
drinking presents 4h after having had a take away intracranial insult. She knows he was not HR=120bpm, BP=70/40mmHg. Exam: pulsatile
meal hypertensive. What was the most likely cause of mass
following a nights heavy drinking. He complains of her in abdomen. What is the most appropriate dx?
nausea and has vomited on several occasions. brother’s death? a. Aortic aneurysm
After the last vomiting episode, he vomited a. Subdural hematoma b. Mesenteric cyst
approximately a cupful of blood. On admission, he b. Subarachnoid hemorrhage c. Umbilical hernia
smells of alcohol, pulse=100bpm, c. Cerebral infarct Ans. The key is A. Aortic aneurism. [There is
BP=140/80mmHg. He has some tenderness in the d. Epidural hematoma features of shock with pulse 120bpm and bp
epigastrium. 70/40mmHg and sudden collapse of the patient
What is your dx? Ans. The key is B. Subarachnoid hemorrhage. with pulsatile mass in abdomen makes the likely
a. Gastric carcinoma [Cerebral aneurisms are recognized association of diagnosis of ruptured aortic aneurism].
b. Mallory-weiss tear PCKD which may lead to subarachnoid 802. A woman prv in good health presents with
c. Esophageal carcinoma hemorrhage]. sudden onset of severe occipital headache and
d. Esophageal varices vomiting. Her only physical sign on exam is a stiff
e. Esophageal varices 798. A 36yo male dx with glioblastoma since last neck. Choose the most likely dx.
f. Peptic ulceration 5m has cerebral edema and is on dexamethasone. a. Subarachnoid hemorrhage
He has diarrhea and vomiting for the last 3days. b. Subdural hematoma
Ans. The key is Mallory-weiss tear. [Repeated He has been suffering from repeated falls today. c. Cerebellar hemorrhage
retching and vomiting is a recognized cause of What could be the possible cause for his falls? d. Migraine
Mallory-weiss tear which is supported here by a. Adrenal insufficiency e. Cerebral embolus
vomiting a cupful of blood]. b. Dehydration Ans. The key is A. Subarachnoid hemorrhage.
c. Dexamethasone therapy [Sudden onset severe occipital headache and
794. A young boy presented with bilateral d. Raised ICP vomiting with stiffness makes the likely diagnosis
periorbital edema, ankle swelling and increase in to be subarachnoid hemorrhage].
body Ans. The key is A. Adrenal insufficiency. 803. A 34yo housemaid presents with headaches
weight. What is the most likely dx? [Prolonged dexamethason use suppresses adrenal in the back of her head for several days and pain
a. Chronic heart failure axis and intercurrent illness can lead to adrenal on
b. Nephrotic syndrome insufficiency (addisonian crisis) in such cases]. flexing her neck. What is the most likely cause?
c. Renal failure a. Subdural hemorrhage
d. Acute heart failure 799. A 2yo child is brought by his mother. The b. Cervical spondylosis
e. Glomerulonephritis mother had hearing impairment in her early c. Subarachnoid hemorrhage
childhood d. Meningitis
Ans. The key is B. Nephrotic syndrome. and is now concerned about the child. What inv e. Cluster headache
would you do? Ans. The key is B. Cervical spondylosis. [Headache
795. A 53yo man with prv hx of COPD presents a. Audiometry in the back of head and pain on flexing neck is
with breathlessness and purulent sputum. O2 b. Distraction testing early feature of cervical spondylosis which
stat=85% c. Scratch test gradually progress to later symptoms like
on air. ABG: PaO2=7.6, PaCOS=7. What is the d. Tuning fork radiculopathies due to root compression in arms
appropriate management for his condition? and hands].
a. 24% oxygen Ans. The key is A. Audiometry. [Conditioned 804. A 40yo man complains of thirst and lethargy.
b. Mechanical ventilation response audiometry 2-4 years, pure tone audio His BP=140/90mmHg, corrected Ca2+=3.7mmol/l.
c. 100% oxygen gram for greater than 5 years, less than 6 months What is the most appropriate management at this
d. Nebulized salbutamol otoacoustic emission or brainstem evoked stage?
response, distraction test for 6-18 months, OHCS a. IV fluids
Ans. The key is A. 24% oxygen. [Patient has ENT]. b. Prednisolone
hypoxemia and hypercapnea (type2 respiratory c. IV hydrocortisone
failure). Hence he should be put on 24% oxygen. 800. An 8yo child presents with recurrent d. Calcium prep
100% oxygen will abolish hypoxic drive and abdominal pain, occasional headaches but Ans. The key is A. IV fluids. [Hypercalcemia is
worsen hypercapnea]. maintains a treated by IV fluids].
good appetite. Exam: normal. CBC, BUE, etc are 805. A 75yo man on digoxin develops weakness in
796. A 34yo man was involved in a RTA and whilst normal. What would you do for her next? the right upper and lower limbs which resolves
in the ambulance his GCS deteriorated and RR a. US abdomen within a few hours. What is the most definitive inv
increased from 30-48. What is the most b. CT head for this condition?
appropriate management for this pt? c. Reassure a. Carotid Doppler
a. IV fluid d. Analgesics b. Angiography
b. Needle thoracocentesis c. CT head
c. 100% oxygen Ans. The key is C. Reassure. [Recurrent abdominal d. Digoxin level
d. Portable XR pain and headaches with no abnormal finding on
Ans. The key is B. Angiography. Probably wrong gain and easy fatigue. Her serum prolactin levels
key! Correct key is C. CT head. are 1100um/l. what is the likely cause of
[Patient on digoxin means he is being yreated for hyperprolactinemia?
atrial fibrillation with heart failure and the cause a. Hypothyroidism
of tia is cardiac in origine! So carotid dopplar or b. Stress
carotid angiography is not the answer. Again for c. Pregnancy
tia investigation of choice is MRI (if not available d. Prolactin secreting pituitary tumor
CT) NICE guideline. So for this question definitove e. PCOS
investigation is CT scan.]. Ans. The key is D. Wrong key. Correct key should
806. A 35yo man presents with balance problems, be A. Hypothyroidism. [ It is likely to be 814. A 68yo man has had increasing dysphagia for
headache, SNHL and loss of coreneal reflex on the hypothyroidism because prolactin level in solid food for 3m and has lost 5kgs in weight.
left side. What is the most definitive inv? hypothyroidism may be 30-200ng/ml which What
a. CT scan of internal auditory meatus means 630-4200 mIU/L. Though weight gain can single inv is most likely to lead to a def dx?
b. Nuclear imagine of the brain occur but fatigue is not a feature of prolactinoma a. Barium swallow
c. MRI of internal auditory meatus unless it is a macroadenoma causing suppression b. CXR
d. MRI brain of other pituitary hormone! given prolactin level c. CT chest
Ans. The key is D. MRI brain. Wrong key! “Most of 1100 IU/L is definitely from a microadenoma (in d. Endoscopy and biopsy
definitive” is MRI internal auditory meatus. macroadenoma prolactin level is > 10,000 MU/L) e. Video-fluoroscopy
[Probable diagnosis is left sided acaustic neuroma. which does not cause fatigue. So the case most Ans. The key is D. Endoscopy and biopsy. [Likely
For this most definitive investigation is MRI likely seems to be hypothyroidism]. cause is malignant stricture for which endoscopic
internal auditory meatus]. 811. A 42yo female had sudden severe headache biopsy is the definitive investigation].
807. A 52yo man has a painful, red, photophobic and vomiting. She took paracetamol and then 815. A 24yo male was trying to move his
right eye with slightly blurred vision and watering collapsed. What is the most likely dx? wardrobe but it fell on his thigh. It was there for a
for 3days. He has had no such episodes in the a. SAH very long
past. On slit lamp examination there are cells and b. Viral encephalitis time before someone was able to remove it.
flare in the ant chamber and pupil is sluggish to c. Meningitis When he was seen in ED he had casts in his urine
react. What is the single most appropriate d. Anaphylaxis but no RBCs. Other inv showed hypocalcemia and
clinical dx? Ans. The key is A. SAH. [Sudden headache, high serum creatinine. What is the cause for
a. Acute close-angle glaucoma vomiting, collapse are suggestive of SAH]. his renal failure?
b. Acute conjunctivitis a. Acetylcholine
c. Acute dacrocystitis 812. Parents of a 3m baby are worried about cot b. Myoglobin
d. Acute iritis death. What advice would you give? c. Myotroponin
e. Corneal foreign body a. Lay on the back with feet towards head end d. Acetyl acetate
Ans. The key is D. acute iritis. [In iritis there is b. Lay on the back with feet towards feet end Ans. The key is B. Myoglobin. [The likely diagnosis
flares and cells in anterior chamber. In iritis pupil c. Lay on side is Rhabdomyolysis where renal failure occurs due
may be irregular but in angle closure glaucoma d. Lay on stomach to myoglobin (myoglobins are released after
pupil is slightly dilated and oval in shape. In iritis Ans. The key is B. Lay on the back with feet breakdown of skeletal muscles)].
there is no halo around light but in glaucoma towards feet end. 816. Anatomical structure to be pierced during
there is halo around light]. 813. A child born at 36wks developed difficulty in surgery midline port during gallstone removal.
808. A 55yo lady with swelling on the abdomen breathing with intercoastal recession and nasal a. External iliac muscle
below the umbilicus on the right side. What is the flaring. His temp is normal but his mother had b. Cricoid cartilage
single most appropriate lymph node? PROM 48h ago. What is the most likely inv that c. Linea alba
a. External iliac LN will lead to tx? d. Rectus sheath muscle
b. Pre-aortic LN a. Blood culture e. Duramater
c. Aortic LN b. CXR f. 1st tracheal cartilage
d. Inguinal LN c. Stool culture g. Conjoined tendon
e. Iliac LN d. Sputum culture h. Intercostal muscles
f. Submental LN Ans. B. CXR.This is wrong key! Correct key is A. Ans. The key is C. Linea alba.
g. Submandibular LN Blood culture. (please see the attachment below 817. A 48yo man has continuous ant chest pain
h. Deep cervical LN where the last line mentions investigations is for which is worse on inspiration and has temp of
Ans. The key is D. Inguinal LN. sepsis). [Probable diagnosis is Neonatal 37.5C
809. A 66yo male presents with painful Pneumonia as there is respiratory signs and since 4wks after a MI. His ESR=45mm/h. What is
swallowing. What is the most likely dx? history of PROM. In extremes of age pneumonia the single most likely explanation for the
a. Nesseria meningitides or sepsis may be with normal temperature or abnormal inv?
b. Cryptococcus neoformans even there may be hypothermia. Here pneumonia a. Acute pericarditis
c. Candida albicans is a part of sepsis and investigation is not CXR but b. Cardiac tamponade
d. Isospora belli blood culture!]. c. Atrial thrombus
e. Mycobacterium avium d. Left ventricular aneurysm
Ans. The key is C. Candida albicans. [Oesophageal e. Dressler syndrome
candidiasis]. The key is E. Dressler syndrome. [Dressler
810. A 45yo lady complains of expressable syndrome usually occurring two to five weeks
galactorrhea, decreased libido and amenorrhea, after the initial event but it can be delayed for as
weight long as three months.  It is characterised by
pleuritic chest pain, low-grade fever and 822. A 15yo boy presents with recurrent b. Continue antibiotics and add IV antifungals
pericarditis]. breathlessness and wheeze especially after c. Stop antibiotics
818.  An 8yo child swallowed 12 tablets of exercise. What is d. Continue only present antibiotics
paracetamol 4h ago. Serum paracetamol levels the most diagnostic inv? Ans. The key is B. Continue antibiotics and add IV
when a. CXR antifungals. [If judicial antibiotic fail to control
tested were at critical level. What would you do b. Lung function test fever next step is to add antifungal agents].
next? c. PEFR 826. A young girl who is known to have T1DM
a. Activated charcoal d. CT scan presented with drowsiness and deep breathing.
b. IV N-acetylcysteine Ans. The key is B. Lung function test. [Assessment Her
c. Gastric lavage of severity of airflow obstruction can diagnose sugar level=20. Her BP=120/80mmHg and her
d. Observation only asthma and COPD]. mucous membranes are dry. What would be the
Ans. The key is B. IV N-acetylcystine. [As 4 hour 823. A 34yo man was walking along the country next appropriate step?
paracetol level is at critical level start IV N- side when an insect bit him. After which he a. Serum urea
acetyltine at once]. started b. Blood culture
Manaement of paracetamol poisoning: to complain of an annular rash spreading c. CT
GI decontamination is recommended in those upwards. d. HbA1c
presenting <4h after OD. Activated charcoal 1g/kg a. Penicillin PO e. ABG
(max 50g) is the treatment of choice, reducing b. Doxycycline PO Ans. The key is E ABG. [Likely diagnosis is DKA
serum levels more than gastric lavage and limiting c. Flucloxacillin PO where ABG is done to demonstrate acidosis].
liver injury. d. Gentamicin PO 827. Removal of a glioma, which single most
• Glucose, U&E, LFT, INR, ABG, FBC, HCO3 e. Ciprofloxacin PO appropriate anatomical structure will be pierced?
–; blood paracetamol level at 4h post-ingestion. f. Antihistamine PO a. Cricoid cartilage
• If <10–12h since overdose, not vomiting, and g. Antihistamine IV b. Rectus sheath muscle
plasma paracetamol is above the line h. Corticosteroid IV c. Duramater
on the graph, start N-acetylcysteine. i. Corticosteroid IM d. Conjoined tendon
• If >8–24h and suspicion of large overdose j. Adrenaline IM e. Intercostal muscles
(>7.5g) err on the side of caution and k. Adrenaline IV Ans. The key is C. Duramater.
start N-acetylcysteine, stopping it if level below l. Atropine IV 828. A child is not breathing and intubation failed.
treatment line and INR/ALT normal. OHCM, 9th m. Reassurance At what anatomical site should the incision be
edition, page 856]. Ans. The key is B. Doxycycline PO. [Described rash made?
819. A pt dx with DVT is taking warfarin. What is is erythema migrans which is characteristic skin a. External iliac muscle
his cut off INR limit? lesion seen in lyme disease. Lyme disease is b. Cricoid cartilage
a. <1 treated with Doxycycline]. c. Linea alba
b. 1-2 824. A 60yo man was brought in by his wife d. Rectus sheath muscle
c. 2-3 complaining of ataxia, urinary incontinence and e. Duramater
d. 3-4 erectile f. 1st tracheal cartilage
Ans. The key is C. 2-3. [This range is appropriate dysfunction. He also complains of rigidity and g. Conjoined tendon
for the prophylaxis or treatment of venous slowing of movement with a pill rolling tremor of h. Intercostal muscles
thromboembolism and reduction of the risk of the hands. What is the likely dx? Ans. The key is B. Cricoid cartilage. [Incision for
systemic embolism for people with atrial a. Parkinson’s disease tracheostomy is done which goes down from
fibrillation and valvular heart disease]. b. Idiopathic parkinson’s disease cricoids cartilage].
820. Inserting a drain in the mid-axillary line. c. Shy-drager syndrome 829. A 10yo child who presented with fx of the
What is the single most appropriate anatomical d. Huntington’s disease radius which was treated with a plaster cast,
structure? Ans. The key is C. Shy-drager syndrome. [Shy- complains of pain. Exam: limb is swollen but warm
a. External iliac muscle drager syndrome is characterized by parkinsonism and well perfused, pulses are present. What
b. Linea alba (rigidity, slowing of movement with hand tremors) should you do next?
c. Rectus sheath muscle plus a more pronounced failure of the autonomic a. Send for repeat XR
d. Conjoined tendon nervous system (urinary incontinence and erectile b. Remove cast
e. Intercostal muscles dysfunction) and there may also cerebellar sign c. Replace cast with more padding
Ans. The key is E. Intercostal muscles. like ataxia be present]. [Please note the d. Give analgesic
821. A 29yo man presents with hx of cough, discrepancy of this question (may be a bad recall)
weight loss and night sweats. Exam: pansystolic that classic pill rolling movement is not a feature Ans. Key is D. Give analgesic.
murmur. of multiple system atrophy (shy-drager
What is the most appropriate dx of underlying dsyndrome)]. 830. A 32yo man has been to Thailand and
cause? 825. A 67yo man being managed for a malignancy returned with cervical lymphadenopathy and
a. Malaria develops neutropenic fever. He has been fever. What
b. HSP commenced on Ticarcillin, Tazobactam and is he most likely suffering from?
c. HIV Gentamicin. He has also recently commenced on a. HIV
d. Dengue fever Meropenem but on the 3rd day his temp still b. EBV
Ans. The key is C. HIV. [Though not mentioned IV remains >39C. 2 blood tests and urine cultures c. Typhoid
drug abusers may have tricuspid regurgitation show no organism. Inv: Hgb=104g/dl, WBC=<0.5, d. Measles
causing pansystolic murmur and HIV also very Plt=15. What will you do next?
common in this group]. a. Continue IV antibiotics and add oral antifungals
Ans. The key is B. EBV. [Cervical lymphadenopathy e. Sclera a. CT scan
mentioned in question which occurs in EBV Ans. The key is A. Iris. [Young man with early b. RBS
infection. In ARS (acute retroviral syndrome) or morning back pain and stiffness is suggestive of c. MRI
primary HIV infection there is generalized seronegative arthritis likely ankylosing spondylitis d. ECG
lymphadenopathy]. where iritis is a common association]. e. ABG
836. A 70yo woman with longstanding anxiety is Ans. The key is B. RBS. [In unconsciousness in
831. A 6yo child presents with edema and mild seen in the OPD. She complains of her heart diabetics, first hypoglycemia should be excluded
proteinuria. No hematuria. What is the most likely skipping by doing RBS].
dx? a beat quite often. This particularly occurs when 841. A 36yo lady comes with hx of early morning
a. PSGN she is trying to get to sleep. The palpitations stiffness of her small joints and with red and
b. Membranous GN are never sustained. What is the most likely painfuleye. What is the single most appropriate
c. Minimal change GN rhythm disturbance? option?
d. RPGN a. SVT a. Iris
Ans. The key is C. Minimal change GN. [Minimal b. VF b. Ciliary body
change GN is usually seen in young children. It c. VT c. Cornea
presents as edema and proteinuria]. d. V-ectopics d. Conjunctivitis
832. An 80yo woman suffering from RA presents e. A-fib e. Sclera
with severe epigastric pain and vomiting. She also Ans. The key is D. V-ectopics. [From the given f. Lichen planus
complains of shoulder tip pain. What is the single options the most likely answer is V-ectopics. If it Ans. The key is E. Sclera. [Ocular manifestation of
most discriminatory inv? occurs in a normal heart though symptomatic it is rheumatoid arthritis is scleritis (red, painfull eye)].
a. US Abdomen benign in nature but if it is secondary to heart 842. A 23yo man comes with 2d hx of sticky
b. Sigmoidoscopy disease like MI it may precipitate to more life greenish discharge from the eyes with redness.
c. Colonscopy threatening arrhythmia like ventricular What is
d. Barium meal fibrillation]. the single most appropriate option?
e. Upper GI endoscopy 837. A 17yo has acute pain around his right eye, a. Iris
f. Erect CXR pain on one side of his face and ear ache too. b. Ciliary body
Ans. The key is F. Erect CXR. [Elderly patient with What c. Cornea
RA is usually on NSAIDs which may lead to is the single most dx? d. Conjunctivitis
perforated peptic ulcer which presents as severe a. Ear wax e. Sclera
epigastric pain, vomiting and shoulder tip pain. b. Ear foreign body Ans. The key is D. conjunctivitis. [Bacterial
Diagnosis is made by seeing free gas shadow c. Dental abscess conjunctivitis can cause sticky greenish discharge].
under the diaphragm on erect CXR]. d. Cellulitis 843. A pt was admitted with erectile dysfunction,
833. A 44yo man went on holiday to Sudan 5wks e. Herpes zoster reduced facial hair and galactorrhea. What is the
ago. He now presents with red urine and fever. Ans. The key is E. Herpes zoster. [It seems to be most probable dx?
Exam: hepatomegaly. What is the most likely dx? herpes zoster of the trigeminal nerve. Onset is a. Hyperprolactinemia
a. Malaria acute, unilateral distribution favours herpes b. Cushing’s syndrome
b. Brucellosis zoster]. c. Pheochromocytoma
c. Leptospirosis 838. A 12yo boy presented with itching in his d. Hyperthyroidism
d. Schistosomiasis hands. Exam: skin is dry and red. His mother is e. Hypoparathyroidism
Ans. The key is D. Schistosomiasis. [Holyday in asthmatic and older brother has hay fever. What Ans. The key is A. Hyperprolactinemia.
Sudan, hematuria, fever and hepatomegaly goes is the single most likely causative factor? 844. A 32yo man has been repeatedly admitted to
most with Schistosomiasis amongst the given a. Dermatitis herpitiformis hospital for what was described as anxiety or
option]. b. Scabies panic attacks and palpitations. On occasions he is
834. A 32yo homosexual comes with hx of weight c. Eczema found to be tremulous and hypertensive. A
loss. Fundoscopy reveals retinal hemorrhages. d. Uremia persistent weight loss is noted. What is the most
What is the single most appropriate option? e. Drug induced probable dx?
a. Mycobacterium avium Ans. The key is C. Eczema. [Itchy lesion and family a. Hyperthyroidism
b. CMV history of asthma and hay fever in 1st degree b. Panic attacks
c. Hemophilus influenze relatives favours the diagnosis of eczema]. c. Pheochromocytoma
d. NHL 839. A 45yo man presented with pruritic purple d. Cushing’s disease
e. Pneumocystic jerovici papules on the flexor surface of his wrist and e. GAD
Ans. The key is B. CMV. [Weight loss in a some Ans. The key is C. Pheochromocytoma.
homosexual is likely to be due to AIDS and CMV white lacy markings on his buccal mucosa. What is 845. A 35yo man with T1DM is dehydrated with
retinopathy with retinal hemorrhage is a the single most likely causative factor? BP of 90/50mmHg. What is the single most
recognized association]. a. ALL appropriate initial inv?
835. A 30yo man comes with hx of early morning b. Lymphoma a. ABG
back pain and stiffness. Exam: red eyes. What is c. Polycythemia b. CBC
the d. IDA c. HbA1c
single most appropriate option? e. Lichen planus d. LFT
a. Iris Ans. The key is E. Lichen planus. [White lacy e. Serum Urea
b. Ciliary body pattern of lesion is characteristic of lichen planus]. Ans. The key is A. ABG. [Dehydration with low BP
c. Cornea 840. A known DM was admitted with sudden LOC. points towards probable DKA. So from the given
d. Conjunctivitis What is the initial inv? options ABG is the best response].
846. In OGTT what is the glucose venous plasma Ans. The key is B. Clean catch of urine. [The a. Blood for C&S
level 2h after glucose intake which indicates clinical features described are consistent with b. ESR
impaired glucose tolerance? urinary tract infection for which clean catch of c. CXR
a. >11.1mmol/l urine is the next best investigation. d. Urine for C&S
b. Between 7.8-11.0mmol/l 851. A 2yo girl presents with a 4d hx of fever that e. CSF analysis
c. Between 8.0-10.9mmol/l started with a cough. Her RR=45bpm, sat=94%, Ans. The key is D. Urine for culture and sensitivity.
d. Between 10.0-11.0mmol/l temp=38.9C, capillary refill time=1s. There are [Dipstick test if show leucocytes or nitrites is
e. Between 7.1-11.0mmol/l crepitations at the left base on auscultation. suggestive of UTI. So to confirm it we should do
Ans. The key is B. Between 7.8-11.0mmol/l. Urine shows negative dipstick. What is the single urine C&S].
847. A young man who has no PMH presented inv most likely to lead to dx? 854. A 3yo boy presents with a 1d hx of being
with jaundice, low Hgb, retics 8% and other a. Blood for C&S unwell. He appears shocked and has 3h old rash
indices b. ESR made
WNL but occasional spherocytes were seen on c. CXR up of urticarial and purpural spots. His RR=30bpm,
blood film. What is the single most appropriate d. Urine for C&S sat=94%, temp=39C, capillary refill time=1s.
inv? e. CSF analysis Urine is clean on dipstick. What is the single inv
a. G6PD enzyme assay Ans. The key is C. CXR. [The features are most likely to lead to dx?
b. Direct coombs test consistent with RTI (probable pneumonia) for a. Blood for C&S
c. Repeat blood film which CXR is the investigation of choice]. b. ESR
d. Indirect coombs test 852. A 3yo girl presents with fever for 2d. She is c. CXR
e. BMA drowsy and had a seizure causing twitching of the d. Urine for C&S
Ans. The key is B. Direct Coombs test. [The direct right side of the body for 4mins. Her RR=30bpm, e. CSF analysis
Coombs test, is used to determine whether the sat=90%, temp=38.9C, capillary refill time=2s. Ans. The key is A. Blood for C&S. [Likely diagnosis
cause of hemolytic anemia, is due Urine negative on dipstick. What is the single inv is septicemia for which Blood culture is the
to antibodies attached to RBCs which are seen in most likely to lead to dx? investigation of choice].
autoimmune-related hemolytic anemia]. a. Blood for C&S 855. A child is dx with VUR. What would you tell
848. A 22yo man came to the hosp after an injury b. ESR his parents?
in his hand while playing basketball. Exam: c. CXR a. Requires antibiotic prophylaxis
avulsion d. Urine for C&S b. Most will require surgery
of extensor tendon from the distal phalanx. What e. CSF analysis c. Most will have kidney scarring by 5yo
is the single most probable deformity? Ans. The key is E. CSF analysis. [It is really very d. Nothing can be done
a. Dinner fork deformity difficult to differentiate between encephalitis and e. Reassure
b. Game keeper thumb meningitis. Encephalitis is mostly viral and in UK Ans. The key is B. Most will require surgery. This is
c. Mallet finger herpes simplex virus is the main cause. Meningitis wrong key! Correct key is A. Require antibiotic
d. Gun-stock deformity also has a viral predominance though less than prophylaxis. [Practically it is established that
e. Garden spade deformi encephalitis.  majority of VUR will cure with time with
Ans. The key is C. Mallet finger. [A finger that Because encephalitis involves infection of the prophylactic antibiotics and need no surgery! Only
bends down at the end joint and cannot be brain itself, symptoms of altered brain function-- a minority needs surgery].
straightened is called a mallet finger. It is caused like confusion or decreased alertness--are usually 856. A 2yo child presents with severe vomiting.
by an injury to the extensor tendon that present, while in cases of meningitis the patient is Exam: mass felt in abdomen. What inv is most
straightens (extends) the finger. A splint worn day initially alert and, though understandably appropriate?
and night for 6-8 weeks will cure the problem in distracted by pain and misery, still in command of a. US
most cases]. their mental processes. b. XR
849. A 28yo man is inv for recurrent lower back As CSF can not differentia between meningitis and c. CT
pain. A dx of AS is suspected. Which of the encephalitis we have to take help of imaging like d. CBC
following MRI. In herpes simplex encephalitis there is Ans. The key is A. US. [The question is incomplete
inv is most useful? characteristic hyperintensity in fronto-temporal and is not suitable for further discussion with such
a. ESR region. little info].
b. XR sacro-iliac joints In the given case there are features of raised 857. A 13yo girl complains of a 2d hx of
c. HLA B27 intracranial pressure like drowsiness and seizer hoarseness of voice a/w dry cough. She feels
d. XR thoracic spine and so we cannot proceed for LP unless guided by feverish. On
e. CT lumbar spine CT (or MRI). On the other hand there is no other direct laryngoscopy, her vocal cords are grossly
Ans. The key is B. XR sacro-iliac joints. [x-ray option that can be diagnostic of the given edematous. What is the single most appropriate
sacroiliac joint shows fusion of both SI joints and condition. So we have to happy with CSF analysis inv?
thin, symmetrical syndesmophytes bridging the as the key though at this moment we have to a. None req
intervertebral disc spaces]. withheld this procedure]. b. Sputum for AFB
850. A 4yo girl is taken by her mother to the ED 853. A 6m boy is admitted with persistent c. Laryngoscopy
and complains of feeling unwell, urinary urgency irritability. He is lethargic and is not feeding as d. Bronchoscopy
andtemp=39C. What is the single next best inv? well as e. XR cervical spine
a. Catheter catch of urine usual. His RR=30bpm, sat=97%, temp=38.0C, Ans. The key is A. None required. [Dx is laryngitis].
b. Clean catch of urine capillary refill time=2s. Urine reveals leucocytes 858.  A 7yo girl is brought by her mother with
c. US on bright red staining of her underpants. She also
d. IVU dipstick. What is the single inv most likely to lead gives a hx that her daughter recently started
e. Suprapubic catch of urine to dx?
taking horse riding lessons. What is the single nose 2wks ago. What is the single most b. Release of common flexor sheath
most appropriate next action? appropriate next action? c. Release of palmar sheath
a.   Local exam a. Check child protection register d. Ulnar nerve release
b. Exam under GA b. Coag profile e. Fasciotomy
c.   Continue regular child care c. Skeletal survey Ans. The key is A. Release of flexor retinaculum.
d.   Inform child protection services d. Continue regular child care [Now case of carpal tunnel syndrome (median
e.    Coag profile e. Inform police nerve compression)].
867. A pt on insulin is booked in for a hernia
Ans. The key is B. Exam under GA. Ans. The key is B. Coagulation profile. [Likely operation. What is the most appropriate
diagnosis is HSP in which coagulation profile will management of
859. A 7d baby whose birth weight was 3.5kg and show significant increase in D-dimer insulin?
now is 3kg. What is the most appropriate next concentration. An activation of coagulation a. Give insulin and saline pre-op
step? including hyperfibrinolysis secondary to the b. Stop insulin for the duration of the op
a. Check child protection register endothelial damage is a typical feature of the c. Give IV insulin + dextrose + saline pre-op
b. Nutritional assessment common types of HSP]. d. Give insulin as usual pre-op
c. Skeletal survey e. None
d. Continue regular child care 863. A 4yo is brought to the ED by ambulance. His Ans. The key is C. Give IV insulin + dextrose +
e. Inform police mother reports that he has been unwell with a saline pre-op.
sore throat for 8h. He is sitting on his mother’s 868. A 35yo male who recently had an
Ans. The key is D. Continue regular child care. [In knee and is tolerating an oxygen mask but looks appendicectomy has got severe pain in his right
1st week the baby loose some weight then start unwell. He has constant noisy breathing and he is big toe. Joint
gain weight again. It is normal. So the option is drooling saliva. His temp=39C. What is the is red and swollen. He consumes 30 units of
continue regular child care]. most imp dx? alcohol/week. What is the most probable dx?
a. Acute asthma a. Rhabdomyosarcoma
860. A 6yo child fell on his nose 2d ago. His b. Bronchiolitis b. Osteoarthritis
parents have now brought him with difficulty in c. Croup c. Gout
breathing. d. Epiglottitis d. Pseudogout
Exam: fever, nasal bones are straight. What is the e. Tonsillitis e. Arthritis
single most likely dx? Ans. The key is C. Gout. [Alcoholic drinks can raise
a. Nasal polyp Ans. The key is D. Epiglottitis. the level of uric acid in the blood].
b. Septal hematoma 869. A 25yo male who recently noticed change in
c. Septal abscess 864. A pt with terminal cancer is being treated his shoe size, he is also constipated, has a
d. Deviated nasal septum with chemo develops tingling and numbness of preference to hot weather, his skin is dry, has
e. Fx nose the severe pain in wrist joint. Joint is red and swollen.
fingertips of both arms. What is the single most What is the most probable dx?
Ans. The key is C. Septal abscess. This is wrong likely cause of the symptoms? a. Chondro-sarcoma
key! Correct key is C. septal hematoma. [Septal a. Bone mets to cervical vertebrae b. Lipo-sarcoma
abscess takes a bit more time and a considerable b. Post-chemo neuropathy c. Gout
portion of caseof septal hematoma is associated c. Hyponatremia d. Pseudogout
with fever]. d. Hypocalcemia e. Ankylosing spondylitis
Ans. The key is D. Pseudogout. [Hypothyroidism
861. A 12yo pt came to the OPD with complains of Ans. The key is B. Post-chemo neuropathy. (change of shoe size due to myxedema (or pedal
fever, malaise, weight loss, anorexia and oedema can be seen in hypothyroidism) , other
productive cough. Exam: temp=39C, 865. An 80yo man has a permanent catheter. features of constipation, cold intolerance, dry skin
pulse=100bpm. His mother says that he has a hx Catheter specimen urine found lots of e-coli. are well known features of hypothyroidism.) has
of What is well recognized association with pseudogout].
recurrent chest infections and he is not thriving the single most appropriate management as he 870. A 45yo woman had her visual acuity checked
well. What is the single most likely causative wants to attend his daughter’s wedding next at her local optician. 12h later she presents to the
organism? week? ED with severe pain and redness in her eye. What
a. Pneumococcal pneumonia a. Change the catheter is the single most appropriate option?
b. Staphylococcus b. Prolonged antibiotics a. Iris
c. Mycobacterium TB c. Bladder wash b. Ciliary body
d. Pseudomonas d. Repeat MSU after wedding c. Ant chamber
e. PCP e. Reassure d. Post chamber
Ans. The key is A. Change the catheter. e. Cornea
Ans. The key is D. Pseudomonas. [Here the likely 866. A 35yo male typist who suffered a scaphoid
diagnosis is cystic fibrosis in which RTI due to fx was treated with a scaphoid cast. After 2wks Ans. The key is C. [In acute angle closure
pseudomonas is a very common association]. when the cast was removed for a review XR, it glaucoma half-dilated pupil is the most likely
was found that he had problems in moving the position which precipitates an acute attack as the
862. A 3yo child brought by his mother. Exam: thumb, index and middle fingers. What would you trabecular meshworks are mostly closed by
bruises on the buttocks. Mother also gives hx of suggest as the management for the recent peripheral anterior synechia of peripheral iris in
runny prb? this position. And mild illuminated darkened room
a. Release of flexor retinaculum like of an optician (also use of mydriatics
accelerates this) or opera (cinema hall) are culprit 876. A 60yo is on tx for IHD, HTN and c. Good pasture’s syndrome
to make this! As the block occurs in anterior hyperlipidemia. During the night he complains of d. MND
chamber it is the likely option here]. wheeze and e. Progressive massive fibrosis
871. A 75yo man who has DM and HTN SOB. Which of the following meds is responsible Ans. The key is B. Lung cancer. [Smoking history,
experiences acute monocular blindness which for that? chest sign and Horner’s syndrome points towards
resolves after a. Amlodipine lung cancer (probable pancoast tumour)].
1h. What is the most likely dx? b. Atenolol 882. A 34yo man had a cold 2d back. He now
a. GCA c. Ramipril presents with right sided facial pain. What is the
b. Optic neuritis d. Simvastatin single
c. Lacunar infarct e. Bendroflumethiazide most likely dx?
d. Pontine hemorrhage Ans. The key is B. Atenolol. [Asthma can be a. Maxillary sinus
e. Amaurosis fugax precipitated by beta blockers]. b. Ethmoid sinus
Ans. The key is E. Amourosis fugax. 877. A 15yo boy who complains of pain in his leg c. Septal hematoma
872. A 26yo presents with prolonged constipation, which has settled with aspirin. What is the most d. Septal abscess
blood on side of stool and very painful defecation. probable dx? e. Allergic rhinitis
PR exam: very painful. What is the single most a. Leomyosarcoma Ans. The key is A. Maxillary sinus. [Probably
likely dx? b. Liposarcoma maxillary sinusitis was intended to say].
a. Ca Colon c. Painful hip 883. A 29yo man with hx of asthma comes with
b. UC d. Exostosis post nasal discharge and bilateral painless nasal
c. CD e. Osteod osteoma blockage. What is the single most likely dx?
d. Anal fissure Ans. The key is E. Osteoid osteoma. [Younger age a. Nasal polyp
e. Constipation of onset and relieved with aspirin favours the b. Septal hematoma
Ans. The key is D. Anal fissure. diagnosis of osteoid osteoma]. c. Septal abscess
873. A 35yo man with painless left testicular 878. A 20yo fit man suddenly developed severe d. Atopic rhinitis
enlargement for the past 6m which is increasing in lower back pain as getting up from bed. What is e. Allergic rhinitis
size the Ans. The key is A. Nasal polyp.
and 3x larger than the right side. There is no single most probable dx? 884. A 24yo man has been found unconscious in
tenderness or redness. What is the most likely dx? a. Paget’s disease an alleyway with a RR=6bpm and HR=60bpm. His
a. Testicular tumor b. Multiple myeloma pupils are constricted. What is the best tx?
b. Hydrocele c. PID a. Methadone
c. Epididymal cyst d. AS b. Naloxone
d. Epididymo-orchitis e. Spondylosis c. Naltrexone
e. Reassure Ans. The key is C. PID [Prolapsed Intervertebral d. Thiamine
Ans. The key is A. Testicular tumour.Doubtful Disc]. [Sudden onset of lower back pain and felt e. Glucose
key!! May be Hydrocele. more during forward bending or similar Ans. The key is B. Naloxone. [Respiratory
874. A middle aged man who has had a hx of movement like getting up from bed favours the depression and miosis points towards opiates
chronic sinusitis, nasal obstruction and blood diagnosis of PID]. poisoning for which antidote naloxone is given].
stained 879. A 60yo man brought to the ED with fx hip, he 885. A 23yo female presents with back pain and
nasal discharge. He now presents with cheek is deaf and has bilateral pedal edema. What is the early morning stiffness, also complaining of eye
swelling, epiphora, ptosis, diplopia, maxillary pain. single most probable dx? problem and her sister has a similar condition.
What is the single most likely dx? a. Paget’s disease What is the single most probable dx?
a. Nasopharyngeal ca b. Osteoporotic fx vertebra a. Paget’s disease
b. Pharyngeal ca c. Secondary b. PID
c. Sinus squamous cell ca d. Multiple myeloma c. Myofacial pain
d. Squamous cell laryngeal ca e. Spondylosis d. AS
e. Hypopharyngeal tumor Ans. The key is A. Paget’s disease. [H/O deafness e. Spondylosis
Ans. The key is C. Sinus squamous cell ca. and evidence of heartfailure like pedal edema Ans. The key is D. AS [Back pain with early
875. A 60yo man with a long hx of smoking and favours the diagnosis of Paget’s disease]. morning stiffness and uveitis points towards the
alcohol presents with nasal obstruction, epistaxis, 880. An 80yo lady presents wih pain on left 6th rib diagnosis of ankylosing spondylitis].
diplopia, otalgia and conductive deafness. What is for a week. It is non-tender on examination. What 886. A 63yo female with a hx of osteoporosis
the single most likely dx? is your most likely dx? suddenly falls on her outstretched hand while
a. Nasopharyngeal ca a. Herpes zoster shopping.
b. Pharyngeal ca b. Costochondritis XR shows fx at distal radius with backward shift of
c. Sinus squamous cell ca c. Bone degeneration the distal fragment. What is the single most
d. Squamous cell laryngeal ca d. Thoracic vertebra compression probable deformity?
e. Hypopharyngeal tumor Ans. The key is C. Bone degeneration. a. Dinner fork deformity
Ans. The key is A. Nasopharyngeal ca. [Nasal 881. A 68yo DM, HTN with a 45pack/year smoking b. Coxavara
obstruction, epistaxis are features with cranial hx, has left sided chest pain increased with c. Mallet finger
nerve palsy diplopia due to 6th nerve, (also 5th breathing. Exam: myosis on left side and wasting d. Cubitus valgus
and 12th cranial nerves are frequently affected. of small muscles of left hand. What is the e. Garden spade deformity
otalgia and conductive deafness from local single most appropriate dx? Ans. The key is A. Dinner fork deformity.
extension as mentioned can occur]. a. Costochondritis 887. A 60yo man presents with severe colicky pain
b. Lung cancer from his right flank radiating to his groin. His
urinalysis reveals trace blood cells. What is the e. Diverticular disease diuretic resistance is not commonly encountered,
single most discrimatory inv? as long as renal function is preserved. However, in
a. US abdomen Ans. The key is D. Cecal carcinoma. [Mass in moderate and severe CHF patients, diuretic
b. XR KUB assending colon and anaemia makes cecal resistance occurs more frequently and often
c. Colonoscopy carcinoma the likely diagnosis from the given becomes a clinical problem. In diuretic resistance
d. Upper GI endoscopy options]. another diuretic is added to get optimal or
e. Laproscopy desirable result. As patient is still in heart failure
Ans. The key is A. US abdomen. [Features are of 892. A 55yo male after gastrectomy developed and pulmonary edema with the usage of loop
ureteric colic. X-ray KUB may miss radiolucent anemia. His MCV=106fl. Exam: loss of diuretic...a thiazide diuretic could be added with
stones so US abdomen is the discriminatory inv]. proprioception loop when response is inadequate. This will
888. A 45yo man has been admitted for an and vibration sense. What is the most likely dx? reduce the volume overload and improve
elective hernia surgery. 3d later he presents with a. IDA breathing].
agitation, b. Folate def
sweating, aggressiveness, and complains of seeing c. Vit B12 def 895. A 31yo man underwent an operation where
snakes on the hosp wall. Chlordiazepoxide d. Anemia of chronic disease his hand was hanging outside the table. After the
has been started for this pt. What is the most Ans. The key is C. Vit B12 def. [gastrectomy –> operation he had wrist drop and sensory loss over
appropriate next step? deficiency of intrinsic factor –> Vitamin B12 def. the dorsum of his hand. Which nerve was
a. Diazepam leading to macrocytic anemia and resulting injured?
b. Acamprosate subacute combined degeneration of cord causing a. Radial
c. Disulfiram loss of proprioception and loss of vibration sense]. b. Ulnar
d. Thiamine c. Median
Ans. The key is D. Thiamine. [Dx is delirium 893. A 26yo male has been operated for d. Axillary
tremens. So following chlordiazepoxide thiamine abdominal trauma and splenectomy was done. On e. Brachial
is the appropriate next step]. the 3rd
889. A woman with a prv hx of pain at the left post-op day the pt developed acute abdominal Ans. The key is A. Radial. [As the man’s hand was
wrist following a fall 4m ago for which she didn’t pain and distention in the upper abdominal area hanging outside the ot table there was pressure
seek with hypotension. On insertion of ryles tubes, 2L on radial nerve at arm level which is similar lesion
any tx now presented with pain in the same wrist of coffee ground fluid was aspirated. What is of Saturday night palsy].
below the thumb and the pain is aggravated the most probable dx?
whenever she holds her baby. What is the cause? a. Acute gastric dilatation 896. What is the mode of spread of chicken pox?
a. Fx radial head b. Reactionary hemorrhage a. Airborne
b. Scaphoid fx c. Subphrenic abscess b. Close contact
c. Carpal tunnel syndrome d. DVT c. Fecal-oral
d. Colles fx e. Left lower lobe atelectasis d. Blood
e. Ulnar fx e. Vector
Ans. The key is A. Acute gastric dilatation. [Acute
Ans. The key is B. Scaphoid frature. gastric dilatation may be caused by: Ans. The key is A. Airborne.
Hyperglycaemia, childbirth, abdominal injury,
890. A 29yo man was involved in an RTA. He application of a spinal cast, rarely, after 897. A 64yo man presents with ipsilateral vertigo,
presents with distended neck veins, clear breath abdominal surgery when the stomach is in ileus tinnitus and left side hearing loss. Exam: Renne
sounds and drink has been taken too soon. The abdomen test +ve and Weber’s lateralizes to the right ear.
and a trachea which is in the midline. His is distended and tender. The patient complains of What is the most appropriate inv?
RR=34bpm, BP=60/0mmHg. What is the most epigastric fullness, nausea, heaviness and a. CT
likely dx? heartburn. There is a succussion splash. The b. MRI brain
a. Simple pneumothorax accumulation of fluid in the stomach may induce c. XR
b. Tension pneumothorax hypovolaemic shock. Reflux of gastric contents d. Audiometry
c. Cardiac tamponade may cause an aspiration pneumonia]. e. None
d. Pericarditis f. Caloric testing
894. A 50yo man presented with increased Ans. The key is B. MRI brain. [Rinne positive, so it
Ans. The key is C. Cardiac tamponade. [Distended breathlessness at rest. He is currently on is not conductive deafness. Again we know if
neck vein, clear breath sound and no tracheal furosemide, weber lateralize to deaf ear it is conductive and
shift and BP of 60/0 points towards cardiac digoxin and isosorbide mononitrate. What drug is lateralized away from deaf ear then it is
tamponade]. going to help him? sensoryneural. So here there is sensoryneural
a. Ramipril hearing loss in left ear. So the most appropriate
891. An elderly woman is found anemia. As part b. Bendroflumethiazide investigation is MRI brain. Why not CT? As
of her exam, she had a barium enema which c. Atenolol auditory involvement including acoustic neuroma
reveals d. Amlodipine is better visualized by MRI].
a mass lesion in the ascending colon. What is the e. Diltiazem
single most appropriate dx? 898. A 67yo man presents to the ED with pain in
a. Sigmoid volvulus Ans. The key is B. Bendroflumethiazide. [This is a his left groin. He suddenly collapses and his is not
b. Anal fissure case of diuretic resistance. Heart failure able to move or lift his leg. He is on alendronate.
c. Sigmoid carcinoma represents the most common clinical situation in What is the dx?
d. Cecal carcinoma which diuretic resistance is observed. In mild CHF, a. Fx of neck of femur
b. Post hip dislocation Spirometry has been done. FEV1/FVC=2.3/3.6. b. Compression of the tibial nerve
c. Fx of shaft of femur After c. Compression of the S1 nerve root
d. Pelvic base fx taking salbutamol, the ratio=2.4/3.7. What is the d. Rupture of Achilles tendom
e. Peripheral vascular disease most likely dx? e. Tx of the medial collateral lig of the ankle
Ans. The key is A. Fx neck of femur. [Use of a. Chronic bronchitis Ans. The key is A. Compression of common
alendronate indicates osteoporosis where b. Asthma peronial nerve. [Foot drop, weakness of extensors
fracture neck of femur is more common]. c. Bronchiectasis of the ankle and toes and diminished pin prick
899. A young male met with an RTA and is d. Lung fibrosis sensation over the dorsum is sugestive of
suspected to have a femur fx. His BP is e. Sarcoidosis compression of common peroneal nerve].
90/60mmHg. What Ans. The key is A. Chronic bronchitis. [Progressive 907. A young man was knocked down during a
is the next immediate action? dyspnoea, wheeze productive cough and the fight in the waiting room of the ED. He is now
a. XR result of spirometry (prebronchodilator FEV1/FVC unconscious and unresponsive. What is the 1st
b. IV fluids ratio of 64% and postbronchodilator FEV1/FVC thing you would do?
c. Put leg splint ratio of 65%] points towards the diagnosis of a. Turn pt and put in recovery position
d. Send bloods for inv chronic bronchitis. Some may think of b. Put airway
e. US bronchiectasis! But in bronchiectasis there is c. Endotracheal intubation
Ans. The key is C. Put leg splint. [Still patient is copious purulent sputums (as cup full of sputum d. Assess GCS
hemodynamically stable and leg splinting takes not table spoon full!!!)]. e. Start CPR
(Thomas splint) 10-15 minutes. This will improve 904. A 62yo man presents with cough, Ans. The key is B. Put airway. [ABC protocol].
alignment and stop the ongoing loss which is breathlessness and wheeze. 24% O2, salbutamol 908. A 52yo man underwent a hemicolectomy.
usually alarming. Some argue for ABC protocol but and After a few days he complains of left ventricular
as the case is stable we can go for splinting first]. hydrocortisone were given. The symptoms pain
900. A 70yo pt presents with cough and SOB. He haven’t improved and so nebulized and fever. ECHO has been done and shows a
stopped smoking cigarettes 2yrs ago but has a bronchodilator systolic murmur. What is the next appropriate
50yr was repeated and IV aminophylline was given. inv?
smoking hx before quitting. CXR=consolidation ABG: pH=7.31, RR=32. What is the next a. CT
and bilateral bihilar lymphadenopathy. What is appropriate management? b. US
the best inv for this pt? a. Nasal IPPV c. CXR
a. LN biopsy b. Intubation and ventilation d. Blood culture
b. Pleural fluid cytology c. LABA e. LFT
c. CT d. Toxapram Ans. The key is D. Blood culture. [Dx a case of
d. MRI e. Amoxicillin PO infective endocarditis. Therefore the next
e. US Ans. The key is A. Nasal IPPV. [Here given case is appropriate investigation is blood culture].
Ans. The key is A. LN biopsy. [likely diagnosis is COPD has following indications of Nasal IPPV i) 909. A 19yo man has exercised induced asthma
lung cancer, so best investigation for this is LN Tachypnea (>24 breaths/min) and ii) and is using a salbutamol inhaler as req and
biopsy]. Hypercapnic respiratory acidosis (pH range 7.10- beclamethasone 400ug BD. He complains that he
7.35)]. has to wake up at night for his inhaler. What is
901. A 27yo pt met with a RTA. While the NGT is the single most appropriate tx?
passing, bowel sounds are heard in the chest. CXR 905. A young girl returns from holidays in Spain. a. Beclo
shows NGT curled. What is the dx? She complains of discharge from her ear and b. Regular salbutamol and budesonide
a. Diaphragm rupture complains of tragal tenderness. Exam: tympanic c. Sodium cromoglycate
b. Aortic rupture membance normal. Aural toilet has been done. d. Oral steroid
c. Splenic rupture What is the next appropriate med? e. Inhaled steroid
d. Bowel rupture a. Antibiotic PO Ans. The key is C. Sodium cromoglycate. [the
e. Liver rupture b. Antibiotic IV patients current complaint is nocturnal asthma for
Ans. The key is A. Diaphramatic rupture. c. Steroid PO which next step is long acting beta blocker!! This
902. A 62yo man dx with T2DM with BMI=33. d. Steroid drop question is probably a bad recall].
Lifestyle modifications have failed to control e. Antibiotic drop with steroid 910. Pt with a long hx of smoking is now suffering
blood Ans. The key is E. Antibiotic drop with steroid. from bronchial ca. histology reveals there are
sugar. Labs: urea=3.6mmol/l, [Discharge from ear and tragal tenderness are sheets of large polygonal or giant MNC. What is
creatinine=89mmol/l. what is the next features of otitis externa. Key treatment is aural the most likely dx?
appropriate management? toileting. Drop advised is Sofradex (Framycetin + a. Squamous cell ca
a. Biguanide dexamethasone) OHCS, 9th edition, page 542]. b. Small cell ca
b. Sulfonylurea 906. A 23yo man sprained his right ankle 6wks ago c. Adenocarcinoma
c. Insulin while playing football. He was tx with a below d. Large cell ca
d. Glitazone knew walking cast. On removal of the cast, the pt e. Oat cell ca
e. Sulfonylurea receptor binder noted to have right foot drop. He has Ans. No key is given!! Correct answer is D. Large
Ans. The key is A. Biguanide. [Patient is obese weakness of extensors of the ankle and toes and cell ca. [Large cell carcinoma is, by definition, a
type 2 diabetic. So biguanide is the tx of choice]. diminished pin prick sensation over the dorsum poorly differentiated malignant epithelial tumor.
903. A pt presents with progressive dyspnea. He of the foot. The ankle jerk is present and plantar It consists of sheets or nests of large polygonal or
complains of cough, wheeze and a table spoonful reflex is flexor. What is the most likely cause of giant multinuclear cells and probably represents
ofmucopurulent sputum for the last 18m. the foot drop? SCC ."]
a. Compression of common peroneal nerve
911. A 27yo man presents with chest pain and mL/min/1.73. The dose for glipizide is 2.5 to 10 deterioration indicates progressive respiratory
respiratory distress. Exam: tachycardia, mg/day. Glyburide and other long-acting failure for which he should be assessed for long
hypotension sulfonylureas are generally not recommended in term O2 therapy.  Long-term oxygen therapy
and neck vein distension. Trachea is deviated to any CKD patient with type 2 diabetes, because of (LTOT) for more than 15 h/day improved mortality
the left side, breathing sounds on right side are the risk of hypoglycemia. Some clinicians and morbidity in a well-defined group of patients
absent and diminished on left side. What is the recommend the use of the meglitinide repaglinide with chronic obstructive pulmonary disease.
next appropriate management? (starting with a dose of 0.5 mg) for nondialysis Requirement of condition to proceed to LTOT is
a. CXR CKD patients since these agents are not renally patient should be stable and on appropriate
b. Right side aspiration (16G) cleared. Nondialysis CKD patients with type 2 optimum therapy (as in given case) and having
c. Left side aspiration (16G) diabetes may be treated with an oral agent, stopped smoking tobacco. Patient should be
d. Right side drain with a small tube (12F) although many patients end up on insulin therapy shown to have a PaO2 less than 7.3 kPa and/or a
e. Left side drain with a small tube (12F) because it is more effective.So it may be that we PaCO2 greater than 6 kPa on two occasions at
Ans. The key is B. Right side aspiration (16G). [The can go for insulin as the answer!!]. least 3 weeks apart. FEV1 should be less than 1.5
features described is diagnostic of right sided litres, and there should be a less than 15%
tension pneumothorax. Next appropriate 914. A 5yo boy was brought to GP with high temp improvement in FEV1 after
management is To remove the air, insert a large- and many vesicles on his back. What is the most bronchodilators. Patients with a PaO2 between
bore (14–16G) needle with a syringe, partially appropriate management? 7.3 and 8 kPa who have polycythaemia, right
filled with 0.9% saline, into the 2nd intercostal a. Topic acyclovir heart failure or pulmonary hypertension may gain
interspace in the midclavicular line on the side of b. Oral acyclovir benefit from LTOT].
the suspected pneumothorax. Remove plunger to c. Oral antibiotics
allow the trapped air to bubble through the d. Topical steroids 917. A 49yo man complains of fullness in his left
syringe (with saline as a water seal) until a chest e. None ear, recurrent vomiting and tinnitus. What is the
tube can be placed. Alternatively, insert a large- most appropriate med?
bore Venfl on in the same location OHCM, 9th Ans. The key is E. None. [A case of chickenpox. a. Buccal prochlorperazine
edition, page 824]. None of the given treatment is used in b. Oral chlorpheniramine
chickenpox. Symptomatic treatment like, c. Oral flupenphenazine
912. A 16wk pregnant pt who was exposed to a acetaminophen if fever, antihistamine and d. Buccal midazolam
child with chicken pox came to GP for help. She calamine lotion is given]. e. IV rantidine
was
tested –ve for varicella antibody. What is the next 915. A woman came with the complaint of pain in Ans. The key is A. Buccal prochlorperazine.
most imp step in management? her right arm when she abducts it. She has [Meniere’s disease Dilatation of the
a. Reassurance recently endolymphatic spaces of the membranous
b. Ig moved to a new house. There is no hx of trauma. labyrinth causes vertigo for ~12h with prostration,
c. Ig + vaccine Wht is the likely cause of her pain? nausea/vomiting, a feeling of fullness in the ear;
d. Vaccine only a. Rupture of the long head of biceps uni- or bilateral tinnitus, sensorineural deafness
e. Acyclovir b. Sprain of the acromio-clavicular ligament (eg fluctuating). Attacks occur in clusters
Ans. The key is B. Ig. [If you are pregnant, come c. Tendinitis of the abductor sheat (<20/month). Cause: A mystery!
contact with chicken pox case, found antibody d. Supraspinatus tendinitis Electrocochleography; endolymphatic space MRI.
negative on blood test you have to take injection e. Shoulder dislocation Prochlorperazine as BuccastemR 3mg/8h PO (1st-
Ig. Ref: patient.info]. line if vomiting) or betahistine 16mg/8h PO or
Ans. The key is D. Supraspinatus tendinitis. chlorthalidone may help. Ref: OHCS, 9th edition,
913. A 68yo woman dx with T2DM and BMI=33. [Tendinitis and partial tears in the supraspinatus page 554].
Lab: GFR=29, urea=13, creatinine=390mmol/L. tendon causes a ‘painful arc’ since as the person
what elevates his arm sideways, the tendon begins to 918. A man had a soft mass on his mandible. Mass
is the next appropriate management? impinge under the acromion throught the middle is freely mobile and has started growing
a. Biguanide part of the arc, and this is usually relieved as the progressively over the past 6m. The mass still
b. Sulfonylurea arm reaches 180 degrees (vertical)]. moves freely. What is the best inv for this pt?
c. Insulin a. FNAC
d. Glitazone 916. An 83yo man with longstanding COPD has b. CT
e. Sulfonylurea receptor binder become progressively breathless over the last c. XR
2yrs. d. MRI
Ans. The key is C. insulin. [Insulin is devoid of He is on salbutamol, ipratropium, salmetarol, e. ESR
significant side effect than Glitazones (like fluid beclomethasone and theophylline. His FEV1<30%.
retention). In renal failure there is reduced GFR What is the next appropriate management? Ans. The key is A. FNAC.
and some fluid retention. It is not desirable that a. Lung transplant
glitazone to cause more fluid retention by causing b. Trial of CPAP 919. A 63yo man has been brought to the hosp
oedema. The oral agents that are thought to be c. Trial of non-invasive ventilation after collapsing during a wedding. His ECG is
relatively safe in patients with nondialysis CKD d. Assessment for long term O2 therapy below.
include short-acting sulfonylureas (eg, glipizide) e. Short course of O2 therapy What is the most likely dx?
and repaglinide. If an oral agent is used, the short-
acting sulfonylurea, glipizide, is the preferred Ans. The key is D. Assessment for long term O2
agent among nondialysis CKD patients who have therapy. [Patient is progressively breathless with
an estimated glomerular filtration rate (eGFR) <30 present FEV1 of <30%. So his respiratory
b. Repeat XR c. Analgesics
c. MRI d. Antibiotics
d. Surgery e. Tetanus prophylaxis
e. None
Ans. The key is B. Wide splint with upward
Ans. The key is A. Immobilization with cast. This is position. [If the limb is kept hanging it will
wrong key! Correct key is C. MRI. [Imaging: aggravate the swelling as fluid shifts to dependant
Request a dedicated ‘scaphoid’ series. If –ve, and part. Here wide splint with upward position will
fracture is suspected MRI has been shown to be help the accumulated blood or fluid to move
sensitive and cost-eff ective. CT is an alternative. If down and will improve the swelling].
neither is available, cast and re-x-ray in 2 weeks.
Ref: OHCS, 9th edition, page-744]. 926. A child was brought in to ED by his parents
922. A 71yo man with a hx of 50yrs of smoking for taking his grand-dad’s meds. There is an extra
presents with cough, hemoptysis, dyspnea and systole in the ECG. Which drug was taken?
chest a. Digoxin
pain. He also has anorexia and weight loss. The dx b. Amitryptiline
of lung cancer has been stabilized. Which c. Atenolol
electrolyte abnormality can be seen? d. Ramipril
a. Hyperkalemia e. Bendroflumethiazide
b. Hypocalcemia
c. Hyponatremia Ans. The key is A. Digoxin. [Both digoxin and
d. Hypernatremia amitryptiline can cause extrasystole].
a. VT e. Hypomagnesemia
b. A-fib 927. A 5yo child came from Ghana 6wks ago. 2d
c. VF Ans. The key is C. Hyponatremia. [Likely diagnosis ago he developed fever, vomiting and neck
d. A-flutter is squamous cell lung cancer (as the patient is stiffness.
e. SVT smoker) causing SIADH and resulting He had taken malaria prophylaxis and had no
hyponatremia]. rash. What is the dx?
Ans. The key is C. VF. [QRS complex is chaotic, a. Cerebral abscess
wide, bizarre and irregular which is characteristic 923. A 56yo man who is hypertensive recently b. Cerebral malaria
of ventricular fivrillation]. underwent a change in meds. 2days later he c. Meningococcal meningitis
developed wheezing. Which drug can cause this? d. SAH
*920. A 75yo war veteran complains of loss of a. Atenolol e. Cerebral tumor
appetite and says he has lost weight over the past b. Ramipril f. Pneumonia
few c. Bendroflumethiazide
months. He says that he has passed some blood in d. Verapamil Ans. The key is B. Cerebral malaria. [Incubation
his urine, however, he had no pain. A recent e. Furosemide period of malaria is 7 – 30 days. Malaria
report shows that PSA >5.5ng/ml. how will you prophylaxis cannot give confirmed protection and
manage this pt? Ans. The key is A. Atenolol. [Beta blockers can there is often failure of prophylaxis. Fever,
a. Radical prostatectomy precipitate asthma attacks]. vomiting, neck stiffness are consistent with
b. TURP cerebral malaria].
c. Cryosurgery 924. A 33yo man has a temp=38.5C, cough and
d. Brachytherapy chest pain on the right side on inspiration. He also 928. A HTN pt on bendroflumethiazide 2.5mg/d
e. Irradiation has has come for his routine checkup. Exam:
purulent sputum. What is the most likely BP=145/85mmHg. Lab: K+=5.9, Na+=137. What is
Ans. The key is A. Radical prostatectomy. This organism to cause pneumonia in this pt? the most appropriate management for this pt?
answer is controversial. [Though PSA is a bit a. Gram +ve diplococcic a. Stop meds
higher than normal it is not confirmatory of b. Coagulase +ve cocci b. Continue same dose
carcinoma! But loss of appetite and weight loss c. PCP cold agglutinins c. Increase the dose
are highly suggestive of cancer in this 75yrs old d. AFB d. Decrease the dose
man. Active Surveillance is offered first for Low e. Gram –ve diplococcic e. Repeat the blood test
Gleason score.esp at age of 75. and any other
treatment depending on extension of tumor will Ans. The key is C. PCP cold agglutinins. It is wrong Ans. The key is E. Repeat the blood test.
be offered after excluding mets]. key! Correct key should be A. Gram +ve [Bendroflumethiazide causes hyponatremia and
diplococci. [It is a case of community acquired hypokalemia. But the findings are opposite which
921. A 19yo boy comes to the ED with pain, pneumonia caused by streptococcus is probably error of test. Hence blood test should
swelling and tenderness 2cm distal to Lister’s pneumoniae]. be repeated to confirm the level of potassium and
tubercle of sodium].
radius. Exam: proximal pressure on the extended 925. A young man’s arm was caught in a machine.
thumb and index finger is painful. XR: no fx. XR showed no fx but arm is very swollen. What is 929. A 65yo man presents with significant weight
What is the next appropriate management for the the best tx? loss and complains of cough, SOB and chest pain.
pt? a. Plaster cast Exam: left pupil constricted, drooping of left
a. Immobilization with cast b. Wide splint with upward position eyelid. What is the most likely dx?
a. Pancoast tumor 931. A pt had passed a 4mm stone in his urine. He
b. Thoracic outlet syndrome has a 3mm stone in the renal pelvis found on US. 935. A 62yo man has been smoking about 15
c. Cervical rib What is the management? cigarettes/day for 45yrs, and has been working as
d. Pneumonia a. ESWL a
e. Bronchogenic ca b. None builder since he was 24yo. He presents with chest
c. Dormier basket pain, SOB, weight loss. CXR shows bilateral
Ans. The key is A. Pancoast tumor. [Pancoast d. Surgery fibrosis and left side pleural effusion. What is the
tumour is the apical lung cancer that is associated e. PCNL best inv that will lead to dx?
with destructive lesions of the thoracic inlet and a. CXR
involvement of the brachial plexus and cervical Ans. The key is B. None. b. Pleural fluid aspiration of cytology
sympathetic nerves (the stellate ganglion) leading  [Stones less than 5 mm in diameter pass c. MRI
to horner’s syndrome]. spontaneously in up to 80% of people. d. Pleural biopsy
 Stones between 5 mm and 10 mm in e. CT
930. A 4yo boy presents with fever, sore throat diameter pass spontaneously in about
and lymphadenopathy. The dx of tonsillitis has 50% of people. Ans. The key is D. Pleural biopsy. [There is
been  Stones larger than 1 cm in diameter asbestos exposure in builders and associated
made. He had 3 episodes last yr. What is the most usually require intervention (urgent smoking greatly increases the possibility of
appropriate management for this pt? intervention is required if complete developing mesothelioma and the given
a. Tonsillectomy obstruction or infection is present). presentation is typical of mesothelioma. Best
b. Paracetamol/ibuprofen investigation is pleural biopsy].
 Two thirds of stones that pass
c. Oral penicillin V
spontaneously will do so within four
d. IV penicillin 936. During a basketball match, one of the players
weeks of onset of symptoms]. Ref:
e. None suddenly collapsed to the ground with coughing
patient.info
and SOB. What is the inv of choice?
932. A 4yo boy presents with fever, severe ear
Ans. The key is B. Paracetamol/ibuprofen. a. CXR
ache, vomiting and anorexia. He also has mod
[Explanation: Drugs: b. CT
tonsillitis. Exam: tympanic membrane bulging. He
 Antipyretic analgesics such as c. MRI
came to the GP a few days ago and was dx
paracetamol and ibuprofen are of value. d. V/Q scan
with URTI. What is the most appropriate dx?
 For most patients, antibiotics have little e. CTPA
a. OE
effect on the duration of the condition b. Acute OM
or the severity of symptoms. The Ans. The key is A. CXR. [Likely diagnosis is
c. Serous otitis
National Institute for Health and Care pneumothorax. So investigation of choice is CXR].
d. Chronic suppurative OM
Excellence (NICE) suggests that e. Mastoiditis
indications for antibiotics include:[1] 937. A 57yo man having HTN on oral anti-HTN.
 Features of marked systemic However, he is finding it difficult to mobilize as he
Ans. The key is B. Acute OM. [High fever, severe
upset secondary to the acute feels dizzy whenever he tries to get up. What is
earache, vomiting, bulging tympanic membrane
sore throat. the most appropriate inv for him?
and H/O associated URTI is highly suggestive of
 Unilateral peritonsillitis. a. Ambulatory BP
acute OM].
 A history of rheumatic fever. b. ECG
 An increased risk from acute c. MRI
933. A 3yo girl presents with complains of sudden
infection (such as a child with d. CXR
right facial weakness and numbness and pain
diabetes mellitus or e. CT
around her ear. There are no symptoms. What is
immunodeficiency). the most appropriate dx?
 Acute tonsillitis with three or Ans. The key is A. Ambulatory BP. [Ambulatory BP
a. SAH
more of the following Centor to document low BP as cause of presenting
b. Bell’s palsy
criteria present: symptom. The case seems to be of postural
c. Stroke
 History of fever hypotension and low BP as a result of given anti
d. TIA
 Tonsillar exudates hypertensive].
e. Subdural hemorrhage
 No cough
 Tender anterior cervical 938. A 33yo female complains of diplopia on
Ans. The key is B. Bell’s palsy.
lymphadenopathy. upright gaze. Exam: ptosis can be seen. There are
[Source: patient.info]. no
934. A 6yo boy fell in the playground and has
Indications for tonsillectomy: other complains or any significant PMH. What is
been holding his forearm complaining of pain.
the most appropriate inv for him?
 The child has five or more episodes of Exam: no
a. Ophthalmoscopy
acute sore throat per year, documented sign of deformity or swelling. However, there is
b. Visual field test
by the parent or clinician. minimal tenderness on exam. What is the dx?
c. TFT
 Symptoms have been occurring for at a. Fx mid radius
d. CT
least a year. b. Fx mid ulnar
e. Checking red reflex
 The episodes of sore throat have been c. Fx neck of humerus
severe enough to disrupt the child's d. Fx shaft of humerus
Ans. The key is E. Checking red reflex. This is a
normal behaviour or day-to-day e. Green stick fx of distal radius
wrong key. Right key should be D. CT. [A case of
functioning.
Ans. The key is E. Green stick fx of distal radius.
3rd nerve palsy. So to delineate the cause CT b. IV calcium gluconate
should be done]. Ans. The key is B. Medial collateral. [The valgus c. IV insulin + dextrose
stress test involves placing the leg into extension, d. Furosemide
939. A tall rugby player was hit in the chest by a with one hand placed as a pivot on the knee. With e. IV 0.9% NS
player of the opponent team. He developed the other hand placed upon the foot applying
breathlessness and his face went blue and purple. an abducting force, an attempt is then made to Ans. The key is B. IV calcium gluconate. [In severe
You have been called to look at him, how will force the leg at the knee into valgus. If the knee is hyperkalemia IV calcium gluconate is given to
you manage him? seen to open up on the medial side, this is protect the heart from cardiac arrest or life-
a. Insert a needle in the 2nd ICS in the mid- indicative of medial collateral ligament damage]. threatening arrhythmias till definitive treatments
clavicular line are arranged].
b. Insert a needle in the 5th ICS in the mid-axillary 941. A 75yo man comes in complaining of
line difficulty in passing urine, poor stream and 945. DM man feels hot, painful lump near the
c. Intubate the pt dribbling at the anal region. What is the most probable dx?
d. Start CPR end of voiding and anorexia. US shows bilateral a. Anal fissure
e. Give oxygen hydronephrosis. What is the cause of these b. Abscess
findings? c. Hematoma
Ans. The key is E. give oxygen. [1. The patient a. BPH d. Wart
went blue and purple!! It is due to catch up of b. Renal stones e. External hemorrhoids
breath and it is not cyanosis by pneumothorax as c. Bladder stones
cyanosis of pneumothorax takes time to develop d. Prostatic ca Ans. The key is B. Abscess. [DM patients are much
and considered as a late feature. Immediate e. UTI prone to infection].
feature is chest pain, tachycardia, tachypnea etc.
2. There are no immediate feature mentioned Ans. The key is D. Prostatic ca. [Elderly patient of 946. A 65yo lady with T1DM for the last 20y
according to which we can say pneumothorax is 75yrs with obstructive symptoms of lower urinary comes with a tender lump near the anal opening.
the diagnosis. tract, bilateral hydronephrosis all can occur in BPH She
3. Collapse in sport’s trauma treatment is ABC. or ca prostate. Anorexia is clincher here. It is a says she also has a fever. What tx should she get?
4. Even if pneumothorax as treatment modality feature of carcinoma rather than BPH]. a. I&D + antibiotics
1st comes oxygen. b. IV antibiotics
942. 2h after an appendectomy, a pt complains of c. C&S of aspirate from swelling
a rapid HR and fever. He says there is also d. Painkillers
abdominal pain and pain in the shoulder area. e. Cautery of swelling
What is happening to this pt?
a. Intra-abdominal bleeding Ans. The key is A. I&D + antibiotics.
b. Anastomotic leak
c. Sepsis 947. An 80yo DM lady presents with redness and
d. Intestinal obstruction swelling over her right foot. It is tender to touch,
warm and glossy. What are the complications this
Ans. The key is A. Intra-abdominal bleeding. pt might develop?
a. Meningitis
943. A 50yo man presents with the complaints of b. Sepsis
recurrent UTI and occasional blood in the urine. c. Ulcer
Some unusual cells have been seen in urine on d. Gangrene
routine exam. Which os the following inv would
you like to carry out now? Ans. The key is D. Gangrene.
a. Cystoscopy
b. Urine C&S 948. After surgery a pt’s left leg has become
c. XR KUB swollen and tender. The diameter of the calf has
d. US increased and passive movements cause pain.
e. CBC What is the most probable dx?
First oxygen mask is put and it takes few
a. DVT
seconds!! Why oxygen? It helps absorb air from
Ans. The key is D. US. It is wrong key! Correct key b. Lymphedema
pleural space (nitrogen) and speeds up the
is A. Cystoscopy. [Likely diagnosis is bladder c. Peripheral vascular disease
absorption of air 4 times as room air. Then make
cancer. Occasional blood in urine and unusual d. Hematoma
the diagnosis on clinical ground and proceed for
cells (suggestive of malignant cells) suggests the e. Superficial thrombophlebitis
needle aspiration.
diagnosis. It is also true that in bladder cancer
there occurs frequent uti like symptoms]. Ans. The key is A. DVT. [Swelling, tenderness and
940. A young woman fell and hit her knee. Exam:
enlarged calf diameter are features of DVT
valgus test +ve. What ligament was most probably
944. A 28yo drug user presents to ED collapsed supported by positive Homan’s test (pain on
injured?
and anuria. His serum K+=7.5mmol/l. CXR shows passive movement)].
a. Ant cruciate
early
b. Medial collateral
pulmonary edema. What is the next appropriate 949. 2h after an appendectomy, a pt complains of
c. Lateral collateral
management for this pt? a rapid HR and fever. He says there is also
d. Post cruciate
a. Urgent hemodialysis
e. Meniscus
abdominal pain and pain in the shoulder area. carcinoma until proven otherwise. Source:
What is the first step in the management? SAMSON notes]. 956. A 65yo man on dexamethasone underwent
a. Maintain IV access and give IV fluids surgery. During and after the surgery, his blood
b. Start IV antibiotics 953. A 68yo pt wakes up with slurred speech and glucose was around 17-19mmol/l. What will you
c. Insert NGT for intestinal decompression right sided weakness. CT shows cerebral infarct. give the pt?
d. Cross match blood What is the most appropriate tx? a. Insulin
e. Emergency exploratory laparotomy a. Aspirin b. Oral hypoglycemic
b. Alteplase c. Remove dexamethasone
Ans. The key is A. Maintain IV access and give IV c. Warfarin d. IV Saline
fluids. [Features given are of internal bleeding! So d. Clopidogrel e. IX dextrose
maintain iv access and giving iv fluid is the first e. Dipyridamole
step in management of this patient]. Ans. The key is D. IV saline. [It is estimated that
Ans. The key is B. Alteplase. It is a wrong key! The adults secrete 75-150mg of cortisol in response to
950. A pregnant woman presents with knee pain correct key is A. Aspirin. [The window period to major surgery and 50mg a day for minor surgery,
on movements. The pain becomes worse at the administer alteplase is 4.5 hours. If we cannot and secretion parallels duration and extent of
end certain this period we cannot proceed for surgery
of the day. Radiology shows decreased joint alteplase]. Patients undergoing a surgical procedure or
space. Labs: CRP=12. What is the 1st line med? responding to stress, trauma, or an acute illness
a. Paracetamol 954. A 73yo man who is recovering from surgery will exhibit an increase in adrenal cortisol
b. NSAIDs on the left carotid artery in his neck. He has production up to 6-fold normal levels. However,
c. Oral steroid slurred in patients on chronic exogenous steroid therapy,
d. Intra articular steroid speech. On protusion of his tongue, the tip atrophy of the hypothalamicpituitary-adrenal
e. DMARDs deviated to the left. What is the single most (HPA) axis may occur through feedback inhibition,
appropriate option? leading to an inability to respond to stress. It has
Ans. The key is A. Paracetamol. [Pain on a. Accessory nerve historically been believed that patients receiving
movement, pain worse at end of day and b. Facial nerve long-term corticosteroids require supplemental,
decreased joint space are characteristic of c. Glossopharyngeal nerve perioperative doses, and that failure to provide
osteoarthritis supported by no significant raise in d. Hypoglossal nerve such coverage in secondary adrenal insufficiency
inflammatory marker (CRP=12). 1st line medicine e. Vagus nerve may lead to an adrenal crisis characterized by
is paracetamol]. hypotension and cardiovascular collapse.
Ans. The key is D. Hypoglossal nerve. [Testing *** So who are telling remove dexamethason are
951. A 68yo man presents with muscle weakness. function of the nerve is performed by asked the wrong!!!***
He is not able to climb stairs. He also complains of subject to stick their tongue straight out. If there patients receiving “physiologic replacement”
mild breathlessness. He says that he sometimes is a loss of innervation to one side, the tongue will doses (<10 mg/ day) do not need additional
feels difficulty in swallowing food. Labs: curve toward the affected side, due to unopposed steroids perioperatively beyond their standard
ALP=216, AST=49, ALT=43, CK=417, ESR=16. What action of the opposite genioglossus muscle. If this regimen. Patients receiving doses exceeding the
is the most likely dx? is the result of alower motor neuron lesion, the normal expected stress response for their surgery
a. Polymyositis tongue will be curved toward the damaged side, (>150 mg/ day of hydrocortisone equivalent for
b. Polymyalgia rheumatic combined with the presence of fasciculations or moderate/major surgery, or >50 mg/ day of
c. Muscular dystrophy atrophy. However, if the deficit is caused by hydrocortisone equivalent for minor surgery) also
d. Esophageal carcinoma an upper motor neuron lesion, the tongue will be do not need additional coverage beyond their
e. Osteoarthritis curved away from the side of the cortical damage, current therapy, because acute
without the presence of fasciculations or immunosuppressive doses are more than
Ans. The key is A. Polymyositis. [Proxymal atrophy]. sufficient to maintain cardiovascular stability
myopathy (not able to climb stairs), mild during the operation.
breathlessness (involvement of thoracic muscles 955. A 24yo woman known to be suffering from Regular daily dose of more than 10 mg
or interstitial lung disease), dysphagia due to panic disorder presents to the hospital with prednisolone within the last three months:
involvement of the oropharyngeal striated tingling i) Minor surgery:
muscles and upper oesophagus)and raised CK and numbness in her fingers. ABG: pH=7.52, 25 mg hydrocortisone at
suggests the diagnosis of Polymyositis]. PCO2=2.2kPa, PO2=11kPa, Bicarb=20. What is the induction
most likely condition? ii) Moderate surgery
952. A 67yo builder presents with a persistent a. Acute metabolic alkalosis Usual pre-operative steroids
nodular lesion on upper part of pinna with some b. Acute resp alkalosis (Hysterectomy) + 25 mg
telangiectasia around the lesion. What is the dx? c. Compensated resp alkalosis hydrocortisone at induction
a. Basal cell d. Compensated metabolic acidosis + 100 mg hydrocortisone/day
b. Squamous cell e. Acute metabolic acidosis
c. Keratocanthoma iii) Major surgery Usual pre-
d. Actinic keratosis Ans. The key is B. Acute respiratory alkalosis. [In operative steroids
e. Bowens disease panic attack there occurs hyperventilation which Major trauma, prolonged + 25 mg
causes washout of CO2 and leads to acute hydrocortisone at induction
Ans. The key is A. Basal cell ca. [Any ulcer which is respiratory alkalosis resulting in raised pH >7.45 surgery + 100 mg
located above the neck is always basal cell (here 7.52), low PCO2 (here 2.2 kPa) with hydrocortisone/day for 2 – 3 days
compensatory decrease in HCO3 (here 20meq/l)].
Surgical patients commonly develop
hyperglycemia related to the hypermetabolic Ans. The key is C. Social anxiety. [Social anxiety Ans. The key is E. LN biopsy.
stress response, which increases glucose disorder is a type of complex phobia. This type of
production and causes insulin resistance. phobia has a disruptive or disabling impact on a 964. A 52yo man known DM presents to ED with
Although hyperglycemia is associated with worse person's life. It can severely affect a person's sudden onset of pain in the left loin and
outcomes, the treatment of hyperglycemia with confidence and self-esteem, interfere with hematuria.
insulin infusions has not provided consistent relationships and impair performance at work or Inv: 8mm stone in left lower ureter. Nifedipine
benefits. Despite early results, which suggested school]. with steroids was prescribed as initial tx with
decreased mortality and other advantages of supportive therapy. He returned complaining of
“tight” glucose control, later investigations found 960. Post gastric ulcer got perforated leading to worsening pain, vomiting with passing of 2
no benefit or increased mortality when bleeding involving the gastro-duodenal artery. stones. Renal function tests indicate impending
hyperglycemia was aggressively treated with Where would fluid accumulate in the cavity? ARF. How will you manage this pt?
insulin. Because of these conflicting data, the a. Left paracolic gutter a. Continue same tx
optimal glucose concentration to improve b. Pelvic cavity b. Start alpha blocker
outcomes in critically ill patients is unknown. c. First part of duodenum c. ESWL
There is agreement, however, that hypoglycemia d. Under the diaphragm d. Percutaneous nephrolithotomy
is an undesirable complication of intensive insulin e. Retroperitoneal e. Percutaneous nephrostomy
therapy and should be avoided.  f. Open surgery
So, “FOR PERIOPERATIVE HYPERGLYCEMIA NO Ans. D. Under the diaphragm. [Correct option
NEED OF INSULIN” just IV saline!!!]. probably A. Left paracolic gutter]. Ans. The key is E. Percutaneous nephrostomy.
[Percutaneous nephrostomy, or
961. A 4yo boy presents with recurrent episodes nephropyelostomy, is an interventional procedure
957. A 61yo man who had stroke 2y ago is on of self limiting spontaneous bleeding. Coag test: that is used mainly in the decompression of the
aspirin. He has RA but suffers from pain and can’t PT renal collecting system. Percutaneous
tolerate it. He is taking senna for constipation. normal, bleeding time normal, APTT prolonged, nephrostomy catheter placement has been the
What is the best med to relieve his pain? Factor VIII decreased. His father and uncle primary option for the temporary drainage of an
a. DMARDs suffer from a similar illness. What is the most obstructed collecting system. Here impending ARF
b. Ibuprofen likely dx? indicates obstructive uropathy].
c. Co-codamol a. Hemophilia A
d. Paracetamol b. Hemophilia B 965. A lady who is alcohol dependent wants to
c. Von willebrand’s disease quit but wants someone to encourage her. What
Ans. The key is B. Ibuprofen. [DMARDs are not d. ITP would you do?
painkillers but prevents disease progression in e. TTP a. Medication
long term. Paracetamol is not enough to control b. Refer to social services
severe pain. Co-codamol will cause constipation. Ans. The key is A. Hemophilia A. [Prolonged APTT c. Refer to psychology
Patient is already taking laxative! So Co-codamol and decreased factor VIII points towards the d. CBT
is not the option. Excluding other Ibuprofen diagnosis of Hemophilia A].
seems to be the best option]. Ans. The key is B. Refer to social services.
962. A 53yo lady presents with hot flash and night
958. A young child was brought by his mother to sweats. Her LMP was last year. She had MI 966. A young girl presented to OBGYN assessment
the OPD complaining that he raised the vol of the recently. What is the most appropriate unit with lower abdominal pain and per vaginal
TV management for her? bleeding after a hx of hysterosalpingograph as a
and didn’t respond to her when she called him. a. Raloxifene part of her infertility tx. Observation:
Exam: tympanic membrane was dull greyish and b. Estrogen BP=90/50mmHg, pulse=120bpm, exam revealed
no shadow of handle of malleus. What is the most c. COCP rigid abdomen. What is the most appropriate
probable dx? d. Evening primrose next inv?
a. Chronic OM e. Clonidine a. CT
b. Acute OM b. XR erect and supine
c. Secretory OM Ans. The key is C. COCP. [COCP has very little c. US abdomen
d. Otitis externa effect in stroke or MI and hence can be used to d. Coag profile
e. Cholesteatoma treat post menopausal symptoms in those e. CXR
patients].
Ans. The key is C. Secretory OM. Ans. The key is C. US abdomen. [Likely cause of
963. A 73yo man who was a smoker has quit bleeding and shock is ruptured fallopian tube for
959. A 48yo woman always socially withdrawn has smoking for the past 3yrs. He now presents with which appropriate next investigation is US
stopped going out of the house. She is afraid to hoarseness of voice and cough since past 3wks. abdomen].
socialize because she fears that people will XR: mass is visible in the mediastinum. What is
criticize her. What is the most probable dx? the best inv to confirm the dx? 967. A 21yo woman who is on COCP had to take
a. Agoraphobia a. Bronchoscopy azithromycin. What should be advised for her
b. PTSD b. Thoracoscopy contraception?
c. Social anxiety c. US a. Using 7d condoms after antibiotics and avoid
d. OCD d. CT thorax pill free break
e. GAD e. LN biopsy
b. Using 14d condoms after antibiotics and avoid d. Liver failure weeks to months. He was given salbutamol
pill free break e. Stroke nebulization and antibiotics and admitted to the
c. Using 7d condoms after antibiotics Ans. The key is B. Benign essential tremor. ward. He died 3d later. CT: patchy infiltrates,
d. No extra precaution [Tremors absent at rest and present on pleural thickening and pleural effusion. Why is this
e. Using 14d condoms after antibiotics outstretched hand and persist on movement are a coroner’s case?
benign essential tremor]. a. Pt got wrong dx or management
Ans. The key is D. No extra precaution. [Before it 972. Pregnant lady had her antenatal screening b. Pt died soon after admission
was thought that antibiotics like azithromycin for HIV and Hep B. what more antenatal inf c. Death could be due to occupational illness
inhibits the enzyme and reduce the efficacy of should Ans. The key is C. Death could be due to
COCP. But later it was established that practically she be screened for? occupational illness.
no significant changes occur and so no need of a. Rubella and syphilis 975. A 26yo lady came with abdominal pain,
any extra precaution]. b. Toxoplasma and rubella vaginal discharge and low grade fever. What is the
c. Syphilis toxoplasma most
968. A 60yo woman presented with radial fx and d. Hep C & E likely dx?
had a colle’s fx and supracondylar fx in the past. e. Hep A & C a. HELLP syndrome
What inv is req to detect her possibility of having Ans. The key is A. Rubella and syphilis. b. Acute PID
the same prb later? 973. A young man has been found in the park, c. Ectopic pregnancy
a. Dexa scan drunk and brought to the ED by ambulance. He d. Appendicitis
b. MRI recently lost his job and got divorced. He thinks Ans. The key is B. Acute PID.
c. Nuclear bone scan nurses are plotting against him. What is the 976. A new screening test has been devised to
d. CT most likely dx? detect early stages of prostate ca. However, the
e. Bone biopsy a. Schizoid personality test
b. Borderline personality tends to dx a lot of ppl with no cancer, although
Ans. The key is A. Dexa scan. [The likely cause of c. Schizophrenia they do have cancer as dx by other standard
these multiple fracture is osteoporosis (post d. Psychotic depression tests. What is this flaw called?
menopausal women) for which we should do e. Paranoid a. False +ve
Dexa scan to establish the diagnosis]. Ans. The key is E. Paranoid personality disorder. b. True +ve
[Patients disease (Paranoid personality disorder) c. False –ve
969. A 43yo woman presents with low mood, loss is responsible for his divorce and loss of job. d. True –ve
of libido, sleep disturbance, tiredness, palpitation, Suspiciousness to wife is reason for divorce]. e. Poor specificity
chest discomfort, irritability and recurrent Ans. The key is C. False negative.
worries. What is the most likely dx? 977. A 26yo political refugee has sought asylum in
a. Seasonal Affective Disorder the UK and complains of poor conc. He keeps
b. Mod depression getting thoughts of his family whom he saw killed
c. Dysthymia in a political coup. He is unable to sleep and
d. GAD feels hopeless about his survival. Because of this
e. Bipolar disorder he is afraid to go out. What is the most likely dx?
a. Acute stress disorder
Ans. The key is B. Moderate depression. [Mild b. PTSD
depression: i)Low mood ii) Anhedonia iii) Guilt iv) c. Social phobia
Hopelessness v) Worthlessness vi) Inability to d. OCD
concentrate. Tx CBT e. GAD
Moderate depression: Features of mild + vii) Poor Ans. The key is B. PTSD. [Keep getting thoughts of
sleep viii) Poor Appetite ix) Poor libido x) Easy his family disaster, insomnia, to avoid similar
fatiguability. Tx Antidepressants circumstances he is avoiding going out these are
Severe depression: Features of moderate + xi) suggestive of PTSD].
Suicidal intensions. Tx ECT]. 978. A 2yo boy presented with gradual swelling of
feet and poor feeding. He has gained weight and
970. Which of the following is true for tamoxifen? has dark urine. What is the single most
a. Increased incidence of endometrial carcinoma appropriate inv?
b. Increased risk of breast ca a. Serum albumin (2nd)
c. Increased risk of osteoporosis b. 24h urinary protein (1st)
d. Increased risk of ovarian ca c. Serum calcium
Ans. The key is A. Increased incidence of d. BUE
endometrial carcinoma. e. Serum glucose
971. A 45yo male complains of tremors in hands. Ans. The key is B. 24 hr urinary protein. [A case of
Exam: tremors are absent at rest but present nephritic syndrome. So the single most
when appropriate investigation from the given options
arms are held outstretched and persist on is 24 hour urinary protein].
movement. What is the most probable dx? 979. A 26yo lady presents with high fever, lower
a. Parkinsonism 974. An elderly man who used to work in the abdominal pain and purulent vaginal discharge.
b. Benign essential tremor shipyard industry presented with cough and SOB She
c. Cerebellar disease few
looks very unwell. What is the most appropriate b. US
management? c. Antibiotics Ans. The key is C. Turner’s syndrome. [Down’s
a. Tetracycline 250mg QD d. IV fluids syndrome and Fragile x syndrome don’t have
b. Doxycycline 100mg BD and metronidazole e. Doppler US primary amenorrhea. Klienfelters syndrome
400mg BD patients are tall males. So the likely diagnosis is
c. IV Ceftriaxone 2g with doxycycline 100mg Ans. The key is A. Urgent exploration. [As sudden Turner’s syndrome].
d. IV ceftriaxone 2g with doxycycline 500mg onset of severe pain likely diagnosis is torsion of
e. Ofloxacin 400mg BD and metronidazole 400mg testis. (infection takes a more prolonged course). 988. A 32yo woman wants reversible form of
BD Next step is urgent exploration]. contraception. She has one child delivered by
Ans. The key is C. IV Ceftriaxone 2g with emergency C-section. She also suffers from
doxycycline 100mg. [The disease is severe 984. An 8wk pregnant woman presents with migraine and heavy periods. What is the most
needing inpatient management with IV persistent vomiting and weight loss. Exam: suitable form of contraception for this lady?
Ceftriaxone 2g + IV doxycycline 100 mg (as Iv HR=110bpm. a. COCP
doxycycline is not licensed to use in uk probably Dehydration was corrected with NS infusion and b. Mini pill
oral doxycycline is given). K+. The condition didn’t improve so IM cyclizine c. IUCD
980. A 39wk pregnanct woman came to labor was given. She is still vomiting. What is the next d. Barrier method
suite 3d after an obstructed labour presents with appropriate management? e. Abstinence
pain a. IV fluids
and swelling of one leg. Exam: leg has blue b. IV antiemetics Ans. The key is C. IUCD. It is wrong key! Correct
mottling and is cold. What is the dx? c. IV steroids key is b. Mini pill. [IUCD causes increased
a. DVT d. Terminate pregnancy bleeding. In migraine we cannot give COCP. So for
b. Post phlebitis syndrome e. Thiamine contraception best option from the given ones are
c. Embolus b. Mini pill though it does not improve
d. Varicose vein Ans. The key is C. IV Steroid. [Steroids may be menorrhagia].
e. Herpes gladiatorum used in patient's of hyperemesis gravidarum
Ans. The key is C. Embolus. refractory to standard therapy].  989. A 45yo known hypertensive man presents
981. An 8yo boy has his tonsils and adenoids with hematuria, proteinuria and edema. What is
removed. On the 7th post-op day, he comes back 985. A 28yo lady presents with dyspareunia and the
to the dysmenorrhea. She is very obese. She now wants definitive dx test for him?
ED with hemoptysis and fever. What is the most reversible contraceptive method. Which of the a. Urine protein
appropriate management? following will be most suitable for her? b. Renal biopsy
a. Admit for IV antibiotics a. Mirena c. Renal function test
b. Prescribe oral antibiotics and discharge b. COCP d. Urine microscopy
c. Packing c. POP e. Serum protein
d. Surgery d. Copper T
e. Reassurance e. Barrier method Ans. The key is B. Renal biopsy. [In nephritic
syndrome glomerulus is affected. So renal biopsy
Ans. The key is A. Admit for IV antibiotic. [Most Ans. The key is A. Mirena. is the definitive diagnostic test].
secondary hemorrhage occurs due to infection
which erodes a vessel. So patient should be 986. A young lady who is 28wks pregnant 990. A 47yo man presents with proteinuria+,
admitted for IV antibiotics]. presents with vaginal bleeding. She has lost about BP=160/95mmHg, small kidneys that have smooth
200ml of renal
982. A 50yo female had swelling in her ankles. She blood. Exam: uterus is tender. Resuscitation has pelvis. What is the most probable dx?
is a known alcoholic. Now she presented with been done. What is the most imp inv to a. GN
breathlessness and palpitations. What is the most establish the dx? b. Chronic pyelonephritis
likely cause of her condition? a. US c. Unilateral renal artery stenosis
a. VT b. CT d. Multiple myeloma
b. SVT c. D-dimer e. ARF
c. A-flutter d. Clotting profile
d. A-fib e. None Ans. The key is A. GN. [In chr. Pyelonephritis US
e. V-ectopics will demonstrate renal scarring and urinalysis will
Ans. The key is A. US. [Antepartum hemorrhage. demonstrate pyuria which are absent here. In
Ans. The key is D. Atrial fibrillation. [Ankle swelling Most important investigation to establish the dx is renal artery stenosis there will be considerable
suggests alcoholic cardiomyopathy and alcoholism US]. difference in kidney size from one another which
is also a well known cause of atrial fibrillation]. is not found here. Here likely diagnosis is GN].
987. A 14yo girl presents with primary
983. A young boy has acute scrotal pain for a few amenorrhea and a short stature. What is the most 991. You are the HO in the hospital and the lab
hours. Exam: one testis is very painful to touch. likely dx? report of a pt shows glucose=4mmol/l,
He a. Down’s syndrome K+=5.2mmol/l,
had this kind of pain before but it was mild and b. Klinefeltner’s syndrome Na+129mmol/l. what is the most appropriate
resolved itself within 30mins. What would you c. Turner’s syndrome management?
do next? d. Fragile X syndrome a. NS 0.9%
a. Urgent exploration e. Normal finding b. NS 0.45%
c. NS 0.9% and insulin Ans. The key is E. Testicular torsion. [Sudden a. Flupenthixol
d. Insulin severe pain, scrotal swelling and higher position b. Fluphenazine
e. Dextrose of the affected testis is diagnostic of testicular c. Haloperidol
torsion. You may get further support from d. Paraldehyde
Ans. The key is A. NS 0.9% [Here there is examining cremesteric reflex which is absent in e. Risperidone
significant hyponatremia with mild hyperkalemia. testicular torsion!!].
Glucose is in lower normal level. Insulin can shift Ans. The key is C. Haloperidol. [If oral preparation
K+ in cells but this mild hyperkalemia does not 995. A 24yo male involved in RTA with XR: fx neck was asked we could give risperidone but in
need this treatment at this level. More over in of humerus. What is the single most associated question asked injectable preparation. Injection
lower blood glucose insulin may cause nerve injury? Risperidone is a depot preparation the action of
hypoglycemia. So the best management option is a. Axillary nerve which takes few days to start. So in acute case it is
A. NS 0.9% which will correct the electrolyte b. Radial nerve not suitable; hence from the given options
imbalance (including K+ level by renal handling)]. c. Median nerve haloperidol is the best choice].
d. Ulnar nerve
992. A 27yo man presents with abdominal pain. 999. A 65yo man complains of hematuria,
He says his urine is dark. Exam: Ans. The key is A. Axillary nerve. frequency, hesistancy and nocturia. He reports
BP=160/105mmHg. that on
What is the most appropriate inv? 996. A 64yo man complains of increasing SOB and certain occasions he finds it difficult to control the
a. US cough for the past 18m. He coughs up a Tbsp of urge to pass urine. Urine microscopy confirms
b. Renal biopsy mucopurulent sputum with occasional specks of the presence of blood but no other features.
c. CT blood. What is the most likely underlying cause? What is the most porbable dx?
d. Urine protein a. Acute bronchitis a. BPH
e. Urine microscopy b. Bronchiectasis b. Bladder ca
c. Chronic bronchitis c. Prostatic ca
Ans. The key is A. US. [Abdominal pain, Dark urine d. Lung cancer d. Pyelonephritis
(hematuria) and hypertension suggests ADPKD. e. Pneumonia e. Prostatitis
The sensitivity of ultrasonography for ADPKD1 is Ans. There are two keys A. BPH and C. Prostatic
99% for at-risk patients older than 20 years]. Ans. The key is C. Chronic bronchitis. [There may carcinoma. [There are no constitutional features
be specks of blood in sputum in both of carcinoma. Bleeding is more common in BPH
993. A 12m child with AIDS is due for his MMR bronchiectasis , acute and chronic bronchitis. and occurs in a minority of cases and much less
vaccination. What is the single most appropriate Duration of disease is 18 months, so it is not acute common in prostatic carcinoma. Features given
action? bronchitis. Again only one tablespoonful sputum are of prostatism only which favours the diagnosis
a. Defer immunization for 2wks does not justify the term copious sputum of of BPH].
b. Don’t give vaccine bronhiectasis in which sputum will be much more 1000. A 60yo man presents with mass in the
c. Give half dose of vaccine like cup-full in amount. So it is chronic bronchitis]. groin. Exam: mass lies below the midpoint of the
d. Give paracetamol with future doses of the inguinalligament and is pulsatile. What is the most
same vaccine 997. A 55yo man who is hypertensive suddenly probable dx?
e. Proceed with standard immunization schedule lost his vision. The retina is pale and fovea a. Direct inguinal hernia
appears as b. Saphenavarix
Ans. The key is B. Don’t give vaccine. [MMR is live a bright cherry red spot. What is the single most c. Femoral hernia
attenuated vaccine which should be avoided in appropriate tx? d. Irreducible hernia
AIDS]. [*** Please note!! In similar question a. Pan retinal photocoagulation e. Femoral aneurysm
where asked about HIV positive child’s vaccination b. Corticosteroids Ans. The key is C. Femoral hernia. Controversial
and the answer was give all vaccines except BCG!! c. Scleral buckling key! May be femoral aneurism . [Hernia below
HIV positive is not in a immunodeficient state and d. Surgical extraction of lens pubic tubercle (below inguinal ligament) is
can receive live attenuated vaccines (except BCG), e. Pressure over eyeball femoral hernia. But below mid-inguinal point and
where as child with AIDS means he has criteria of pulsatile mass may be femoral aneurism as
immunodeficiency already for which all live Ans. The key is E. Pressure over eyeball. [In central well!!].
attenuated vaccines should be avoided]. [Please retinal artery occlusion (CRAO) retina becomes
note the two terms “HIV positive” and “with pale and fovea becomes cherry red. Hypertension 1001. An 82yo man has woken up with incoherent
AIDS”]. is a major cause of CRAO. Apply direct pressure speech and difficulty in finding the right words.
for 5-15 seconds, then release. Repeat several Exam: otherwise normal, good comprehension.
994. A young man presents with sudden, severe times. Ocular massage can dislodge the embolus Which anatomical site is most likely to be
pain and swelling in the scrotum. Exam: one testis to a point further down the arterial circulation affected?
seems higher than the other. What is the most and improve retinal perfusion]. a. Broca’s area
probable dx? b. Wernicke’s area
a. Varicocele 998. A 32yo man with schizophrenia and a hx of c. Midbrain
b. Hematocele violence and distressing auditory hallucinations d. Parietal cortex
c. Testicular tumor was e. Pons
d. Epidiymo-orchitis admitted to the ward with aggressive behavior Ans. The key is A. Brocha’s area. [A person with
e. Testicular torsion and has already smashed his room. He is refusing expressive aphasia will exhibit halting and
any oral meds. What is the single most effortful speech. Speech may only include
appropriate injection? important content words. Word comprehension is
preserved. The person may still be understood, above and lateral to the pubic tubercle. It is 3. Paranasal sinusitis.
but sentences will not be grammatical. This reducible. On applying pressure on the internal 4. Pulmonary infiltrates (may be transient).
contrasts with receptive or Wernicke’s aphasia, ring 5. Histological confirmation of vasculitis
which is distinguished by a patient's inability to there is no cough impulse seen. What is the most with extravascular eosinophils.
comprehend language or speak with probable dx?
6. Mononeuritis multiplex or
appropriately meaningful words though fluency a. Direct inguinal hernia
polyneuropathy.
may be preserved]. b. Indirect inguinal hernia
The presence of four out of six of these features
c. Femoral hernia
has a high specificity and sensitivity for the
1002. A 25yo woman has a recent cough, d. Strangulated hernia
diagnosis of CSS.
hoarseness and swelling in the neck. There are e. Femoral aneurysm
In the given case there are 1. Bell’s palsy
several nontender swellings on both sides of her Ans. The key is B. Indirect inguinal hernia.
(mononeuritis multiplex) 2. Asthma 3. Multiple
neck. She has lost 13kgs. She takes recreational [Swelling in the groin; mass just above and lateral
soft shadows on CXR (pulmonary infiltrates) 4.
drugs. to the pubic tubercle means inguinal hernia. It is
Eosinophilia. Presence of these 4 features are
What is the most probable dx? reducible. On applying pressure on the internal
diagnostic of CSS. In CSS 70% patient is p-ANCA
a. Thyrotoxicosis ring there is no visible cough impulse; it means
positive!].
b. Hyperthyroidism the hernia enters through deep ring, and enters
c. Vocal cord nodules scrotum passing through the superficial ring. That
1010. A 50yo man complains of visual prbs and
d. Carcinoma bronchus means it is indirect inguinal hernia].
dull pain in the left eye. Fundoscopy reveals
e. TB
papilloedema. He was dx with MS 2yrs ago. There
Ans. The key is E. TB. [Cough, wt. loss, non-tender 1007. A 34yo male presents with headache and
is no consensual light reflex of the right eye.
cervical lymphadenopathy, and immunodeficient vomiting. Exam: temp=38.5C, neck stiffness,
What is the single most likely defect?
state as a result of recreational drug use all these discharge
a. Paracentral scotoma
favours the diagnosis of TB. Laryngeal from left ear and right sided hyper-reflexia with
b. Mono-ocular field loss
involvement may be the cause of horseness]. an extensor plantar response. What is the most
c. Homonymous upper quadrantanopia
1003. A 30yo woman presents with acute likely dx?
d. Central scotoma
headache. She complains of seeing halos a. Cerebral tumor
e. Homonymous lower quadrantanopia
especially at night. b. Meningitis
Ans. The key is B. Mono-ocular field loss.
What is the single most likely defect? c. Cerebellar tumor
a. Paracentral scotoma d. Cerebral abscess
1011. A 54yo pt wakes up with right sided
b. Mono-ocular field loss e. Normal pressure hydrocephalus
weakness. His current medication is
c. Tunnel vision Ans. The key is D. Cerebral abscess. [Otitis media
bendroflumethiazide
d. Central scotoma can lead to cerebral abscess].
for HTN. Pulse=92bpm, BP=160/90mmHg. CT
e. Cortical blindness
shows left cerebral infarct. What is the most
Ans. The key is C. Tunnel vision. [The diagnosis is 1008. A 26yo male presents with speech
appropriate tx?
glaucoma (headache and haloes especially at difficulties. Exam: nystagmus. Which anatomical
a. Alteplase
night). In glaucoma there occurs tunnel vision]. site is most
b. Aspirin
likely to be affected?
c. Clopidogrel
1004. A 35yo man presents with a headache that a. Midbrain
d. Dipyridamole
worsens on bending his head forward. What is the b. Pons
e. Simvastatin
most likely dx? c. Cerebellum
Ans. The key is A. Alteplase. It is a wrong key!
a. Chronic sinusitis d. Cerebrum
Correct option is B. Aspirin. [In stroke window
b. SAH e. Vestibule cochlear nerve
period to use alteplase is 4.5 hours. In the given
c. Migraine Ans. The key is C. Cerebellum. [Difficulty speech
history we can no way certain the time of stroke
d. Cluster headache (cerebellar dysarthria) and nystagmus are
and we can not proceed for alteplase unless
e. Tension headache suggestive of cerebellar lesion].
window period is confirmed. Hence we cannot
Ans. The key is A. Chronic sinusitis.
give alteplase but Aspirin].
1009. A 75yo man presents with Bell’s palsy. His
1005. A 20yo man presents with painful PMH is significant for late onset asthma and heart
1012. A 33yo man presented to the GP with hx of
swallowing. Exam: trismus and unilateral failure. He also reports to have consulted his GP
headaches and photophobia. The GP examines
enlargement of his for generalized rash prv. CXR: multiple soft
him
tonsils. The peritonsilar region is red, inflamed shadows and CBC: eosinophilia. What is the single
and finds a rash and is now ringing you at the
and swollen. What is the most appropriate tx? most likely positive antibody?
hospital for advice. What would you advice the
a. Oral antibiotics a. P ANCA
GP?
b. IV antibiotics and analgesics b. C ANCA
a. Send pt home
c. I&D with antibiotics c. Anti Ro
b. Start IV benzylpenicillin
d. Analgesics with antipyretics d. Anti DS DNA
c. Conduct LP
e. Tonsillectomy e. Anti centromere
d. Start IV ceftriaxone
Ans. The key is C. I&D with antibiotics. [The dx is Ans. The key is A. p-ANCA. [The dx is Charg Strauss
Ans. The key is B. Start IV benzylpenicillin. [Before
peritnsillar abscess and tx is I&D with antibiotics]. Syndrome (CSS). There are six criteria for dx of
hospitalization IV benzylpenicillin. In hospital
CSS:
Ceftriaxone can be given but not in calcium
1006. A 40yo manual worker presents with a 1. Asthma (wheezing, expiratory rhonchi). containing fluid instead give Cefotaxime (NICE).
swelling in the groin. Exam: mass is found to be 2. Eosinophilia of more than 10% in According to OHCM hospital management is
just peripheral blood.
Cefotaxime if <55yrs and Cefotaxime + Ampicillin Ans. The key is D. ECT. [ [In question it is 1021. An asymptomatic 56yo man who has never
if >55yrs ]. mentioned she has severe depression. In consumed alcohol came for a routine checkup.
1013. An 89yo pt has lung cancer. His severedepression the tx is ECT]. Exam:
Na+=122mmol/l. What is the tx for this? increased skin pigmentation, spider angioma,
a. Demeclocycline 1017. A 20yo woman with no prv hx of ear cardiomegaly, S3 gallop, liver firm with 8cm span,
b. Vasopressin complains, presents with 1d hx of severe pain in no ascites. He is in the risk of which condition?
c. Restrict fluids the right a. Cerebellar degeneration
d. Reassure ear which is extremely tender to examine. What is b. Werniecke’s encephalopathy
Ans. There are two keys. A. Demeclocycline and C. the single most likely dx? c. Renal failure
Restrict fluids. *both are correct!![• a. Chondromalasia d. Hepatoma
Asymptomatic chronic hyponatraemia fluid b. Furuncle e. Hepatic vein thrombosis
restriction is often sufficient if asymptomatic, c. Myringitis Ans. The key is D. Hepatoma. [Increased skin
although demeclocycline (ADH antagonist) may be d. OE pigmentation, features of chronic liver disease
required. Ref: OHCM, 9th edition, page 686]. e. OM (spider angioma), Cardiomyopathy (cardiomegaly,
Ans. The key is D. OE. [Extreme tenderness to S3 gallop), Enlarged liver these are suggestive of
1014. A 25yo woman who is 11wks pregnant had examine indicate otitis externa]. Haemochromatosis. (OHCM: Slate-grey skin
central abdominal pain for 36h. The pain is now pigmentation; signs of chronic liver disease;
colicky. There is no vaginal bleeding. She has 1018. A couple has just finished their detox hepatomegaly; cirrhosis; dilated cardiomyopathy;
vomited once and has had an episode of loose regime and wants a drug with a pharmacological osteoporosis; also endocrinopathies are features
motion. She looks ill, temp=37.8C and there is action to of haemochromatosis). If cirrhosis, 22–30% get
rebound tenderness in the RIF. What is the most serve as a deterrent when they take alcohol. hepatocellular cancer].
probable dx? What drug is the appropriate choice?
a. Salpingitis a. Disulfiram 1022. A 39yo male presents with visual
b. PID b. Acamprosate symptoms. Ophthalmoscopy shows papilloedema.
c. Appendicitis c. Vitamin supplement Which
d. Ovarian torsion d. Naloxone anatomical site is most likely to be affected?
e. Uterine fibroid e. Naltrexone a. Optic nerve
Ans. The key is B. Acamprosate. This is a wrong b. Optic disc
Ans. The key is C. Appendicitis. [No vaginal key! Correct key is A. Disulfiram. [Acramposate is c. Optic radiation
bleeding and pain above pelvis makes pregnancy not a deterrent (which keeps away from drinking d. Occulomotor nerve
complication less likely. Loose motion, low grade by making it unpleasant) but Disulfirum!] e. Optic chiasma
temperature and positive Mc Burney’s sign makes
the diagnosis of appendicitis more probable]. 1019. A 68yo woman presents to the ED with Ans. The key is B. Optic disc.
confusion. Temp=39.3C and productive cough.
1015. A 42yo man presents with stroke. He is not Sputum is 1023. A 75yo man has been attending the clinic
able to walk straight and his speech is slurred. rusty colored after 2 days. CXR shows right lower for lower urinary tract symptoms. His mood is
What lobe consolidation. What is the most likely very low
is the initial appropriate inv? organism? and he says he feels unhappy, anxious and unable
a. CT brain a. Streptococcus pneumonia to sleep. He has been dx with moderate
b. PET brain b. Staphylococcus aureus depression. What tx would be most effective for
c. MRI brain c. Coxiella burnetti this pt?
d. Carotid angiography d. Mycoplasma pneumonia a. Amitriptyline
e. Monitor for 24h Ans. The key is A. Streptococcus pneumonia. b. Citalopram
[Typical presentation of streptococcal c. CBT
Ans. The key is C. MRI brain. [Ataxia and slurred pneumonia]. d. Dosulepin
speech are features of cerebellar lesion which is e. Diazepam
posterior fossa organ. For posterior fossa lesion 1020. A 70yo man with prostatic cancer has had
MRI is the investigation of choice! So key is severe acute back pain waking him up at night for Ans. The key is B. Citalopram. [Mild depression =
correct. It is C. MRI brain]. 6wks. What is the most appropriate inv? CBT, Moderate depression = Antidepressant,
a. MRI spine Severe depression & Psychotic depression = ECT,
1016. A 24yo woman has severe depression 3m b. Radionuclide bone scan Amitryptiline and Dosulepine causes urinary
after the birth of her first child. She is c. DEXA scan retention (which is comperatively less in
breastfeeding d. Serum ALP concentration citalopram) so not suitable in a patient with lower
but is otherwise unable to look after the baby and e. Serum calcium concentration urinary symptoms. Hence Citalopram is the
is convinced that her family is likely to kill her. Ans. The key is B. Radionuclide bone scan. [[MRI is preferred option].
She has no interest in anything and keeps crying. good for soft tissue but not for bone. If it was
What is the most appropriate tx? radiculopathy, spinal cord compression or 1024. A 48yo pt after surgical removal of
a. Fluoxetine prolapsed disc creating pressure on nerve MRI mandibular ca presents with perioral paresthesia
b. Citalopram would be fine but not for bony metastasis. Here and severe
c. CBT investigation of choice is radionuclide bone scan]. pain which is not relieved by oral morphine. What
d. ECT is the next step in treating this pt?
e. Haloperidol a. Oral amitryptiline (2nd line)
b. Oral oxycodone
c. PCA 1028. A 27yo lady after C-section developed d. Fluids via NGT
d. IV morphine epigastric pain after 8h. What is the appropriate e. Fluids PO
e. Fentanyl patch inv? Ans. The key is A. Albumin infusion. [Here serum
f. Gabapentine (1st line) a. ABG albumin is 17. So the patient developed ascites
b. Coag profile and oedema secondary to reduced plasma
Ans. The key is F. Gabapentine. [This pain is likely c. Liver enzyme colloidal osmotic pressure. Albumin infusion can
due to nerve injury during operation and both d. Liver biopsy help rise the colloidal osmotic pressure and
amitryptilin Ans. The key is C. Liver enzyme. [Epigastric pain is improve the patients condition].
a warning sign of HELLP syndrome. So to rule out
1025. A 34yo man was slapped over his right ear HELLP syndrome Liver enzymes should be done]. 1033. A 2yo with atrophy of the buttocks,
in a fight. There is blood coming from his external distended abdomen with frequent offensive
auditory canal and he has pain, deafness and 1029. A 35yo woman presents with visual smelly stool.
ringing in his ears. What is the most appropriate problems. CT brain reveals pituitary tumor. What Choose the single most likely inv?
initial inv? is the a. Upper GI endoscopy
a. CT single most likely defect? b. Endomyseal/alpha glidin antibody
b. MRI a. Homonymous hemianopia c. Sweat test
c. Otoscopy b. Homonymous upper quadrantopia d. Colonscopy
d. Skull XR c. Bitemporal hemianopia e. Stool culture
e. Facial XR d. Cortical blindness Ans. The key is B. Endomyseal/alphagliadin
Ans. The key is A CT. It is a wrong key! Correct key e. Homonymous lower quadrantopia antibody. [Atrophy of buttock due to fat
is C. Otoscopy. [Traumatic perforation of the Ans. The key is C. Bitemporal hemianopia. malabsorption and distended abdomen
tympanic membrane (TM) can cause pain, [pitutary tumour by pressing optic chiasma causes (sometimes with everted umbilicus) with frequent
bleeding, hearing loss, tinnitus, and vertigo. bitemporal hemianopia]. offensive smelly stools are seen in coeliac disease.
Diagnosis is based on otoscopy. Treatment often Hence endomyseal/alpha gliadin antibody should
is unnecessary. Antibiotics may be needed for 1030. A 45yo heroin addict was involved in a car be done].
infection. Surgery may be needed for perforations crash and is now paraplegic. During the 1st week
persisting > 2 mo, disruption of the ossicular of 1034. A 78yo woman is brought to the hospital
chain, or injuries affecting the inner ear. hospital stay he cried everyday because he complaining of back pain and is referred to the
Reference: Merck Manual (Prefessional version) couldn’t remember the accident. What is the surgeon.
[www.merckmanuals.com/professional/ear,- most She has been saying that her mother is due to visit
nose,-and-throat-disorders/middle-ear-and- likely dx? her today and that somebody must have
tympanic-membrane-disorders/traumatic- a. PTSD broken her lower back as she is in agony. Labs:
perforation-of-the-tympanic-membrane]. b. Severe depression creatinine=295mmol/l, calcium=3.03mmol/l.
c. Organic brain damage Which inv is most likely to lead to a dx?
1026. A 45yo man has developed an annular rash Ans. The key is C. organic brain damage. a. US KUB
with a scaly edge on his thigh. The rash has been b. XR Spine
spreading over the last 3wks. He has some 1031. A pt with T1DM has a fundus showing c. IVU
general aches and pains. What is the single most micro-aneurysm and hard exudate. What is the d. Bence-Jones Protein
useful investigation? single e. Mental state exam
a. ANA most likely dx? Ans. The key is D. Bence-Jones protein. [Severe
b. Biopsy lesion a. Macular degeneration back pain, high calcium level and renal
c. Lyme antibodies b. Hypertensive retinopathy impairment are typical of multiple myeloma].
d. Skin scrap for mycology c. MS
e. Skin swab for bacteria d. Diabetic background 1035. A 40yo woman presents with dysphagia.
Ans. The key is C. Lyme antibodies. [Spreading e. Proliferative DM retinopathy Exam: febrile with erythema and middle neck
annular rash suggests erythema migrans of lyme Ans. The key is D. Diabetic background swelling.
disease]. retinopathy. [Microenurism, hard exudates are What is the best management strategy?
seen in background diabetic retinopathy]. a. IV antibiotics and drainage
1027. A 80yo man with prostatic cancer has b. Antipyretics
confusion, thirst, abdominal pain and an 1032. A 62yo man has multiple liver mets due to c. XR neck
abnormal ECG. adenocarcinoma with an unknown primary. He is d. Endoscopic diverticulectomy
What is the most appropriate inv? deeply jaundiced and has ascites with edema upto e. I&D
a. MRI spine the buttocks. He is now drowsy and his family Ans. The key is A. IV antibiotics and drainage.
b. Radionuclide bone scan are worried that he is not drinking enough. His [Neck abscess. Treated with IV antibiotics and
c. DEXA scan meds include: haloperidol 1.5mg, lactulose 10ml. drainage].
d. Serum ALP concentration Bloods taken 3d ago: electrolytes normal,
e. Serum calcium concentration urea=6.5mmol/l, creatinine=89mmol/l, 1036. A young lady presents with gradually
Ans. The key is E. Serum calcium concentration. calcium=2.04mmol/l, albumin=17g/L, total worsening headaches, visual disturbance, and lack
[Confusion, thirst, abdominal pain and ECG bilirubin=189mmol/l. What is the single most of
changes (marked shortening of QT interval) are appropriate management of his fluid intake? energy. MRI shows 15mm tumor in the pituitary
features of hypercalcemia. So E. Serum calcium a. Albumin infusion fossa. What is the tx of choice?
concentration is the likely option]. b. Crystalloids IV a. Radiotherapy
c. Crystalloids SC b. Octreotide
c. Reassurance and f/u after 6m 1041. A 79yo man who is being treated with GnRH the local community mental health team
d. Surgery antagonist for proven adenocarcinoma of the (CMHT)/adolescent unit or Eating Disorder Unit
e. Chemotherapy prostate attends a follow up session. What is the (EDU) if available. So the key is A. Eating disorder
Ans. The key is D. Surgery. [Hormonically active most appropriate inv? clinic]. OHCS, 9th edition, page-349.
tumour, tumour causing pressure symptoms (like a. Serum AFP
headache, visual disturbance) and large size of b. Serum PSA 1045. A 36yo woman has an injury to the right
tumour are indications for surgical removal]. c. Serum acid phosphates conc external laryngeal nerve during a thyroid surgery.
d. Serum ALP isoenzyme conc What
1037. A man with dementia has an ulcerative e. Trans rectal US symptom would be expected in this pt?
lesion on his forehead. He wants it removed so ‘it Ans. The key is B. Serum PSA. [Serum PSA is used a. Stridor
can to assess the progress of treatment. (If a man’s b. Hoarseness
help improve his memory’. Wife says he is not fit PSA level rises after prostate cancer treatment, c. Aphonia
to give consent. What will you do? his doctor will consider a number of factors d. Dysphonia
a. Get letter signed from the GP before recommending further treatment. e. Aphasia
b. Get letter signed from the wife Additional treatment based on a single PSA test is Ans. The key is D. Dysphonia. [External laryngeal
c. Get letter signed from the pt not recommended. Instead, a rising trend in PSA nerve (smaller, external branch of the superior
d. Refer to psychiatrist to assess the mental level over time in combination with other laryngeal nerve) lesion causes mono toned voice
capacity to give consent findings, such as an abnormal result on imaging (loss of ability to produce pitched sound) that is
Ans. The key is D. Refer to psychiatrist to assess tests, may lead a man’s doctor to recommend dysphonia].
the mental capacity to give consent further treatment)].
1042. A middle aged woman has some weakness 1046. A 75yo woman has weakness of the left side
1038. A pt with flame shaped hemorrhage on long of hand after an injury. Which vertebra will be the of her face. She has had a painful ear for 48h.
term tx with nifedipine. What is the single most lowest to be included on cervical XR to dx the There
likely dx? injury? are pustules in the left ear canal and on the
a. Macular degeneration a. C7/T1 eardrum. What is the single most likely dx?
b. HTN retinopathy b. C8/T1 a. Chronic serous OM
c. MS c. C5/C6 b. Herpes zoster infection
d. DM background d. C6/C7 c. Impacted earwax
e. Proliferative DM retinopathy Ans. The key is A. C7/T1. d. Perforation of eardrum
f. SLE 1043. A 50yo man with a known hx of stroke. He e. Presbycusis
Ans. The key is B. HTN retinopathy. [Stages of can’t remember anything about his life. What is Ans. The key is B. Herpes zoster infection. [A case
hypertensive retinopathy: the of Ramsay Hunt syndrome defined as an acute
1. Grade 1: Silver wiring single most likely defect? peripheral facial neuropathy associated with
2. Grade 2: 1 + AV nipping a. Homonymous hemianopia erythematous vesicular rash of the skin of the ear
3. Grade 3: 2 + flame shaped b. Homonymous upper quadrantanopia canal, auricle (also termed herpes zoster oticus),
hemorrhage c. Bitemporal hemianopia and/or mucous membrane of the oropharynx
4. Grade 4: 3 + optic disc edema d. Binasal hemianopia caused by Herpes zoster infection].
+ macular star]. e. Homonymous lower quadrantanopia
Ans. The key is B. Homonymous upper 1047. An 8wk baby boy is noted to be jaundiced.
1039. A pt whose pain is not relieved by oral quadrantanopia. [Memory processing occur in He is breast-feeding well and has gained 300g
codeine. What is the best management? temporal lobe. So as the patient lost memory his since
a. Oral oxycodone temporal lobe is involved. We know temporal birth. His stools are yellow and his urine is pale
b. Co-codamol lobe lesion causes “homonymous upper straw colored. What is the most likely dx?
c. PCA quadrantanopia” which is the key here!] a. Galactosemia
d. IV morphine b. Biliary atresia
e. Oral morphine 1044. An 18yo girl has been dx with anorexia c. G6PD deficiency
Ans. The key is E. Oral morphine. nervosa and has mild depressive symptoms. She d. Breast milk jaundice
has cut e. Congenital viral infection
1040. A 6wk baby with vomiting, irritability and down her food intake for the last 18m and Ans. The key is A. Galactosemia. [Biliary atresia
palpable mass in the abdomen on feeding. Choose exercises 2h everyday. Her BMI=15.5, causes obstructive picture where stools are pale
thesingle most likely inv? BP=90/60mmHg. What would be the single most and urine becomes dark. Hemolytic disorder like
appropriate management? G6PD or spherocytosis causes appearance of
a. Upper GI endoscopy a. Refer to eating disorder clinic jaundice in 1st24 hours. In breastmilk jaundice,
b. Barium meal b. Refer to psychodynamic therapy jaundice develops in 2nd week. Also congenital
c. US c. Refer to acute medical team viral infection (TORCH) occurs in 1st 24 hours. Here
d. CT abdomen d. Prescribe antidepressant there is failure to thrive also. So likely cause in the
e. Barium enema Ans. The key is C. Refer to acute medical team. presented case is Galactosemia].
Ans. The key is C. US. [Palpable mass in the This is a wrong key! Correct key is A. Eating
abdomen “ON FEEDING” (just after feeding the disorder unit. [Anorexia nervosa is moderate 1048. A lady developed breast abscess after
mass becomes more prominent and can be when BMI is 15-17.5 as in given case. In moderate delivery. What is the most likely organism?
palpated more easily) is suggestive of pyloric anorexia (BMI 15–17.5, no evidence of system a. Staph aureus
stenosis. US is used to diagnose this]. failure) routine referral can be to b. Staph albus
c. GBS
d. Strep pyogenes e. Trigeminal neuralgia Ans. The key is B. GAD. [In GAD patient is worried
e. Strep faecalis about different number of events every day.
Ans. The key is A. Staphylococcus aureus. Ans. D. Temporomandibular joint pain. Almost everything triggers the anxiety].
[Temporomandibular joint pain may cause pain in
1049. A 32yo man suffering from MS presents ear and teeth grinding is a recognized cause of 1056. Which of the following is not a degenerative
with blurring of vision. Ophthalmoscopy shows this symptom]. corneal disease?
pallor of a. Band keratopathy
the optic disc. Which anatomical site is most likely 1053. A 42yo lady had corrective surgery for b. Marginal dystropathy
to be affected? cyanotic congenital heart disease at the age of 3y, c. Fatty/lipid degeneration
a. Optic nerve after a d. Mooren’s ulcer
b. Optic disc palliative operation during infancy. There is a e. Keratoconus
c. Optic radiation parasternal impulse and an early diastolic Ans. The key is D. Moorens ulcer. [All other
d. Trigeminal murmur. options are degenrative disease of cornea except
e. Oculomotor nerve What is the most probable dx? Mooren’s ulcer which is a type of ulcerative
Ans. The key is A. Optic nerve. [ MS--optic a. Aortic regurgitation keratitis].
neuropathy which affects optic nerve and long b. Ischemic mitral regurgitation
standing optic neuropathy can be seen as pallor of c. Aortic stenosis 1057. A 30yo man presents to hosp complaining
optic disc in opthalmoscope]. d. Pulmonary stenosis that his urine has been very dark recently,
e. Pulmonary regurgitation resembling
1050. A 23yo man presents with severe pain in coffee at worst. He has been under the weather
the right flank radiating to his groin. He is rolling Ans. The key is E. Pulmonary regurgitation. [In the 2wks back and had taken a few days off work
about present day, some patients with tetralogy of with a sore throat and coryzal symptoms. Urine
on the floor. An IVU confirms a stone in the ureter Fallot have survived for longer than 15-20 years dipstick in hosp returns highly positive for blood
which is 8mm in size. Which tx modality will after their first operation. The major problem and protein. He is admitted for supportive
be most effective? encountered by these individuals is the management and is scheduled for a renal biopsy,
a. Fluids and alpha blockers development of pulmonary valvular regurgitation. which shows mesangial proliferation with a
b. ESWL It appears that a number of these individuals positive immune-flurescence pattern. What is the
c. CCB require pulmonary valve replacement (ref: most probable dx?
d. Dormier basket Medscape)]. a. Membranous glomerulonephropathy
e. PCNL b. SLE
Ans. The key is B. ESWL. [Though for 8 mm stone 1054. A 45yo lady presents with hx of double c. Wegener’s granulomatosis
we can use medical expulsive therapy but for this vision and facial numbness. Which anatomical site d. Post – strep GN
patient with agonizing pain “most effective” is e. IgA nephropathy
therapy seems to be ESWL]. most likely to be affected?
a. Cerebral cortex Ans. The key is D. Post- strep GN. [Nephritic
1051. A 37yo woman believes that her neighbours b. Trigeminal nerve picture, H/O sorethroat 2 weeks before
have been using her shower while she is away c. Oculomotor nerve presentation and biopsy reports are suggestive of
from d. Brain stem post streptococcal glomerulonephritis].
home. Her 42yo partner is convinced about this e. Basal ganglia
and calls the police. What term best describes Ans. The key is D. Brain stem. [Cerebral cortex will 1058. A 65yo lady presents with a 6h hx of facial
this situ? not cause features of cranial nerve lesions. droop and weakness in the left side of her body.
a. Capgras syndrome Trigeminal nerve: ophthalmic and maxillary What single agent will she be prescribed for her
b. Cotard syndrome divisions are pure sensory hence it is also not the whole life?
c. Delusion of persecution answer. Oculomotor does not cause facial a. Clopidogrel
d. Folie a deux numbness. These features are not of basal ganglia b. Altepase
e. Munchausen syndrome lesion also. These features only can be explained c. Aspirin
Ans. The key is D. Folie a deux. [Folie a deux is by cranial nerve involvement in brainstem lesion]. d. Labetalol
symptoms of a delusional belief
and hallucinations that are transmitted from one 1055. A 30yo woman has experienced Ans. The key is A. Clopidogrel. [Ongoing
individual to another as here from wife to restlessness, muscle tension and sleep symptoms are considered as stroke (unless
husband]. disturbance on most completely resolved within 24 hours). Hence
days over the last 6m. She worries excessively Clopidogrel is the answer here as they want life
1052. A 45yo woman has dull pain in her right ear about a number of everyday events and activities long!].
which has been present for several weeks. There and is unable to control these feelings which are
is impairing her ability to hold down her job. 1059. A 10yo boy is brought to the hosp with a
no discharge. Chewing is uncomfortable and her What is the most likely dx? rash over his buttocks a/w abdominal pain and
husband has noticed that she grinds her teeth a. Panic disorder vomiting. In the ED, he is accompanied by his
during sleep. The eardrum appears normal. What b. GAD mother and stepfather. His mother had left him
is the single most likely dx? c. Pheochromocytoma for the weekend with the stepfather and was
a. Dental caries d. Acute stress disorder called to come back from holiday as he started to
b. Mumps e. Social phobia have some hematuria with the rash. Social
c. OM services had been notified on arrive to hospital.
d. Temporomandibular joint pain What
is the most probably dx? d. Tramadol 1067. A 45yo woman has recently been dx with
a. NAI MS and has been started on oral steroids. She is
b. ITP Ans. The key is D. Tramadol. brought to the hosp after having ingested 100
c. HSP paracetamol tablets 4h ago. She is refusing all
d. ALL 1064. A 62yo farmer presents with a persistent med tx. What is the next best step?
e. HUS firm irregular lesion on upper part of pinna which a. Observe
grew b. Refer to psychiatrist to assess pts ability to
Ans. The key is C. HSP. [Rash over buttock, over the last few months. What is the most refuse tx
abdominal pain and vomiting, blood in urine or appropriate dx? c. Gastric lavage
stool suggest HSP]. a. Basal cell d. Activated charcoal
b. Squamous cell e. Refer to social worker
1060. A man with hx of fall had confusion and c. Keratocanthoma
laceration mark on the head. Which is the most Ans. The key is B. Refer to psychiatrist to assess
appropriate vessel affected? Ans. The key is A. Basal cell. [Any ulcer which is pt’s ability to refuse tx.
a. Basiliary artery located above the neck is always basal cell
b. Middle meningeal artery carcinoma until proven otherwise. (samson 1068. A 44yo obese pt with findings:
c. Vertebral artery note)]. FBS=6mmol/l, OGTT=10mmol/l. What is the most
d. Diploic vein likely dx?
1065. A 24yo schizophrenic has been under a. Impaired glucose tolerance
Ans. The key is B. Middle meningeal artery. antipsychotic tx for the last 1 yr and now b. Diabetes insipidus
[Considerable head injury and lucid interval complains of ED Erectyl Dysfunction).Which drug c. T1DM
(several episodes of transient confusion)points is most likely to have caused this? d. T2DM
towards extradural hematoma which occurs a. Fluoxetine e. MODY
frequently from bleeding from middle meningeal b. Citalopram
artery. Subdural hematoma usually occur due to c. Clozapine Ans. The key is A. Impaired glucose tolerance.
bleeding vein]. d. Haloperidol [Impaired glucose tolerance is 2-hours glucose
e. Risperidone level 140-199 mg/dl (7.8 to 11.0 mmol/l) on the
1061. A 72yo lady is drowsy and her relatives 75-g oral glucose tolerance test].
want to take her home. She has been prescribed Ans. The key is D. Haloperidol. This is a wrong key.
diazepam 2.5mg. What is the best delivery route? Correct key is E. Risperidone. [Haloperidol, raises 1069. A child distressed with fever, stridor and
a. Oral serum prolactin levels to 20~40 ng/ml in unable to swallow saliva. His RR=40bpm. What is
b. IV therapeutic doses but risperidone causes around the
c. IM 171 ng/ml]. initial step that needs to be taken?
d. Per rectal a. Examine throat
e. SC 1066. What is the most likely dx based on this ECG b. Secure airway
c. Keep him laid flat
Ans. The key is D. Per rectal. d. IV penicillin

1062. A nonsmoker who has worked in coal mines Ans. The key is B. Secure airway. [Probable
for 20yrs presents with gradually increasing SOB, epiglottitis. So airway should be secured].
limited exercise tolerance and a dry cough. His
CXR shows round fibrotic tissue demonstrating a 1070. A pt presents with hemoptysis 7d after
mixed restrictive and obstructive ventilator defect tonsillectomy and adenoidectomy. What is the
with irreversible airflow limitation and next step
reduced gas transfer. What is the single most of management?
appropriate dx? a. Explore again
a. Churg-strauss syndrome b. Pack it
b. Cryptogenic organizing c. Oral antibiotics and discharge
c. Extrinsic allergic alveolitis d. Admit and IV antibiotics
d. Good pasture’s syndrome e. Ice cream and cold fluid
e. Progressive massive fibrosis
f. Molluscum Ans. The key is D. Admit and IV antibiotics.
[secondary hemorrhage can occur due to
Ans. E. Progressive massive fibrosis. [Coal miners infection causing sloughing out of part of arterial
pneumoconiosis]. wall due to infection. It is managed by iv
a. Normal antibiotics folloing admitting the patient to
1063. A pt was complaining of pain within 6h after b. VT hspital].
his appendectomy for gangrenous appendix. c. Sinus Tachycardia
What d. WPW syndrome 1071. A 55yo man presents with swelling at the
med is the best option for his pain relief? e. A-fib angle of the mandible which is progressively
a. IV morphine increasing
b. Diclofenac per rectal Ans. The key is C. Sinus Tachycardia. in size and it’s mobile for 6m. What is the most
c. PCA probable dx?
a. Benign parotid 1075. A 54yo lady presents with sudden severe
b. Mandible tumor pain in the left half of her skull. She also 1079. A 65yo man presents with dyspnea 3d after
c. Tonsillar carcinoma complains of an MI. On auscultation he has a pansystolic
pain around her jaw. What is the most likely next murmurat the apex radiating to the axilla. What is
Ans. The key is A. Benign parotid. [Mandibular step? the most likely dx?
tumour or tonsillar carcinoma would not be so a. CT a. Ruptured papillary muscle
mobile after such time period and Benig parotid is b. MRI b. Ventricular aneurysm
likely diagnosis]. c. Fundoscopy c. Pericarditis
d. ESR d. Pericardial effusion
1072. A 61yo man, known smoker, comes to the e. Temporal artery biopsy e. VSD
hospital with complaints of painless hematuria,
urgency and dysuria. He has been worried about Ans. The key is D. ESR. [Elderly lady with severe Ans. The key is A. Ruptured papillary muscle.
his loss of weight and reduced general activity. pain in the left half of her skull and pain around [Pansystolic murmur in apex radiating to axilla
Which inv would be diagnostic of his condition? jaw is suspicious of Giant cell or Temporal may be due to mitral regurgitation secondary to
a. Urine microscopy arteritis. ESR if markly raised supports the papillary muscle rupture].
b. IVU diagnosis of temporal arteritis].
c. CT 1080. A 64yo man with multiple myeloma has
d. Cystoscopy 1076. A 7yo school boy has been dx with been vomiting since the past 2days. Labs:
e. US abdomen meningococcal meningitis. What is the advice for Ca2+=3.2mmol/l, K+=5mmol/l, Na+=149mmol/l
f. KUB schoolmates and staff? and PCV=55%. What is the most appropriate
g. Cystoscopy with biopsy a. Rifampicin for the whole class and family next step?
h. Mid stream urine for culture b. Rifampicin for the whole school and family a. IV insulin
i. Trans rectal US c. Meningococcal vaccine for the family b. IV calcium gluconate
d. Benzylpenicillin c. IV fluids
Ans. The key is G. Cystoscopy with biopsy. e. IV cefotaxime d. IV bisphosphonates
[Painless hematuria in elderly man is due to Ca of e. Oral bisphosphonates
bladder usually and in bladder cancer there are Ans. The key is A. Rifampicin for the whole class
UTI like symptoms like urgency, dysuria. Also and family. [In contacts of meningococcal Ans. The key is C. IV fluids. [Multiple myeloma
constitutional symptom like weight loss and meningitis chemoprophylaxis is given with itself is a cause of vomiting and also associated
reduced genral activity is feature of carinoma]. Rifampicin]. hypercalcemia can cause sickness. As the patient
is vomiting for 2 days there may be considerable
1073. An 8wk pregnant lady is brough to the ED 1077. A pt came with dyskaryosis to the OPD. She dehydration and also the hypercalcemia needs
due to severe vomiting. She was administered IV is a heavy smoker and alcoholic. Cervical smear treatment with IV fluid].
fluids and oral anti-emetics. She still can’t shows abnormal cells. What is the best advice for
tolderate anything orally. What is the next best her? 1081. A 30yo man from Australia returned from a
tx? a. Colposcopy business trip to Indonesia 6d ago presenting with
a. IV feeding b. Biopsy complaints of fever, joint and muscle ache and
b. IV antiemetics c. Endocervical sample headache, in particular behind the eye for the
c. Termination of pregnancy d. Repeat after 4m past 2 days. What is the most probable dx?
d. PPI e. None a. Malaria
e. IV steroid f. Cone biopsy b. Chicken pox
c. TB
Ans. The key is IV antiemetics. [If oral antiemetics Ans. The key is A. Colposcopy. [Patient with d. Lyme’s disease
are not tolerated IV antiemetics are tried]. dyscariosis should go for colposcopy]. e. Dengue

1074. A 48yo man presents with bone pain. Labs: 1078. Pt with pain and swelling in left leg and Ans. The key is E. Dengue. [Fever, arthralgia,
ALP=high, phosphate=normal. What is the most thigh up to the level of inguinal ligament. Where myalgia, headache these are common in dengue
likely dx? is the fever. Particularly retro-orbital pain is well
a. Osteoporosis level of occlusion? recognized feature of dengue fever].
b. Osteomalacia a. Femoro-popliteal artery
c. Paget’s disease b. Left common iliac artery 1082. A lady came for OBGYN assessment unit
d. Fx c. Aortoiliac artery with hx of 8wk pregnancy and bleeding per vagina
e. Myeloma d. Femoral artery for
e. Profound femoral artery last 2 days. On bimanual exam, uterus =8wks in
Ans. The key is C. Paget’s disease. [Bone pain, high size. On speculum exam, cervical os is closed.
ALP and normal phosphate are suggestive of Ans. There are two keys B. Left common iliac How do you confirm the viability of the fetus?
pagets disease. In osteoporosis Ca+, PO4³- and artery. and D. Femoral artery. Correct key is B. a. Transvaginal US
ALP are normal. In osteomalacia Ca+ low, PO4³- Left common iliac artery. [General rule is b. Serum BHCG
low and ALP is high. In Pagets disease Ca+ and occlusion occurs in proximal bifarcation. So the c. Urinary BHCG
PO4³- normal and ALP markedly raised]. correct key is B. Left common iliac artery. That is d. Abdominal US
occlusion in the bifarcation of left common iliac e. Per speculum exam
artery (as femoral artery bifarcates more distally
to superficial and deep branches)].
Ans. The key is A. Transvaginal US. [Viability of the hypokalemia, hypomagnesemia and next step?
fetus is better assessed with transvaginal US. TVU hypocalcemia]. a. TFT
can be done throught pregnancy. It should be b. Tensilon test
avoided once the membrane is ruptured to avoid 1087. A 70yo hypertensive white british man on c. Fundoscopy
chance of fetal infection]. thiazide diuretics needs a 2nd drug to control his d. Autoantibodies
BP. e. EMG
1083. A 24yo lady has been low after the death of Which one of the following is the best choice for
her husband and had stopped contacting her him? Ans. The key is A. TFT. [Features are suggestive of
family. a. Amlodipine (CCB) Grave’s disease hence TFT].
She was started on SSRI tx and starts feeling b. Enapril (ACEi)
better after a few months. On discontinuating the c. Propranolol (BB) 1091. A 41yo man presents with longstanding foul
meds she starts feeling that she has developed d. Increase dose of diuretic smelling ear discharge and progressive hearing
cancer just like her husband. What is the most e. Prazocin (Alpha blocker) loss.
appropriate next step? Otoscopy showed perforation of the pars flacida
a. Continue SSRI Ans. The key is B. Enalapril. [In patients >55 yrs and a mass in the upper part of the middle ear.
b. Add TCA CCB. If CCB not suitable (oedema, heart failure) What is the most likely dx?
c. Neuropsychiatric analysis thiazide is preferred. In patients <55yrs ACEI or a. ASOM
d. CBT ARB. Now if patient is on CCB or Thiazide 2nd drug b. CSOM
e. Antipsychotics should be added it should be one of ACEI or ARB c. Acquired cholesteatoma
and not one from CCB or Thiazide]. d. Congenital cholesteatoma
Ans. The key is C. Neuropsychiatric analysis. e. Barotrauma
1088. A 74yo lady who has had a stroke in the
1084. A 24yo male who is sexually active with past has an indwelling catheter for 10m. She Ans. The key is C. Acquired cholesteatoma. [In
other males with hx of discharge per urethra. Dx presents congenital cholesteatoma there is usually no
of with bluish-purple discoloration of the catheter perforation. In acquired cholesteatoma
chlamydia has been made. What is the possible bag. What is the most likely explanation for this? perforation is in pars flaccida].
complication if left untreated? a. Normal change
a. Orchitis b. Catheter degradation 1092. A 9yo child presented with a rash on his
b. Balanitis c. Acidic urine skin which didn’t respond to antibacterial
c. Epididymo-orchitis d. Alkaline urine ointment.
d. Acute abdomen e. Bacterial colonization of the urinary tract What med should be added next?
a. Corticosteroid
Ans. The key is C. Epididymo-orchitis. Ans. The key is E. Bacterial colonization of the b. Antifungal
urinary tract. [Purple Urine Bag Syndrome]. c. Emollient
1085. A person doesn’t go outside the home d. Permethrin
because he thinks that people will look at him and 1089. A 62yo man has slow palpitations and the e. Coal tar
talk following ECG. What is the most likely dx?
about him. He finds it difficult to associate with Ans. The key is B. Antifungal. [If a lesion is
his peers in a restaurant or under social settings. suspected to be infected should be given
What is the most likely dx? antibiotic. but if no response next step is to add
a. Agoraphobia an antifungal].
b. GAD
c. Panic disorder 1093. A young boy has a hx of epistaxis.
d. Adjustment disorder CBC=normal, except APTT=47s. What is the most
e. Social phobia likely dx?
a. Hemophilia
Ans. The key is E. Social phobia. b. ITP
c. Sickle cell
1086. A 63yo man presented with sudden onset d. HUS
of severe dyspnea, orthopnea, raised JVP and e. Thalassaemia
bilateral
a. Sinus bradycardia
basal crackles 3d after an episode of MI. A dx of Ans. The key is A. Hemophilia. [A young boy with
b. 1st degree heart block
acute congestive cardiac failure was made and epistaxis and prolonged APTT is a probable case of
c. Mobitz type 1 block
IV furosemide was started for this pt. What hemophilia. Factor VIII and IX should be offerred
d. Mobitz type 2 block
electrolyte abnormality is expected? to confirm the diagnosis].
e. Complete heart block
a. High Na+, Low K+
b. Low Na+, High K+ 1094. A 29yo young man presents with
Ans. The key is C. Mobitz type 1 block. [Gradual
c. Low Na+, Low K+ complaints of recurrent attacks of diarrhea. He
prolongation of PR interval followed by a drop
d. High Na+, High K+ says his stools
beat].
e. Low Na+, Normal K+ contain blood and mucos. Sometimes he has low
grade fever. What is the most appropriate inv
1090. A 29yo woman presents with lid lag, lid
Ans. The key is C. Low Na+, Low K+. [Frusemide for his condition?
retraction and diplopia. What is the most
causes hyponatremia, hypochloremic alkalosis, a. Stool culture
appropriate
b. Plain abdominal XR the single most imp immediate management of 1101. A lady comes with a missing IUCD thread.
c. Per rectal exam this pt? Her LMP was 2wks ago. What is the single most
d. Barium enema a. IV MgSO4 appropriate next step in management?
b. IV diazepam a. Abdominal US
Ans. The key is D. Barium enema. [Features are c. Immediate C-section b. Prescribe contraceptives
suggestive of IBD, so barium enema is the most d. IV phenytoin c. CT
relevant investigation among the given options]. e. MgSO4 bolus d. Serum BHCG
f. IV lorezepam e. Vaginal exam
1095. A 26yo young man presents with hx of Ans. The key is A. Abdominal US. It is wrong key!
passing loose stools for the past 2m. He says his Ans. The key is A. IV MgSO4. It is a wrong key! Correct key is E. Vaginal exam. [LOST THREAD:
stools Correct key is E. MgSO4 bolus. [Treat a first First confirm the thread is not in position by
contain blood and mucus and are a/w abdominal seizure with 4g magnesium sulfate in 100mL 0.9% vaginal examination. If not present ultrasound
pain. He undergoes a colonscopy after which saline IVI should be arranged to locate the device.
he was started on tx. What is the most over 5min + maintenance IVI of 1g/h for 24h. If ultrasound does not locate the device and there
appropriate tx for his condition? Beware  respiration. is no definite history of expulsion then abdominal
a. Mesalazine  If recurrent seizure give 2g IVI magnesium X-ray should be performed to look for an
b. Corticosteroids sulfate over 5 min. extrauterine device.
c. Infliximab  Check tendon reflexes and respiratory rate Expulsion should not otherwise be assumed.
d. Cyclosporine every 15min. Hysteroscopy can be helpful if ultrasound is
 Stop magnesium sulfate IVI if respiratory rate equivocal.
Ans. The key is A. Mesalazine. [Features and tx <14/min or tendon reflex Surgical retrieval of an extrauterine device is
after colonoscopy suggests dx of ulcerative colitis loss, or urine output <20mL/h. Have IV calcium advised].
for which initial treatment option is mesalazine]. gluconate ready in case of
MgSO4 toxicity: 1g (10mL) over 10 min if 1102. A 32yo woman presents with hx of lower
1096. A 52 yo male with poorly controlled DM has respiratory depression. abdominal pain and vaginal discharge. She had
now presented to his GP with pain in the ear.  Use diazepam once if fits continue (eg 5–10mg her
Exam:skin around the ear is black in color and slowly IV). If seizures continue, menses 4wk ago. She has a temp of 38.6C. What
there was foul smelling discharge from the ear. Pt ventilate and consider other causes (consider CT is the most suitable dx?
also scan). OHCS, 9th edition, page-49]. a. Acute appendicitis
had conductive hearing loss. What is the most b. Acute PID
probable dx? 1099. A 24yo lady with BMI=30 complains of facial c. Endometriosis
a. Carbuncle hair growth and hx of amenorrhea. FSH=10.9, d. Ectopic pregnancy
b. Folliculitis prolactin=400IU, estradiol=177.8mmol/l, e. UTI
c. Malignant OE progesterone=normal, LH=33.2. What is the most
d. Cholesteatoma probable dx? Ans. The key is B. Acute PID. [Fever, lower
e. Furuncle a. PCOS abdominal pain and vaginal discharge are features
b. Pregnancy of PID].
Ans. The key is C. Malignant OE. [In some cases, c. Cushing’s disease
otitis externa can spread to the outer ear and d. CAH 1103. A 40yo female was on COCP which she
surrounding tissue, including the bones of the jaw e. POF stopped 6m ago. But she has not had her periods
and face. This infection is known as malignant since
otitis externa. Thogh the term malignant is used it Ans. The key is A. PCOS. [Clinical features (obesity, then. Labs: FSH=22, LH=24, prolactin=700,
is not a cancer]. hirsutism, amenorrhea etc.)are consistent with estradiol=80. What is the most appropriate dx?
PCOS. We know in PCOS LH:FSH ratio is 2:1 or 3:1. a. Hypothalamic amenorrhea
1097. A 55yo male has been admitted for elective Here LH is 33.2 and FSH is 10.9 which also b. Post pill amenorrhea
herniorraphy. Which among the following can be supports diagnosis of PCOS]. c. Prolactinoma
the reason to delay his surgery? d. Pregnancy
a. Controlled asthma 1100. A 17yo girl with a lump in her breast was e. Premature ovarian failure
b. Controlled A-fib seen in the clinic. Exam: the lump was free and
c. DVT 2yrs ago mobile Ans. The key is E. Premature ovarian failure. [ FSH
d. DBP 90mmHg and not attached to the skin. Her mother wants and LH are raised in ovarian failure; an FSH level
e. MI 2m ago further tests done. What should be the next ≥20 IU/l in a woman aged around 40 with
step? secondary amenorrhoea indicates ovarian failure.
Ans. The key is E. MI 2m ago. [Following MI at a. CT In POF there is also a mild to moderate rise in
least 6 months should be elapsed for elective b. US breast prolactin].
operation. Prior this time surgery is associated c. Punch biopsy
with increased mortality]. d. Reassure and send home 1104. A 25yo woman presents with a single lump
e. Stereotactic biopsy in the breast and axilla. The lump is mobile and
1098. A 21yo female in her first pregnancy at hard
38wks was brought to the ED with generalized Ans. The key is B. US breast. [Described lump is in consistency. The US, mammogram and FNA
tonic fibroadenoma. Next investigation is either US turn out to be normal. What is the most
clonic seizure. IV MgSO4 was given but fits was breast or mammography]. appropriate inv to confirm the dx?
not controlled. She is having fits again. What is a. FNAC
b. MRI b. Hematoma Ans. The key is B. Take swab. This is a wrong key!
c. Punch biopsy c. Fibroids Correct key is A. Repeat smear in 6 months. [Ref:
d. Genetic testing and counselling d. Adenomyosis Samson notes].
e. Core biopsy e. Incomplete abortion
Ans. The key is C. Fibroids. [Regular heavy 1113. A 37yo infertile lady with 5cm subserosal
Ans. The key is E. Core biopsy. [The lump is menstruation in a uterus of 14 wk size is highly and 3cm submucosal fibroid is trying to get
suspicious but US, mammogram and FNA turn out suggestive of fibroid]. pregnant.
to be normal. So most appropriate investigation Which is the most suitable option?
to confirm diagnosis is core biopsy. It is often the 1109. A 29yo at 38wks GA presents with a 2h hx a. Clomifen therapy
preferred biopsy method because it is accurate of constant abdominal pain. She then passes b. IVF
and does not involve surgery]. 100ml of c. Myomectomy
blood per vagina. What is the next appropriate d. Hysterectomy
*1105. A 37yo lady stopped taking COCP 18m ago inv? e. IU insemination
and she had amenorrhea for 12m duration. Labs: a. USS Ans. The key is C. Myomectomy. [Fibroids are
FSH=8, LH=7, prolactin=400, estradiol=500. What b. CTG preventing from her being pregnant. So
is the cause? c. Clotting screen myomectomy is the most suitable option to let
a. Hypothalamic amenorrhea d. Hgb her get pregnant].
b. PCOS e. Kleihauer Betke test
c. Prolactinoma Ans. The key is A. USS. [Ultrasonogram scan can 1114. A young tall man and his wife are trying for
d. Post pill amenorrhea detect placental abrubtion and fetal heart beat as babies and present at the infertility clinic. On inv
e. POF well. So it is the preferred key over CTG]. the
Ans. The key is D. Post pill amenorrhea. [Post pill man has primary infertilitiy and azoopermia. What
amenorrhea= failure to resume menses within 6 1110. A 26yo woman had amenorrhea for 10wks other inv should be done?
months of stopping pill, but she was amenorrheic and is pregnant. She experiences hyperemesis. a. Testosterone
for 12 months!!]. Now b. LSH
she presents with vaginal bleed. Exam: c. FSH
1106. A lady with a firm smooth breast lump in uterus=16wks, closed os. What is the most d. Estradiol
outer quadrant had a FNAC done. Results showed probable dx? e. Karyotyping
borderline benign changes. She also has a fam hx a. Thyrotoxicosis Ans. The key is E. Karyotyping. [The likely
of breast cancer. What is the your next? b. Hyperemesis gravidarum diagnosis is Klinefelter’s syndrome for which
a. Mammography c. Twins Karyotyping should be done to make the diagnosis
b. US d. Wrong dates established].
c. Core biopsy e. Molar pregnancy
d. Genetic testing and counselling Ans. The key is E. Molar pregnancy. [In 1115. A woman who is on regular COCP presented
e. Punch biopsy hytaditidiform mole uterus becomes more in size to you for advice on what to do as she has to now
Ans. The key is D. Genetic testing and counselling. than actual gestational age and due to production start to take a course of 7d antibiotics. What
Probably wrong key! Correct one should be C. of large amount of gonadotrophin by moles would you advice?
Core biopsy. [Genetic testing is done for patient suffers from severe vomiting i.e. a. Continue regular COC
asymptomatic patients in their 30s and if positive hyperemesis. There also occurs vaginal bleeding]. b. Continue COCP and backup contraception using
SERMs are given which cause significant reduction condoms for 2d
in cancer . 1111. A pregnant woman of G2, GA 11wks c. Continue COCP and backup contraception using
Since patient has firm breast lump in upper outer presents with heavy vomiting, headache and condoms for 7d
quadrant ( most common location for cancer) and reduced urine d. Continue COCP and backup contraception using
FNAC has shown borderline benign changes plus output. Urine analysis shows ketonuria. Choose condoms for 2wks
patient is high risk for cancer , so I must go for the next best step? Ans. The key is D. Continue COCP and backup
core biopsy to confirm whether it is cancer or a. US contraception using condoms for 2wks. This is
not]. b. Oral fluid replacement wrong key! Correct key should be A. Continue
1107. A pt presents with mild dyskaryosis. 1y ago c. Serum BHCG regular COCP. [Before it was thought that
smear was normal. What is the most appropriate d. Parental anti-emetics antibiotics alter the gut flora and ethinyloestradiol
next step? e. IV fluids is not conjugated. There is more ethinyloestradiol
a. Cauterization Ans. The key is E. IV fluids. [Hyperemesis passed in the stool. So pregnancy and
b. Repeat smear gravidarum with oliguria and ketonuria. IV fluid breakthrough bleeding can occur. But later it was
c. Swab and culture should be started to prevent renal failure]. established that except for strong enzyme inducer
d. Cone biopsy like Rifampicin and Rifabutin other antibiotics
e. Colposcopy 1112. A pt had inflammatory changes on cervical practically does not reduce potency of COCP and
Ans. The key is E. Colposcopy. [In mild dyscariosis smear. There is no vaginal discharge, no pelvic hence except only rifampicin or rifabutin for other
colposcopy should be done with HPV test]. pain antibiotics no additional precautions are needed]. 
and no fever. What is the next step?
1108. An African lady presents with heavy but a. Repeat smear in 6m 1116. A lady presents with hot flashes and other
regular periods. Her uterine size correlates to b. Take swab symptoms of menopause. What is the tx option?
14wks c. Treat with antibiotics a. Raloxifen
pregnancy. What is the most appropriate dx? d. Colposcopy b. HRT
a. Blood dyscrasia e. Cone biopsy c. Bisphosphonate
d. COCP can only be done if it is due or overdue, not in any Ans. The key is B. DIC. [It is not HELLP syndrome
e. Topical estrogen time or not as needed basis]. as liver enzyme is not elevated. Acute abdominal
Ans. The key is B. HRT pain may indicate concealed abruption placenta
1121. A 20yo woman has had abdominal pain in which may lead to DIC].
1117. A 28yo woman at 34wks GA for her first the LIF for 6wks duration. Over the past 48h, she 1125. A 36wk pregnant woman presents with
pregnancy attends antenatal clinic. Her blood has sudden onset of uterine pain and bleeding, uterus
results: severe abdominal pain and has a fever of 39.1C. is
Hgb=10.6, MCV=95, MCHC=350. What do you do Pelvic US shows a complex cystic 7cm mass in tender, no prv LSCS. What is the most appropriate
for her? the LIF. What is the most likely dx? cause?
a. Folate a. Endometriosis a. Preeclampsia
b. Dextran b. Dermoid cyst b. DIC
c. Ferrous sulphate c. Ovarian ca c. Placental abruption
d. None d. Tubo-ovarian abscess d. Placental previa
e. IV FeSO4 e. Ectopic pregnancy e. Ectopic pregnancy
f. Explain this physiologic Ans. The key is D. Tubo-ovarian abscess. f. Missed abortion
hemodynamic anemia g. Ectropion
g. Blood transfusion 1122. A woman is 16wk pregnant and she is Ans. The key is C. Placental abruption. [Painful
Ans. The key is F. Explain this physiologic worried about abnormal chromosomal anomaly in bleeding at 36th week is placental abruption
hemodynamic anemia. [According to NICE, cut her (either revealed or mixed type)].
offs for iron supplements: child. What is the definitive inv at this stage?
at booking (8-10 weeks)- if less than 11  a. Amniocentesis 1126. A 28wk pregnant woman presents with
at 28 weeks and further- if less than 10.5 b. CVS (Chorionic Villous Sampling) uterine bleeding after sexual intercourse. What is
if less than these values=> give iron]. c. Parents karyotyping the
1118. A 34yo woman who never had fits or high d. Coomb’s test most appropriate cause?
BP developed fits 6h after delivery of a term e. Pre-implantation genetic dx a. Preeclampsia
healthy Ans. The key is A. Amniocentesis. [Amniocentesis b. DIC
child. What is the most likely dx? is done between 14 to 16 weeks of gestation. CVS c. Placental abruption
a. Eclampsia is done prior to 15 weeks. So the option is A. d. Placental previa
b. Preeclampsia Amniocentesis]. e. Ectopic pregnancy
c. Epilepsy f. Missed abortion
d. Pulmonary embolism 1123. A 28yo lady with a fam hx of CF comes for g. Ectropion
e. Pregnancy induced HTN genetic counselling and wants the earliest Ans. The key is G. Ectropion. [Post coital bleeding
Ans. The key is A. Eclampsia. [In eclampsia seizers possible dx can be either placenta previa or cervical
occurs within 24 hours mostly and majority within test for CF for the baby she is planning. She is not ectropion. But as ectropion is commoner in
12 hours. Also single seizure doesn’t support in favor of termination. What would you pregnancy so it is the option here].
epilepsy strongly. So likely answer is A. recommend for her?
Eclampsia]. a. CVS (Chorionic Villous Sampling) 1127. A 6wk pregnant woman presents with
b. Amniocentesis abdominal pain. She has prv hx of PID. What is the
1119. A 30yo lady who already has one child c. Pre-implantation genetic dx most
through a prv C-section demands a reversible d. Chromosomal karyotyping likely dx?
contraception. She presently experiences heavy e. Maternal serum test a. Preeclampsia
and painful periods. What is the most f. Reassure b. DIC
appropriate contraceptive you will recommend Ans. C. Pre-implantation genetic dx. [Patient is not c. Placental abruption
for her? yet pregnant but planning for pregnancy. Earliest d. Placental previa
a. COCP possible diagnostic test for CF is Pre-implantation e. Ectopic pregnancy
b. POP genetic dx. Preimplantation genetic f. Missed abortion
c. Implanon diagnosis (PGD) is a procedure used prior to g. Ectropion
d. Danazol implantation to help identify genetic defects Ans. The key is E. Ectopic pregnancy. [H/O PID is a
e. Mirena within embryos created through in vitro recognized cause of ectopic pregnancy. Also pain
f. IUCD fertilization to prevent certain diseases or without bleeding at 6th week support the
Ans. The key is E. Mirena. disorders from being passed on to the child]. diagnosis of ectopic pregnancy].
1120. A 32yo woman comes with intermenstrual
bleeding. Her last cervical smear was 1y ago and 1124. A 39yo woman in her 36th week GA with 1128. A 33wk pregnant woman presents with
was acute abdominal pain is rushed for immediate vaginal bleeding, low Hgb, low plt, increased
negative. What test would you recommend for delivery. bilirubin,
her initially? Her report: BP=110/60mmHg, Hgb=low, AST normal, APTT & PT increased. What is the
a. Colposcopy bilirubin=22, AST=35, Plt=60, APTT=60, PT=30, most likely dx?
b. Cervical smear Fibrinogen=0.6. What is the cause? a. Preeclampsia
c. Endocervical swab a. Pregnancy induced hypertension b. DIC
d. Transvaginal US b. DIC c. Placental abruption
e. Pelvic CT c. HELLP syndrome d. Placental previa
Ans. The key is B. Cervical smear. This is a wrong d. Acute fatty live e. Ectopic pregnancy
key! Correct key is C. Endocervical swab. [Smear e. Obstetric cholestasis f. Missed abortion
g. Ectropion e. Mirena e. Digoxin
Ans. The key is B. DIC. Ans. The key is B. COCP. [In irregular period: COCP
except the contraindications for it and in that case Ans. The key is E. Digoxin. [Digoxin toxicity causes
1129. A 25yo lady at her 28th week GA came for POP should be used]. diarrhea, vomiting, dehydration, xanthopsia
check up. Her BP=160/95mmHg, protein in (yellow halos around light), bradicardia. History of
urine=6g/d. 1134. A 32yo presents with heavy blood loss, US: atrial fibrillation is also a clue of digoxin use].
What is the most likely dx? uterine thickness>14mm. What is the most
a. Essential HTN appropriate tx for her? 1138. A 33yo lady who is a drug addict wants to
b. Gestational HTN a. Mefenemic acid quit. She says she is ready to stop the drug abuse.
c. Chronic HTN b. COCP She is supported by her friends and family. What
d. Preeclampsia c. POP drug tx would you give her?
Ans. The key is D. Preeclampsia. d. IUCD a. Benzodiazepines
e. IU system (mirena) b. Diazipoxide
1130. A 32yo woman has a hx of spontaneous Ans. The key is E. IU system (mirena). [Simple c. Lithium
abortions at 6wks, 12wks, and 20wks. She is now endometrial hyperplasia without atypia responds d. Methadone
keen to high-dose progestogens, with repeat histology e. Disulfiram
to conceive again. Which of the following would after three months. This can be effectively
you prescribe for the next pregnancy? delivered by the levonorgestrel intrauterine Ans. The key is D. Methadone. [Methadone is an
a. MgSO4 system (IUS). Source: patient.info]. opioid medication. Methadone reduces
b. Aspirin withdrawal symptoms in people addicted to
c. Warfarin 1135. A 37yo woman presents with heavy heroin or other narcotic drugs without causing the
d. Mefenemic acid bleeding. Inv show subserosal fibroid=4cm and "high" associated with the drug addiction and
e. Heparin intramural thus used to help a drug addict to quit from drug
Ans. The key is B. Aspirin. [Patient of fibroid=6cm. Which is the most appropriate tx? addiction].
antiphospholipid syndrome had 3 fetal losses and a. UAE
no thromboembolic event and now wants to be b. Abdominal hysterectomy 1139. A 50yo lady has been suffering from chronic
pregnant. In such case prophylactic treatment c. Hysteroscopic Myomectomy RA and is on methotraxate and naproxen. Her CBC
with low dose aspirin is indicated]. d. Vaginal Hysterectomy shows microcytic anemia. What is the most likely
1131. A 6yo child presents with hx of recurrent e. Abdominal myomectomy cause?
jaundice. Between the episodes he is totally fine. Ans. The key is E. Abdominal myomectomy. [As a. Anemia of chronic disease
Mother gives hx of jaundice being brought about patient is young we should go for myomectomy. b. GI hemorrhage
by ongoing infections. What is the most likely As hysteroscopic myomectomy is suitable for c. Menorrhagia
dx? mainly submucosal fibroids. We should go for
a. Hereditary spherocytosis Abdominal myomectomy which will deal with Ans. The key is B. GI hemorrhage. [Anemia of
b. G6PD deficiency both subserosal and intramural fibroids]. chronic disease is mostly normocytic and
c. Thalassemia methotrexate causes folate deficiency which may
d. Sickle cell disease 1136. A woman with sickle cell disease complains lead to macrocytosis. So for this microcytic
e. Congenital storage disorder of heavy menstrual blood loss. What is the most anemia NSAIDs induced GI hemorrhage is the
Ans. The key is B. G6PD deficiency. [There are appropriate tx? most likely cause].
some precipitating factors of hemolysis causing a. COCP
jaundice in patients with G6PD deficiency among b. Mirena 1140. A 15yo male noticed swelling on the left
which infection is a recognized one]. c. Depot provera knee following a fall while playing. The swelling
d. Copper IUS has not
1132. A 42yo woman who smokes 20 cigarettes/d e. Transdermal patch subsided in spite of rest and analgesia. Exam: full
presents with complains of heavy bleeding and Ans. The key is C. Depot provera. [Hormone and knee movement with slight tenderness. He has
prolonged menstrual period. What is the most barrier methods are all acceptable choices but painless palpable mass in left inguinal region.
appropriate tx for her? intrauterine devices are not recommended, as What is the most probable dx?
a. Tranexemic acid they may be associated with uterine bleeding and a. Osteosarcoma
b. COCP infection. Depot contraceptive (Depo-Provera®) is b. Ewing’s sarcoma
c. Mefenemic acid safe and has been found to improve the blood c. Chondrosarcoma
d. IUCD picture and reduce pain crises. Source: d. Lymphangiosarcoma
e. Norethisterone patient.info]. e. Osteodosteoma
Ans. The key is D. IUCD. This is wrong key. Correct
key is A. Tranexemic acid. 1137. A 70yo woman is admitted with diarrhea, Ans. There are two keys i) Osteosarcoma ii)
vomiting and dehydration. Exam: yellow visual Ewing’s sarcoma. But the likely correct key is
1133. A 17yo senior school girl with complain of halos in probably A. Osteosarcoma. [Both occurs in
prolonged irregular menstrual period and heavy her eyes, ECG=bradycardia. She has a hx of children and young adults but the occurrence of
blood chronic A-fib. Which drug causes the above painful symptom during playing is usually a
losses. What is the most appropriate tx for her? mentioned side effects? feature of osteosarcoma. In case of osteosarcoma
a. Mefenemic acid a. Nifedipine teenagers who are active in sports often complain
b. COCP b. Ramipril of pain in the lower femur, or immediately below
c. POP c. Atenolol the knee. If the tumor is large, it can present as
d. IUCD d. Lithium overt localised swelling. Sometimes a sudden
fracture is the first symptom, because affected IOP is normal. Red reflex is absent in both eyes. Ans. The key is E. Alpha 1 antitrypsin deficiency.
bone is not as strong as normal bone and What is the single most likely dx? [In those with unexplained liver disease with or
may fracture abnormally with minor trauma]. a. Cataract without respiratory symptoms should be
b. DM retinopathy evaluated for AATD]. 
1141. A 45yo female looking pale has bluish c. Hypermetropia
discoloration of hands whenever she goes out in d. Macular degeneration 1149. A 65yo man had a bowel resection 5d ago.
the cold. e. HTN retinopathy He is anuric and breathless. His
She has also noticed some reddish spots on her BP=150/110mmHg.
body. She has symmetrical peripheral Ans. The key is A. Cataract. [Prolonged use of He has crackles at both lung bases and sacral
arthropathy for the last yr. What is the most steroid (here in RA) is a known cause of cataract]. edema. Bloods: K+=6.8mmol/l, urea=58mmol/l,
probable dx? creatinine=600umol/l. What is the single most
a. RA 1145. An elderly man with recently dx HF has appropriate immediate management?
b. Osteosarcoma been treated with diuretics. He now develops a. Bolus of 20U insulin
c. Limited systemic sclerosis severe b. Calcium resonium enema
d. Diffuse systemic sclerosis joint pain in his left ankle with swelling and c. Dextrose-saline infusion
e. Chondrosarcoma redness. What is single most likely inv? d. 5% dextrose infusion
a. XR of bone e. 10U insulin, 50ml of 50% dextrose infusion
Ans. The key is C. Limited systemic sclerosis. b. Plasma RF Ans. The key is E. 10U insulin, 50ml of 50%
[Raynaud’s phenomena, telangiectasia, c. Joint fluid uric acid crystals dextrose infusion. [There is renal failure with
sclerodactyly indicates to the dx of CREST d. ESR hyperkalemia. First we have to save heart from
syndrome or Limited systemic sclerosis]. Ans. The key is B. Plasma RF. It is a wrong key. arrest or life threatening arrhythmia by shifting K+
Correct key should be C. Joint fluid uric acid into cell. 10U insulin in 50 ml 50% dextrose will
1142. A 60yo female has pain and stiffness in her crystals. [Thiazide diuretics may increase uric acid help doing this].
right hip joint. Pain is not severe in the morning levels in blood causing or precipitating gout and
but joint fluid may show uric acid crystals]. 1150. A 25yo woman presents with a painful
increases as the day progresses. She has noticed shallow ulcer on the vulva. What inv has to be
some nodules in her hands. Inv: Hgb=low. What 1146. A 60yo lady with a hx of HTN and suffering done?
is the most probable dx? from RA since the last 10y now presents with hot, a. HSV antibodies
a. RA swollen and tender knee joint. What inv would b. Syphilis serology
b. Osteoarthritis you do for her? c. Swab for hemophilus ducreyi
c. Gout a. XR d. Urine culture
d. Pseudogout b. C&S of joint aspirate e. Blood culture
e. Multiple myeloma c. US Ans. The key is C. Swab for hemophilus ducreyi.
d. MRI [Herpes simplex virus causes multiple painful
Ans. The key is B. Osteoarthritis. [Stiffness and e. CT vesicles and syphilis ulcer is painless. As here
pain of right hip increasing as the day progresses, Ans. The key is B. C&S of joint aspirate. [RA itself single painful ulcer probable dx is Chancroid
nodules in hands (Hebarden or Bouchard’s is a risk factor for joint infection and there are caused by Hemophilus ducreyi].
nodes), anemia (probably secondary to prolonged possibilities to increase the chance of septic
NSAIDs use) indicates osteoarthritis]. arthritis with the use of probable steroid which 1151. A child was admitted with fever, generalized
lowers immunity and further facilitates infection skin lesion, some of them are weeping lesions and
1143. A 30yo female has chronic diarrhea, mouth of joints for which C&S of joint aspirate should be some of them are crusted. What is the most
ulcers and skin tags. She complains of visual prbs, done]. probable dx?
low back pain and morning stiffness. Inv: ESR & a. Varicella
CRP=raised, Hgb=10mg/dl. What is the most 1147. A 34yo man after an RTA was brought to b. Impetigo
probable dx? the ED. He has BP=50/0mmHg and chest wall with c. Drug reaction
a. SLE asymmetrical movement, RR=34bpm. What would d. Contact dermatitis
b. Reactive Arthritis be the initial action? e. Scabies
c. Gout a. IV fluid infusion Ans. The key is B. Impetigo. This is probably a
d. Pseudogout b. Intubation and ventilation wrong key! Likely correct key should be A.
e. Seronegative arthritis c. CT chest Varicella. [Please consider dx of impetigo if
d. Transfer to ITU specifically mention honey (or yellow or golden)
Ans. The key is E. Seronegative arthritis. [Chronic coloured lesion or distribution described to be on
diarrhea, mouth ulcers and skin tags are features Ans. The key is B. Intubation and ventilation. face and limbs predominately. Otherwise consider
of IBD. There is a well known association between chickenpox. Though key is impetigo here, it is
IBD and seronegative arthritis (particularly AS). 1148. A 7yo presented with chronic cough and is wrong key. Likely correct option here is varicella].
Here low back pain, visual problem, morning also found to be jaundiced on exam. What is the
stiffness, raised ESR and CRP, low hemoglobin most likely dx? 1152. A pt comes with 6m hx of painless bilateral
indicates the diagnosis of Seronegative arthritis]. a. Congenital diaphragmatic hernia swelling of the face which has been progressively
b. Congenital cystic adenomatoid malformation increasing in size. On routine CXR, he is found to
1144. A 28yo woman has been on tx for RA for c. Bronchiolitis have perihilar lymphadenopathy. What is the
3yrs. She has gradual loss of vision in both eyes. d. RDS most probable dx?
Her e. Alpha 1 antitrypsin deficiency a. Chronic sialadenitis
b. Thyroid adenoma
c. Carcinoma of salivary gland 1157. A 15m baby girl presented to the ED with hematuria and fluctuating urea levels. A dx of
d. Adenoid cystic carcinoma difficulty in breathing. Exam: she has intercostal renal vein thrombosis is made. What is the most
e. Mikulicz’s disease recessions and a wheeze. Temp=normal. What is likely cause for renal vein thrombosis?
Ans. The key is E. Mikulicz’s disease. [Mikulicz’s the most likely dx? a. Protein C deficiency
disease (MD) is a well-known disorder a. URTI b. Vasculitis
characterized by enlarged lacrimal and parotid b. Pneumonia c. Loss of antithrombin III
glands caused by infiltration with lymphocytes. c. Bronchiolitis d. High estrogen levels
When no specific cause is found it is called d. RDS e. Stasis
Mikulicz’s disease; and if secondary to disease like e. Alpha 1 antitrypsin deficiency Ans. The key is C. Loss of antithrombin III.
sarcoidosis it is termed as Mikulicz’s syndrome]. Ans. The key is C. Bronchiolitis. [Asthma and
bronchiolitis has similar sign symptoms. The 1162. A 36yo woman presented with massive
1153. A woman has widespread metastasis from a difference is in pathology. In asthma there is bleeding from multiple sites. Lab: fibrin
carcinoma. She presented with severe back pain. bronchoconstriction but in bronchiolitis there is degradation
Where do you expect the cancer to be? oedema of the airway wall. In infants and younger products: +++, plt=30, bleeding time=prolonged,
a. Lungs children bronchiolitis is commoner than asthma PT=prolonged, APTT=prolonged. What is the
b. Cervix and it is seen that many of the children with most likely dx?
c. Ovary bronhiolitis usually followed by asthma]. a. Hemophilia
d. Uterus b. DIC
e. Breast 1158. An 8yo boy develops a seizure affecting his c. ITP
Ans. The key is E. Breast. [In female breast and right arm, seizure lasts for several mins. He d. Factor V leiden
lung cancer and in male prostate and lung cancer doesn’t e. Warfarin
are most common to metastasize to bone]. remember anything what happened. On his CT: Ans. The key is B. DIC.
lesion in left hemisphere. What is the most
1154. A 10yo child has got progressive bilateral probable dx? 1163. A study was done amongst 2 hosp for the
hearing loss. He has started to increase the TV a. Epilepsy equal number of cancer pts. It was noted that
volume. b. Space occupying lesion hosp A
All other examination is normal. What is the most c. Dementia had the higher rate of mortality than hosp B for
likely dx? d. Huntington’s chorea treated cancer pts. What is the study done here
a. Wax e. Intracranial HTN classified as?
b. Foreign body Ans. The key is B. Space occupying lesion. [This is a. Retrospective
c. Bilateral OM with effusion complex partial seizure due to space occupying b. Observational
d. SNHL lesion]. c. Cohort
e. Meningitis due to meningococcus d. Case study
Ans. The key is C. Bilateral OM with effusion. 1159. A 28yo female presented with complains of Ans. The key is C. Cohort study. [A cohort is a
difficulties in swallowing liquids only. She also group of people who share a common
1155. A child had a patchy rash following tx for suffers from recurrent chest infection in the past characteristic or experience within a defined
sore throat & cervical LN enlargement. Which is few months. What is the most probable dx? period (e.g., are born, are exposed to a drug or
the a. Foreign body vaccine or pollutant, or undergo a certain medical
most appropriate antibiotic? b. Plummer vinson syndrome procedure].
a. Ampicillin c. Achalasia cardia
b. Erythromycin d. Peptic stricture 1164. A 17yo girl comes to see her GP after having
c. Cefuroxime e. Esophageal carcinoma unprotected sex 2d ago. She asks if her GP can
d. Metronidazole Ans. The key is C. Achalasia cardia. [Dysphagia to explain to her how this prescribed procedure
e. Tetracycline both solid and liquid or mostly to liquid are would work by helping her not to get pregnant.
Ans. The key is A. Ampicillin. [Infectious common feature of achalasia. In achalasia there a. It helps to prevent implantation
mononeucleosis can present with sorethroat and may occur aspiration due to regurgitation during b. It helps in preventing or delaying ovulation
lymphadenothy like tonsillitis and if treated with lying down to sleep which may result recurrent c. It causes an early miscarriage
ampicillin leads to eruption of patchy rashes]. chest infection and cough]. d. It releases progesterone and stops ovulation
e. It causes local enzymatic reaction
1156. A child with a hx of asthma is brought to ED 1160. Mother having 2 children with CF. What is Ans. The key is A. It helps to prevent implantation.
with a cut on knee and sprained on her left wrist. the risk of getting another baby? Probably key is suggesting IUCD wich helps to
Which is the best analgesic for her? a. 1:2 prevent implantation and not the pills as both the
a. Paracetamol b. 1:8 available pill causes a delay in ovulation. [The
b. NSAIDs c. 1:4 time required for the egg to travel to the uterus
c. Cocodemol d. 1:16 and implant is usually between 7 and 10 days].
d. Ibuprofen e. 1:1 [There are two types of pill:
Ans. The key is A. Paracetamol. [NSAIDs including Ans. The key is C. 1:4. [Cystic fibrosis is an 1. 1.5 mg of levonorgestrel pill. It should be taken
ibuprofen can precipitate asthma and Cocodamol autosomal recessive disease. If both parents are within 72 hours (three days). It is thought to work
is also advised to avoid in asthma (due to its carrier there is 1:4 chance of risk of occurrence of mainly by preventing or delaying the release of an
codeine content). So paracetamol is the best the disease for each child]. egg from your ovary, which normally happens
option here]. each month (ovulation).
1161. A 14yo boy has been dx with nephrotic 2. Ulipristal acetate (brand name ellaOne®) is a
syndrome. 5d later he presents with flank pain, type of emergency contraceptive pill that was
launched in the UK in 2009. It is taken as one c. Delayed verbal development e. GAD
single tablet. Take the pill as soon as possible after d. Normal development Ans. The key is A. Conversion reaction.
unprotected sex. The earlier you take the pill, the e. Delayed social development [Conversion  reaction is sometimes applied to
more effective it is. It can be taken up to 120 Ans. The key is D. Normal development. patients who present with neurological
hours (five days) after having unprotected sex. It symptoms, such
is a type of hormone which seems to work by 1169. A young child, 3yo, has presented with as numbness, blindness, paralysis, or fits, which
stopping or delaying release of an egg (ovulation). vomiting for 3d. Exam: mild-mod dehydration. are not consistent with a well-established organic
IUCD prevent implantation. As she is sexually What is cause, and which cause significant distress. It is
active hymen is ruptured and IUCD can be given his ABG profile likely to show? thought that these symptoms arise in response to
to her. But with IUCD chance of ectopic pregnancy a. pH low, PCO2 low stressful situations affecting a patient's mental
is much more!]. b. pH low, PCO2 high health].
c. pH high, PCO2 low
1165. A 2d baby’s mother is worried about the d. pH high, PCO2 high 1173. A 25yo man has been suffering from
baby’s hearing. Mother has a hx of conductive Ans. The key is D. pH high, PCO2 high. [Prolonged breathlessness and wheeze for 3m. He has been
hearing vomiting causes loss of H+, Cl- and Na+. There is taking
loss. What is the most appropriate test? also hypokalemia. There occur metabolic salbutamol 2puffs as required. In the last 2 wks his
a. Brain stem evoked response alkalosis. Compensation for metabolic alkalosis symptoms have worsened and he has to take
b. CT occurs mainly in the lungs, which retain carbon salbutamol more frequently during the day time.
c. Fork test dioxide (CO2) through slower breathing, He also complains of excessive dyspnea at
d. MRI or hypoventilation (respiratory compensation) night. What drugs or regimen would you like to
e. Reassure leading to high PCO2]. add?
Ans. The key is A. Brain stem evoked response. [A a. Prednisolone
BAER (brainstem auditory evoked response) test 1170. A 68yo woman has been admitted with b. Fluticasone + salbutamol inhaled
can help to diagnose hearing loss and nervous poor appetite, weight loss, poor concentration c. Beclomethasone inhaled
system disorders, especially in newborns, young and self d. Montelukast PO
children, and others who may not be able to neglect for 3wks. She has not been eating or e. Salmetrol PO
participate in a standard hearing test]. drinking adequately and has rarely left her bed. Ans. The key is C. Beclomethasone inhaled.
She [Patient was in step 1. As not controlled next step
1166. A healthy 8yo boy had antibiotic tx for is expressive suicidal ideas and is convinced that is addition of inhaled corticosteroid].
meningitis. Initially he wasn’t resuscitated. What people are out to kill her. She has been on
will be antidepressant therapy for the past 3m with no 1174. A 64yo man who was exposed to asbestos
the outcome if he receives full tx? improvement. What is the most appropriate tx? for 40yrs presents with weight loss and chest
a. He will recover fully to his prv health a. Anti depressants pain.
b. He will have hearing impairment b. CBT The dx of mesothelioma has been made. He
c. He will have brain abscess c. Interpersonal therapy develops SOB and XR=pleural effusion. What is
d. He will have encephalitis d. ECT the
Ans. The key is A. He will recover fully to his prv e. Antipsychotics most appropriate management?
health. Ans. The key is D. ECT. [Patient has symptoms of a. Thoracocenthesis
severe depression with persecutory delusion b. Chest drain
1167. A pt presented with jaundice, fever and making the diagnosis of psychotic depression c. Radiation therapy
upper abdominal pain within 24h after removal of which is treated by ECT]. d. Pneumonectomy
gallstone by ERCP. The cholangiography was done e. Chemotherapy
and it was patent. What is the possible cause 1171. A 78yo retired teacher was admitted for a Ans. The key is E. Chemotherapy. It is probably a
of his complaints? hernioplasty procedure. After the operation he wrong key! Correct key should be A.
a. Biliary infection became agitated, aggressive and confused. What Thoracocentasis. [As mesothelioma responds very
b. Acute pancreatitis is the most appropriate management? poor to chemotherapy and life expentancy is also
c. Perforation a. Diazepam very poor Thoracocentasis is the appropriate
Ans. The key is B. Acute pancreatitis. [Though b. Chlordiazepoxide procedure to improve patients distress (when it
biliary infection is possible but acute pancreatitis c. Vit B becomes more resistant then chest drain)].
is more common complication of ERCP and d. Clozapine
absence of any obstruction (stone or tumour e. Thiamine 1175. A 72yo presents with polyuria and
which helps biliary infection to occur) makes Ans. The key is B. Chlordiazepoxide. [Probable polydipsia. The fasting blood sugar is 8 and
cholangitis less likely]. delirium tremens]. 10mmol/l.
1172. A 25yo girl saw a tragic RTA in which a BP=130/80mmHg and the level of
1168. A mother presents with her 14m child. He young boy was killed. The night of the event she cholesterol=5.7mmol/l. There is
holds furniture and other things to help him stand couldn’t microalbuminuria. What is
and walk. He can say ‘mama’ and ‘papa’. He sleep and the day after she suddenly lost her the single most appropriate next management?
makes eye contact and smiles. He can transfer vision. She was prv fine and there was no hx of a. ACEi and sulfonylurea
objects from one hand to another. He responds to medical or psychological prbs. What is the dx? b. Statin and biguanide
his name. what do you interpret from his a. Conversion c. Statin and glitazone
development? b. Somatization d. Insulin and ACEi
a. Delayed gross motor development c. PTSD e. Statin and ACEi
b. Delayed fine motor development d. Dissociation
Ans. The key is E. Statin and ACEI. [1st we shall Ans. The key is B. Needle thoracocentesis. [The
recommend modification of lifestyle so no case may be either pneumothorax or massive 1184. A 72yo man presents to the ED with chest
diabetic drug now. But for microalbuminuria we pleural effusion. Thoracocentasis is recommended pain. The following ECG was taken. What is the
shall start ACEI as it is renoprotective and also for both]. most
statin which is recommended as cholesterol is likely dx?
>5mmol/l]. 1180. A 7yo child is being inv for TB. His parents
don’t agree for taking a BAL. what other sample
1176. A 49yo woman presents to the OPD. Her will
oral glucose test after 2h of glucose intake vs show growth of the organism?
plasma a. Blood test
level in 2 different tests are 6mmol/l and b. Throat swab
10mmol/l. This situation can be categoraized as c. Gastric washing
a. Impaired glucose tolerance d. Mantoux test
b. Impaired fasting glucose e. CSF
c. T1DM
d. T2DM Ans. The key is C. Gastric washing. a. Anterior MI
e. Metabolic syndrome b. Inferior MI
Ans. The key is A. Impaired glucose tolerance. 1181. A 51yo man had a MI a few days ago. He c. Lateral MI
[The WHO defines someone as having pre- developed breathlessness. Echo was done and d. Posterior MI
diabetes if they have: showeda pansystolic murmur. What can be the e. NSTEMI
 A fasting blood glucose of less than 7 cause of this symptom?
mmol/L; AND a. Ruptured papillary muscle Ans. The key is E. NSTEMI.
 A blood glucose of 7.8 mmol/L or more b. Acute pericarditis
but less than 11.1mmol/L after a two- c. Dresslers syndrome 1185. A 36yo woman has recently spent a lot of
hour oral glucose tolerance test]. d. Malignant VT money on buying clothes. She goes out almost
e. Ventricular aneurysm every
1177. A white English man with a past hx of MI is Ans. The key is A. Ruptured papillary muscle. night with her friends. She believes that she
a known HTN and DM. He is currently on aspirin, [Papillary muscle rupture causes mitral knows better than her friends, so she should
statin and metformin. What would you add to the regurgitation causing pancystolic murmur leading choose
tx? to features of heart failure like breathlessness]. the restaurant for eating out with her friends. She
a. ACEi gave hx of having low mood at 12y. What is
b. Diuretic 1182. A 61yo man was found with K+=7.5 and ECG the dx?
c. Insulin with prolong QRS complex. What is the best a. Mania
d. Beta blocker possible tx option? b. Depression
e. CCB a. Dialysis c. Bipolar affective disorder
b. IV calcium gluconate d. Borderline personality disorder
Ans. The key is A. ACEI. c. IV insulin and dextrose e. Dysthymia
d. Salbutamol nebulizer Ans. The key is C. Bipolar affective disorder. [In
1178. A 57yo man who had MI a few months ago e. Loop diuretics bipolar disorder there is depressive disorder
has been having a low mood. A dx of moderate Ans. The key is B. IV calcium gluconate. [In this alternate with mania and the present case is in
depression has been established. Which high level of potassium we have to shift this manic stage of bipolar disorder].
medication is the best tx for him? potassium into cells to reduce plasma level and
a. SSRI save heart from arrest or life threatening 1186. A homeless lady presents with cough and
b. TCA arrhythmia. For keep the heart safe option of first fever. She complains of night sweats and weight
c. MAOi choice in such emergency is IV calcium gluconate! loss.
d. Benzodiazepam However it does not shift or reduce potassium CXR has been done and shows opacity. What is
e. Mood stabilizer level but protects heart from arrhythmia and buy the next appropriate management?
time for definitive measure to take place!]. a. AFB
Ans. The key is A. SSRI. [Among SSRIs Sertraline is b. Mantoux test
the drug of choice. If SSRI cannot be used 1183. A 38yo man presents with acute infection of c. IFN gamma testing
Mirtazapine is recommended as next skin in the leg. Dx of cellutitis has been made. d. Bronchoscopy
antidepressant]. What e. CT
meds should be prescribed? Ans. The key is A. AFB. [Cough, fever, night sweat
1179. A 12yo presents with chest pain. Exam: a. Penicillin + Flucloxacillin and weight loss are very suggestive of TB. So we
tachycardia, hypotension, dilated neck veins and b. Metronidazole + erythromycin should do AFB as her next investigation].
the c. Vancomycin + metronidazole
trachea is not centrally placed. What is the next d. Ceftriaxone + terbinafine 1187. A 32yo woman presents with malaise
appropriate management? e. Ceftriaxone + flucloxacillin fatigue and fever. She complains about weight
a. Portable XR Ans. The key is A. Penicillin + Flucloxacillin. loss. Exam:
b. Needle thoracocenthesis [Cellulitis is usually caused by bacteria, such as malar rash with sparing of nasolabial fold can be
c. Chest drainage staphylococci or streptococci that are commonly seen. What is the most appropriate inv?
d. ABG present on the skin. So Penicillin + Flucloxacillin a. Anti ds DNA
e. CTPA should be given]. b. Anti histone
c. Anti centromere ideas, elation, over activity and disinhibition, or Ans. The key is B. Give 0.25ml in 1000U
d. Anti Jo low mood with lack of energy and social epinephrine IM. [Stridor is one of the indication of
e. Anti Scl70 withdrawal. What is the most probable dx? IM epinephrine in anaphylaxis].
Ans. The key is A. Anti ds DNA. [Features are a. Bipolar affective disorder
suggestive of SLE. So most appropriate b. Dysthymia 1195. A terminally ill pt with metastatic carcinoma
investigation from the given options is A. Anti ds c. Mania presents with dysphagia and difficulty in
DNA]. d. Hypomania swallowing. What is the best possible tx?
e. Cyclothymia a. Nystatin suspension
1188. A 75yo man presents with back pain. Inv: Ans. The key is A. Bipolar affective disorder. [Flight b. Amphotericin B IV
plasma cells are found. What is the most probable of idea, elation, over activity and disinhibition are c. PO fluconazole
dx? features of mania and low mood, lack of energy d. Cotrimazole
a. Multiple myeloma and social withdrawal are features of depression. e. Analgesic
b. AS Alternating mood swings with depression and Ans. The key is C. PO fluconazole. [Treatment of
c. Disc prolapse mania are chracteristic of Bipolar disorder]. carcinoma can predispose to development of
d. Leukemia oesophageal candidiasis which is treated as
e. Myelofibrosis 1192. Healthy parents have 2 children, a child follows: Oral fluconazole (200 – 400mg) daily for
Ans. The key is A. Multiple myeloma. [H/O back with CF and a healthy child. They want to have 14 to 21 days (if needed IV fluconazole can also be
pain with presence of plasma cells on inv. Are another given].
highly suggestive of Multiple myeloma]. child. What are the chances of that child being a 1196. A couple attends their GP because of
carrier? marital problems. The wife states that her
1189. A 45yo woman presents with complains of a. 1:4 husband is having affairs although she has no
abdominal pain and blood in stool. She brings the b. 1:2 proof of this. The husband states that she even
stool sample from home but has never been able c. 2:3 had him followed by a private detective and this is
to produce a sample at the hospital. Her urine d. 1:8 putting considerable strain on their marriage.
and blood tests are normal. Exam: multiple scars e. 1:16 What is the most likely dx?
on the abdomen consistent with laparoscopies Ans. The key is B. 1:2. [As the parents have a child a. Fregoli syndrome
and appendectomy. She insists on getting further with cystic fibrosis and they are healthy both of b. Cotard syndrome
inv although no abnormalities are found. them are carrier. It is an autosomal recessive c. Mood disorder
What is the most likely dx? disease where if both parents are carrier mode of d. Ekbom syndrome
a. Malingering inheritence is as follows: Chance of being healthy e. Othello syndrome
b. Somatization child 1:4, Chance of being diseased 1:4 and Ans. The key is E. Othello syndrome. [Othello
c. Hypochondriasis chance of being carrier 1:2]. syndrome is delusion of infidelity (cheating,
d. Conversion disorder adultery, or having an affair) of a spouse or
e. Munchausen syndrome 1193. A 64yo man believes a female newscaster is partner].
Ans. The key is E. Munchausen syndrome. communicating directly with him when she turns
[Munchausen syndrome is a psychiatric factitious a 1197. A 65yo lady who is on thiazide suffers from
disorder wherein those affected feign disease, page. What kind of delusions is he suffering from? falls in the morning. What is the cause for her
illness, or psychological trauma to a. Persecutory symptoms?
draw attention,sympathy, or reassurance to b. Control a. Orthostatic hypotension
themselves]. c. Grandeur b. TIA
d. Nihilistic c. Epilepsy
1190. A 36yo woman contacts the police to notify e. Reference Ans. The key is A. Orthostatic hypotensin.
them she was responsible for a recent disastrous Ans. The key is E. Delusion of reference. [Thiazide diuretic is associated with orthostatic
flood with loss of lives. What kind of delusions is [Delusions of reference – A neutral event is hypotension].
she suffering from? believed to have a special and personal meaning.
a. Persecutory For example, a person with schizophrenia might 1198. A boy was admitted with partial thickness
b. Poverty believe a billboard or a celebrity is sending a burn, what is your next step?
c. Guilt message meant specifically for them]. a. Escharectomy
d. Nihilistic b. Dressing
e. Reference 1194. A 7yo girl with allergy became acutely c. Burst blisters
Ans. The key is C. Delusion of guilt. [Delusions of unwell while visiting a friend’s house and has d. Local antibiotics
guilt or sin (self-accusation): This type of been e. Refer to burn unit
delusions involve feeling guilty or remorseful for brought immediately to the ED. She is fully Ans. The key is E. Refer to burn unit.
no valid reason.  An example would be someone conscious but has got stridor, wheeze and
that believes they were responsible for a war in erythematous rash. She is receiving oxygen. What 1199. A 28yo man presents with a 2h hx of rapid
another country or hurricane damage in another is the single immediate management? palpitations. He feels a little light headed but is
state.  In this case, the person believes that they a. Check airway patency and prepare intubation otherwise well. Exam: pulse=170bpm and regular,
deserve to be punished for their sins and place full b. Give 0.25ml in 1000U epinephrine IM BP=100/68mmHg. He has had 2 similar
blame on themselves]. c. Give 10mg chlorphearamine IM episodes in the past. What is the most likely
d. Give 50ml hydrocortisone IM rhythm disturbance?
1191. A 27yo man presents with symptoms e. Obtain secure IV access a. SVT
characterized by alternating mood swings a/w b. VF
flight of c. VT
d. V-ectopics demarcated from the surrounding yellow adipose Ans. The key is B. Mumps. [Here sudden onset
e. A-fib tissue]. and rise of temperature are points indicating the
Ans. The key is A. SVT. [Palpitation, light diagnosis of Mumps].
headedness with a tachycardia of 170bpm that is 1204. A young boy presented with peri-oral
regular are most likely a SVT]. blisters. Some of which are weeping and others 1208. A 6wk baby has a blue mark near coccyx
are since birth. His mother is worried. What would
1200. A child has hypothyroidism. What feature is crusted. What is the single most appropriate dx? you do?
a/w it? a. Impetigo a. Reassure
a. Microglossia b. Varicella zoster b. Coag profile
b. Prolonged neonatal jaundice c. Shingles c. Karyotyping
c. Undescended testis d. Scabies d. Skeletal survey
d. Anal tag e. Herpes simplex e. CT
e. Left soft palate Ans. The key is A. Impetigo. [Distribution of Ans. The key is A. Reassure. [This is a benign
Ans. The key is B. Prolonged neonatal jaundice. varicella zoster is centripetal (more in central area condition known as “Mongolian spot” or
[Prolonged neonatal jaundice is a well known (trunk) and less in limbs and face. On the other “Mongolian blue”. Mongolian spot refers to a
feature of hypothyroidism]. hand if blisters are present mainly on face or macular blue-gray pigmentation usually on the
limbs and/or described as golden, yellow or honey sacral area of healthy infants. Mongolian spot is
1201. A 2wk girl presents with E-coli which is coloured crusts are impetigo]. usually present at birth or appears within the first
confirmed by urine culture. What is the most weeks of life. Mongolian spot typically disappears
appropriate 1205. A 39yo man comes with umbilicated spontaneously within 4 years but can persist for
next inv? papules on his face. His CD4 count is measured to life].
a. US be 35.
b. IVU What is the single most appropriate option? 1209. A man presents with inoperable carcinoma
c. CT kidney a. Mycobacterium avium and back pain. His pain has been well controlled
d. BUE intercellular with
e. MCUG b. CMV morphine but he develops vomiting. Morphine
Ans. The key is US. [Infants and children who have c. Streptokinase was stopped and he was started on
had a lower urinary tract infection should undergo d. Toxoplasmosis metoclopramide and fentanyl patches. He then
ultrasound (within 6 weeks) only if they are e. Pneumocystis jerovici develops neck stiffness and fever. What is the
younger than 6 months or have had recurrent f. Moluscum contagiosum cause of these symptoms?
infections. NICE (Probably to exclude VUR wich is Ans. The key is F. Molluscum contagiosum. [CD4 a. Metoclopramide
a cause of recurrent UTI)]. count 35 is too low indicating immunodeficiency b. Fentanyl
where molluscum contagiosum occurs more c. Morphine
1202. A lady from Asia presented with lump in her easily. Umbilicated papules are feature of d. Meningitis
neck. FNAC has been done and revealed lesions molluscum contagiosum]. e. Metastasis
with caseous material in the center surrounded by Ans. The key is A. Metoclopramide.
fibrosis. What is the most probable dx? 1206. A 45yo man is admitted to ED with [Extrapyramidal effects like neck stiffness is
a. Thyroid carcinoma excruciating pain in the right leg. Exam: limb is common side effect of metoclopramide and also
b. TB lymphadenitis pale and there may occur rise of temperature].
c. Lymphoma dorsalis pedis and posterior tibial pulses are
d. Inf Mono absent. Pulse=88bpm, irregular and he has a 1210. A 51yo man has become increasingly
e. Mesothelioma pansystolic murmur at apex. What is the most fatigued for the past 10m. PE: no abnormal
Ans. The key is B. TB lymphadenitis. [Central probable dx? findings. Labs:
caseous necrosis with sourrounded fibrosis is a. Thromboangitis Obliterans Hgb=9.2, Hct=27.9%, MCV=132fl, plt=242,
diagnostic of TB lymphadenitis. The most common b. Sciatica WBC=7.59. Which of the following morphologic
cellular components seen are epithelioid cell c. DVT findings is most likely to be present on
clusters. Lymphocytes and langhans giant cells are d. Atherosclerosis examination of his peripheral blood smear?
also seen]. e. Embolus a. Hypersegmented neutrophils
Ans. The key is E. Embolus. [Pansystolic murmur b. Nucleated RBC
1203. A 32yo woman has undergone a biopsy for at apex indicates mitral regurgitation and irregular c. Blasts
a breast lump. The report says: a well pulse indicates AF which can lead to thrombus d. Hypochromic, microcytic RBC
circumscribed formation what probably is responsible for this e. Schistocytes
lump with clear margins and separated from the embolus in leg]. Ans. No key is given! Probable key is A.
surrounding fatty tissue. What is the most Hypersegmented neutrophils. [Only anemia with
appropriate interpretation of this report? 1207. An 18yo man has a smooth, tender swelling macrocytosis is with normal examination findings
a. Fibroadenosis extending from the ear to the angle of the jaw of makes Megaloblastic aneamia to be most likely
b. Ca Breast sudden onset. Temp=38.5C. What is the single cause where PBF shows hypersegmented
c. Mammary abscess most likely dx? neutrophils].
d. Fibroadenoma a. Dental caries
e. Fat necrosis b. Mumps 1211. A 9yo girl with weekly abdominal pain and
Ans. The key is D. Fibroadenoma. [Fibroadenomas c. OE occasional headaches but not a/w vomiting or
are the most common benign tumor of the female d. OM diarrhea. She maintains a good appetite. Lab:
breast. White , well-circumscribed mass is clearly e. Temporomandibular joint pain normal. CBC, BUE, etc are normal. Exam: no
abnormality as found and the abdomen was soft a. B1 deficiency b. Ciprofloxacillin
and non-tender. What would you do for her b. B12 deficiency c. Flucloxacillin
next? c. Folate deficiency d. Ceftazidime
a. US abdomen d. B6 deficiency e. Benzyl penicillin
b. CT thorax e. Alcohol withdrawal Ans. The key is A. Amoxycillin. [For acute
c. LFT Ans. The key is A. B1 deficiency. [Presence of suppurative otitis media 1st line antibiotic in UK is
d. Reassure confusion and nystagmus indicates Wernicke’s Amoxycillin. Ref: patient.info].
e. Analgesics encephalopathy caused by thiamin (vit. B1)
Ans. The key is D. Reassure. [Probable case of deficiency]. 1219. A 26yo man strongly believes that every
abdominal migraine a benign condition for which elderly man he meets is his father. Although they
reassurance is appropriate. Abdominal migraine 1215. A 23yo female presented with a swelling of look
is a form of migraine seen mainly in children. It is her neck that moved upwards on protrusion of different, he is sure it is father wearing different
most common in children ages five-nine years old, tongue. What is the next appropriate inv? disguises. What kind of delusions is this man
but can occur in adults as well. The diagnostic a. FNAC suffering from?
criteria for abdominal migraine are: b. Punch biopsy a. Delusion of persecution
A. At least five attacks fulfilling criteria B–D. c. Core biopsy b. Erotomania
B. Attacks of abdominal pain lasting 1-72 hours d. MRI neck c. Delusion of grandeur
(untreated or unsuccessfully treated). e. Radioactive thyroid scan d. Delusion of doubles
C. Abdominal pain has all of the following Ans. The key is A. FNAC. [The diagnostic e. Delusion of reference
characteristics: investigation is FNAC[Ref: wikipedia]. Then we Ans. The key is D. Delusion of doubles. [The
C1. Midline location, periumbilical, or poorly palpate for thyroid tissue. If absent we do any one Fregoli delusion, or the delusion of doubles, is a
localized of following to confirm the absence of thyroid rare disorder in which a person holds a delusional
C2. Dull or ‘just sore’ quality tissue: ultrasonography, CT scanning, thyroid belief that different people are in fact a single
C3. Moderate or severe intensity scanning or MRI. If there is normal thyroid tissue person].
During abdominal pain at least two of the we go for more aggressive Sistrunk's operation
following: but if there is no normal thyroid tissue [in that 1220. A 26yo passed a 4mm stone in his urine. On
Anorexia case thyroglossal cyst itself is the only source of US a 3mm stone is found in the renal pelvis. What
Nausea thyroid tissue] we do conservative surgery and is the single most appropriate management?
Vomiting preserve some portion of cyst with thyroid tissue]. a. ESWL
Pallor b. None
Not attributed to another disorder (1)]. 1216. A 34yo man from Asia presented with 5m c. Open Surgery
hx of productive cough, night sweats and weight d. Conservative
1212. A 54yo male pt DM with BMI=33 who has loss. Ans. The key is D. Conservative. [Increased fluid
been treated using dietary control up till now His CXR reveals some shadowing in the left upper intake is advised].
presents zone. What is the single most discriminating
to his GP with a fasting blood sugar of 14mmol/l inv? 1221. A 35yo man has had acute pain and swelling
and creatinine=90mmol/l. Urine shows a. AFB for sputum below the mandible on the left side for 2h. The
glycosuria. No other abnormalities are found. b. CXR swelling occurred after eating a large meal. What
What is the best next step in management? c. CT is the single most likely dx?
a. Biguanide d. TFT a. Laryngocele
b. Sulfonylurea e. US abdomen b. Ranula
c. Insulin Ans. The key is A. Sputum for AFB. [Features are c. Neck abscess
d. Sugar free diet suggestive of PTB for which most discriminating d. Parotid calculus
e. ACEi inv. Is Sputum for AFB]. e. Submandibular calculus
Ans. The key is A. Biguanide. [Patient is obese Ans. The key is E. Submandibular calculus. [Pain
type2 diabetic with normal renal function for 1217. A prv healthy 23yo presented a week hx of and swelling below mandible is due to stone in
whom biguanide is the treatment of choice]. bloody diarrhea and abdominal pain with cramps submandibular salivary gland duct. Pain and
and fever. Exam: tenderness in lower abdomen. swelling increased after large meal as the saliva
1213. What are the side effects of thiazide What is the most appropriate dx? produced during meal could not come out of the
diuretics? a. Celiac disease gland due to stone].
a. Hypocalcemia b. Colorectal polyps
b. Hyponatremia c. UC 1222. A 45yo man has had impaired vision and
c. Hypernatremia d. Laxative abuse pain on eye movement in his left eye over the last
d. Hyperkalemia e. Gastroenteritis 5d.
Ans. The key is B. Hyponatremia. [Thiazide causes Ans. The key is E. Gastroenteritis. [A short history He also notes loss of color vision in the same eye.
hypercalcemia, hypokalemia and hyponatremia]. (one week) of bloody diarrhea, abdominal pain In the left eye, the visual acuity is up to
with cramps, fever and tenderness in lower counting fingers. When the pupil is stimulated
1214. A 46yo man who is a heavy drinker is abdomen is indicative of gastroenteritis]. with light, it dilates. His fundus is normal. What is
brought to the ED in a drowsy state. He is the single most appropriate clinical dx?
responding 1218. A 10yo boy presents with irritability, sudden a. Acute dacryocystitis
vaguely to questions. Exam: nystagmus and onset of pain and discharge from the right ear. b. Acute iritis
hyperreflexia. MCV=103fl. What is the most likely Which antibiotic would be the 1st line of tx? c. Papillitis
cause for his cognitive impairment? a. Amoxicillin d. Retrobulbar neuritis
e. Scleritis a. Below elbow full plaster of paris nearer is D. Left main stem, post. descending
Ans. The key is D. Retrobulbar neuritis. b. Below elbow split plaster of paris artery].
[Presentation can be described by either optic c. Closed reduction of fx
neuritis or retrobulbar neuritis. In optic neuritis d. Elasticated support bandage 1230. A 55yo female presented with anemia and
there is disc pallor. As in this case disc is normal it e. Open reduction and internal fixation dysphagia. There is a feeling of something stuck in
is a case of retrobulbar neuritis]. the throat. The esophagus can’t be negotiated
Ans. The key is C. Closed reduction of fracture. beyond the crico-pharynx. What is the most
1223. A 56yo pt has been dx with MS. She This is a wrong key! Correct option seems to be A. probable dx?
presents with a positive Romberg’s test. She also Below elbow full plaster of paris. [Older people a. Foreign body
has are less likely to go under closed reduction even if b. Plummer vinson syndrome
weakness and loss of sensations in all her 4 limbs. the dorsal angulation is not anatomic (like 20 c. Pharyngeal carcinoma
Which site is most likely to be affected? degrees!). Less than 10 degree dorsal angulation d. Barret’s esophagus
a. Cerebral cortex does not require even closed reduction]. e. Esophageal carcinoma
b. Cerebellum
c. Cervical spinal cord 1227. A 16yo girl who is normally fit and well Ans. The key is B. Plummer Vinson syndrome.
d. Thoracic spinal cord attends her GP complaining of heavy and painful [The picture fits two D/D. 1. Plummer Vinson
e. Brain stem periods. syndrome 2. Oesophagial carcinoma. Lower
She is requesting tx for these complaints. She oesophagial stricture is more common for Ca. So
Ans. The key is C. Cervical spinal cord. denies being sexually active. Select the most presenting case is likely a case of Plummer Vinson
[Quadriplegia is the feature of cervical cord appropriate management for her menorrhagia? syndrome. It can be differentiated by the type of
lesion]. a. Antifibrinolytics (tranexamic acid) anemia. If it is IDA dx is Plummer Vinson
b. COCP syndrome].
1224. A 58yo man suddenly becomes shocked c. Endometrial ablation
several days after suffering an acute ant MI. His d. IUS progestrogens (mirena) 1231. A pt is on cancer tx with dexamethasone.
CXR e. NSAIDS (mefenamic acid) According to her biochemical results her
shows a large globular-shaped heart and clear K+=normal
lung fields. What is the single most likely Ans. The key is A. Antifibrinolytics (tranexamic and her Na+=low. What is the dx?
explanation for the abnormal inv? acid). It is a wrong key. Correct answer is E. a. Addisons
a. Acute pericarditis NSAIDs (mefenamic acid). b. Dexamethasone side effect
b. Cardiac tamponade c. Dilutional hyponatremia
c. Atrial thrombus 1228. A 67yo lady with an ulcer on the anal
d. Left ventricular aneurysm margin. Which is the single most appropriate LN Ans. The key is C. Dilutional hyponatremia.
e. Dressler syndrome involved?
a. External iliac LN 1232. A diabetic has been prescribed a long acting
Ans. The key is B. Cardiac tamponade. [Shock in a b. Pre-aortic LN hypoglycemic in the morning and short acting in
post MI patient with globular heart on X-ray and c. Aortic LN the
clear lung field indicate Cardiac tamponade]. d. Inguinal LN evening. He takes a regular lunch, but has been
e. Iliac LN having hypoglycemic attacks at around 4pm
1225. A 56yo alcoholic man who has increased the each day. What is the most appropriate
amount of alcohol he is using wants to attend his Ans. The key is D. Inguinal LN. [Anal canal below intervention?
daughter’s wedding that is in 2wks. He is now pectinate line is drained into superficial inguinal a. Recommend a heavier lunch
coming to you for help. How would you help him? lymph nodes]. b. Review morning drug
a. Acamprosate c. Review evening drug
b. Refer to clinical psychologist 1229. A branch of the dominant coronary artery d. Review both drug
c. Refer to GP that supplies the inferior portion of the septum. e. Reassure
d. Despiramine What
e. Refer to community mental health support is the single most appropriate option? Ans. The key is B. Review morning drug. [As there
group. a. Septal branches is hypoglycemic attacks at evening morning dose
b. Obtuse marginal branches needs to be adjusted].
Ans. The key is A. Acamprosate. [If the patient c. Circumflex artery
stop alcohol without any supportive treatment d. Left main stem, post descending artery 1233. A male pt presented with blood and mucus
there will occur withdrawal symptoms. For the e. Diagonal branch in stool. He has also noticed weight loss but has
presented situation Acamprosate can help by no
stopping alcohol without producing withdrawal Ans. The key is D. Left main stem, post descending hx of altered bowel habits. What is the dx?
symtoms (by restoring brain chemical artery. [Here no option is satisfactorily correct! By a. Carcinoma of cecum
derangement caused by alcohol which is dominant coronary artery we mean that coronary b. Carcinoma of descending colon
responsible for withdrawal symptoms)]. artery which gives of the branch of posterior c. Carcinoma of sigmoid colon
descending artery. Mostly it is right coronary d. Carcinoma of rectum
1226. An 80yo woman fell over at her nursing artery and if there is left coronary dominance,
home. XR shows fx of radius with <10degree of posterior descending artery is the branch of Ans. The key is A. Carcinoma of cecum.
dorsal circumflex artery and not direct branch of left
angulation. What is the single most appropriate main artery. However the only option that goes
tx?
1234. A 22yo man keeps having persistent and pulse and BP with no abnormal neurological sign.
intrusive thoughts that he is a dirty thief. No Ans. The key is D. Give all vaccines except BCG What is the next step in her management?
matter vaccine. [There are two terms HIV +ve (not a. Admission for observation
what he tries these thoughts keep coming to him. immune deficient yet) and AIDS (immune b. CT brain
Any attempt to avoid these thoughts leads to deficient state). In HIV +ve case live attenuated c. MRI head
serious anxiety. What is the most likely dx? vaccines can be given except BCG and if AIDS all d. Reassurance and discharge home
a. Schizophrenia live attenuated vaccines are avoided]. e. XR skull
b. OCD
c. PTSD 1238. A 36yo man has been dx with DI. What Ans. The key is B. CT brain. [To exclude any
d. Mania electrolyte picture is expected to be seen? possible intracranial bleeding].
e. Psychotic depression a. High serum Na, low serum osmolarity, high
urine osmolarity 1243. A 30yo woman is taking tx for asthma. She
Ans. The key is B. OCD. [Though here no obsession b. Low serum Na, low serum osmolarity, high has a HR=130bpm and peak expiratory flow
or thoughts but no compulsion of activity but urine osmolarity rate=400.
there is compulsion of thoughts as well which c. Low serum Na, high serum osmolarity, high What is the most appropriate management?
makes the diagnosis as OCD]. urine osmolarity a. Atenolol
d. High serum Na, high serum osmolarity, low b. Digoxin
1235. A 45yo female comes to the ED while urine osmolarity c. Review drugs
having a generalized tonic clonic seizure and she e. Normal Na, normal serum osmolarity, normal
has urine osmolarity Ans. The key is C. Review drugs. [Oral beta agonist
having difficulty breathing and is cyanosed. What frequently causes tachycardia and palpitations].
is the tx option for her? Ans. The key is D. High serum Na, high serum
a. Secure airways osmolarity, low urine osmolarity. 1244. A pt presents with a mask face. He also has
b. IV diazepam gait prbs. Which class of drug is causing this?
c. IV phenytoin 1239. The artery that supplies the ant right a. Anti-depressant
d. Oxygen mask ventricular wall. What is the single most b. Anti-psychotic
appropriate c. Anti-HTN
Ans. A. Secure airways. [Breathing difficulty with option?
cyanosis may indicate saliva, blood, foreign body a. Acute marginal branch Ans. The key is B. Anty-psychotic. [Anti-psychotics
(as denture), fall back of tongue etc. for which 1st b. Left ant descending artery by reducing dopamine can precipitate or induce
action is to secure airway]. c. Coronary sinus parkinsonism].
d. Circumflex artery
1236. A 30yo man is becoming concerned about e. Right coronary artery 1245. A 16yo boy came home from boarding
the safety of his family. He has been checking the school with a cough. His CXR showed bilateral
locks of the door every hour during the night. He Ans. The key is A. Acute marginal branch. consolidations. What is the most likely organism
becomes very anxious if his wife tries to stop which would have caused his symptoms?
him. What is the most likely dx? 1240. A 55yo male presents to the ED after an a. Legionella pneumophilia
a. Paranoid delusion RTA with breathlessness, engorged neck veins and b. Mycoplasma pneumonia
b. PTSD a c. Mycobacterium TB
c. Social phobia dull percussion note on the right side of his chest. d. Pneumocystis jiroveci
d. OCD Exam: pulse=140bpm, BP=80/50mmHg. What e. Pseudomonas aeruginosa
e. GAD is the most likely dx?
a. Hemothorax Ans. The key is B. Mycoplasma pneumonia.
Ans. The key is D. OCD. [Obsessive compulsive b. Hemopneumothorax [Mycoplasma is common in military barrack,
disorder (OCD) is a mental health condition where c. Tension pneumothorax prison or boarding dwellers].
a person has obsessive thoughts and compulsive d. Simple pneumothorax
activity. 1246. After an MI, a man presents with
An obsession is an unwanted and unpleasant Ans. The key is B. Hemopneumothorax. pansystolic murmur which is radiating to the
thought, image or urge that repeatedly enters a axilla. What is the
person's mind, causing feelings of anxiety, disgust 1241. A 32yo woman presents with complaints of dx?
or unease. having low back pain. She is taking analgesics for a. Tricuspid regurgitation
A compulsion is a repetitive behaviour or mental it. All inv are normal. What will you advice her? b. Mitral regurgitation
act that someone feels they need to carry out to a. Bed rest c. Aortic stenosis
try to temporarily relieve the unpleasant feelings b. Physiotherapy d. Mitral stenosis
brought on by the obsessive thought]. c. Advice to be more active
d. Admit Ans. The key is B. Mitral regurgitation. [MI can
1237. A 6wk baby has been dx as HIV+ve. Which lead to papillary muscle rupture causing mitral
immunization plan will you opt for him? Ans. C. Advice to be more active. regurgitation].
a. Don’t give any vaccine
b. Give all vaccines except live attenuated 1242. A 32yo woman suffers an episode of severe 1247. A 34yo labourer developed severe pain in
vaccines occipital headache with vomiting and LOC. She is his lower back after lifting a sack of sand. He also
c. Give only BCG vaccine brought to the hosp where she is found to be complains of shooting pain down his leg. The GP
d. Give all vaccines except BCG vaccine conscious and completely alert. Exam: normal has prescribed him complete bed rest, with
painkillers and also scheduled an MRI for him. a. Anorexia nervosa 1255. The artery that runs along the left AV
What is the most likely dx? b. Bipolar groove. What is the single most appropriate
a. Peripheral vascular disease c. OCD option?
b. Intervertebral disc prolapse d. Bulimia a. Left internal mammary artery
c. Hairline fx of the spine b. Left anterior descending artery
d. Sprain of the back muscles Ans. The key is D. Bulimia. [BMI 22, even though c. Circumflex artery
e. Muscle injury unhappy, hypokalemia, like to have dinner in an d. Left main stem (LMS) post descending artery
expensive restaurant (probable binge eating) e. Diagonal branch
Ans. The key is B. Intervertebral disc prolapse. suggest the diagnosis of bulimia].
[Intervertebral disc prolapsed can cause severe Ans. The key is C. Circumflex artery.
back pain following heavy lifting and 1252. A 59yo pt has been dx with HTN. His BP has
radiculopathy (pressure on spinal nerve root) can been >160/90mmHg on 3 separate occasions. His 1256. A 26yo man presents with painless
cause this shooting pain which can be biochemical profile is as follows: Na+=145mmol/l, hematuria. He has no other complaints and on
demonstrated by MRI spine]. K+=6.2mmol/l, creatinine=112umol/l, examination
urea=5.7mmol/l. What is the most appropriate no other abnormality is found. What is the most
1248. A young man returns to his hostel and gets anti-HTN drug for him? appropriate initial inv to get to a dx?
headache and lethargy. Now presents with fever. a. Amlodipine a. Cystoscopy
There are crepitations on the auscultation of lung. b. Bendroflumethiazide b. Midstream urine for culture
What is the most likely organism which would c. Ramipril c. Abdominal US
have caused his symptoms? d. Lorsartan d. MRI spine
a. Legionella pneumonia e. Propranolol e. Coag screening
b. Mycoplasma
c. Staphylococcus Ans. The key is B. Bendroflumethiazide. [Thiazide Ans. The key is C. Abdominal ultrasound.
d. Streptococcus reduces Na+ and K+ level. So in this picture of [Abdominal US to exclude polycystic disease,
upper normal Na+ with hyperkalemia thiazide malignancy, tumour, urolithiasis etc].
Ans. The key is A. Legionella pneumonia. seems to be appropriate].
[Legionella is common in hostel, hotel, hospital, 1257. A pt, 50yo smoker and heavy drinker,
nursing home where it spread through their water 1253. A 22yo girl had a fight with her boyfriend presents with complaints of racing heart. A 24h
system, aircondition etc.]. and then took 22 tabs of paracetamol. She was EKG
commenced on N-acetyl cysteine and she was comes out normal. What is your next step in
1249. A pt is about to undergo surgery. Her medically fit to go home the following day. Which management?
Hgb=8.9g/dl and MCV=70. What is the best option of the following does she require? a. Echo
for her? a. OPD referral to relationship counselor b. Reassure
a. Inv and postpone the surgery b. OPD referral to psychiatrist c. Stress test
b. Transfuse and proceed with surgery c. Inpatient referral to psychiatrist
c. Transfuse and defer surgery d. Inpatient referral to psychologist Ans. The key is B. Reassure. [Racing heart or
d. Continue with surgery palpitation is a common phenomenon in
Ans. The key is C. Inpatient referral to psychiatrist. alcoholics which is not serious or harmful. So
Ans. Key is A. Investigation and postpone the [Suicidal ideation is abnormal and if discharged reassure the patient].
surgery. [For elective operation only proceed if Hb there is chance of repeated suicidal attempts. So
% > 10 g/dl. If Hb% < 10 g/dl then defer the inpatient referral to psychiatrist is needed for this 1258. A 36yo woman came with uterine bleeding.
operation and investigate first. If Hb% < 8 g/dl patient]. Vaginal US reveals uterine thickness=12mm. what
also must be transfused. Samson note]. is
1254. A 74yo man presents with sudden onset of the most probable dx?
1250. A 24yo male presents with discomfort in with right sided weakness and slurred speech. He a. Cervical ca
the groin area and scrotal swelling. Exam: scrotal also has loss of sensation over the right side of the b. Endometrial ca
skin is body and visual field defects. CT shows c. Ovarian ca
normal. What would be the next best step? ischemic stroke. What is the most appropriate d. Breast ca
a. Urgent US management? e. Vaginal ca
b. Urgent surgery a. Alteplase
c. OPD referral b. Streptokinase Ans. The key is B. Endometrial Ca. [Actually it is
d. Antibiotics c. Nimodipine endometrial thickening and in general 12 mm
d. Aspirin means hyperplasia which may suggest
Ans. The key is C. OPD referral. [A case of inguinal e. Labetolol endometrial Ca in this woman].
hernia or hydrocele. In both torsion or epididymo-
orchitis scrotal skin will be inflammed Ans. The key is A. Alteplase. [Patient has acute 1259. A 30yo woman has PID which was treated
(erythematous and oedematous). Should be onset of symptoms and time of onset is also with metronidazole and cephalosporin. It is
referred to Surgery OPD]. known. So considering window period of 4.5 getting
hours from onset of symptoms can be given worse. What is the next best inv?
1251. A 22yo girl unhappy about her weight with alteplase]. a. Endocervical swab
BMI=22. She likes to have her dinner in an b. US
expensive restaurant. She does excessive c. Laparotomy
shopping. K+=3.3. What is the dx? d. High vaginal swab
Ans. The key is D. Secondary PPH. [Secondary PPH Ans. The key is A. Repeat vaginal examination in
Ans. The key is B. US. [Probable tubo-ovarian occurs from 24 hours after delivery. Usually 4h.
abscess]. occurs between 5 and 12 days].
1268. A 36yo pregnant woman comes for
1260. A pregnant woman had hit her chest 3wks 1264. A 22yo lady who is in her last trimester of evaluation with her husband. Her husband has
ago. Now she is 24wks pregnant and presents pregnancy comes with hx of exposure to a child dx been
with with complaining of morning sickness, easy
left upper quadrant mass with dimpling. What is chicken pox 1d ago. She was investigated and was fatiguability and even intermittent abdominal
the most probable dx? +ve for varicella antibody. What is the single pain. What
a. Breast ca most appropriate management? is the husband suffering from?
b. Carcinoma a. Give varicella Ig a. Ganser syndrome
c. Fibroadenoma b. Quarantine b. Couvade syndrome
d. Fibroadenosis c. Give varicella vaccination c. Pseudo-psychosis
e. Fatty necrosis of breast d. Oral acyclovir d. Stockholm syndrome
e. Reassure e. Paris syndrome
Ans. The key is E. Fatty necrosis of the breast.
Ans. The key is E. Reassure. [If you have Ans. The key is B. Couvade syndrome. [Cauvade
1261. A pregnant pt with Rh –ve who hasn’t been antibodies in your blood, this means you have had syndrome, also called sympathetic pregnancy, is a
prv sensitized delivers her first baby without any chickenpox in the past, or have been immunised. proposed condition in which a partner
prbs. What would be the latest time to administer No further action is then needed]. experiences some of the same symptoms and
anti-sensitization? behavior of an expectant mother].
a. 6h PP 1265. A 22yo woman who is 20wk pregnant came
b. 24h PP with pain and bleeding per vagina. Exam: os is not 1269. A woman comes to the ED complaining of
c. 48h PP open. What is the single most likely dx? pain in the right side of the abdomen, she has
d. 72h PP a. Threatened abortion 7wks
e. 5d PP b. Missed abortion amenorrhea. Her pregnancy test is +ve and US
c. APH scan shows an empty uterus. What is the next
Ans. The key is D. 72h PP. d. Miscarriage step?
e. Inevitable abortion a. Laparoscopy
1262. A 30yo primigravida who is 30wks GA b. HCG measurements
presents to the L&D with absent fetal Ans. The key is A. Threatened abortion. c. US
movements. She d. Laparotomy
also complains of severe headache, heartburn and 1266. A 32yo lady G1, 28wks GA came to her ANC e. Culdo-centhesis
seeing floaters before her eyes for the last with a concern about pain relief during labour.
few days. Exam: BP=170/110mmHg, urine She
protein=++++, rock hard uterus, no visible signs of has no medical illnesses and her pregnancy so far
fetal movements. Choose the single most likely has been uncomplicated. She wishes to feel Ans. B. HCG measurements. This is a wrong
dx? her baby being born but at the same time she
key! Correct key is A. Laparoscopy.
a. Abruption of placenta 2nd pre-eclampsia wants something to work throughout her labour.
b. Antepartum hemorrhage What method of pain relief best matches this [Surgery should be offered to those
c. Placenta previa lady’s request?
d. Primary PPH a. C-section women who cannot return for follow-up
e. IUFD b. Pudendal block after methotrexate or to those who have
f. Abruption of placenta due to trauma c. Entonox
d. TENS any of the following:
Ans. 2 keys A. Abruption of placenta 2nd pre- e. Pethidine
eclampsia. E. IUFD.
 Significant pain.
Ans. The key is C. Entonox. [This is a mixture of
1263. A 38yo woman, 10d post partum, presents oxygen and nitrous oxide gas. Gas and air won't  Adnexal mass ≥35 mm.
to her GP with a hx of passing blood clots per remove all the pain, but it can help to reduce it
vagina and make it more bearable].  Fetal heartbeat visible on scan.
since yesterday. Exam: BP=90/40mmHg,  Serum hCG level ≥5000 IU/L.
pulse=110bpm, temp=38C, uterus tender on 1267. A primipara at fullterm in labor has passed
palpation show and the cervix is 3cm dilated. What is the
and fundus is 2cm above umbilicus, blood clots ++ single A laparoscopic approach is preferable.
+. Choose the single most likely dx? most appropriate management for her labor?
a. Abruption of placenta 2nd preeclampsia a. Repeat vaginal examination in 4h A salpingectomy should be performed,
b. Concealed hemorrhage b. CTG unless the woman has other risk factors
c. Primary PPH c. IV syntocin drip
d. Secondary PPH d. Repeat vaginal examination in 2h for infertility, in which case a
e. Retained placenta e. Induction of labour
salpingotomy should be undertaken].
f. Scabies
[If HCG >6000IU/L and an intrauterine gestational
sac is not seen, ectopic pregnancy is very likely, as
is the case if HCG 1000–1500IU/L and no sac is a. Abdominal US
seen on transvaginalUltrasound]. b. Anti-phospholipid screen 1279. A woman who is 7wks pregnant presents
c. CBC with excessive and severe vomiting and put on IV
1270. A 23yo woman who has had several recent d. Transvaginal US fluids and anti-emetic (ondansteron). She is
partners has experienced post-coital bleeding on e. Laparoscopy complaining of severe headache and can’t take
gentle contact. What is the single most likely oral
cause of her vaginal discharge? Ans. The key is D. Transvaginal US. [D/D: ectopic fluids. What is the most appropriate
a. Cervical ca pregnancy, renal colic, torsion of ovarian cyst, management?
b. Cervical ectropion appendicitis etc. Transvaginal US will give better a. Termination of pregnancy
c. CIN results in case of lower abdominal pain]. b. TPN
d. Chlamydial cervicitis c. Feeds via NGT
e. Gonococcal cervicitis 1275. A pt is at term and in labor, the membranes d. P6 acupressure
have ruptured, the liquor contains meconium but e. IV hydrocortisone
Ans. D. Chlamydia cervicitis. the CTG is normal. The cervix is 3cm dilated. What
is the single most appropriate action? Ans. The key is A. Termination of pregnancy. This
1271. A 68yo woman presents with post-coital a. BP monitoring is wrong key. Correct key is E. IV hydrocortisone.
bleeding following her first episode of sexual b. CTG [Termination is the last resort! Before it IV
intercourse in 10yrs. What is the single most likely c. C-section hydrocortisone is tried. 1st oral antiemetics if fail
cause that has led to post-coital bleeding? d. Fetal scalp blood sample perenteral antiemetics if fail iv hydrocortisone.
a. Endometrial ca e. Internal rotation Last resort is termination].
b. Atrophic vaginitis
c. Endometrial polyp Ans. The key is D. Fetal scalp blood sample. This is 1280. A young lady with primary amenorrhea has
d. Cervical ca wrong key. Correct key is CTG. [Till CTG is normal normal LH, FSH, estradiol and prolactin. Choose
e. Cervical ectropion we should not switch to other option but the
continuing CTG]. single most likely dx?
Ans. The key is B. Atrophic vaginitis. a. PCOS
1276. A pt is at term and labor. The head has been b. POF
1272. A 28yo woman 8wks GA had PID treated delivered and you suspect shoulder dystocia. c. Absent uterus
prvly and now comes with vaginal bleeding, rigid What d. Absent ovaries
abdomen, BP=80/50mmHg, pulse=140bpm. What is the single most appropriate action? e. Turner’s syndrome
is the most probable dx? a. C-section
a. Threatened abortion b. Episiotomy Ans. The key is C. Absent uterus. [Normal LH, FSH,
b. Miscarriage c. External rotation estradiol and prolactin rule outs PCOS (increased
c. Missed abortion d. Fetal scalp blood sample LH, increased FSH, normal oestrogen. LH:FSH ratio
d. Tubal pregnancy e. Instrumental delivery is 2:1 or 3:1), POF (in POF, LH & FSH raised, FSH >
e. Inevitable abortion 20 IU/L), Absent ovary will lead to low estradiol,
Ans. The key is B. Episiotomy. high FSH and LH, Turner’s syndrome: gonadal
Ans. The key is D. Tubal pregnancy. [Previous PUD streaks, as absent ovaries].
is a risk factor for tubal pregnancy]. 1277. A 29yo female at 28wks GA presents to you
with complains of hard stools and constipation for 1281. An obese lady presents with primary
1273. A 34yo primigravida who is 16wk GA comes last 2wks. CTG shows fetal tachycardia. What is amenorrhea. She has high LH, normal FSH and
for routine antenatal check up. Her the single most appropriate tx? slightly high
BP=160/100mmHg. She has a hx of repeated a. Oral laxatives prolactin levels. Choose the single most likely dx?
childhood UTI. What is the most likely cause of b. Fiber diet a. PCOS
her c. Phosphate enema b. POF
high BP? d. Lactulose c. Hypothyroidism
a. Essential HTN e. Reassure d. Pregnancy
b. Chronic pyelonephritis e. Primary obesity
c. Acute pyelonephritis Ans. The key is B. Fiber diet. [Changes to diet and
d. Pre-eclampsia lifestyle are often recommended as the first Ans. The key is A. PCOS. [High LH, high or normal
e. Chronic UTI treatment for constipation. It includes high fiber FSH with slight rise in prolactin levels in an obese
diet and plenty of fluid, regular exercise etc.]. lady is suggestive of PCOS].
Ans. The key is B. Chronnic pyelonephritis. [35% of
childhood UTI is associated with VUR and many of 1278. A 16yo girl presents with heavy bleeding. 1282. A 38yo lady presents with amenorrhea has
them develop renal scarring and chronic What is the most appropriate initial inv? very high LH and FSH levels, normal prolactin and
pyelonephritis causing hypertension]. a. Endometrial sampling low estradiol. Choose the single most likely dx?
b. Transvaginal US a. PCOS
1274. A 24yo woman has had lower abdominal c. Hysteroscopy b. POF
pain for 12h. She is otherwise well. She is at d. Pelvic US c. Hypothyroidism
10wks GA e. Exam under anesthesia d. Pregnancy
in a planned pregnany. What is the single most e. Menopause
appropriate test to inv the cause of acute Ans. The key is D. Pelvic US. [Pelvic or transvaginal
abdomen in this lady? ultrasound are same thing].
Ans. The key is B. POF. [High LH and FSH, normal non fertile. So UAE is the only suitable option
prolactin and low estrogen in secondary here]. 1291. At birth, a baby boy at 38wks GA weighs
amenorrhea in a lady under age 40 is highly 1.8kgs. He has hepato-splenomegaly and a rash.
suggestive of POF]. 1287. A pt comes with sudden loss of vision. Blood test show raised level of bilirubin and liver
Exam: high BP. Fundoscopy: retina appears enzymes. What is the most likely dx?
1283. A 77yo publican was admitted for an swollen. a. Galactosemia
appendectomy. Post-op he becomes confused, Which blood vessel occlusion is involved? b. Biliary atresia
agitated a. Branch RVO c. G6PD deficiency
and starts to pick at things. He is then given an IV b. Branch RAO d. Rh incompatibility
drug which settles this confusion. Which of the c. CRAO e. Congenital viral infection
following drugs was given for his confusion? d. CRVO
a. Diazepam Ans. The key is E. Congenital viral infection.
b. Chlordiazepoxide Ans. The key is D. CRVO. [Congenital infections affect the unborn fetus or
c. Thiamine newborn infant. They are generally caused by
d. Vit B 1288. A 2yo girl has had a temp=39C, poor viruses that may be picked up by the baby at any
appetite, abdominal pain and urinary frequency time during the pregnancy up through the time of
Ans. The key is B. Chlordiazepoxide. [Patient is for 3d. delivery. The more common viruses linked to
alcoholic. On admission abstinence from alcohol What is the single most appropriate inv? congenital infections include the Cytomegalovirus
caused this withdrawal symptom. Agitation, a. Catheter specimen of urine for culture (CMV), Herpes, Rubella (German measles),
confusion and pick at things are suggestive of b. Clean catch urine specimen for culture Parvovirus, Varicella (chickenpox), and
delirium tremens which is treated with c. CBC Enteroviruses].
Chlordiazepoxide]. d. KUB US
e. Supra-pubic aspirate of urine for culture 1292. A 12yo boy with T1DM has poor long-term
1284. A 65yo lady presents with dyspareunia. control. He is unconscious, hyperventilating and
What will you give her for her condition? Ans. The key is B. Clean catch specimen of urine dehydrated. His blood glucose is 28mmol/l. What
a. HRT for culture. is the single most imp initial tx?
b. COCP a. Albumin IV
c. Estrogen gel 1289. A child with T1DM who is not compliant b. Bicarbonate IV
d. Testosterone gel with meds and eats a lot. He thinks that he is c. Insulin IV
short in d. Insulin SC
Ans. No key is given. Likely key is C. Estrogen gel. his class. He is not happy. What would you do e. Saline 0.9% IV
[Seems to be atrophic vaginitis for which estrogen next?
gel can be given]. a. Refer to psychologist Ans. The key is E. Saline 0.9% IV. [Probable
b. Refer to pediatrician diabetic ketoacidosis. Initial treatment is IV fluid
1285. A 35yo lady with subserosal fibroid=4cm c. Refer to GP (0.9% saline)].
and submural fibroid=6cm is planning for a child. d. Refer to social services
Which way will you remove the fibroids? e. Change type of insulin 1293. A 30yo woman on OCP presents with
a. Laproscopy dilated tortuous veins crossing her abdomen to
b. Vaginal myomectomy Ans. The key is A. Refer to psychologist. [Not join the
c. Abdominal myomectomy compliant with medicine, eating a lot, thinking of tributaries to SVC. What is the single most likely
d. Drugs being short and being unhappy these are cause?
e. Reassure psychological issues. So he should be referred to a. Intra-abdominal malignancy
psychologist]. b. Ovarian cyst
Ans. The key is B. Vaginal myomectomy. Probably c. Fibroids
wrong key! Correct key should be C. Abdominal 1290. An 8yo boy with a BMI=28 was admitted to d. Ascites
myomectomy. [Subserosal fibroid is not suitable a surgical ward following a MVC. He was found to e. DVT
to treat with vaginal myomectomy. Abdominal have glycosuria. When he recovered from his
myomectomy can deal with both subserosal and injury the glycosuria resolved. What is the single Ans. The key is A. Intra-abdominal malignancy.
submural fibroid]. most appropriate follow-up inv?
a. Fasting blood glucose conc 1294. An 84yo woman with drusen and yellow
1286. A 32yo presents with heavy blood loss, US: b. Glycosylated hemoglobin - HbA1c spots in the center of retina. What is the single
uterine thickness>14mm. What is the best c. OGTT most
possible d. Random blood glucose conc likely dx?
management for her? e. Serum cortisol conc a. Macular degeneration
a. COCP b. HTN retinopathy
b. UAE Ans. The key is A. Fasting blood glucose c. MS
c. Hysteroscopy myomectomy concentration. [The boy had glycosuria while d. DM background
d. Abdominal myomectomy treating in hospital following a MVC (motor e. Proliferative DM retinopathy
e. Endometrial ablation vehical crush). It may be due to stress related
cortisol release which later became nil as the boy Ans. The key is A. Macular degeneration. [In early
Ans. B. UAE. [COCP will not resolve the case. recovered from trauma. But yet to certain that he disease, the macula shows yellowish-colored
There is no fibroid so no myomectomy. is not diabetic fasting blood glucose concentration subretinal deposits called “drusen”].
Endometrial ablation may render the young lady should be done in follow up visit].
1295. A pt presents with headache, blurring of management? a. Sulfasalazine
vision and acuity loss. On fundoscopy, dots and a. Admission for parental b. Paracetamol
blots antibiotics c. Metronidazole
were noted with huge red swollen optic disc. b. Nasal decongestant d. Ibuprofen
What is the most probable dx? c. Oral amoxicillin Ans. The key is C. Metronidazole. It is a wrong key.
a. CRAO d. OPD review Correct key should be A. Sulfasalazine. [Drug of
b. Branch RAO e. Packing of ear 1st preference is Steroids, then
c. CRVO f. Surgical intervention immunosuppressants, then amynosalicylates
d. Optic atrophy g. Syringing ENT (such as sulfasalazine). Metronidazole is used if
there is infective complications like infected
Ans. The key is C. CRVO. Ans. The key is C. Oral amoxicillin. [Small perianal fistula Ref: patient.info].
perforations are usually heal by themselves within
1296. A 64yo DM has come for a routine eye 6-8 weeks and doctors often prescribe antibiotics 1303. A 62yo prv shipyard worker complains of
check up. Fundoscopy: new vessels all over the to prevent infections during this healing period]. breathlessness and chest pain for 6m. He has now
retina. developed a large pleural effusion. Which is the
What is the most appropriate management? 1300. A 45yo man has noticed difficulty hearing single best diagnostic inv?
a. Strict sugar control on the telephone. He is concerned because his a. ABG
b. Regular eye check ups father b. Bronchoscopy
c. Non urgent referral to specialist has been moderately hard of hearing since middle c. CXR
d. Laser photocoagulation age. BC=normal. An audiogram shows d. Pleural biopsy
e. Insulin moderate hearing loss in both ears across all e. Transfer factor
frequencies. What is the single most likely dx?
Ans. The key is D. Laser photocoagulation. a. Acoustic neuroma Ans. The key is D. Pleural biopsy. [Shipyard worker
[Neovascularization suggests proliferative diabetic b. Menieres’ disease is exposed to asbestos and the likely diagnosis
retinopathy which is treated with laser c. Noise induced deafness here is mesothelioma for which single best
photocoagulation]. d. Otosclerosis investigation is pleural biopsy].
e. Presbyacusis
1297. A 25yo primigravida of 8wk GA presents 1304. A 67yo man presents with a hx of increasing
with severe lower abdominal pain, vaginal Ans. The key is D. Otosclerosis. [This patient has confusion and drowsiness. He had a fall 2wk ago.
bleeding and conductive hearing loss. So it is not acoustic CT
passage of clots. The internal os is open. What is neuroma. Meniere’s disease has symptoms like head reveals a chronic subdural hematoma. What
the most likely dx? vertigo, tinnitus, fullness, along with hearing loss. is the best management for this pt?
a. Appendicitis There is no history of noise pollution not a. Craniotomy
b. Placental abruption presbycausis as his age is 45. So his deafness fits b. Burr hole drainage
c. Ectopic pregnancy more with otosclerosis. These are by exclusion. c. Conservative management
d. Abortion Points in favour of otosclerosis: i) Conducting d. Excision and biopsy
hearing loss. ii) Age of 45 yrs iii) Moderate hearing
Ans. The key is abortion. [Up to 24 weeks loss in both ears across all frequencies]. Ans. The key is B. Burr hole drainage.
termination of pregnancy is abortion. Ref:
patient.info] 1301. The biological parents of a child with CF 1305. A 45yo male with epigastric discomfort has
come to you to know about the chances of their been given triple therapy. He has now returned
1298. A man developed intense pain after using future children with the same disease. What after
the end of a pencil to scratch his inner ear. He would you say to them? 4wks of epigastric discomfort. What inv would
took a. There is a 1:4 chance that your future child will you do for him?
out the pencil from his ear and realized the end of have this disease a. ECG
the pencil with the rubber part is still stuck in b. All their unaffected children will be carriers of b. H.pylori breath test
his ear. What is the most appropriate CF c. Endoscopy and biopsy
management? c. Nothing can be predicted d. US
a. Remove with a hook d. It can 100% dx antenatally
b. Instill olive oil Ans. The key is A. There is a 1:4 chance that your Ans. The key is C. Endoscopy and biopsy. This is a
c. Remove GA future child will have this disease. [It is autosomal wrong key! Correct key is B. H. pylori breath test.
d. Remove with magnet instrument recessive disease. As one child is affected both
e. Do syringing parents are carrier. So in this case next child will
be normal in 1:4 case, carrier in 1:2 cases and
Ans. The key is A. Remove with a hook. [Place a diseased in 1:4 cases].
hook behind the object and pull it out].
1302. A 14yo boy presents with recurrent
1299. A 16yo boy presents with acute pain in the abdominal pain, malaise and weight loss over 6m.
right ear and little bleeding from the same ear. He Exam:
had been in a boxing match and had sustained a vague mass is felt in RIF. Colonoscopy shows
blow to the ear. There is little amount of blood transmural inflammation and granulomata. What 1306. A 13yo boy with umbilical pain for the last
in the auditory canal and a small perforation of is 12h presents with anorexia, nausea and has not
the eardrum. What is the most appropriate the most appropriate management? passed a bowel motion 24h. What is your dx?
a. Acute appendicitis a. IV aminophylline
b. IBD b. IV magnesium sulphate Ans. The key is B. Cheek. [It also involve soft
c. IBS c. IV salbutamol palate but not hard palate. So cheek is more
d. Meckel’s diverticulum d. IM adrenaline complete answer than palate. Conjuctiva is more
e. Muscle strain e. IV adrenaline affected in ophthalmic division of trigeminal
f. Ovarian cysts nerve].
g. PID Ans. The key is B. IV magnesium sulphate.
h. Psoas hematoma 1313. A 52yo man presents with visual
i. Pyelonephritis 1309. A 49yo man first presented with increasing hallucinations and features of cognitive
j. Uretric calculus difficulty in swallowing. Several months later he impairment. What is
developed weakness in his right foot. Now he can the most likely dx?
Ans. The key is D. Meckel’s diverticulum. [In no longer feed himself, he chokes on food and a. Frontotemporal dementia
appendicitis pain shifts to rt iliac fossa]. has become confined to a wheelchair. What is the b. Lewy body dementia
most likely dx? c. Delirium tremens
1307. A 46yo man with tachycardia has the a. Cerebral tumor d. Alzheimer’s disease
following ECG. What is the most likely dx? b. Myasthenia gravis e. Huntington’s disease
c. Lambert-Eaton syndrome Ans. The key is B. Lewy body dementia. [Visual
d. Motor neuron disease hallucinations and cognitive impairment are
e. Cerebro-vascular disease suggestive of Lewy body dementia. Hallucination
Ans. The key is D. Motor neuron disease. [The is prominent feature of LBD than Alzheimers.
picture is of amyotrophic lateral sclerosis with Again cognitive impairment goes more with LBD
bulbar onset, so it has very bad prognosis]. while prominent early memory loss symptom
suggests Alzheimer’s].
1310. A 10yo boy with lower abdominal pain for
the last 10d presents with a hx of passing 6-8 1314. A 40yo woman who has recently returned
loose from working in the middle east complains of
stools. Temp=38.8C. He is tender in the right thirst,
lower quadrant and has an anal fistula. Choose episode of loin pain, urinary frequency, dysuria
the and has passed a urinary stone. All inv are
single most likely cause of abdominal pain. normal. She plans to return to the Middle East in
a. IBD a month’s time. What is the single best advice
b. IBS to prevent recurrent stone formation?
c. Pyelonephritis a. Drink less milk
d. Uretric calculus b. High fibre diet
e. Gastroenteritis c. Increase fluid intake
Ans. The key is A. IBD. [Likely diagnosis is crohn’s d. Low calcium diet
disease which is often associated with perianal e. Low protein diet
fistula formation].
Ans. The key is C. Increased fluid intake. [ Risk
1311. A 28yo woman with hx of drug addiction factors for renal stones include being overweight,
wants to start a family and have a baby. She certain foods, some medications, and not drinking
would like enough fluids].
to stop taking heroin and asked for something to
help her stay away from it. What drug tx would 1315. A 32yo man presents with 3d of scrotal
you give her? pain. Exam: thickening o the left testis and it is hot
a. Naloxone to
a. SVT
b. Acamprosate touch. What is the most appropriate
b. VT
c. Methadone management?
c. Mobitz I heart block
d. Chlordiazepoxide a. Analgesia
d. Atrial fibrillation
e. Naltrexone b. Reassurance
e. WPW syndrome
Ans. The key is C. Methadone. [Methadone is a c. Antibiotics
powerful synthetic analgesic drug which is similar d. Referral to surgeon
Ans. The key is E. WPW syndrome. [Short PR
to morphine in its effects but less sedative and is
interval and slurred delta wave indicates WPW
used as a substitute drug in the treatment of Ans. The key is C. Antibiotics. [scrotal pain of 3
syndrome].
morphine and heroin addiction]. days with thickening of testis which is hot to
touch suggests epididymoorchitis, which is
1308. A 24yo male is admitted with acute severe
1312. A pt with vesicles in the maxillary divison of treated with antibiotics].
asthma. Tx is initiated with 100% oxygen,
trigeminal nerve. Which area of mucus membrane
nebulized
will be involved? 1316. A 34yo woman presents with truncal
salbutamol and ipratropium bromide nebulizers
a. Palate obesity, easy bruising, hyperglycemia, high BP and
and IV hydrocortisone. Despite initial tx there is
b. Cheek depression. Which of the following inv’s will be
no improvement. Which is the next step in
c. Cornea most helpful in localizing the cause for Cushing’s
management?
d. Conjunctiva syndrome?
a. Serum cortisol Ans. The key is D. Psoriatic arthropathy. [Psoriatic c. Lies below and lateral to the pubic tubercle
b. 24h urinary cortisol arthritis initially can affects the toes very similar d. Only passes through the superficial inguinal ring
c. Low dose dexamethasone suppression test to gout. There can be sacroilitis and onycholysis. e. Passes through the deep inguinal ring
d. High dose dexamethasone suppression test Ref: patient.info].
e. Overnight dexamethasone suppression test Ans. The key is E. Passes through the deep
Ans. The key is D. High dose dexamethasone 1321. Which of the following conditions requires inguinal ring. [Ingunal hernia lies mostly above
suppression test. [High dose dexamethasone operative management? and medial (occasionally lateral particularly when
suppression test can identify whether pituitary a. Cellulitis small) to pubic tubercle. It first enters the inguinal
adenoma or ectopic ACTH producing tumour]. b. Dyshidrosi canal through deep inguinal ring and then enters
c. Erysipelas the scrotum through the superficial inguinal ring].
1317. A 32yo man develops hematuria 2wks after d. Fournier’s gangrene
a sore throat. What is the dx? e. Lymphangitis 1325. A 56yo woman with hx of breast cancer 10y
a. Post infection nephritis Ans. The key is D. Fournier’s gangrene. [Fournier's ago has undergone radical mastectomy and
b. IgA nephropathy gangrene: A horrendous infection of the genitalia axillaryLN removal, now complains of swollen
c. Membranous nephritis that causes severe pain in the genital area (in the upper limb 3wks after an insect bite. The bite site
d. Glomerulonephritis penis and scrotum or perineum) and progresses is
from erythema (redness) to necrosis (death) of better but gross edema is still present. What is the
Ans. The key is A. Post infection nephritis. tissue. Gangrene can occur within hours]. cause?
[Hematuria 2 wks after sorethroat indicate post a. Lymphedema
infection nephritis while hematuria after few days 1322. A 55yo pt presents with collapse and b. Breast Ca
of sorethroat indicate IgA nephropathy]. complains of abdominal pain that radiates to the c. Allergy
back. An d. Filariasis
1318. An elder man who has anorexia, prostate expansile abdominal mass is felt on examination Ans. The key is A. Lymphedema. [As during
symptoms and HTN. There are small kidneys on and the pt is in shock. What is the single most mastectomy axillary clearance is done, lymphatics
US. likely dx? on that upper limb is compromised. So there
What is the dx? a. Ruptured aortic aneurysm occurs lymphedema].
a. Hypertensive renal disease b. Renal colic
b. Prostate ca c. Trauma 1326. A homeless person is found wandering on
c. BPH d. Endocarditis the street. He had ataxic gait, nystagmus and
e. Atheroma opthalmoplegia. He looked unkempt and his
Ans. The key is A. Hyperensive renal disease. clothes had a sweaty odour. He had a dry mucous
[Small kidneys suggest chronic renal failure and Ans. The key is A. Ruptured aortic aneurysm. membrane with a BP=118/70mmHg and
uremia here is the cause of anorexia. As elder he PR=90bpm. Blood sugar level=8. Alcohol breath
may have associated prostate symptoms from 1323. A house-bound 78yo man with severe COPD test= -
concomitant BPH]. has had a gradual deterioration over recent ve. What would the most imp initial inv?
months a. IV insulin
1319. A 55yo woman with breast ca which has and is now breathless at rest. He is on maximal b. Vit B complex
spread to lung, liver and bone now presents with inhaled medical therapy. Result: pH=7.36, c. Bolus IV 0.9%NS
increasing constipation, weakness, thirst and PaCO2=5.9kPa, PaO2=6.9kPa. What is the single d. IV dextrose
anorexia for the past 3d. Her only medication is most appropriate additional tx? e. Antibiotics
haloperidol for hiccoughs. Today she is a. Aminophylline PO
disorientated and has left sided weakness. What b. ACEi PO Ans. The key is B. Vit B complex. [This is
is the c. Antibiotic PO wernicke’s encephalopathy due to alcohol
most likely dx? d. Oxygen withdrawal, so Vit B complex should be checked.
a. Brain mets e. Steroid PO This occurs mainly due to thiamine deficiency].
b. Hypercalcemia
c. Liver failure Ans. The key is D. Oxygen. [In patients with 1327. A 34yo man has supra-orbital pain and
chronic hypoxaemia, LTOT should usually be tenderness and developed tenderness over the
Ans. The key is A. Brain mets. [The patient has prescribed after appropriate assessment, when maxilla.
clinical hypercalcemia, but the neurological the PaO2 is consistently at or below 7.3 kPa (55 He also has mild fever. What is the single likely
features of disorientation and left sided weakness mm Hg) when breathing air during a period of cause for these symptoms?
can only be explained with brain metastasis]. clinical stability. Clinical stability is defined as the a. Acute sinusitis
absence of exacerbation of chronic lung disease b. GCA
1320. A 22yo man presents with a red, hot, for the previous five weeks. The level of c. Trigeminal neuralgia
swollen, metatarsal phalangeal joint, sarcoilitis PaCO2(which may be normal or elevated) does d. Maxillary carcinoma
and not influence the need for LTOT prescription].
onycholysis. What is the single most likely cause Ans. The key is A. Acute sinusitis. [There is
of his condition? 1324. A 79yo man has a swelling of the right groin supraorbital pain in frontal sinusitis and
a. Gout which was clinically dx to be indirect inguinal tenderness over maxilla in maxillary sinusitis. Also
b. RA hernia. What is the single feature of the hernia mild fever may present in acute sinusitis].
c. Reiter’s syndrome sac that would confirm the dx?
d. Psoriatic arthropathy a. Comes through femoral ring 1328. A 51yo woman presents with painful tongue
b. Doesn’t pass through the deep inguinal ring and complains of tiredness. She is pale and has
angular stomatitis and a smooth red tongue. thickness burns over the arms and legs and had e. Hep B vaccine once and Ig
There is no koilonychea. Choose the single cell soot in the mouth and nose. His breathing has
type become noisy. What is the single most immediate Ans. The key is B. Hepatitis B full vaccine and Ig.
you will find on the blood film. management?
a. Numerous blast cells a. Nebulized adrenaline 1337. A man suffers from Herpes Zoster affecting
b. Oval macrocytes b. Nebulized salmetarol and oxygen his face. Which of the following mucos membrane
c. Spherocytes c. Needle cricothyrodotomy is to be affected?
d. Microcytic hypochromic d. Oropharyngeal airway a. Cheek
e. Mexican hat cells e. Intubation of airway b. Cornea
f. Erythrocytes c. Conjunctiva
Ans. The key is B. oval macrocytes. [Macrocytes Ans. The key is E. Intubation of airway. [Soot in d. Oropharynx
are two types, round and oval. Oval macrocytes the mouth and nose and noisy breathing occurs in e. Palate
are seen in megaloblastic anemia which occurs inhalation injury where intubation of the airway is
due to Vit B12and/or folic acid deficiency. indicated]. Ans. The key is C. Conjunctiva. [If maxillary or
Glossitis (painful tongue), angular stomatitis and ophthalmic division is not mentioned but only
smooth tongue are though characteristic feature 1333. A new born baby is borught with mentions face it means ophthalmic branch more
of iron deficiency but also known feature of pansystolic murmur at sternal border but the commonly around eye. Herpes zoster
megaloblastic anemia. Absence of koilonychea baby is not ophthalmicus can cause following lesions
also favours megaloblastic anemia]. cyanosed. What is the dx? blepheritis, conjunctivitis, keratitis, anterior
a. VSD uveitis etc. As question wants mucous membrane
1329. A 24yo woman presents with tingling and b. ASD and among the mentioned lesions only
twitching of her fingers followed by throbbing c. TOF conjunctiva is mucous membrane the answer is
unilateral headache. What is the most likely dx? d. PDA conjunctiva].
a. Tension headache
b. Migraine Ans. The key is A. ventricular septal defect. 1338. A 34yo man sustains a fx to shaft of femur
c. Cluster headache [Pancystolic murmur in sternal border without after falling from the roof of his house. Exam:
d. TIA cyanosis is indicative of VSD]. distal pulses are not palpable. Which vessel is
e. SAH damaged?
1334. A woman complaining of diarrhea, a. Femoral artery
Ans. The key is B. Migraine. [The tingling and abdominal pain and fatigue. All the tests are b. Circumflex femoral artery
twitching of her fingers are aura and the migraine found to be c. Profundafemoris artery
is with aura]. normal. What is the cause? d. Popliteal artery
a. Somatization e. Obturator artery
1330. A young child dx with chicken pox. Usually b. Conversion f. Dorsalispedis artery
goes to day care. What is the most appropriate c. Hypochondriasis
advice? Ans. The key is A. Femoral artery.
a. Child should be admitted to hospital straight Ans. The key is A. Somatization. [Somatization
away disorder is patients presenting with any physical 1339. A 9yo child doesn’t play with his peers and
b. Isolate the child from parents and siblings at symptom and frequent medical visits in spite of has collected 200 cars. He doesn’t respond to any
home negative investigations]. criticism. What is the dx?
c. Advice that he can go back to nursery when the a. Autism
rash is crusted over 1335. A 26yo man has returned from NY to the UK b. Personality disorder
and noticed weight loss, night sweats, c. Schizophrenia
Ans. The key is C. Advice that he can go back to temp=37.5C d. Rett syndrome
nursery when the rash is crusted over. [At this and cervical lymphadenopathy. He also has e. Social anxiety
stage patient is no more infectious]. splenomegaly. What is the dx?
a. TB Ans. The key is A. Autism.
1331. A 7yo boy is brought by his mother. There b. Lymphoma
are multiple perioral and forehead vesicles. Some c. Bronchial carcinoma 1340. A 63 yo man with vague but persistent pain.
vesicles are crusted and some are not. The face is d. Bronchitis On endoscopy: columnar epithelium was found to
hot. What is the most likely dx? be pouched into muscularis. What is the dx?
a. Varicella zoster Ans. The key is B. Lymphoma. [Splenomegaly a. Adenocarcinoma
b. Herpes zoster favours lymphoma]. b. Adenoma
c. Fungal infection c. Peptic ulcer
d. Impetigo 1336. A mother got infected with Hep B during d. H. pylori infection
e. Psoriasis pregnancy. Her child is born and she is worried
about Ans. The key is A. Adenocarcinoma.
Ans. The key is B. Herpes zoster. the risk of infection to the baby with Hep B. What
would you give to the baby? 1341. A 24yo man after a head injury presents
1332. A 5yo boy is rescued from a burning a. Hep B Ig only with difficulty dressing himself, difficulty in writing
building and is presented to the ED. He has 5% b. Hep B full vaccine and Ig and inability to differentiate the fingers of his
partial c. Hep B vaccine only once hand. Which part of the brain is most likely to be
d. Nothing until immune status is checked affected?
a. Frontal lobe 1346. An 89yo man presents with carcinoma of
b. Parietal lobe posterior oropharynx. Which is the single most ***1351. A 4yo boy ingested his grandmother’s
c. Temporal lobe appropriate LN involved? medicine and has developed dilated pupil. What is
d. Occipital lobe a. Pre-aortic LN the cause?
e. Brainstem b. Aortic LN a. Amitryptiline
c. Submental LN b. Paracetamol
Ans. The key is B. Parietal lobe. [Dressing apraxia, d. Submandibular LN c. Iron
agraphia and finger agnosia are features of e. Deep cervical LN d. Digoxin
parietal lobe lesions].
Ans. The key is E. Deep cervical LN. Ans. The key is A. Amitryptiline. [TCA causes
1342. A 16yo boy in boarding school feels unwell. mydriasis].
He developed cough and rash. His CXR showed 1347. A young boy presented to the OPD 12wks
bilateral consolidations. What is the cause of his after renal transplantation with fever and pain in 1352. A 46yo male presents with confusion and
symptoms? lower abdomen. Renal functions were deranged. drowsiness. What is the most likely dx?
a. Staph aureus Renal biopsy showed immune cell infiltrate and a. Cryptococcus neoformans
b. Legionella tubular damage. What is the most probable dx? b. Toxoplasma gondii
c. Mycoplasma a. Pyelonephritis c. HSV
d. Streptococcus b. Chronic graft rejection d. CMV
Ans. The key is C. Mycoplasma. [Rash, bilateral c. Acute rejection e. Candida albicans
consolidation favours mycoplasma pneumonia. d. Drug toxicity
Mycoplasma is common in military barrack, prison e. Graft vs host disease Ans. The key is A. Cryptococcus neoformans. [The
or boarding dwellers]. Ans. The key is C. Acute rejection. [Acute rejection key is doubtful as Toxoplasmosis is most common
usually occurs in the first few months after cns lesion in AIDS].
1343. A 10yo boy is brought to the ED 10h after transplant, but may occur up to a year after
injury to the foot. It was punctured with a metal transplant. Features presented are typical of 1353. A child has developed rash after the tx of
spike that passed through his shoe. What is the acute graft rejection]. penicillin. What will be the cause of rash?
next best step? a. Drug reaction
a. Ig 1348. A 56yo lady presents with a pathological fx b. Kawasaki
b. Ig and vaccine of T11 vertebra. There is found to be an c. Inf Mono
c. Vaccine only underlying metastatic lesion. What is her most
d. Clean the wound common primary ca? Ans. The key is A. Drug reaction.
e. Antibiotics a. Lung
b. Breast 1354. A child comes with recurrent joint pain,
Ans. The key is D. Clean the wond. [“NEXT” best c. Uterine multiple bruises, swollen ankle and unable to
step is clean the wound]. d. Brain move his legs. What is the inv of choice?
Ans. The key is B. Breast. [In female breast and a. ESR
1344. A 56yo male presents with persistent lung cancer and in male prostate and lung cancer b. RF
watery diarrhea. What is the most likely dx? are most common to metastasize to bone]. c. Clotting factors
a. Treponema pallidum
b. Nesseria meningitides 1349. A 6m infant has breast milk jaundice. He is Ans. The key is C. Clotting factors. [Probable
c. Cryptosporidium otherwise feeding well and is not dehydrated. diagnosis is hemarthrosis with the disease
d. Staph aureus What would his LFTs look like? hemophilia].
e. Pseudomonas aeruginosa a. Total bilirubin:40, conjugated bilirubin<5%
b. Total bilirubin:300, conjugated bilirubin 85% 1355. A 66yo man has renal colic. He has also
Ans. The key is C. Cryptosporidium. c. Total bilirubin:500, conjugated bilirubin>85% presented with acute onset pain in his knee in the
d. Total bilirubin:400, conjugated bilirubin<85% past. What is the single most likely cause for renal
1345. A 2yo girl has frequency, urgency and failure?
burning micturition. She has some supra pubic Ans. The key is A. Total bilirubin:40, conjugated a. SLE associated GN
tenderness. Which one of the following is the bilirubin<5%. [Brest milk jaundice is characterized b. Hypercalcemia
most appropriate initial inv? by indirect hyperbilirubinemia]. c. HTN
a. Supra pubic aspiration of urine for C&S d. Hyperuricemia
b. Clean catch of urine for C&S 1350. A 29yo man took a tour of Japan and also e. Hyperoxaluria
c. USG travelled to other parts of Asia, developed fever,
d. IVU petechie and rash on his body. He didn’t take Ans. The key is D. hyperuricemia. [Knee pain is
e. MCUG malaria prophylaxis prior to travel. What is the due to gout and renal colic may occur from uric
most likely dx? acid stone (urate stone)].
Ans. The key is B. Clean catch of urine for C&S. a. Malaria
[The features are consistent with lower UTI for b. HSP 1356. A boy with a hx of recurrent swollen tender
which clean catch of urine for c&s should be c. HIV joints on both knees and elbows and not able to
done]. d. Dengue fever participate in sports. What is the inv of choice to
e. ITP dx?
a. RF/ASO titre
Ans. The key is D. Dengue fever. b. Clotting factor
c. ESR a. Acute cystitis Ans. The key is D. PCP. [IV drug abuser, weight
b. Bladder ca loss, breathlessness and dry cough with interstitial
Ans. The key is B. Clotting factor. [Haemarthrosis c. Renal vein thrombosis shadowing indicate PCP pneumonia in an AIDS
causing disability]. d. Acute pyelonephritis patient].
e. Ureteric calculus
1357. A 26yo man is referred for gastroscopy 1365. A 27yo female who had a RTA 7m back now
because of a hx of several months of dyspepsia. Ans. The key is E. Ureteric calculus. [In ureteric complaints of attacks of sudden onset rotational
He has routine bloods checked and is found to stone there is radiation of pain from loin to groin vertigo which comes on with sharp movements of
have a serum calcium level=3.2mmol/l with a or scrotum. There also occurs hematuria]. the head and neck. Which of the following would
venous bicarbonate level of 33mmol/l. Renal and be most helpful?
LFT are both ormal. CXR is normal. What is the 1361. A 55yo man is having slow growing ascites. a. Caloric testing
most likely cause of his hypercalcemia? When we tap the peritoneal fluid the protein is b. Hallpikes maneuver
a. Melanoma <25 and it is clear and yellow. What could be the c. Gutenbergers test
b. Metastatic malignancy origin for ascites? d. Meniere’s test
c. Milk alkali syndrome a. Budd-Chiari e. Otoscopy
d. Primary hyperparathyroidism b. Gastrinoma
e. Sarcoidosis c. Hepatoma Ans. The key is B. Hallpikes maneuver. [Vertigo
d. TB which comes on with sharp movement of the
Ans. The key is C. Milk alkali syndrome. [In e. Pancreatitis head and neck points towards the diagnosis of
medicine, milk-alkali syndrome is characterized by benign positional vertigo which can be
high blood calcium caused by taking in too much Ans. The key is A. Budd-Chiari syndrome. [In demonstrated by Hallpikes maneuver].
calcium and absorbable alkali; common sources of gastrinoma there is no ascites! In hepatoma there
calcium and alkali are dietary supplements taken will be exudative ascites as in TB and pancreatitis. 1366. A man rescued from a building on fire
to prevent osteoporosis and antacids. If Budd-Chiary syndrome causes transudative presented with unconsciousness without any
untreated, milk-alkali syndrome may lead to ascites and the likely option here]. evidence of burns or external injury or soot. What
kidney failure or death]. would you do next?
1362. A 7yo boy presents with his mother to GP a. 100% oxyen inhalation
1358. A 3yo boy presents with difficulty in walking surgery. His mother describes he had presented b. 24% oxygen by mask
and skin lesions. What is the most likely causative this since 3wks ago. He had not experienced any c. Hyperbaric oxygen in a hyperbaric chamber
agent? trauma. No other symptoms a/w the condition. d. Intubation
a. Strep pyogenes Exam: non tender swollen ankles bilaterally. There e. Refer to specialist unit
b. Rubella virus is no rash or lesion. He is otherwise well. Which
c. Parvovirus single test would be the best as an initial Ans. The key is A. 100% oxygen inhalation. [In CO
d. Papovirus assessment? poisoning 100% oxygen is the initial management
e. Paramyxovirus a. Plasma electrolytes and also there is a role of hyperbaric oxygen in a
b. Albumin hyperbaric chamber].
Ans. The key is C. Parvovirus. [Sometimes there c. Total serum protein
may occur arthropathy for which children gets d. Anti-streptolysin 1367. A pt has had 1 ep of depression and 2 eps of
difficulty to walk. There occurs macular mania over the last year and now presents with
morbiliform rash in parvovirus infection following Ans. The key is B. Albumin. [Likely cause of ankle depression. He is on anti-depressants. What
disappearance of rash of the cheeks]. oedema from hypoalbuminema]. additional pharmacological tx would now act as a
prophylaxis for his condition?
1359. A pt after his house fire came with 1363. In lyme disease, which complication is most a. Antidepressants
hematemesis with erosion/ulcer of esophagus likely to lead to collapse? b. Antipsychotics
and on examination there is 55% burn and on a. Dilated CM c. Mood stabilizers
endoscopy there is a stomach/gastric erosion and b. AV block d. No additions req
soot in the mouth. What is the tx? c. Mild encephalitis
a. PO PPI d. Meningitis Ans. The key is C. Mood stabilizers.
b. IV PPI e. Myocarditis [Antidepressant alone can lead to manic attack
c. PPI and antibiotic and to prevent this a mood stabilizer is required].
d. H. pylori test Ans. The key is B. AV block.
e. Tracheal intubation 1368. A man presented with a purplish swelling at
1364. A 30yo pt came to the OPD with complaint the anal area. It is acutely painful and he
Ans. The key is E. Tracheal intubation. [If a burnt of breathlessness and dry cough. He has lost 5kgs complains of constipation for the last 2m. What is
patient gets soots in mouth and /or nose tracheal in 2m. He is an IV drug abuser. Inv: CXR=bilateral the most appropriate management?
intubation is done. GI ulceration (here interstitial shadowing. What is the single most a. I&D
oesophageal and gastric ulcers and erosions) due likely causative organism? b. I&D + antibiotics
tu burns are known as curlings ulcers]. a. Klebsiella c. Reassure
b. TB d. Analgesia
1360. A 40yo man complains of severe colicky loin c. Chlamydia pneumonia e. Sclerotherapy
pain that radiates to his scrotum. He is noted to d. PCP
have microscopic hematuria. No masses are e. Chlamydia psitacci Ans. The key is B. I&D + antibiotics.
palpated. What is the single most likely cause?
1369. A pt came to the ED after he had banged his Ans. The key is C. Echo. [To rule out any valvular b. CT head
car quite a few times on reversing. He was lesion (following papillary muscle rupture) or c. MRI spine
complaining of seeing double while he tried to septal lesion]. d. Nuclear bone scan
look back during the process of reversing the car, e. XR thoracolumbar spine
he also complains of double vision on looking at 1373. A 73yo stroke pt has been on aspirin for
an outward gaze. Which nerve is involved? 2yrs. He now presents with epigastric pain and is Ans. The key is C. MRI spine. [There is neurological
a. Abducent nerve asking for a tx. What is the most appropriate deficit so to exclude spinal involvement MRI is the
b. Trochlear nerve management? single most appropriate investigation].
c. Oculomotor nerve a. Laparotomy
d. Optic nerve b. NSAIDs 1378. A pregnant lady at her 39wk GA present
e. Trigeminal nerve c. Omeprazole with eclampsia. Soon after her arrival in the
d. Morphine labour suit, IV MgSO4 and IV hydralazine has been
Ans. The key is A. Abducent nerve. [Lateral rectus e. Tramadol prescribed. The pt then develops another fit in the
palsy]. hosp and maintenance dose of MgSO4 has been
Ans. The key is C. Omeprazole. started. What is your next step in management?
1370. A pt had a stroke. Now, there is left sided a. Mg SO4 bolus
weakness and right side facial numbness. CT 1374. A 2yo girl is brought to the ED by her b. Delivery of baby
shows ischemic stroke. Which one would you mother. The child is screaming that there is c. MgSO4 loading dose
prescribe? something in her ear and she appears agitated. d. Diazepam
a. Alteplase Exam: a plastic bead is seen inside the ear. What
b. Aspirin is the best method of removal? Ans. The key is A. MgSO4 bolus.
c. Clopidogrel a. Forceps
d. Heparin b. Hook 1379. A man suffering from Influenza A since 5d
e. Warfarin c. Under general anesthesia ago. CXR: pneumonia. What organism is
d. Syringing responsible for pneumonia in this pt?
Ans. The key is A. Alteplase. [Some say CT detects e. Magnet a. Hemophilius influenze
ischaemic stroke beyond window period which is b. Klebsiella
not correct and it is able to detect stroke much Ans. The key is C. Under general anaesthesia. c. Staphylococcus aureus
earlier and hence it’s place is there in the d. Streptococcus pneumonia
treatment with alteplase]. 1375. During antenatal visits, the following tests e. Pseudomonas
are routinely offered to all pregnant mothers
1371. A young boy presents with fever and cough. apart from HIV and Hep B? Ans. The key is C. Staphylococcus aureus. [Post
His father was dx with TB a week ago. The parents a. Rubella and syphilis flue pneumonia is usually caused by
don’t want him to have a BAL under anesthesia. b. Syphilis and toxoplasmosis staphylococcus aureus].
Which other samples can be taken for dx? c. Hep C & thalassemia
a. Urine d. CMV and rubella 1380. A pt admitted due to repeated attacks of
b. Blood e. Sickle cell anemia and Hep pancreatitis presents with dementia and loss of
c. CSF proprioception in the legs. What is the most
d. Gastric washing Ans. The key is A. Rubella and syphilis. appropriate tx?
e. Sweat a. Thiamine
1376. A 32yo male complains of tremors b. Pyridoxine
Ans. The key is D. Gastric washing. [A young boy is everytime he tends to use his muscles and when c. Cobolamin
able to cough up sputum unless it is dry cough he is pointing at objects. No complaints at rest. d. Lipase
where suspected TB is diagnosed by broncho- His father complained of similar problems. What e. Antibiotics
alveolar-lavage (BAL). As in dry cough TB bacteria is the most probable dx?
contaminate saliva and goes to stomach with a. Parkinsonism Ans. The key is C. Cobalamin. [Pancreatic
swallowed contaminated saliva Gastric-washing b. Lithium toxicity insufficiency can lead to malabsorption and B12
can bring out bacteria and detected thereby. So c. Thyrotoxicosis (cobalamin) deficiency. Dementia and diminished
gastric-washing is an alternative to BAL in dry d. Benign essential tremor vibratory sensation and proprioception in the
cough]. lower extremities were the most common
Ans. The key is D. benign essential tremor. [In objective findings of cobalamin deficiency].
1372. A 50yo man came to the hosp a few months benign essential tremor there is no rest tremor
after he had a MI. Exam: everything normal, S1 but tremor occurs during movement of the 1381. A man after MI presented with sudden
and S2 were heard on auscultation, but there is a affected limb]. breathlessness and dyspnea. Exam: scattered
new pan-sytolic murmur. What is the most pansystolic murmur all over the precordium.
appropriate inv of choice? 1377. A 40yo woman with breast cancer has back What is the next inv that will lead to dx?
a. ECG pain which keep her awake at night. She blames it a. ECG
b. 24h ECG on a gym session she had 2wks ago. She now has b. Echo
c. Echo difficulty in climbing stairs. There is tenderness c. CT
d. CXR over the right thoracic spine. She has diminished d. Blood culture
e. CT fine touch and temp sensation in her right foot. e. CXR
What is the single most appropriate inv?
a. Bone density scan
Ans. The key is B. echo. [Sudden breathlessness change. Unilateral loss of vestibular function and
and dyspnea suggest mitral regurgitation hearing associated with nausea and vomiting 1390. A 7yo boy presents with proptosis and
secondary to papillary muscle rupture but on the which is preceded by viral infection]. periorbital edema. What is the immediate action
otherhand pansystolic murmur all over the that needs to be taken?
precordium suggest VSD! However in either case 1386. A pt with celiac disease from birth, now as a. IV morphine and immediate ophthalmoscopy
required investigation is echo]. an adult presented with some abdominal b. IV morphine
symptoms. The biopsy shows infiltration of the c. Observation only
1382. During a laparoscopic procedure, a trochar gastric epithelium by lymphocytes. What is the
is inserted halfway between the umbilicus and the most likely dx? Ans. The key is A. IV morphine and immediate
ant superior iliac spine. What are the structures a. Lymphoma ophthalmoscopy.
most likely to be pierced? b. Diverticular disease
a. Rectus sheath c. Lynch syndrome 1391. A schizophrenic man complains that he can
b. Linea alba d. Gastric TB hear voices talking about him and telling him to
c. External oblique aponeurosis e. Peritoneal tumor end his life by cutting his throat. He only hears
d. Internal oblique and transverse abdominal them when he wakes up from sleep and not at
e. Both C and D Ans. The key is A. Lymphoma. [Coeliac disease other times. What type of hallucinations is he
leads to an increased risk of both adenocarcinoma having?
Ans. The key is D. Internal oblique and transversus and lymphoma of gut]. a. Somatic
abdominis. b. Kinesthetic
1387. A 55yo man presented with hot, raised, c. Hypnogogic
1383. A pt, a small child presented with URTI and tender area of skin on his right leg. He is febrile d. Hypnopompic
later developed fever, earache and tympanic with rigors. He has been started on flucloxacillin. e. Lilliputian
membrane is dull. What is the likely dx? What other meds will you add? Ans. The key is D. Hypnapompic hallucination.
a. OM a. Ciprofloxacin [Hallucinations during going for sleep is
b. OE b. Gentamicin hypnagogic and hallucinations during wake up
c. Glue ear c. Metronidazole from sleep is hypnapompic hallucinations].
d. Perforation of the tympanic membrane d. Benzylpenicillin
e. Referred ear ache e. Ceftriaxone 1392. A 28yo woman complains of hearing
strange voices in her bedroom as she is falling
Ans. The key is A. OM. [Fever, earache and dull Ans. The key is D. Benzylpenicillin. [Abscess, asleep in the night. She says there is no one in the
tympanic membrane following urti are pointing cellulitis, furuncle these lesions are usually caused room except her. On evaluation she has no other
the diagnosis of acute otitis media]. by Staphylococcus and streptococcus. So problems. What is she suffering from?
Benzylpenicillin should be added with a. Delusion of persecution
1384. A 72yo male who is a regular smoker has flucloxacillin to cover streptococcus]. b. Cotard syndrome
come to the ED with complaints of loss of weight c. Hypnogogic hallucinations
and loss of appetite. He also complains of 1388. A 65yo man has incurable bronchial cancer. d. Lilliputian hallucinations
odynophagia. Exam: actively bleeding ulcer on He is unable to cough up his secretions. This is e. Schizophrenia
right tonsil. What is the most appropriate dx? leading to a distressing cough. Which of the
a. Tonsillar ca following drugs is most likely to help him? Ans. The key is C. Hypnagogic hallucinations.
b. Vincent’s angina a. Scopolamine [Hallucinations during going for sleep is
c. Irritant ingestion b. Xanomeline hypnagogic and hallucinations during wake up
d. Paracoccidiodmycosis c. Aceclidine from sleep is hypnapompic hallucinations].
e. Herpes simplex infection d. Pilocarpine
e. Cevimiline 1393. A 32yo man on psychiatric meds presents
Ans. The key is A. Tonsillar ca. with coarse tremors and diarrhea. What is the
Ans. The key is A. Scopolamine. [Any secretory most likely alt to the drug causing the prb?
***1385. A pt with regular episodes of SNHL, condition of lung is reduced and improved with a. Lithium
vertigo and tinnitus lasting >30min. Neurological scopolamine]. b. Diazepam
exam=normal. What is the likely dx? c. Haloperidol
a. Meniere’s disease 1389. A pt presented after eating a seafood dish d. Valproate
b. Acoustic neuroma at a local restaurant. He complains of difficulty in e. Citalopram
c. Otosclerosis breathing. His speech is slurred and his
d. Benign positional vertigo BP=85/55mmHg. What would be the most Ans. The key is D. Valproate. [Diarrhea and coarse
e. Labrynthitis appropriate next step? tremors are well known side effects of valproate].
a. IV adrenaline
Ans. The key is A. Meniere’s disease. [SNHL b. IM adrenaline 1394. A man is brought to the ED after he was
vertigo andtinnitus lasting 30 minutes are c. SC adrenaline stabbed in the chest. Chest is clear bilaterally with
consistent with Meniers disease. Here attacks are d. PO chlorpheniramine muffled heart sounds. BP=60/0mmHg,
episodic that is symptoms resolves but hearing e. IV chlorpheniramin pulse=120bpm, JVP is raised. What is the most
loss is progressive. In acoustic neuroma hearing Ans. The key is B. IM adrenaline. [Patient is getting probable dx?
loss is not episodic. Otosclerosis also not episodic anaphylactic shock with difficulty of breathing and a. Pulmonary embolism
and conductive type. Benign positional vertigo, no slurred speech (probably from tongue swelling) b. Cardiac tamponade
hearing loss and vertigo with sharp posture which are indications of IM adrenaline]. c. Pericardial effusion
d. Hemothorax found to have lid lag as well. She also complains chest, BP=100/76mmHg, pulse=110bpm. He has a
e. Pneumothorax of her heart racing at times. Which test will help swollen abdomen with shifting dullness.
in dx? a. Gastric ca
Ans. The key is B. Cardiac tamponade. [Lungs clear a. Tensilon test b. Mallory-weiss tear
bilaterally excludes effusion and pneumothorax. b. 24h ECG c. Esophageal ca
Muffled heart sound and low bp with raised jvp c. TFT d. Esophageal varices
points towards the diagnosis of cardiac d. Schimmer test e. Esophagitis
tamponade]. e. Young Helmholtz ophthalmoscopy f. Peptic ulceration
Ans. The key is C. TFT. [Difficulty in looking Ans. The key is D. Esophgeal varices. [Spider nevi
1395. A 64yo alcoholic who has been dx with liver upwards is due to superior rectus or inferior and ascites suggest advanced liver disease. Here
cirrhosis presents with a massive ascites. What is oblique palsy and palsy of one or more extra- in alcoholic middle aged man having large
the mechanism of fluid accumulation in a pt with ocular muscle can occur in thyrotoxicosis which is hematemesis with above mentioned features are
liver disease? known as Ballett’s sign. There is lid lag in very suggestive of bleeding from esophageal
a. Cirrhosis thyrotoxicosis and racing heart is due to varices secondary to portal hypertension in liver
b. Portal HTN paroxysmal atrial fibrillation caused by cirrhosis].
c. Hypoalbuminemia hyperthyroidism].
d. Liver failure 1404. A 23yo woman presents with a 1cm small
e. Hepatic encephalopathy 1400. A young anxious mother of a 10m boy smooth, firm, mobile mass in her left breast. She
comes to you and requests a test for CF. What is is very anxious. What is the most appropriate inv?
Ans. The key is C. Hypoalbuminemia. [Mechanism the most appropriate inv? a. Mammography
of fluid accumulation (in general, not only ascites a. Sweat test b. US breast
is hypoalbuminaemia]. b. Heel prick test c. FNAC
c. Breath test d. Mammography and US
1396. A 38yo man presented to ED with severe d. CXR
pain in upper abdomen. He has already taken Ans. The key is B. US breast. [This is fibroadenoma
course of triple therapy and now had elective Ans. The key is A. Sweat test. [The sweat test of the breast (breast mouse) for which US of
endoscopy 2d ago. He is in shock. What is the measures the concentration of chloride that is breast is the appropriate investigation and if it
most probable dx? excreted in sweat. It is used to screen for cystic reveal a fibrous lump we shall go for FNAC.
a. Ca esophagus fibrosis (CF). Due to defective chloride channels, Another point a doctor should know that for any
b. Barret’s esophagus the concentration of chloride in sweat is elevated suspicious breast lesion radiology should must be
c. Mediastinitis in individuals with cystic fibrosis]. done before options of biopsy and investigation of
d. Ca stomach choice in radiology is US scan before the age of 35
1401. A 22yo Greek man presents with rapid and mammography after the age of 35. In that
Ans. The key is C. Mediastinitis. anemia and jaundice following tx of malaria. He is term also for this case the investigation of choice
noted to have Heinx bodies. Choose the single is US breast].
1397. A 68yo man who is a known case of liver most likely cause from the given options?
cirrhosis has developed ascites. What is the a. G6PD deficiency 1405. A pt was admitted with abdominal pain,
mechanism for the development of ascites? b. Anemia of chronic disease diarrhea, pigmented palmar creases and buccal
a. Portal HTN c. Pernicious anemia mucosa. What is the most probable dx?
b. Hypoalbuminemia d. IDA a. Addison’s disease
c. Congestive heart failure e. Vit B12 deficiency b. Cushing syndrome
d. Liver failure c. Pheochromocytoma
Ans. The key is A. G6PD deficiency. [G6PD d. Hyperthyroidism
Ans. The key is A. Portal hypertension. (glucose-6-phosphate dehydrogenase) deficiency e. Hypoparathyroidism
[Development of ascites is more due to portal exacerbated by administration of oxidant drugs
hypertension in a patient with liver disease]. (e.g., primaquine, dapsone, quinidine) can also Ans. The key is A. Addison’s disease. [Abdominal
result in Heinz bodies]. pain, diarrhea, pigmented palmar creases and
1398. A man feels mild discomfort in the anal buccal mucosa are well known features of
region and purulent discharge in underpants. 1402. A 65yo has terminal cancer and his pain is addisons disease].
What is the most likely dx? relieved by a fentanyl patch but he now complains
a. Feacal incontinence of shooting pain in his arm. Which of the following 1406. A 36yo pt came with diarrhea, bleeding,
b. Anal abscess will add to his pain relief? weight loss and fistula. What is the single most
c. Fistula in ano a. Gabapentin likely dx?
d. Anal tags b. Radiotherapy a. Colorectal ca
e. Rectal Ca c. Amitryptiline b. Celiac disease
Ans. The key is C. Fistula in ano. [Abscess is with d. Morphine c. CD
severe pain which may reduce after it bursts with Ans. The key is A. Gabapentin. [Shooting in the d. UC
purulent discharge. But here mild discomfort and arm indicates neuropathic pain for which both e. IBS
purulent discharge favours the diagnosis of amytriptiline and gabapentin can be used].
fistula]. Ans. The key is C. CD. [Bloody diarrhea can occur
1403. A 45yo male alcoholic presents after a large in both CD & UC. Also weight loss is common! but
1399. A 38yo female presents with difficult in hematemesis. He has some spider naevi on his fistula formation is unique feature of CD].
looking upward and on examination she was
1407. A 45yo man has undergone detox and now • Mononeuritis multiplex or pain, arthritis, proteinuria and hematuria are well
wants a drug to stop him from craving alcohol. polyneuropathy. known features of HSP].
What med would be that drug of choice? The presence of four out of six of these features 1415. A pt is on loop diuretics. What effect do
a. Disulfiram has a high specificity and sensitivity for the loop diuretics produce?
b. Acamprosate diagnosis of CSS]. a. Low Na+, low K+
c. Thiamine b. Low Na+, normal K+
d. Naloxone 1411. A 28yo woman comes with sudden onset c. Normal Na+, normal K+
e. Diazepam vomiting and pain per abdomen. Exam: mobile d. High Na+, low K+
Ans. The key is B. Acamprosate. [Disulfirum is a swelling in the right iliac fossa. What is the most e. High Na+, high K+
deterrent which does not reduce craving. probable dx?
Acramposate by restoring deranged brain a. Ectopic pregnancy Ans. The key is A. Low Na+, Low K+. [Loop
chemical reduces craving]. b. Tubo-ovarian abscess diuretics causes hyponatremia and hypokalemia].
c. Acute appendicitis
1408. A 68yo man awoke to find that he is unable d. Ovarian torsion 1416. A 6yo girl is being investigated for renal
to close his left eye and dribbling saliva from the e. Diverticulitis failure. She is found to have a congenital
left angle of his mouth. What is the single most abnormality of the insertion of the ureters into
appropriate option? Ans. The key is D. [As there is no history of the urinary bladder. What is the single most likely
a. Facial nerve amenorrhea ectopic pregnancy is less likely. So dx cause for renal failure in this pt?
b. Glossopharyngeal nerve is possible ovarian torsion]. a. SLE
c. Hypoglossal nerve b. PKD
d. Optic nerve 1412. A 68yo man on tx for an irregular heart beat c. Wilm’s tumor
e. Vagus nerve comes to the ED. He has palpitations for the last d. Acute tubular necrosis
3h. Exam: pulse=regular, 154bpm. Carotid sinus e. Reflux nephropathy
Ans. The key is A. Facial nerve. [Features are massage settled his pulse down to 80bpm. What
suggestive of left sided facial nerve palsy]. is the most likely rhythm disturbance? Ans. The key is E. Reflux nephropathy. [Reflux
a. SVT nephropathy is kidney damage (nephropathy) due
1409. A 19yo female dx with trichomonas b. V-fib to urine flowing backward (reflux) from the
vaginalis. LMP was 10d ago. What is the best c. VT bladder toward the kidneys; the latter is called
antibiotic tx? d. V-ectopics vesicoureteral reflux (VUR)].
a. Erythromycin e. A-fib
b. Vancomycin 1417. A 76yo man is in the CCU 2d after an acute
c. Metronidazole Ans. The key is A. SVT. [Regular tachycardia MI. He tells you that he had an episode of rapid
d. Penicillin responding to vagal maneuver is likely SVT]. pounding in the chest lasting for about 2mins. He
e. Clarithromycin remains conscious throughout. What is the most
f. Doxycycline 1413. A 43yo man with a hx of hospital admissions likely rhythm?
g. Fluconazole talk about various topics, moving from one loosely a. SVT
h. Clotrimazole connected topic to another. What is the most b. VF
likely dx? c. VT
Ans. The key is C. Metronidazole. a. Psychosis d. V-ectopics
b. Mania e. A-fib
1410. A 35yo man has been given a dx of allergic c. Schizophrenia
rhinitis and asthma. Exam: peripheral neuropathy d. Pressured speech Ans. The key is C. VT. It is wrong key. Correct key
with tingling and numbness in a ‘glove and e. Verbal diarrhea should be D. V-ectopics. [Ref: OHCM]
stocking’ distribution. Skin elsions are present in
the form of tender subcutaneous nodules. The pt Ans. The key is B. Mania. [Flight of idea is common 1418. A 49yo man comes with hx of cough and
is responding well to corticosteroids. What is the in mania]. SOB. His CD4 count is measured as 350. CXR
single most appropriate dx? shows lobar consolidation. What is the single
a. AS 1414. An 18yo girl presents with rash on her most appropriate option?
b. Churg-strauss syndrome trunk, abdominal pain, arthritis, proteinuria and a. Mycobacterium avium intercellular
c. Crytogenic organizing hematuria. What is the most probable dx? b. CMV
d. Extrinsic allergic alveolitis a. TTP c. Streptococcus
e. Tropical pulmonary eosinophilia b. ITP d. Toxoplasmosis
c. HSP e. Pneumocystis jerovici
Ans. The key is B. Churg-strauss syndrome. [The d. HUS
American College of Rheumatology has identified e. Measles Ans. The key is C. Streptococcus. [Features are
six criteria for the diagnosis of CSS:[3] consistent with lobar pneumonia].
• Asthma (wheezing, expiratory rhonchi). Ans. The key is C. HSP. [Henoch-Schönlein purpura
• Eosinophilia of more than 10% in (HSP) is an acute immunoglobulin A (IgA)– 1419. A 32yo woman with prv hx of PID now
peripheral blood. mediated disorder characterized by a generalized presents with severe abdominal pain. Her LMP
• Paranasal sinusitis. vasculitis involving the small vessels of the skin, was 8wks ago. What is the most probable dx?
• Pulmonary infiltrates (may be transient). the gastrointestinal (GI) tract, the kidneys, the a. Ectopic pregnancy
• Histological confirmation of vasculitis joints, and, rarely, the lungs and the central b. Ovarian torsion
with extravascular eosinophils. nervous system (CNS). Rash on trunk, abdominal c. Hematometrium
d. Chronic PID a. Cerebral tumor d. Viral pneumonia
e. Cholecystitis b. Pituitary adenoma e. Chlamydia psitacci
c. Cerebellar abscess
Ans. The key is A. Ectopic pregnancy. [PID is a risk d. Huntington’s chorea Ans. The key is A. Legionella. [H/O travel (staying
factor for ectopic pregnancy]. e. Parkinsonism in AC and watersystem of hotel), lymphopenia,
decreased Na+, bi-basal consolidation are well
1420. A 25yo who is 38wks pregnant presents to Ans. The key is A. Cerebral tumour. known features of legionnaires disease].
the labour ward with a hx of fewer fetal
movements than usual during the evening. She 1424. A 70yo lady on Raloxifene for osteoporosis 1228. A 20yo student came to the OPD with
also says that abdominal contractions are coming has recently to the UK from Australia. She now complains of headache, malaise, dry cough, joint
every few minutes and she is having a blood presents with severe chest pain, SOB and pain and vomiting. Exam: temp=39C. CXR: patchy
stained show per vagina for the last few minutes. suddenly collapsed in the ED. What is the single consolidation. What is the single most likely
Exam: cervix is fully affaced, 9cm dilated, cephalic most appropriate dx? causative organism?
presentation and station is +1. Choose the single a. MI a. Pneumococcal pneumonia
most likely dx? b. Aortic dissection b. Mycoplasma
a. APH c. Pulmonary embolism c. Klebsiella
b. Concealed hemorrhage d. Costochondritis d. Streptococcus
c. Labour e. Pneumothorax e. PCP
d. IUFD
e. IUGR Ans. The key is C. Pulmonary embolism. Ans. The key is B. Mycoplasma.
[Prolonged air travel is a risk factor for pulmonary
Ans. The key is C. Labour. [Abdominal embolism]. 1229. A 45yo man presented to his GP with vague
contractions coming every few minutes, having symptoms of headache, proximal muscle
blood stained show per vagina, fully effaced cervix 1425. A 35yo woman complains of hoarseness of weakness and nocturia. Test results show him to
with dilatation of 9 cm, cephalic presentation and voice 3h after partial thyroidectomy. She had no be severely HTN (230/130mmHg) and
station +1 suggests that the patient is in labour]. hx of phonation probs before the surgery. What is hypokalemic. What is the most probable dx?
the single most appropriate inv? a. Addison’s disease
1421. A 30yo woman has a painless lump in the a. Laryngoscopy b. Conn’s disease
outer aspect of her left breast. She has had a prv b. Bronchoscopy c. Familial hyperaldosteronism
breast lump. Her grandmother had breast cancer c. CT neck d. Cushing’s disease
at 70yrs. She has a 1cm smooth, firm, discrete, d. CXR e. Cushing’s syndrome
mobile lump in the other quadrant region of the e. Barium swallow
left breast. What is the single most likely dx? Ans. The key is B. Conn’s disease. [High BP is often
a. Breast abscess Ans. The key is A. Laryngoscopy. [Probable the only presentation of Conn’s syndrome. Loss of
b. Breast carcinoma diagnosis is recurrent laryngeal nerve palsy]. K+ in urine leads to hypokalemia which in turn
c. Breast cyst causes muscle weakness and polyuria particularly
d. Fibro-adenoma 1426. A 40yo pt came to OPD with complaint of nocturia].
e. Sebaceous cyst fever, pleuritic chest pain, productive cough and
painful vesicles around the lips. Exam: temp=38C. 1230. A man says his insides are rotting and
Ans. D. Fibroadenoma. [H/O previous brest lump He has a hx of splenectomy last yr. What is the nobody has buried him. Which term best
and presently with a lump of 1 cm size which is single most likely causative organism? describes his condition?
smooth, firm (not hard), discrete and mobile (not a. Pneumococcal pneumonia a. Delusion of nihilism
fixed) suggests fibro-adenoma]. b. Staphylococcus b. Delusion of guilt
c. Klebsiella c. Delusion of persecution
1422. A 38yo woman is in the ED following an OD d. Streptococcus d. Incongruent affect
of her meds. She doesn’t need med tx for the OD. e. Chlamydia psitacci e. Clang association
She says she wishes to be discharged. What is the
single most appropriate management? Ans. The key is A. Pneumococcal pneumonia. Ans. The key is A. Delusion of nihilism.
a. Community psychiatric nurse visit [painful vesicles around the lips is well known
b. Psychiatric OPD review the next day association of pneumococcal pneumonia. Also 1431. A man with chronic cough presents with
c. Prescribe anti-depressants pleuritic chest pain and productive cough are copious purulent sputum. What is the single most
d. Admission under the mental health act present in pneumococcal pneumonia]. dx?
e. Discharge and allow to go home a. Bronchitis
1227. A 37yo male pt who recently returned back b. Bronchiectasis
Ans. The key is D. Admission under the mental to UK from UAE attends the OPD with complaint c. COPD
health act. [There is every chance of repeat over of dry cough, breathlessness and anorexia. d. Pneumonia
dose. So she should be admitted under mental According to him he had flu like symptoms a week e. Emphysema
health act]. ago. He is slightly confused. Inv: lymphopenia &
decreased Na+. CXR: bi-basal consolidation. What Ans. The key is B. Bronchiectasis.
1423. A 63yo male presents after having had a is the single most likely causative organism?
seizure. Exam: alert and oriented. Exam: a. Legionella 1432. A 32yo man working in a shipyard comes
inattention on the left side and hyperreflexia of b. Chlamydia pneumonia with SOB. Exam: dullness on left side of the chest,
the arm. What is the most probable dx? c. PCP pain in left side of chest, pleuritic rub and crackles
been heard on the same side. What is the single 1436. A 25yo pt came to the OPD with complaint 1440. A 35yo is agitated and euphoric. He claims
most likely dx? of fever, malaise, breathlessness, cough and to be helping the prime minister with economic
a. Pericarditis anorexia. His gf has got similar symptoms. He had policy, although this is not true when checked.
b. Pleurisy hx of sore throat and ear discharge a month ago. What is the most likely dx?
c. Pleural effusion What is the single most likely causative organism? a. Mania
d. CCF a. Legionella b. Schizophrenia
e. TB b. Mycoplasma c. Hypomania
c. Chlamydia pneumonia d. Drug induced personality disorder
Ans. The key is C. Pleural effusion. [Shipyard d. PCP e. Delusion of grandeur
worker are exposed to asbestos and presenting e. Chlamydia psitacci Ans. The key is E. Delusion of grandeur. Key is
case has developed mesothelioma causing pleura wrong! Correct key should be mania. [Agitated,
thickening and pleural effusion. Though it is usual Ans. The key is C. Chlamydia pneumonia. euphoric and delusion of grandiosity makes the
that in pleural effusion pleural rub reduces or likely dx to be “Mania”. Agitation and euphoria
becomes absent but it is also possible to get 1437. A 72yo male presents with acute confusion. are not feature of delusion of grandiosity but
pleural rub even in pleural effusion and the like He has been in the hosp for 2wks having been mania].
diagnosis is pleural effusion here]. treated for a DVT. The nurses have noticed that 1441. A 20yo student who recently visited Asia
he became increasingly drowsy. Exam: small scalp came to the OPD with complains of low grade
1433. A 67yo female presents with balance prbs. laceration, a GCS of 8 and bilateral up-going fever, night sweats, anorexia and productive
Exam: nystagmus on left lateral gaze, a loss of the plantar response. cough. Inv: CXR=cavitatory lesions in upper lobes.
left corneal reflex and reduced hearing in the left a. Infection toxicity What is the single most likelt causative organism?
ear. What is the most likely dx? b. Delirium tremens a. Mycoplasma
a. Meniere’s disease c. Extradural hematoma b. Klebsiella
b. Acoustic neuroma d. Subdural hematoma c. TB
c. Cerebral abscess e. Electrolyte imbalance d. PCP
d. Pituitary tumor
e. Gentamicin Ans. The key is D. Subdural hematoma. [Even Ans. The key is C. TB. [Low grade fever, night
trivial head trauma can lead to subdural sweats, anorexia, dry (initially) or productive
Ans. The key is B. Acoustic neuroma. [nystagmus, hematoma. Presence of small scalp laceration, cough, and upper lobe cavitary lesions are highly
loss of corneal reflex, hearing loss, balance confusion and becoming increasingly drowsy suggestive of TB].
problem are well known feature of acoustic Glasgow coma scale of 8 are suggestive of
neuroma]. subdural hematoma]. 1442. A 35yo man with T1DM is dehydrated with
BP=90/50mmHg. What is the single most
1434. A 22yo man reports a 2d hx of hoarseness 1438. A 50yo DM pt came to the OPD with appropriate initial inv?
of voice. He denies any weight loss but he has complaint of fever, muscle ache, dry cough and a. ABG
been smoking for 4yrs. What is the single most anorexia. Inv: CXR=upper lobe cavitation. What is b. CBC
appropriate inv? the single most likely causative organism? c. HbA1c
a. None a. Legionella d. LFT
b. Laryngoscopy b. Mycoplasma e. BUE
c. Bronchoscopy c. Staphylococcus
d. BAL d. Klebsiella Ans. The key is A. ABG. [The likely dx is DKA.
e. CXR e. Streptococcus Among the given options ABG is the most
appropriate inv. To demonstrate acidosis].
Ans. The key is B. Laryngoscopy. It is a wrong key! Ans. The key is D. Klebsiella. [Upper lobe
Correct key is A. None. [If horseness is of >3 cavitation favours Klebsiella pneumonia. Also it is 1443. A 45yo woman presents with pruritis. Exam:
weeks in man >50 yrs and smoker and heavy well known that staphylococcal and klebsiella skin pigmentation. Inv: raised ALP and presence of
drinker to rule out cancer do CXR and\or pneumonia are more common in DM than normal anti-mitochondrial antibodies. What is the single
laryngoscopy- NICE guideline]. person]. most likely dx?
a. Psoriasis
1435. A 34yo IVDA (intravenous drug addict) 1439. A 20yo man complains that all his b. Scabies
presents with a 4m hx of productive cough. He movements are being watched. Sometimes he c. Atopic eczema
has lost 10kgs. What is the single most feels as though his actions are being controlled by d. Dermatitis herpetiformis
appropriate inv? his radio. At other times he is aware of voices e. Hyperthyroidism
a. Sputum for AFB describing what he is doing. What is the most f. Primary biliary cirrhosis
b. Laryngoscopy probable dx?
c. Bronchoscopy a. Mania Ans. The key is F. Primary biliary cirrhosis.
d. CT neck b. Drug induced psychosis [Pruritus, skin pigmentation (increased amounts
e. CXR c. Delusion of control of melanin, widely dispersed throughout both
d. Schizophrenia epidermis and dermis), raised ALP, and presence
Ans. The key is A. Sputum for AFB. [In IVDA e. Korsakoff psychosis of AMA are diagnostic of primary biliary cirrhosis].
immunity becomes low and easily gets infected
with TB]. Ans. The key is D. Schizophrenia. 1444. A 60yo man complains of tiredness,
lethargy and itching that is severe after a hot
bath. He also has nocturia, polyuria and nausea
and vomiting. Exam: pallor, pigmentation and c. Coronary sinus c. IDA
generalized edema. What is the single most likely d. Circumflex artery d. Uremia
dx? e. Right coronary artery e. Drug induced
a. Hyperthyroidism
b. Lichen planus Ans. The key is B. Left ant descending artery. Ans. The key is A. Lymphoma. [Cervical
c. Lymphoma lymphadenopathy, lethargy, night sweats and
d. Eczema 1449. Which virus is transmitted by the fecal-oral itching are well known features of lymphoma].
e. Liver failure route?
f. CRF a. Hep C 1454. A 25yo male presents with fever and pain in
b. Coxsackie virus the right lower thigh of 1m duration. Exam: lower
Ans. The key is F. CRF. [Given picture is typical of c. Dengue third of his thigh is red, hot and tender. The XR
CRF]. d. None of the above showed new bone formation. What is the most
probable dx?
1445. A 30yo man complains of vague pain in the Ans. The key is B. Coxsackie virus. a. Osteosarcomoa
loin with BP=140/90mmHg. He is found to have b. Ewing’s sarcoma
proteinuria and hematuria. What is the inv to 1450. A 40yo woman presented with generalized c. Tuberculus arthritis
confirm the dx? itching and tiredness for few months. She gave a d. Exotosis
a. Abdominal US hx of heavy menstrual periods. Exam: pallor. What e. Fibrosarcoma
b. ANCA is the single most likely causative factor?
c. ANA a. IDA Ans. The key is B. Ewing’s sarcoma.
d. Urine microscopy and culture b. Lichen planus
e. Stool culture c. Dermatitis herpitiformis 1455. A 76yo man presents with sore throat, local
d. Eczema irritation by hot food, dysphagia and a sensation
Ans. The key is A. Abdominal US. [The likely dx is e. Uremia of a lump in his throat. He has a 20y hx of
ADPKD for which US is diagnostic investigation]. smoking. What is the single most likely dx?
Ans. The key is A. IDA. [IDA is one of the cause of a. Nasopharyngeal ca
1446. A 54yo man comes with sudden onset of pruritus. Heavy periods, pallor and tiredness b. Pharyngeal ca
palpitations and breathlessness. His HR=164bpm. further supports the diagnosis]. c. Sinus squamous cell ca
What is the single most appropriate tx in the d. Squamous cell laryngeal ca
acute phase? 1451. A 7yo child presents with lesions on the e. Hypopharyngeal ca
a. Adenosine trunk. Exam: some lesions are weeping and others
b. Metaprolol are crusted with a red base. What is the causative Ans. The key is B. Pharyngeal Ca.
c. Verapamil organism?
d. Amiodarone a. Herpes simplex 1456. A 42yo female who is obese comes with
b. Varicella zoster severe upper abdominal pain and right shoulder
Ans. The key is A. Adenosine. [Common c. Rubella virus tip pain with a temp=37.8C. She has 5 children.
arrhythmias we encounter are AF, SVT, VT in d. Herpes zoster What is the most probable dx?
exams. Here no suggestive feature for AF and SVT a. ERCP
is the commonest presentation as in described Ans. The key is B. Varicella zoster. b. LFT
case. So first we shall give adenosine to establish c. Serum amylase
the diagnosis]. 1452. An 87yo woman with a hx of HTN has acute d. MRCP
breathlessness. She has a RR=32bpm, widespread e. US abdomen
1447. A 29yo woman has developed an itchy scaly lung crackles, pulse=120bpm, BP=160/90mmHg
rash particularly over her wrist with fine white and elevated venous pressure. Her peripheral O2 Ans. The key is US abdomen. [5 “f”. Fat, female,
streaks overlying the lesion. Her nails have ridges sat=85%. What is the single most appropriate fair, fourty, fertile = cholecystitis].
and her buccal mucosa is lined with a lacy white initial management?
pattern. What is the single most likely dx? a. IV antibiotics 1457. A 37yo laborer comes with hx of redness of
a. Psoriasis b. IV furosemide left eye with foreign body sensation in the same
b. Scabies c. Nitrate infusion eye. What is the single most appropriate option?
c. Urtericaria d. Neb. Salbutamol a. Ciliary body
d. Dermatitis herpetiformis e. 100% oxygen b. Sclera
e. Hyperthyroidism c. Conjunctivitis
f. Lichen planus Ans. The key is E. 100% oxygen. [Most appropriate d. Cornea
initial management is E. 100% oxygen.Oxygen e. Iris
Ans. The key is F. Lichen planus. [“Lacy white saturation is low. So we have to give oxygen
pattern” is used as a diagnostic description of initially]. Ans. The key is D. Cornea. [Redness with foreign
lichenplanus]. body sensation of eye in a labourer are most likely
1453. A 25yo man presented with painless due to foreign body in cornea].
1448. The artery that runs in the ant inter- cervical lymphadenopathy with lethargy, night
ventricular groove. What is the single most sweats and itching. What is the single most likely 1458. An 11yo boy came to the hospital with pain
appropriate option? causative factor? after falling off his bicycle. XR= fx at distal radius
a. Acute marginal branch a. Lymphoma with forward angulations. What is the single most
b. Left ant descending artery b. Polycythemia probable dx?
a. Dinner fork deformity 1463. A man presented with carcinoma of the a. Doxycycline
b. Cubitus valgus bladder and has been working in factories. He b. Mefloquine
c. Gun stock deformity wants to know what dye has caused it. What is c. Proguanil
d. Garden spade deformity the single most likely cause? d. Quinine
e. Genu valgus a. Aniline e. Artesunate
Ans. The key is D. Garden spade deformity. b. Asbestos Ans. The key is B. Mefloquine which is a wrong
c. Latex key! None of the given option is correct!!
1459. A middle aged man with a lump in front of d. Silica [Shuffners dot indicate either vivax or ovale
his neck which moves up while he’s swallowing. infection and the hepatic cycle only can be
US shows a mass replacing the left lobe of thyroid. Ans. The key is A. Aniline. [Aniline dye is a well eradicated by primaquine. So none of the given
And spread to the sternocleidomastoid and known cause of bladder cancer]. drugs are the option!! It is a bad recall].
adjacent muscles. What is the most probable dx?
a. Thyroid ca 1464. A 62yo man presents with left sided hearing 1468. A 50yo man presents with flight of ideas
b. Pharyngeal pouch loss and tinnitus. He also complains of vomiting which are rambling and disinhibited. He is
c. Bronchus ca and headache. Exam: papilledema and SNHL in distractable, confused and overactive. What is the
d. Thyroid cyst the left ear. What is the single most likely dx? most likely dx?
e. Larynx ca a. Meningioma a. Dementia
b. Nasopharyngeal ca b. Mania
Ans. The key is A. Thyroid Ca. c. Acoustic neuroma c. Schizophrenia
d. Pharyngeal ca d. Psychosis
1460. A 28yo male complains of severe pain while e. Meniere’s disease e. Acute confusional state
trying to grasp any object. It started since he
participated in skiing and had a fall and caught his Ans. The key is C. Acoustic neuroma. [SNHL, Ans. The key is B. Mania. [Flight of ideas,
thumb in the matting. Exam: rupture of the ulnar tinnitus, papilledema (raised intracranial pressure) dysinhibition, distractibility, confusion and
collateral ligament of MCP joint of the thumb. are suggestive of acoustic neuroma]. overactivity are features of mania].
What is the single most probable deformity?
a. Dinner fork deformity 1465. A HIV +ve 55yo man presents with painless 1469. A pt presents with a lid lag, bulging eyes,
b. Game keeper thumb lymphadenopathy, fever, night sweats and weight opthalmoplegia and thyroid bruit. What inv will
c. Mallet finger loss. What is the most probable dx? you do?
d. Gun stock deformity a. Hodgkin’s lymphoma a. TFT
e. Garden spade deformity b. NHL b. Eye sight
c. ALL c. Tensilon test
Ans. The key is B. Game keeper thumb. d. AML d. US
[Gamekeeper's thumb is an insufficiency of the e. CML e. FNAC
ulnar collateral ligament (UCL) of the
metacarpophalangeal (MCP) joint of the thumb]. Ans. The key is B. NHL. [NHL is more likely Ans. The key is TFT. [TFT should be done. Why not
diagnosis in AIDS or immunodeficient state]. FNAC? Thyrotoxicosis is benign and not
1461. A 25yo male had an injury to the knee while malignant].
playing football. XR=condylar fx of tibia. What is 1466. A 22yo man says that he can hear the voice
the single most probable deformity? of his deceased uncle telling him that he is being 1470. A 30yo lady complaining of right ear
a. Dinner fork deformity spied on. The pt is distressed by this becoming deafness with decreased corneal reflex and past
b. Gibbus low in mood and anxious and has not left the pointing. Acoustic analysis shows SNHL. What is
c. Cubitus valgus house for 2wks. He is starting to drink increasing the next most appropriate inv to do?
d. Garden spade deformity quantities of alcohol. He is noticed to have a. CT brain
e. Genu valgus thought-block and passivity phenomena. What is b. CT acoustic canal
the single most suitable med to treat his c. MRI brain
Ans. The key is E. Genu valgus. symptom? d. MRI acoustic canal
a. Diazepam e. PET brain
1462. A 50yo man presents with itching after hot b. Disulfiram
shower with dizziness, chest pain after exercise. c. Fluoxetine Ans. The key is MRI brain. This is wrong key!
Exam: splenomegaly. What is the single most d. Lithium Correct key is D. MRI acoustic canal.
likely causative factor? e. Olanzapine
a. ALL 1471. A 29yo woman who returned from Egypt
b. Lymphoma Ans. The key is E. Olanzapine. [Auditory 2wks ago now presents with difficulty in
c. Polycythemia hallucination, social withdrawal, thought block are breathing, chest pain, cough and purulent sputum
d. Scabies features of schizophrenia. So olanzapine is the with an episode of blood staining. She is on
e. Eczema drug to be prescribed from the given option]. COCPs. What is the most likely dx?
a. Pulmonary embolism
Ans. The key is C. Polycythemia. [Itching after hot 1467. A middle aged Asian presents with b. Pneumonia
shower; dizziness and angina due to episodes of fever with rigors and chills for last 1yr. c. Lung abscess
hyperviscosity and splenomegaly are well known Blood film: ring form of plasmodium with d. Pneumothorax
features of polycythemia]. schuffners dots in RBCs. What is the drug to e. Pulmonary edema
eradicate this infection?
Ans. The key is B. Pneumonia. [Purulent sputum is given omeprazole 40mg. What is the single best
the clincher to differentiate pneumonia from 1476. A 26yo man presented with abdomen initial inv?
pulmonary embolism here]. distension and pain. His stools have been mucoid a. Hydrogen breath test
and sometimes blood stained. What is the most b. Gastroscopy
1472. A 60yo pt recovering from a surgery for appropriate inv? c. Barium meal
toxic goiter is found to be hypotensive, cyanosed a. Stool C&S d. None
in the recovery room. Exam: neck is tense. There b. Gastroscopy
is oozing of blood from the drain. What is the c. IgG tissue transglutaminase Ans. The key is B. Gastroscopy.
most probable dx? d. Barium meal 1481. A 35yo woman who usually has 4 days mid-
a. Thyroid storm e. Jejunal biopsy cycle bleeding, had her period 10d ago. She has
b. Reactionary hemorrhage now presented with spots of blood. Her smear
c. Secondary hemorrhage Ans. The key is D. Barium meal. [Probable was normal 6m ago. Exam: cervical ectropion
d. Primary hemorrhage diagnosis is Crohn’s disease. Can be demonstrated which doesn’t bleed on touch. What would you
e. Tracheomalacia by barium meal]. do?
a. Cervical smear
Ans. No key is given. Correct key is B. Reactionary 1477. An 83yo elderly woman presented in the ED b. Endocervical swab
hemorrhage. [Hemorrhage within 1st 24 hours with cough, fever and sneezing. Tx was given but c. US guided biopsy
which usually occurs due to dislodgement of clot she became confused and again presented with d. Laparotomy
or slippage of a ligature]. above said symptoms. What is the cause of her e. Transvaginal US
condition? f. Punch biopsy
1473. A 40yo woman has had varicose vein a. Aspiration due to confusion g. Serum estradiol
surgery, planned as a day pt. After the op, she is b. Alveolar damage due to drugs h. Colposcopy
distressed by repeated retching and vomiting. Her c. Drug toxicity
pain is currently well controlled. What is the best d. Pneumothorax Ans. The key is A. Cervical smear. Wrong key!
management strategy? Correct key is colposcopy! [Cervical smear can
a. Tramadol Ans. The key is A. Aspiration due to confusion. only be done on scheduled time and not in on
b. Co-codamol need basis. In such case if investigation is needed
c. IM morphine 1478. A 37yo man presents with some raised colposcopy can be done].
d. IV ondansetron lesions on the shin. He came with cough and also
e. PO ondansetron complains of arthralgia. Exam: bilateral hilar *1482. A 7yo boy presents with epistaxis of 2h
lymphadenopathy and erythema nodosum is duration. The bleeding has been controlled. Inv:
Ans. The key is D. IVondensatron. [As there is present. What is the single most likely cause? Plts=210, PT=13, APTT=42, bleeding time=normal.
repeated retching and vomiting pt. cannot keep a. CD Which of the following is the most likely dx?
oral medication down. So IV ondansatron]. b. UC a. Hemophilia
c. Sarcoidosis b. Von willebrand disease
1474. A pt with renal failure has serum K+=7.5, d. Streptococcal infection c. ITP
raised creatinine and broad complex tachycardia. e. TB d. Vit K deficiency
What is the most appropriate management? e. Liver disease
a. Calcium gluconate Ans. The key is B. UC. It is a wrong key! Correct f. Anatomical defect
b. Sodium bicarbonate key is C. Sarcoidosis. [Cough, arthralgia, bilateral
c. Dialysis hilar lymphadenopathy and erythema nodosum Ans. The key is F. Anatomical defect.
d. Furosemide are suggestive of sarcoidosis].
e. Sotalol 1483. A pregnant woman returns from Sudan,
1479. A young lady with cervical ectropion bleeds now presenting with intermittent fever, rigor and
Ans. The key is A. Calcium gluconate. [Calcium on touch. What is the most appropriate next inv? seizures. What is the dx?
gluconate don’t shift K+ to cell or reduce a. Transvaginal US a. TB
potassium level but it prevents arrythmogenic b. Cervical smear b. Malaria
action of raised K+ till definitive measure is taken]. c. Punch biopsy c. Meningitis
d. Serum estradiol d. Lyme disease
1475. An 18yo lady in her 30th wk of pregnancy is e. Colposcopy
brought to the hospital in an altered sensorium. Ans. The key is B. Malaria. [Intermittent fever is
She is taking slow, shallow breaths and her breath Ans. B. Cervical smear. Wrong key! Correct key is seen in malaria. In meningitis fever is not
has a fruity smell. An ABG: pH=7.20, urine E. colposcopy.[Screening test cervical smear is intermittent].
ketones: +ve. What is the most probable dx? only done in scheduled time and not in on
a. HONK demand basis. So if it is scheduled now then it can 1484. A pt is unresponsive and cyanosed. What is
b. DKA be the option otherwise Colposcopy should be the the most definitive 1st step in management?
c. HELLP syndrome key!! This is definitely an incomplete question]. a. Chest compressions
d. PIH b. Check airway
e. GDM 1480. A 28yo man with recent onset of dyspepsia c. Call 999
after eating spicy food and alcohol consumption. d. Mouth to mouth
Ans. The key is B. DKA. [Shallow breath and fruity H. pylori fecal antigen was negative. He returns e. Recovery position
smell with acidosis in ABG and positive ketone after 1m with similar symptoms despite being
body in urine suggests the diagnosis of DKA]. Ans. The key is B. Check airway.
dexamethasone 16mg daily.What is the single 1492. A retired ship worker has pleural effusion
1485. A man was bitten by a drug addict and most likely cause of her weakness? and pleural thickening on right side with bilateral
comes to the hosp with a wound. What inv should a. Paraneoplastic neuropathy lung shadowing. What would you do to improve
be undertaken? b. Progression of brain tumor his symptoms?
a. Hep C c. PID at L2/L3 a. Aspiration
b. Lyme disease d. Spinal cord compression b. Chest drain
c. Hep B e. Steroid induced myopathy c. Chemotherapy
d. Syphilis
e. Hep A Ans. No key is given! The likely correct option is D. d. Diuretic
Spinal cord compression. [Brain metastasis
Ans. The key is C. Hepatitis B. induced cerebral oedema can explain blurring of Ans. The key is C. Chemotherapy. It is a wrong
vision secondary to raised intracranial pressure. key! Correct key is B. Chest drain. [Respond to
1486. An 18yo woman says that she can’t walk Rest of the features including weakness can well chemotherapy and life expectancy is poor in
around as she is very big for that room. What is explain spinal cord compression]. mesothelioma. To improve symptoms chest drain
the most likely hallucination? should be undertaken].
a. Extracampine visual hallucinations 1490. A 78yo woman presents with unilateral
b. Liliputian visual hallucinations headache and pain on chewing. ESR=70mm/hr. 1493. An 88yo woman is a known smoker. She
c. Alice in wonderland syndrome She is on oral steroids. What is the appropriate had an attack of MI 2y back and is known to have
d. Hypnagogic hallucinations additional therapy? peripheral vascular disease. She presents with an
a. Bisphosphonates irreducible herniation over the incision region of a
Ans. The key is B. Lilliputian visual hallucination. b. HRT surgery which she underwent in her childhood.
[B. Liliputian visual hallucinations and C. Alice in c. ACEi What is the most appropriate tx?
wonderland syndrome are same]. d. IFN a. Truss
e. IV steroids b. Elective herniorrhaphy
1487. A middle aged lady presented with fever, c. Urgent herniorrhaphy
altered sensorium, bleeding gums and jaundice. Ans. The key is A. Bisphosphonates. [Oral steroid d. Elective herniotomy
Labs: deranged renal function tests, normal can lead to decrease in bone mineral density. To e. Reassure
PT/APTT, fragmented RBCs and low plts. What’s overcome this Bisphosphonate can be
the most likely dx? administered]. Ans. The key is B. Elective hernioraphy. [Truss can
a. Cholesterol emboli not be used as hernia is irreducible; urgent
b. HUS ***1491. A 48yo woman is admitted to the ED herniorrhaphy in strangulation; elective
c. TTP with a productive cough and mod fever. She often herniotomy- in herniotomy the hernia will not
d. Hepatorenal syndrome has central chest pain and she regurgitates subside or recur; only reassure is not an option as
e. Sepsis undigested food most of the time but doesn’t irreducibe hernia may become strangulated or
suffer from acid reflux. These symptoms have may develop intestinal obstruction].
Ans. The key is C. TTP. [Fever and altered been present for the last 3.5m which affects her
sensorium suggest the diagnosis of TTP]. daily food intake. CXR: air-fluid level behind a 1494. A 72yo woman who is taking loop diuretics
normal sized heart. What is the single most likely for left ventricular failure. She now is suffering
1488. A child came to the ED with severe asthma dx? from palpitations and muscle weakness. What is
and not responding to salbutamol nebulizer and a. Pharyngeal pouch the electrolyte imbalance found?
vomiting many times. What is the most b. Hiatus hernia a. Na+=130mmol/L, K+=2.5mmol/L
appropriate management? c. Bulbar palsy b. Na+=130mmol/L, K+=5.5mmol/L
a. Salmetrol d. Achalasia c. Na+=140mmol/L, K+=4.5mmol/L
b. Montelukast e. TB d. Na+=150mmol/L, K+=3.5mmol/L
c. Prednisolone e. None
d. Budesonide inhaler Q. 1. What is the key?
e. Oxygen Q. 2. What are the points in favour? Ans. The key is A. Na+=130mmol/L,
f. IV salbutamol K+=2.5mmol/L. [Loop diuretics causes
Ans. 1. The key is B. Hiatus hernia. This is a wrong hyponatremia and hypokalemia].
Ans. The key is A. Salmeterol which is a wrong key. Correct key should be D. Achalasia.
key!! The correct option is F. IV salbutamol. [In Ans. 2. Points in favour: Aspiration pneumonia 1495. A young woman who is a marathon runner
acute attack there is no place for salmeterol. In due to retained food and fluid in oesophagus. In comes with secondary amenorrhea. Inv: normal
the given case most appropriate management is achalasia usually there is no acid reflux. Dysphagia LH, FSH and estradiol, prolactin=600. What is the
IV salbutamol]. for both food and drink. Air-fluid level behind most likely dx?
heart. a. Hypothalamic amenorrhea
1489. A 73yo woman with skeletal and brain mets Why it is not hiatus hernia? Ans. Differentiating b. Pregnancy
from breast ca has worsening low back pain and point:-i) In hiatus hernia usually you will get c. PCOS
blurring of vision. She has weakness of her legs, associated GORD ii) Also in hiatus hernia there d. Prolactinoma
minimal knee and absent ankle tendon reflexes, a may be nausea or vomiting. e. Anorexia
palpable bladder, a power of 2/5 at the hip, 3/5 at Why it is not pharyngeal pouch? Ans. In
the knee and ankle, and tenderness over the 2nd pharyngeal pouch there will be halitosis. Ans. The key is D. Prolactinoma. It is a wrong key.
lumbar vertebra. There is reduced sensation in Correct key is A. Hypothalamic amenorrhea.
the perineum. She has been started on [Generally in hypothalamic amenorrhea there is
slight low level of LH, FSH and Oestrogen and mild 1501. A 23yo man is having difficulty in speaking
elavation of prolactin]. following a stab wound to the right of his neck. On Ans. The key is B. Simvastatin. It is a wrong key.
being asked to protude his tongue, the tip Correct key should be LMWH. [Specific treatment
1496. A 4yo child comes with a sprain in his foot. deviated to the right. Which anatomical site is means treatment particularly adapted to the
Hx reveals that the child has had recurrent most likely to be affected? special disease being treated. LMWH is vital to
admissions to the hosp due to severe asthma. a. Facial nerve prevent further events or deterioration and seems
What is the most appropriate analgesic? b. Hypoglossal nerve to be the most specific drug in the given scenario].
a. Diclofenac sodium c. Vagus nerve
b. Ibuprofen d. Trigeminal nerve 1506. A 69yo woman presents with a sudden
c. Paracetamol e. Glossopharyngeal nerve onset of weakness of her right arm and leg. She is
d. Codeine known to be hypertensive. There has been no
Ans. The key is B. Hypoglossal nerve. [In headache, LOC, visual, speech or sensory
Ans. The key is C. Paracetamol. hypoglossal nerve palsy the tongue will be curved symptoms. Exam: BP=180/90mmHg, pulse=100
toward the damaged side, combined with the and regular heart sounds, no carotid bruit. Higher
1497. A 34yo pregnant woman, 38wk GA is in presence of fasciculations or atrophy]. mental function tests are normal. No apraxia or
labor. She had a long 1st stage and troublesome neglect. Speech, swallowing and sensation are
2nd stage, has delivered a baby. After her 1502. A girl presents with signs of normal. There are no visual field defects. There is
placenta was delivered she had a convulsion. hyperventilation. What is the most likely ABG a mild facial weakness sparing the forehead. The
What is the most probable management? derangement? right arm and leg are flaccid and weak. Reflexes
a. MgSO4 IV a. pH increased, PCO2 increased and tone are normal. There is a right extensor
b. Diazepam IV b. pH decreased, PCO2 increased plantar response. What is the most likely cause of
c. IV fluid c. pH increased, PCO2 decreased this pt’s symptoms?
d. Hydralazine IV d. pH decreased, PCO2 decreased a. Cardioembolic stroke
e. Anti-epileptic b. Lacunar stroke
Ans. The key is A. MgSO4 IV. [Eclampsia, tx is IV Ans. The key is A. pH increased, PCO2 increased. c. Right internal carotid artery atheroembolic
MgSO4]. This is a wrong key. Correct key should be C. pH stroke
increased, PCO2 decreased. d. Right internal carotid artery dissection
1498. A 23yo woman presents with offensive e. Right vertebral artery atheroembolic stroke
vaginal discharge. Vaginal pH=4.5. What is the 1503. A pt presents with skin pigmentation, Ans. The key is B. Lacunar stroke. [Weakness of
most likely organism? diarrhea, vomiting, abdominal pain and postural right arm and leg. So lesion is on left side. Hence
a. Gardenella hypotension. What electrolyte abnormality is C,D and E can not be the option. Also no AF or
b. Trichomonas likely to occur? carotid artery disease and features are very much
c. Candida a. Na+=130, K+=6.5 consistent with lacunar infarct].
d. Mycoplasma b. Na+=130, K+=2.5
c. Na+=13, K+=6.0 1507. A 34yo man has an intermittent epigastric
Ans. The key is A. Gardenella. d. Na+=140, K+=8 pain for 3wks. It is worse by food but helped by
e. Na+=130, K+=1.5 some tablets he obtained from the pharmacy. He
1499. A 62yo man has had ano-rectal pain had a similar episode 3yrs ago and his doctor gave
aggravated by defecation for 3d. Rectal exam: Ans. The key is A. Na+=130, K+=6.5. [In Addison’s him a course of 3 types of tablets at the time.
purple, tender lump at the anal verge. Flexible disease there is low Na+, low Cl-, low CO2 and What is the most appropriate next inv?
aigmoidoscopy: normal rectal mucosa and hard raised K+]. a. Abdomen US
feces. What is the best management strategy? b. Barium meal
a. Anal hematoma 1504. A 10yo boy develops nasal bleeding. What c. Serum H.Pylori antibodies
b. Anal fissure is the best way to stop the bleeding from the d. C13 urea breath test
c. Rectal ca nose? e. Upper GI endoscopy
d. Diverticulitis a. Pressure over base of the nose
e. Angiodysplasia b. Ice packs Ans. The key is D. C13 urea breath test. [Patient
c. Pressure over the soft tissues was allright for 3yrs after eradication therapy.
Ans. The key is A. Anal hematoma. d. Nasal packing Now symptoms again may indicate recurrence of
e. Surgery H. Pylori infection which can be demonstrated by
1500. A 43yo presents with severe vertigo on C13 urea breath test. Serum antibody persist (IgG)
moving sidewards whilst sleeping. What test Ans. The key is C. Pressure over the soft tissues. for long and not reliable for recheck].
would you do to confirm the dx?
a. Hallpikes maneovure 1505. A pt came to the hosp with a complaint of 1508. A girl with sickle cell anemia has painful
b. Romberg’s test severe chest pain lasting for >1h. Following ECG bleeding and vaso-occlusive crisis during her
c. Trendelenburg test test, pt revealed to have ST depression. He was periods. What is the best possible management
d. Heel-shin test already on aspirin. What is the most specific tx for for this pt?
this pt? a. COCP
Ans. The key is A. Hallpike maneovure. [Hallpike a. GTN b. Tranexamic acid
maneovure is the preferred method to detect b. Simvastatin c. Copper IUS
benign positional vertigo]. c. Clopidogrel d. UAE
d. BB e. Depot provera
e. LMWH
Ans. The key is E. Depot provera. [Hormone and e. Fronto-temporal dementia personality, cognition and physical skills at the age
barrier methods are all acceptable choices but Ans. The key is C. Huntington’s disease. [A person of 35-45yrs and there is chorea. In vascular
intrauterine devices are not recommended, as with Huntington's disease may appear to have a dementia there is multiple progressive infarcts
they may be associated with uterine bleeding and lack of drive, initiative and concentration. and so neurological features may be associated
infection. Involuntary jerking or writhing movements with features of other vascular disease].
Depot contraceptive (Depo-Provera®) is safe and (chorea). Typical presentation is between 35-55
has been found to improve the blood picture and yrs]. 1516. A 38yo woman with hemophilia who
reduce pain crises]. received several blood transfusions a few years
1513. A 54yo man has collapsed suddenly ago presents with irritability and increasing
1509. A 70yo pt comes with swelling in the following a headache. He has hypertension and memory deficit. She is unable to speak properly.
parotid region for the last 10y. Exam: gland is soft takes warfarin for prosthetic heart valve. GCS=4 He is on anti-TB tx. What is the single most likely
and cystic. Choose the most probable dx? and dilated left pupil. What is the single most dx?
a. Pleomorphic adenoma likely dx? a. Creutzfeldt Jacob disease
b. Carcinoma of the salivary glands a. Ant circulation stroke b. Drug toxicity
c. Mikulicz’s disease b. Post circulation stroke c. Vascular dementia
d. Adenoid cystic carcinoma c. Intracerebral hemorrhage d. HIV associated dementia
e. Parotid duct stones d. Intracerebellar hemorrhage e. Space occupying lesion
e. Pontine hemorrhage.
Ans. The key is D. Adenoid cystic carcinoma. Ans. The key is D. HIV associated dementia. [Blood
[Pleomorphic adenoma is firm in consistency not Ans. The key is C. Intracerebral hemorrhage. transfision is the clue for HIV transmission.
soft and cystic. Also adenoid cystic carcinoma [Headache, collapse, and warfarine use makes the Immunodeiciency is also responsible for TB].
progresses slowly and 15 year cervival is 40%]. dx of intracerebral hemorrhage most likely.
“Pointers to bleeding (unreliable!): Meningism, 1517. An 18yo girl has menorrhagia and
1510. A 74yo man has been admitted unconscious severe headache, and coma within hours.” dysmenorrhea and requires contraception. What
with no hx. He has a GCS=6 and a dilated left puil OHCM]. drug will you give her?
which becomes insensitive to light. What is the a. COCP
single most likely dx? 1514. A 5wk breast fed baby whose birth weight b. Mirena coil
a. Extradural hematoma was 3.5kg and is now 4.5kg is thriving well but is c. Copper T
b. Meningitis deeply jaundiced. What is the most likely dx? d. UAE
c. Opioid OD a. Galactosemia e. Depo provera
d. Pontine hemorrhage b. Breast milk jaundice
e. SAH c. Thalassemia Ans. The key is A. COCP. It is a wrong key. Correct
Ans. The key is E. SAH. [Extradural hematoma is d. Sickle cell disease key should be B. Mirena coil.
very rare in elderly and there occurs lucid interval; e. Congenital storage disorder
Fteatures are not consistent with meningitis. 1518. A pt of tuberculous abscess with the hx of
Opioid and pontine hemorrhage causes myosis. So Ans. The key is B. Breast milk jaundice. [Breast prv abscess drainage presented with fever and
likely dx here is SAH]. milk jaundice is a type of neonatal jaundice tenderness between L2/L3 vertebra. Which is the
associated with breastfeeding. It is characterized best inv for this pt?
1511. A 27yo man presents to the ED with 2d hx by indirect hyperbilirubinemia, presents in the a. XR
of severe headache and pyrexia (38.9C). CT: first or second week of life, and can persist for as b. CT
petechial hemorrhage in the temporal and inf long as 12 weeks before spontaneous resolution. c. US
frontal lobes. What is the most likely dx? There is normal thrive and weight gain. In d. MRI
a. Brain abscess galactosemia there will be lethargy, vomiting, e. Blood culture
b. Meningococcal meningitis diarrhea and failure to thrive].
c. Cerebral malaria Ans. The key is D. MRI. [Vertebral collapse from
d. Herpes simplex encephalitis 1515. A 71yo man with no prv immediate hx is TB infection is better seen on MRI].
e. New variant CID brought to the ED by his wife who says he has
become progressively more forgetful, tends to 1519. A 4yo child presents with repeated chest
Ans. The key is D. Herpes simplex encephalitis. lose his temper and is emotionally labile. There is infections. He has yellow discoloration of sclera
[Petechial hemorrhage in the temporal and no hx of infectious disease or trauma. What’s the and the mother gives a hx of diarrhea as well.
inferior frontal lobes are characteristic of Herpes single most likely dx? What is the single inv most likely to lead to a dx?
simplex encephalitis]. a. Pic’s dementia a. Sweat chloride test
b. Fronto-temporal dementia b. Anti-endomysial antiboides
1512. A 44yo woman with memory loss, poor c. Huntington’s disease c. LFT
concentration and inability to recognize d. Alzheimer’s disease d. Jejunal biopsy
household projects. She has right-handed e. Vascular dementia e. TFT
involuntary writhing movement. There is strong
fam hx of similar complain. What is the single Ans. The key is D. Alzheimer’s disease. [Pic’s Ans. The key is A. Sweat chloride test. [Repeated
most likely dx? dementia and fronto-temporal dementia are chest infections, and jaundice suggests dx of cystic
a. Pic’s dementia similar in some way (first personality change then fibrosis. In the hepatobiliary system which is
b. Wilson’s disease dementia) but in pic’s there is odd social behavior damaged in about 25 to 30% of the cystic fibrosis
c. Huntington’s disease like violating etiquettes and says vulgar. In patients, malfunctioning secretion can cause liver
d. HIV associated dementia huntington’s disease there is changes in disease and gall stones, leading to pain and
jaundice. For cystic fibrosis sweat chloride test located in the abdomen. What is the best e. Sinus tachycardia
can be done]. management plan for this pt?
a. Short trial of HCG Ans. The key ia A. Atrial fibrillation. [In A-
1520. An 82yo woman has been admitted from a b. Orchidectomy fibrillation fibrillatory f waves replaces p waves
nursing home with dense hemiplegia and c. Orchidopexy and R – R interval are irregular].
homonymous hemianopia. She is dysphasic. What d. Reassurance
vessel is most likely to be involved? e. IV testosterone 1528. The ECG of an 80yo pt of IHD shows
a. Ant cerebral artery sawtooth like waves, QRS complex of 80ms,
b. Mid cerebral artery Ans. The key is C. Orchidectomy. [Ectopic testis is ventricular rate of 150bpm and regular R-R
c. Post cerebral artery prone to develop testicular cancer and therefore interval. What is the most probable dx?
d. Internal carotid artery it should be surgically removed]. a. A-fib
e. Post inf cerebellar artery b. A-flutter
*1525. A 56yo male who presented with epilepsy c. SVT
Ans. The key is B. Mid cerebral artery. like symptoms has been dx with an intracranial d. Mobitz type 1 2nd degree heart block
[Hemiplegia, homonymous hemianopia, dysphasia space occupying lesion. He now complains of e. Sinus tachycardia
these are common features of mid cerebral artery thirst and mild dehydration. His blood glucose is
stroke]. also increased. What is the single most Ans. The key is B. Atrial flutter. [Sawtoothlike
appropriate immediate tx? waves and regular R-R interval are diagnostic of
1521. A pt is dx with SIADH. Choose the a. Insulin atrial flutter].
appropriate biochemical change. b. IV fluids
a. Plasma Na+ decrease and urine osmolarity c. Stop dexamethasone 1529. A man brings his wife into the ED after
increase d. Stop sodium valproate and change to another finding her unconscious at home. He says at
b. Plasma Na+ decrease and urine osmolarity anti-epileptic breakfast time she had complained of sudden
decrease severe headache. What is the most appropriate
c. Plasma Na+ increase and urine osmolarity Ans. The key is B. IV fluids. It is wrong key. Correct inv?
decrease key should be A. Insulin. [Hyperglycemia is not a a. MRI
d. Plasma Na+ increase and urine osmolarity feature of ICSOL per se but indicates b. XR
increase dexamethasone therapy. For dexamethasone c. CT brain
induced hyperglycemia lose insulin is the d. Carotid Doppler
Ans. The key is A. Plasma Na+ decrease and urine treatment. So answer should be A. insulin. Also
osmolarity increase. for mild dehydration IV fluid is not indicated. IV Ans. The key is C. CT brain. [Probable SAH. Among
fluid is not indicated even in moderate the given option most appropriate is CT brain].
1522. A newborn that is electively intubated at dehydration. It is only indicated in severe
birth and is due for surgery 48h after birth. The dehydration]. 1530. A 68yo lady with T2DM. Which drug should
condition was suspected on antenatal US on CXR. be prescribed?
What is the most likely dx? 1526. A mother brings her newborn to the hosp a. Biguanides
a. CF concerned about a blue patch on the buttocks. b. Sulphonyl urea
b. Congenital diaphragmatic hernia The newborn is of mixed race and was delivered c. Insulin
c. Congenital cystic adenomatoid malformation normally. What is the most appropriate d. Lifestyle modifications
d. RDS management?
e. Alpha 1 antitrypsin deficiency a. Reassurance Ans. The key is A. Biguanide. [This is an
b. CBC incomplete question and BMI is needed to decide
Ans. The key is B. Congenital diaphragmatic c. XR whether biguanide or sulphonylurea be
hernia. d. Plt count prescribed].

1523. A 63yo male undergoes abdominal surgery. Ans. The key is A. Reassurance. [The diagnosis is 1531. In a laparoscopic mesh repair for hernia,
On Monday morning, 3d post-op, repeat samples mongolian blue or spot. Mongolian spot refers to when the trochar is inserted at midpoint between
confirm serum K+=7.1mmol/l. His ECG shows a macular blue-gray pigmentation usually on the umbilicus and ischial spine. What structure will be
broad QRS complexes. Which one of the following sacral area of healthy infants. Mongolian spot is pierced?
can be used as an effective tx for this pt’s usually present at birth or appears within the first a. Linea alba
hyperkalemia? weeks of life. Mongolian spot typically disappears b. Rectus muscle
a. Calcium chloride IV spontaneously within 4 years but can persist for c. Conjoint tendon
b. Calcium gluconate IV life. It is a benign condition requiring no d. External and internal oblique muscles
c. Insulin subcutaneously intervention]. e. Inguinal ligament
d. Furosemide IV
Ans. The key is B. Calcium gluconate IV. [Calcium 1527. The ECG of a 65yo shows absent P waves, Ans. The key is D. External and internal oblique
gluconate does not shift potassium into cells or narrow QRS complex, ventricular rate of 120bpm muscles.
reduce its level but prevents its arrythmogenic and irregular R-R interval. What is the most
effect on heart and buys time till definitive probable dx? *1532. A 48yo man has intermittent left sided
measures are taken]. a. A-fib lower abdominal pain and feels generally unwell.
b. A-flutter He has lost his appetite and has lost weight.
1524. A 25yo man attended in urological OPD has c. SVT Temp=38.3C and he has BP=190/100mmHg. What
single testis. He was inv and other testis was d. Mobitz type 1 2nd degree heart block is the single inv most likely to lead to dx”?
a. Colonoscopy Ans. The key is E. Pseudomonas. chronic cough and forgetfulness. He is on long
b. Endomysial antibodies term antipsychotic meds. What is the single most
c. Fasting serum glucose conc 1537. An obese woman with hx of migraine likely dx?
d. TFT presented with heavy bleeding during a. Shy drager syndrome
e. US abdomen menstruation which is painful and needs b. Parkinsonism
contraception too. What is the best possible c. Huntington’s chorea
Ans. The key is E. US abdomen. [Probable management for this pt? d. Tardive dyskinesia
diagnosis is diverticulitis and preferred a. COCP e. Akathisia
investigation from given list is US abdomen]. b. Mirena coil
c. Copper T Ans. The key is B. Parkinsonism. [Antypsychotic
1533. A man with DM comes to the ED after he d. UAE drugs can cause parkinsonism].
collapsed at home. His GCS=10. What should be e. Depo provera
the next initial inv for this man? 1542. A 34yo female presented with vomiting
a. Capillary blood sugar Ans. The key is B. Mirena coil. preceded by an occipital headache of acute onset.
b. MRI head Exam: conscious and alert with photophobia but
c. CT head 1538. A 2yo fell on outstretched hand on no neck stiffness. CT: normal. What is the most
d. Serum electrolytes playground. He presents with pain on base of the appropriate further management?
thumb. XR=no fx. What is the single most likely a. CT brain with contrast
Ans. The key is A. Capillary blood sugar. [It may be dx? b. Repeat CT brain in 24h
either hypoglycemic or hyperglycemic coma a. Colles fx c. CSF exam
which can be detected by checking capillary blood b. Head of radius d. Cerebral angio
sugar]. c. Mellet finger e. MRI brain
d. Scaphoid fx
1534. A 60yo DM pt presented with easy e. No fx Ans. The key is C. CSF exam. [Probable
fatigability, weakness and numbness of hands and subarachnoid hemorrhage. CT may not show any
swollen feet. Exam: pedal edema, sensory Ans. The key is D. Scaphoid fx. [Scaphoid fracture abnormality in early stage in some instances and
neuropathy and palpable liver and spleen. Urine: is often missed in initial x-ray]. in that case we shall do CSF examination 12 hours
proteinuria. US abdomen: enlarged kidney. Renal later to see xanthochromia].
biopsy: amorphous homogenous substance that 1539. A pt was admitted with increased
stained red with congo-red. What is the dx? frequency of passing urine, increased thirst, 1543. A lady with post ileo-colectomy closure of
a. DM retinopathy weakness and muscle cramps. What is the most stoma has a small 4cm swelling around the stoma.
b. Sarcoidosis probable dx? What is the most appropriate management of the
c. Wilms tumor a. Conn’s syndrome swelling?
d. Amyloidosis b. Cushing’s syndrome a. Local exploration of swelling
e. Glycogen storage disease c. Pheochromocytoma b. Exploratory laparotomy
d. Hyperthyroidism c. Open laparotomy and re-closure
Ans. The key is D. Amyloidosis [Amyloidosis is a e. Hypoparathyroidism d. Abdominal binder
common cause of organomegaly. Also in diabetic e. Truss
nephropathy kidneys may be enlarged]. Ans. The key is A. Conn’s syndrome. [There is f. Laparotomy with mesh repair
hypokalemia in Conn’s syndrome. Increased
1535. A 75yo man has urinary symptoms of frequency of passing urine and increased thirst Ans. The key is A. Local exploration of swelling.
hesitancy, frequency and nocturia. Rectal exam: are from nephrogenic DI resulted from
large hard prostate. What is the most appropriate hypokalemia and hypokalemia also causes 1544. A 64yo woman has been brought by her son
inv? weakness and muscle cramps]. for psychiatric evaluation. She says that she has
a. CA 125 stopped living with her husband because she is
b. CA 153 1540. A 69yo male presented with sudden onset convinced it is someone else posing to be him.
c. CA 199 of dysphagia. He is neither able to swallow liquid What kind of delusions is she suffering from?
d. CEA nor solid, he recently had a denture fitting. What a. Delusion of reference
e. PSA is the most probable dx? b. Delusion of control
a. Foreign body c. Delusion of guilt
Ans. The key is E. PSA. [Urinary symptoms and b. Plummer vinson syndrome d. Delusion of persecution
hard prostate on PR suggests prostatic cancer c. Achalasia cardia e. Delusion of doubles
forwhich PSA should be done]. d. Esophageal rupture
e. Esophageal ca 1 Ans. The key is B. Delusion of control. This is a
1536. A child suffering from CF developed wrong key! The correct key is E. Delusion of
pneumonia. Which organism is responsible for Ans. The key is A. Foreign body. [Sudden onset of doubles. [Delusion of control is a false belief that
this pneumonia? dysphagia to both liquid and solid and recent another person, group of people, or external force
a. H. influenza history of fitting denture suggests foreign body controls one's general thoughts, feelings,
b. Klebsiella (denture) in oesophagus]. impulses, or behavior. The delusion of doubles, is
c. S. aureus a rare disorder in which a person holds a
d. S. pneumonia 1541. A 62yo man with chronic schizophrenia delusional belief that different people are in fact a
e. Pseudomonas presents with a mask like face and involuntary pill single person who changes appearance or is in
rolling movement in both hands. He complains of disguise].
e. Cortical blindness
1545. A 19yo man with known hx of OM presents 1549. A 55yo woman suffered from an acute MI
with headache, lethargy, sweating and shivering. 5d ago. While she was in the hosp the pt Ans. The key is E. Cortical blindness. [Cortical
What is the single most likely dx? developed features of pulmonary edema and blindness is the total or partial loss of vision in a
a. Furuncle heart failure. What is the most probable cause of normal-appearing eye caused by damage to the
b. Meningitis her present condition? brain's occipital cortex. Rarely, a patient with
c. Myringitis a. VSD acquired cortical blindness may have little or no
d. Nasopharyngeal tumor b. Ruptured papillary muscle insight that they have lost vision, a phenomenon
e. OM c. Pericarditis known as Anton–Babinski syndrome].
Ans. The key is B. Meningitis. [OM often can be a d. A-fib
cause of intracranial extension leading to cerebral e. Re-infarction 1554. An elderly lady presents with confusion. She
abscess or meningitis]. is apyrexial but complains of dysuria for 2d
Ans. The key is B. Ruptured papillary muscle. [Post duration. What is the def dx inv?
1546. A 46yo woman has gained weight. She has MI rupture of papillary muscl is responsible for a. Blood culture
sensitivity to cold. Her pulse = regular at 50bpm valve failure (mitral regurgitation) causing b. Urine nitrates
and heart=enlarged. What is the single most pulmonary oedema and heart failure]. c. CT head
likely underlying mechanism for this condition d. ECG
a. Autoimmune 1550. A 76yo woman presents with deep stroke e. IVU
b. Degenerative 6h ago. What would the immediate tx be?
c. Congenital a. Aspirin 75mg Ans. The key is B. Urine nitrates. [Dysurea is
d. Infective b. Aspirin 300mg suggestive of uti. UTI also can lead to confusion.
e. Nutritional c. Streptokinase Urine nitrates if positive is diagnostic of UTI].
d. IV heparin
Ans. The key is E. Nutritional. This is a wrong key. e. Dipyridamole 200mg 1555. A 40yo woman on chemotherapy for
Correct key is Autoimmune hypothyroidism. [The metastatic breast carcinoma now presents with
most common cause of hypothyroidism in UK is Ans. The key is B. Aspirin 300mg. painful swallowing. Exam: she has white plaques
autoimmune hypothyroidism. In developing on top of friable mucosa in her mouth and more
countries it is often nutritional (iodine 1551. A 19yo man accuses his friend of making his seen on esophagoscopy. What is the most
deficiency).The diagnosis is autoimmune right arm swing out at a stranger. What is the best effective tx for this pt?
hypothyroidism (in hypothyroidism heart is often term to describe his condition? a. Antispasmodic
gets enlarged)]. a. Control b. H2 blocker
b. Persecution c. Antibiotics
1547. A 70yo man presents with a punched out c. Guilt d. Antifungals
ulcer between his toes. He is a heavy drinker and d. Reference e. I&D
smoker. Exam: ulcer is yellow and the foot turns e. Grandeur
red when dangling off the bed. What is the single Ans. The key is D. Antifungal. [Features are
most likely dx? Ans. The key is A. Control. [Delusion of control is a consistent with oesophageal candidiasis which is
a. Arterial ischemia ulcer false belief that another person, group of people, treated with Oral fluconazole (200-400 mg daily)
b. Malignancy or external force controls one's general thoughts, or IV fluconazole for 14-21 days].
c. Neuropathic ulcer feelings, impulses, or behavior].
d. Pressure ulcer 1556. A 43yo woman has suffered with heavy
e. Venous stasis ulcer 1552. A 26yo man with hx of hereditary periods for many years and has tried many
hemorrhagic telengectasia is planning to start a medical tx without success. She is constantly
Ans. The key is A. Arterial ischemic ulcer. family. What is the mode of inheritance? flooding and at times can’t leave her house due to
a. AD with incomplete penetrance heavy bleeding. She has completed her family of 5
1548. A 65yo woman complains of a painful b. Autosomal co-dominant children and her last blood test showed
discharging ulcer above her ankle on the inner c. AR with incomplete penetrance Hgb=8.9g/dl. She feels that she can’t cope with
side of her left lower leg. Exam: the base of the d. AD the bleeding anymore and her husband is asking
ulcer is red and covered by a yellow fibrous tissue. e. AR for a tx that can guarantee success. What is the
The border is irregular. The skin is tight. What is most appropriate management to improve
the single most likely dx? Ans. The key is D. AD [Hereditary hemorrhagic menorrhagia in this pt?
a. Arterial ischemia ulcer telangiectasia is of autosomal dominant mode of a. Endometrial ablation
b. Malignancy inheritance]. b. Hysterectomy
c. Neuropathic ulcer c. Hysteroscopic/Laser resection of fibroids
d. Pressure ulcer 1553. A 50yo man with a known hx of stroke is d. Myomectomy
e. Venous stasis ulcer unable to get out of his house because he can’t e. UAE
find where the door is. He refuses help from his
Ans. The key is E. Venous stasis ulcer. [Some wife and says he is not blind. What is the single Ans. The key is B. Hysterectomy. [As family is
people with venous insufficiency develop stasis most likely defect? complete hysterectomy is the best option to stop
dermatitis. Blood pools in the veins of the lower a. Paracentral scotoma bleeding and it also helps not to get endometrial
leg. Fluid and blood cells leak out of the veins into b. Tunnel vision carcinoma].
the skin. It is usually seen above ankle and inner c. Total blindness
side of leg]. d. Central scotoma
1557. A man on antipsychotic meds develops 1561. A 50yo man complains of dysphagia after e. Anti centromere
features of retinitis pigmentosa. Which drug is eating bread. Barium swallow reveals a lower
most likely to cause these symptoms? esophageal ring. What is the most appropriate tx? Ans. The key is B. C ANCA. [ARF and recurrent
a. Thioridazine a. Reassurance epistaxis and hemoptysis suggests Wegener’s
b. Haloperidol b. Antispasmodics granulomatosis for which C ANCA is most
c. Chlorpromazine c. Dilatation of the LES specific].
d. Risperidone d. Endoscopic diverticulectomy
e. I&D 1566. A woman is admitted to the hosp for
Ans. The key is A. Thioridazine. [Thioridazine and elective abdominal hysterectomy. 2m ago she was
other antipsychotics (neuroleptics, dopaminergic Ans. The key is C. Dilatation of the LES. dx with DVT and pulmonary embolism and was
antagonists) can cause degenerative retinopathies started on warfarin. What is the most appropriate
with histological, electrophysiological and 1562. A 48yo nulliparous woman feels tired all the preop measure you will take on this occasion?
symptomatological features similar to those of time. Her periods are regular but have always a. Continue warfarin
primary retinitis pigmentosa]. lasted for at least 10d. Choose the single most b. Stop warfarin
appropriate intial inv? c. Stop warfarin and start heparin
1558. Pt with low Hgb, MCV=76, angular a. High vaginal swab d. Increase warfarin dose e. Add heparin
stomatitis, red tongue, and koilonichea. What is b. Serum Hgb conc
the most probable dx? c. TFT Ans. The key is C. Stop warfarin and start heparin.
a. Folate def d. None
b. B12 def e. Abdominal US 1567. This condition affects middle aged women
c. Iron def Ans. The key is B. Serum Hb conc. [Feeling tired all more than men and is characterized by low mood,
d. Vit E def the time and prolonged period suggest anemia. So early morning waking, loss of libido, tiredness and
e. Hemolytic anemia serum Hb should be done initially]. suicidal intention last for at least 2wks. What is
the most probable dx?
Ans. The key is C. Iron deficiency anemia. [Low 1563. A man got his hand caught in machinery at a. Bipolar affective disorder
MCV, angular stomatitis, red tongue and work. The fingers are swollen but the XR shows no b. Dysthymia
koilonichea are characteristic of iron deficiency fx. What is the most appropriate management? c. Major depressive disorder
anemia]. a. Splint d. Schizoaffective disorder
b. Put in plaster e. Recurrent brief depression
1559. A pt with sudden severe eye pain, red eye, c. Broad arm sling for 1wk
visual blurring, acuity of only finger counting, d. Elevate in high sling for 2d Ans. The key is C. Major depressive disorder.
nausea, vomiting with a shallow ant chamber that e. Neighbor strapping [Given case is severe depression (major
is hazy on shining a torch. What is the dx? depressive disorder).
a. CRVO Ans. The key is D. Elevate in high sling for 2d. Mild depression: i)Low mood ii) Anhedonia iii)
b. Acute closed angle glaucoma [Extravasetion of blood caused this swelling along Guilt iv) Hopelessness v) Worthlessness vi)
c. Uveitis with inflammation for which best option is elevate Inability to concentrate. Tx CBT
d. Iritis in high sling for 2d which will help improve Moderate depression: Features of mild + vii) Poor
e. Open angle glaucoma symptom by gravitational shift of fluid from hand]. sleep viii) Poor Appetite ix) Poor libido x) Easy
fatiguability. Tx Antidepressants
Ans. The key is E. Open angle glaucoma. This is a 1564. A 39yo woman presents with symptoms Severe depression: Features of moderate + xi)
wrong key! The correct key is B. Acute closed recurring annually characterized by depressed Suicidal intensions. Tx ECT
angle glaucoma. [Sudden severe eye pain, red mood, being socially withdrawn hypersomnia, lack Psychotic depression: Features of severe + xii)
eye, visual blurring, acuity of only finger counting, of enjoyment in life, last for several months. What Hallucinations xiii) Delusions xiv) Guilt xv) Nihilistic
nausea, vomiting, shallow anterior chamber that is the most likely dx? delusion. Tx ECT].
is hazy on shining torch are all classic presentation a. Seasonal Affective Disorder
of acute closed angle glaucoma]. b. Mod depression 1568. A 10yo boy has fallen from a tree and
c. Dysthymia injured his right chest. He has pain and difficulty
1560. A pt who works in a pet shop has d. GAD breathing. He is tachypenic and tender with an
temp=37.5C, dyspnea, chest pain and cough. CXR: e. Bipolar disorder area of paradoxical chest wall movement on the
patchy consolidation. What is the most suitable right side. What is the single most likely dx?
tx? Ans. The key is A. Seasonal Affective Disorder. a. Diaphragmatic rupture
a. Amoxicillin [Depression associated with late autumn and b. Flial chest
b. Tetracyclin winter and thought to be caused by a lack of c. Fx ribs
c. Erythromycin light]. d. Hemothorax
d. Clarithromycin e. Tension pneumothorax
e. Penicillin 1565. A 75yo man presents with ARF. He has been
troubled by recurrent epistaxis but over the last Ans. The key is B. Flial chest. [In flial chest multiple
Ans. The key is A. Amoxicillin. This is a wrong key! 3wks he reports to have coughed up blood too. adjacent ribs are broken in multiple places,
Correct key is D. Clarithromycin. [OHCM, 9th What is the single most likely positive antibody? separating a segment, so a part of the chest wall
edition]. a. P ANCA moves independently. Pain, difficulty breathing,
b. C ANCA tachypnea and tender area of chest wall
c. Anti Ro movement suggest flial chest].
d. Anti DS DNA
1569. A 37yo woman had an elective LSCS 1d ago. b. Flail chest
You are called to see her as she becomes SOB 1573. A woman became acutely SOB in the c. Fx ribs
with left sided chest pain and a cough. She has recovery bay and is coughing after GA. d. Ruptured esophagus
had 3 children, 2 born by LSCS. Exam: she has Auscultation: reduced air entry at the right lung e. Tension pneumothorax
reduced air entry at left lung base. Her base and diffuse wheeze. Observation:
observations include sat=92% on air, HR=88bpm, BP=112/76mmHg, temp=37.8C and Ans. The key is A. Diaphragmatic rupture. [Bowel
BP=105/84mmHg, pulse=120bpm, temp=37.2C. sat=91% in air. Choose among the options which sound present on auscultation of chest following
Choose among the options which C-section C-section complication has she developed? RTA causing lower chest injury is suggestive of
complications has she developed? a. Aspiration pneumonitis diaphragmatic rupture].
a. Aspiration pneumonia b. Spontaneous pneumothorax
b. Aspiration pneumonitis c. Endometritis 1577. A lady presents with a swelling below the
c. Spontaneous pneumothorax d. Pulmonary embolism groin crease that can be reduced. There is no med
d. Pulmonary embolism e. Tension pneumothorax hx of note. What is the most probable dx?
e. DVT a. Inguinal hernia
Ans. The key is A. Aspiration pneumonitis. [Diffuse b. Strangulated hernia
Ans. The key is D. Pulmonary embolism. [Absence wheeze and raised temperature favours the c. Testicular tumor
of wheeze and near normal temperature makes diagnosis of aspiration pneumonitis]. d. Epidydimal cyst
aspiration as unlikely diagnosis and favours the e. Femoral hernia
diagnosis of pulmonary embolism]. 1574. A 23yo female presents with paresthesias
and loss of distal pulses in her arms. She is noted Ans. The key is E. Femoral hernia.
1570. A pt presents with increasing retrosternal to be hypertensive. She describes feeling unwell a
pain and dysphagia for both solids and liquids month prior with fever and night sweats. What is 1578. A 32yo woman of 38wks GA attends the
over 18m but denies weight loss. Chest is clear. the most probable dx? antenatal day unit with pain in the suprapubic
What is the most likely dx? a. Kawasaki disease area that radiates to the upper thighs and
a. Achalasia b. Takayasu arteritis perineum. It is worse on walking. Her urine
b. Pharyngeal carcinoma c. Buerger’s disease dipstick showed a trace of protein but no white
c. Esophageal spasm d. Embolism cells, nitrates or blood. What’s the most likely dx?
d. Esophageal stricture e. Raynaud’s phenomenon a. Braxton hicks contractions
Ans. The key is B. Takayasu arteritis [Takayasu b. Round ligament stretching
Ans. The key is C. Esophageal spasm. This is a arteritis is a granulomatous inflammation of the c. Symphasis pubis dysfunction
wrong key. Correct key should be A. Achalasia. aorta and its major branches. Criteria: d. Labor
[Dyspagia for both solids and liquids or dysphagia Age of 40 years or younger at disease onset e. Complicated femoral hernia
to mostly liquids are features of Achalasia. Claudication of the extremities
Increasing dysphagia is characteristic of achalasia Decreased pulsation of 1 or both brachial arteries Ans. The key is C. Symphysis pubis dysfunction.
(Esophagial spasm does not cause progressive but Difference of at least 10 mm Hg in systolic blood [Symptoms can include:
intermittent dysphagia]. pressure between arms pain over the pubic bone at the front in the centre
Bruit over 1 or both subclavian arteries or the pain across one or both sides of lower back
1571. A 70yo man presents with a fluctuant abdominal aorta pain in the area between vagina and anus
swelling of the scrotum which feels like worms Arteriographic narrowing or occlusion of the (perineum)
when he is standing but regresses when he lies entire aorta, its primary branches, or large Pain can also radiate to thighs.
down. What is the most probable dx? arteries in the upper or lower extremities that is Cause: stiffness of pelvic joints or the joints
a. Varicocele not due to arteriosclerosis, fibromuscular moving unevenly at either the back or front of
b. Hematocele dysplasia, or other causes]. pelvis during pregnancy. Physiotherapy is helpful].
c. Testicular ca [Ref: NHS]
d. Epidydimal cyst 1575. A 35yo woman presents with mass in the
e. Saphena varix groin. Exam: mass found just below and lateral to 1579. A 45yo mechanic presents with a reducible
the pubic tubercle. There is no cough impulse and swelling in the groin, impulse on coughing is
Ans. The key is A. Varicocele. [Fluctuant swelling it is irreducible. What is the most probable dx? present. He has mild dragging pain in the
of the scrotum which feels like worms are a. Direct inguinal hernia abdomen, otherwise he’s normal. What is the
suggestive of varicocele]. b. Strangulated hernia best management strategy?
c. Femoral hernia a. Truss
1572. A 52yo woman has had a swelling in the d. Saphenavarix b. Elective herniorrhaphy
neck, hoarseness and stridor-both inspiratory and e. Femoral aneurysm c. Urgent herniorrhaphy
expiratory for 2m. What is the most probable dx? d. Elective herniotomy
a. Ca larynx Ans. The key is C. Femoral hernia. [Mass below e. Reassure
b. Ca thyroid and lateral to the pubic tubercle is suggestive of
c. Vocal chord nodules femoral hernia]. Ans. The key is D. Elective herniotomy. This is
d. Ca bronchus wrong key! Correct key is B. [Elective
e. Thyrotoxicosis 1576. A 30yo woman has injured her left lower herniorrhaphy. [elective herniotomy is done in
chest in a RTA. She has BP=80/50mmHg, young children preferably at 3-6 months of age.
Ans. The key is B. Ca thyroid. [There isno features pulse=120bpm. Auscultation of chest=bowel Presenting case should be treated by elective
of hyperthyroidism. So it is not thyrotoxicosis but sounds present. What is the single most likely dx? herniorrhaphy. Herniotomy = only resection of
Ca thyroid]. a. Diaphragmatic rupture sac; Herniorrhaphy = resection of sac + repair of
posterior wall of inguinal canal; Hernioplasty = tubercle but it is not absolute and it is possible for Ans. The key is B. Gastric cancer. [Non-tender
resection of sac + posterior wall repair + Mesh femoral hernia to present as mass below and mass in epigastrium, conjunctival pallor (anemia),
reinforcement]. medial to pubic tubercle. So the presented case is weight loss, vomiting of undigested food mixed
a case of femoral hernia]. with blood due to pyloric obstruction by cancer
1580. A 25yo man present with a mass in the mass and particularly Japanese (highest incidence
groin after heavy lifting. Exam: mass is found just 1583. A camel rider sustained a kick to the lateral of gastric cancer due to taking smoked fish) are
above and medial to the pubic tubercle. It is side of his right leg just below the knee caused by almost diagnostic of gastric cancer].
reducible. On applying pressure on the internal the camel stick. The site is slightly bruised and
ring, cough impulse is still present. What is the tender to touch. During physical examination, he 1586. A 45yo man, known to be chronically
most likely dx? is unable to either dorsiflex or evert the foot. addicted to alcohol, presents in the ED and
a. Direct inguinal hernia There is loss of sensation over the front and outer reports two episodes of vomiting fresh bright red
b. Indirect inguinal hernia half of the leg and dorsum of the foot. If these blood in the previous 6h. He estimated the
c. Femoral hernia observations are the result of damage to a nerve volume blood vomited at each bout to be more
d. Strangulated hernia bundle, which is the most likely nerve affected? than 500mls. Clinical exam: the radial
e. Femoral aneurysm a. Lateral popliteal pulse=120/min, BP=90/60mmHg. There is no
b. Peroneal mass or tenderness in the epigastrium. The liver is
Ans. The key is A. Direct inguinalhernia. [On c. Tibia palpable for 3 cm below the costal margin and not
occlusion of deep inguinal ring if cough impulse d. Sural tender. The patient is not jaundiced. The
still palpable (actually more appropriate is visible) physician resuscitates the patient with oxygen by
on medial to occluded ring it is direct inguinal Ans. The key is B. Peroneal. [Inability of dorsiflex face mask, rapid infusion of intravenous normal
hernia]. and foot eversion with sensory loss over front and saline while he requests for haemoglobin level
{Cough impulse negative means after occluding outer half of leg and dorsum of foot are seen in and whole blood for transfusion. Which is next
deep ring there will be no visible cough impulse peroneal nerve injury]. appropriate step in management?
and positive means there will be visible cough a. Barium Swallow
impulse. Negative cough impulse=indirect hernia; 1584. A 46yo woman presents with sudden b. Exploratory laparotomy
positive cough impulse=direct inguinal hernia]. episode of abdominal pain which started about 2h c. CT scan of the abdomen
ago. The pain is located in the epigastrium and d. Upper gastrointestinal endoscopy
1581. A 35yo woman presents with a swelling in radiates to her back. She has vomited twice since Ans. The key is D. Uppergastrointestinal
the neck. The swelling has increased in size the onset of attack. The pain is made worse by endoscopy. [The likely diagnosis is bleeding
gradually over the last two years and the patient lying flat on her back and she is more comfortable oesophageal varices which should be diagnosed
feels she has difficulty with breathing. Exam: mass sitting up and bending forwards. She was by endoscopy and if needed stappling can be
measures 8cm by 10 cm, soft and not warm to informed of the presence of gallstones in her gall done with endoscope].
touch. It moves with deglutition. Which is the bladder four weeks earlier when she reported
most appropriate management of this mass? pain in the right hypochondrium. The oral 1587. A 42yo woman reports to the surgeon that
a. Partial thyroidectomy temp=39C, BP=120/80mmHg and the radial she is worried about a lump that she feels the
b. Oral thyroxine pulse=118/min. There is no jaundice but there is right breast. The surgeon observes a 2 cm by 3 cm
c. Oral propylthiouracil marked tenderness in the epigastrium both on mass in the right lower quadrant of the breast.
d. Excision biopsy deep and superficial palpations. Which is the most There are no associated skin changes and the
appropriate inv for the cause of the patient’s mass has limited mobility. There is no discharge
Ans. The key is A. Partial thyroidectomy. pain? from the nipple. There is no axillary lymph node
[Gradually increased swelling in the neck which a. Plain abdominal X-ray enlargement. Examination of the left breast and
moves with deglutition is thyroid enlargement b. Serum Amylase axilla was completely normal. A mammogram
and as it is causing pressure symptom like c. Serum bilirubin report suggests the presence of
difficulty in breathing a partial thyroidectomy d. Barium Swallow microcalcifications. Which is the most appropriate
should be performed]. next step in the management of this pt?
Ans. The key is B. Serum amylase. [Epigastric pain a. Observation for one year and repeat the
1582. A 46yo laborer reports swelling in the right radiating to back, worse on lying flat and comfort mammography
groin. The non-painful swelling is observable in on bending forward are classic presentation of b. A needle-guided biopsy of the breast
both the erect and the recumbent positions. acute pancreatitis in which serum amylase is c. Excision biopsy of the breast
Exam: non-tender irreducible 4 cm mass in the increased]. d. Partial mastectomy
right groin below and on the medial side of the
inguinal ligament. Which is the most likely dx in 1585. A 75yo Japanese woman reports repeated Ans. The key is B. A needle-guided biopsy of the
this pt? episodes of vomiting of undigested food mixed breast.
a. Indirect inguinal hernia with blood. She has lost 5 kgs in weight over the
b. Femoral hernia last one month. Clinical exam: shows a frail 1588. A 45yo man presents with a mass on the
c. Saphenous vein varicocoele woman with mild conjuctival palor. Exam: non- right side of the face. The mass was first observed
d. Hydrocoele tender slightly mobile mass in the epigastric three months ago but has recently become visibly
Ans. The key is C. Saphenous vein varicocele. This region. Which is the most likely dx? larger. He feels pain over the mass and is unable
is a wrong key! Correct key is B. Femoral hernia. a. Colon cancer to blow a whistle. Clinical examination shows that
[Sephanous vein varicocele is readily reducible b. Gastric cancer the mass is likely to be the parotid gland. An oral
with recumbent position and reducible. So c. Gall bladder cancer examination shows a foul smelling discharge from
presented case is not sephanavarix. Usual location d. Oesophageal cancer the duct of the gland and gentle probing shows
of femoral hernia is below and lateral to pubic that it is stenosed at the meatus. Which of the
following features suggests that the mass might As he gets older, the child may use his hands to He wishes that his father does not suffer any
be malignant? help him get up, looking as if he is 'climbing up his psychological distress caused by the knowledge of
a. Presence of pain legs'. This is called 'Gower's sign']. a terminal diagnosis. Which one of the following
b. Recent enlargement ethical principles supports the son’s request?
c. Facial nerve palsy 1592. A previously healthy, 10m female child a. Patient autonomy
d. Stenosed duct meatus presents to your clinic with a 1-day history of high b. Beneficence
fever, runny nose and conjunctivitis. The child c. Justice
Ans. The key is C. Facial nerve palsy. [Due to looks unwell and is irritable. Exam: child's d. Non-maleficence
malignant infiltration]. oropharynx shows that it is inflammed and there
are small white spots on the oral mucosa. Which Ans. The key is D. Non-maleficence.
1589. A 6yo boy presents with jaundice following is the most likely dx? [Nonmaleficence means non-harming or inflicting
treatment with sulphathiazole. Investigations a. Kawasaki disease the least harm possible to reach a beneficial
suggest that the jaundice is due to haemolysis b. Parvovirus infection outcome. Harm and its effects are considerations
caused by G6DP deficiency. Which is true c. Herpes zoster and part of the ethical decision-making process in
regarding etiology of G6DP deficiency? d. Measles the NICU. Short-term and long-term harm, though
a. Inherited as autosomal dominant condition unintentional, often accompany life-saving
b. Inherited as sex-linked dominant condition Ans. The key is D. Measles. [Koplick’s spots are treatment in the NICU.
c. Inherited as sex-linked recessive condition characteristic of measles].
d. Results from auto-antibodies to red cell 1596. A 23yo single male was brought to
antigens 1593. A 3d term, breast-fed infant is brought by Emergency exhausted and frightened. His father
the mother who reports that the child has not tells you that his son, who was previously healthy,
Ans. The key is C. Inherited as sex-linked recessive been active and not feeding well. She also notices had, for no apparent reason, a sudden attack of
condition. jaundice, which was not present at birth and is fear, dizziness, sweating, palpitations and the
increasing. Exam: the temp=35.4°C, and the liver feeling that his heart is going to stop beating. The
1590. A 5yo previously healthy child has a 1-day is palpable 2 cm below the costal margin. Which is symptoms started to decrease gradually after
history of severe pain in the throat, breathing the most likely dx? about 10 minutes. Which is the most likely dx?
difficulties and fever. On examination you find an a. Rhesus isoimmunisation a. Panic attack
anxious, septic-looking child with drooling of b. Inadequate breast milk b. Delirious state
saliva and stridor. Which is the most appropriate c. Congenital biliary tract obstruction. c. Alcohol withdrawal phenomena
initial management? d. Sepsis d. Social phobia
a. Intubation under general anaesthesia
b. Insertion of nasogastric tube Ans. The key is D. Sepsis. [Not active, not feeding Ans. The key is A. Panic attack. [a sudden feeling
c. Fluid resuscitation and antibiotics IV well, increasing new onset jaundice and of acute and disabling anxiety; often fear of
d. Anteroposterior & lateral neck x-ray hypothermia are suggestive of neonatal sepsis]. death].

Ans. The key is A. Intubation under general 1594. A 65yo woman with DM, HTN and normal 1597. A 30yo woman, G2P1, at 37 weeks
anesthesia. [Acute epiglottitis. Should intubate to kidney function underwent a total right hip gestation mentions that her 3-year-old son has
save from closure of airway]. replacement. She had massive haemorrhage just developed chickenpox. She is not certain
during the operation and was given 8 units of whether she has had the disease herself. Which is
***1591. A 6yo boy has been noticed to have packed RBC. The blood pressure dropped to 60/40 the next step in management?
problems with co-ordinating his voluntary mm Hg for about two hours before it was a. Administration of varicella-zoster immune
movements over the last two years. He has a corrected with blood transfusion. Two days after globulin IM
waddling gait and needs to support himself on his the surgery the serum creatinine level rose to 4.2 b. Measurement of varicella IgM level
hands when rising from the floor. He has larger mg/dl (normal <1.5 mg/dl), BUN was 50 mg/dl c. Acyclovir tablets orally
calves than other boys but he runs more slowly. (normal 10-20 mg/dl) and potassium 5.0 mmol/L d. Measurement of varicella IgG level
Which is the most likely dx? (normal 3.5-5.0 mmol/l). There were brown Ans. The key is D. Measurement of varicella IgG
a. Myotonia granular casts in the urine sediment. Which is the level. [If previous infection is doubtful do varicella
b. Myasthenia gravis most likely cause of this complication? IgG level].
c. Duchenne muscular dystrophy a. Diabetic nephropathy
d. Muscular atrophy b. Malignant hypertension 1598. A 24yo primigravida presents to the ED with
c. Acute tubular necrosis a history of 8-week amenorrhoea followed by
Ans. The key is C. Duchenne muscular dystrophy. d. Interstitial nephritis heavy vaginal bleeding and severe, crampy
[The child having difficulty with walking, running, abdominal pain. Exam: HR=110/min and
jumping and climbing stairs. Walking may look Ans. The key is C. Acute tubular necrosis. BP=120/80mmHg. The uterus is bulky. The cervix
different with a 'waddling' type of walk. The boy [Hypotension even for some minutes or few hours is dilated and there is active bleeding from the
may be late in starting to walk (although many can readily lead to acute tubular necrosis which is cervical os, but no tissue has been expelled.
children without DMD also walk late). evident here by uremia and further supported by Which of the following is the most likely dx?
When you pick the child up, you may feel as if he brown granular cast in the urine sediment]. a. Inevitable abortion
'slips through your hands', due to looseness of the b. Threatened Abortion
muscles around the shoulder. 1995. A 78yo pt is diagnosed with metastatic lung c. Incomplete abortion
The calf muscles may look bulky, although they cancer; there is no cure for his condition. His son d. Missed Abortion
are not strong. tells the physician that in the case of a diagnosis
of cancer, the physician must not tell his father.
Ans. The key is A. Inevitable abortion. [when os is a. Cardiac tamponade Ans. The key is E. Acute limb ischemia. [AF may be
closed threatened and when os is opened b. Diaphragmatic rupture the cause of thrombus leading to embolic acute
inevitable abortion (No tissue has been expelled)]. c. Fx ribs limb ischemia].
d. Tension pneumothorax
1599. A 46yo woman comes for a routine e. Traumatic rupture of aorta 1606. A 50yo woman complains of several months
gynaecological visit. On pelvic examination, a 1- hx of weakness and difficulty climbing stairs.
cm red, granular lesion is noted on the posterior Ans. The key is E. Traumatic rupture of the aorta. Exam: fissuring of the skin of her hands. CXR:
cervical lip, which is firm and bleeds on contact. [A traumatic aortic disruption is caused by a rapid pulmonary fibrosis. What is the single most likely
Which is the next best step for establishing a dx? acceleration (or deceleration) causing a tear in the positive antibody?
a. Cervical cytological smear aorta. Normally this is immediately fatal, but a. Anti Jo1
b. Punch biopsy those who survive may show a widened b. Anti Scl 70
c. Transvaginal ultrasound mediastinum on CXR. This can be confirmed with c. Anti Ro
d. Colposcopy CT scan or angiography of the aorta and requires d. Anti dsDNA
prompt surgical correction. Stable the e. Anti centromere
Ans. The key is B. Punch biopsy. haemodynamics and surgical correction. Note: pts Ans. A. Anti Jo1. [Anticentromere would be
with diaphragmatic rupture usually presents days present in limited scleroderma and Anti-Scl 70
1600. A 31yo woman, G5P4, who has after trauma with vague symptoms, history is would be present in diffuse scleroderma. Anti-
amenorrhoea for 12 weeks and a positive vital]. DSDNA would be positive in SLE, along with Anti-
pregnancy test presents to the ED with vaginal Ro. Anti-Ro is also positive in Sjogren’s syndrome
bleeding. Symphysial-fundal height measurement 1603. A 36yo woman presents with swelling in the and scleroderma. This is a case of Polymyositis
corresponds to 22 weeks gestation. Ultrasound groin. Exam: swelling is diffuse and soft and lies because none of the others would have the
examination reveals bilateral cystic masses. No below the inguinal ligament. It empties with muscular weakness that is present in this case.
fetal parts are seen during the examination. The minimal pressure and refills with release. There is The antibody of choice would be Anti Jo1
cervix is closed. Which is the most likely dx? a cough impulse and it disappears on lying down. antibody].
a. Tubal pregnancy On the calf of the same leg there are varicosities
b. Endometriosis on the medial aspect. What is the most likely dx? 1607. A 65yo woman complaining of symptoms
c. Hydatidiform mole a. Varicose vein suggestive of Raynaud’s phenomenon and
d. Threatened abortion b. Varicocele difficulty in swallowing. Exam: painful lesions on
c. Saphena varix her finger tips and facial telangiectasis. What is
Ans. The key is C. Hydatidiform mole. [In molar d. Femoral hernia the single most likely positive antibody?
pregnancy uterus is more enlarged than e. Inguinal hernia a. Anti Jo1
gestational age and on US no fetal part but cystic Ans. The key is C. Saphena Varix. [Below inguinal b. Anti Scl 70
masses are seen]. ligament, empties with minimal pressure and c. Anti Ro
refills with release, presence of cough impulse d. Anti ds DNA
The final set (1601-1700) is done by DR. ARIF and disappearance on lying down are suggestive e. Anti centromere
SIDDIQUI and EDITED by me. Thanks for Dr. of caphena varix supported by presence of Key: Anti-Centromere (E)
Siddiqui for his kind contribution! varicosity in same leg]. Reason: Anti-centromere antibody would be
present in CREST syndrome which is also called
1601. A married 25yo woman presents with 6h hx 1604. A man presents with a swelling above the Limited scleroderma. Features of given case are
of abdominal pain located in the LIF. The pain is groin crease in the abdomen. He has not had any consistent with CREST syndrome and hence the
persistent, of increasing intensity and not med problems of note. What is the most probable answer is E. Anti centromere antibody].
radiating first experienced while she was lying dx?
down. She feels giddy when she tries to stand a. Inguinal hernia 1608. A 6yo boy presented about 4h ago with
erect. The last menstrual period was 6 weeks ago. b. Spigelian hernia acute severe pain on the testis with the left half
The radial pulse=130/min and BP=80/40 mmHg. c. Testicular tumor slightly higher than the right. Pain was not
Pelvic US shows free intra-peritoneal fluid. What d. Epidydimal cyst relieved by any strong analgesic. What is the
is the most appropriate next step in e. Irreducible hernia initial
management? management?
a. Immediate laparoscopy. Ans. B. Spigelian Hernia. [A Spigelian hernia (or a. Give strong analgesic
b. Immediate laparotomy. lateral ventral hernia) is a hernia through the b. IV NS and monitor vital signs
c. Pregnancy test (urine or serum). spigelian fascia, which is the aponeurotic layer c. Reassure
d. Observation for 24 hours in the ICU between the rectus abdominis muscle medially, d. Immediate surgical referral
and the semilunar line laterally. e. Cover with antibiotics
Ans. The key is B. Immediate laparotomy. [As the
patient is in shock it is ruptured ectopic 1605. A 70yo man presents with acutely painful, Ans. The key is D. Immediate surgical referral. [Dx
pregnancy. So the next step is immediate pale paralysed and pulseless left leg. He is noted torsion of testis].
laparotomy]. to have a-fib. What is the most probable dx?
a. Intermittent claudication 1609. A 60yo man is brought to the ED in an
1602. A 40yo man has fallen off a roof. He is b. Cardiovascular syphilis agitated state. He is lashing out violently. Which
shocked and has chest pain. There is a delay c. Buerger’s disease drug in low dosage due to its relative lack of
between the radial and femoral pulse. His d. Chronic limb ischemia autonomic side effects is a drug of choice in the tx
CXR=widening of the mediastinum. What is the e. Acute limb ischemia of agitation in this pt?
single most likely dx? a. Haloperidol
b. Diazepam Ans. The key is A. Lithium. [Lithium causes tremor, or that one of their organs has stopped
c. Fluoxetine GI distress (vomiting) along with Diabetes functioning].
d. Clozapine insipidus].
e. Chlorpromazine 1617. A 38yo man has just returned from a
1614. A 24yo man presents with painless holiday where he went swimming everyday. For
Ans. The key is A. Haloperidol. It is a wrong key. hematuria. No other complaint and no the last few days he has had irritation in both
Correct key is Diazepam. [Haloperidol has abnormality is found on physical exam. What is ears. Now his right ear is hot, red, swollen and
autonomic side effects]. the most appropriate initial inv which is helpful to acutely
get a dx? painful. What is the single most likely dx?
1610. A 32yo woman of 40wks gestation attends a. Coag screening a. Foreign body
the antenatal day unit with sudden onset b. MSU b. Impacted earwax
epigastric pain with nausea and vomiting. She is c. Cystoscopy c. OE
clinically jaundiced. Her biochemistry results show d. MRI spine d. OM
a raised bilirubin, abnormal liver enzymes, high e. Abdominal US e. Perforation of eardrum
uric acid and hypoglycemia. What’s the most
likely dx? Ans. The key is E. Abdominal US. [Painless Ans. C. Otitis Externa. [The swimming history,
a. Acute fatty liver of pregnancy hematuria in a young male without any other irritation in both ears and ear being hot, red,
b. Obstetric cholestasis findings on history or examination often suggests swollen and painful indicates inflammation of the
c. Cholecystitis Polycystic Kidney Disease]. external acoustic meatus called Otitis Externa. It
d. HELLP syndrome isn’t otitis media because of the lack of Tympanic
e. Acute hepatitis 1615. A 29yo woman presents to her GP with membrane signs].
troublesome heavy periods. The med tx that she
Ans. The key is A. Acute Fatty Liver of Pregnancy. has tried have made little difference. She is known 1618. A healthy 2yo boy is brought to the ED
[When jaundice is present in pregnancy, AFLP to have large uterine intramural fibroids. You having cut his hand playing in the garden. He has
should be high on the differential. Pain, nausea, confirm that she is currently trying for more a 2cm clean laceration. He has not received any
vomiting, jaundice, fever with elevated liver children. Select the most appropriate routine immunizations as his parents are
enzymes and bilirubin is clinically indicative of management concerned about possible side effects. There are
AFLP. Also can have elevated INR, TLC and for menorrhagia in this pt? no contraindications to immunizations. What is
hypoglycaemia. a. Danazol the single most appropriate follow up inv?
b. Endometrial ablation a. Courses of DPT vaccine
1611. A 24yo man believes his bowels are blocked c. Hysterectomy b. Courses of DT
and his life is in ruin. What kind of delusion is he d. Hysteroscopic resection of fibroids c. Single inj of DPT vaccine
suffering from? e. Myomectomy d. Single inj of DT
a. Persecutory e. Single inj of tetanus Ig
b. Factitious Ans. E. Myomectomy. [Chance of subsequent Ans. Courses of DPT Vaccine. [The child is
c. Guilt pregnancies is better after myomectomy]. unimmunized and has no contraindication to
d. Nihilistic vaccination. Keeping in mind his age, single
e. Hypochondriacal 1616. A 30yo schizophrenic female attacks her injections would be useless and would not help
mother believing that aliens have replaced her the patient. Courses of DPT vaccine would be the
Ans. The key is D. Nihilistic. [The man believes his with an exact double. What condition is she best choice in this case].
bowels are blocked and his life is ruined is an suffering from?
example of nihilistic delusion]. a. Capgras syndrome 1619. A 6wk child has hx of frequent vomiting
b. Ganser syndrome which became worse during the last weeks. He
1612. A 75yo man with declining vision, cornea c. Todd syndrome has no fever, recently he has passed stool only
and pupils are normal, fundus shows obscured d. Fregoli syndrome once every 2-3d. What inv will you do to confirm
margins. What is the single most likely dx? e. Cotard syndrome the dx?
a. Macular degeneration a. Abdominal US
b. HTN retinopathy Ans. Capgras Syndrome. [Capgras syndrome is an b. Barium meal
c. MS irrational belief that a familiar person or place has c. Erect XR abdomen
d. DM background been replaced by a duplicate. d. Feed test
e. Proliferative DM retinopathy Ganser syndrome is a fictitious disorder in which a e. Reassure
Ans. The key is A. Macular degeneration. [In a patient deliberately acts as if he has a physical or Ans. The key is A. Abdominal US. [The age and
75yrs old man with normal cornea and pupils and mental illness when he doesn’t have it. symptoms points towards pyloric stenosis].
papilloedema suggests macular degeneration]. Todd syndrome/Alice In Wonderland
syndrome/Lilliputian syndrome is a disorienting 1620. A 30yo woman had an IUCD inserted 8-9m
1613. A man under psychiatric tx develops GI neurological condition affecting human ago. Now on routine follow up the thread is
distress and tremors. Which drug is most likely to perception of size, shape and time. missing. Uterine US showed no IUCD in the
cause these symptoms? Fregoli syndrome is a delusion of doubles, a uterus. What is the best management?
a. Lithium delusional belief that different ppl are infact a a. Laparoscopy
b. Diazepam single person in disguise or change appearance. b. Pelvic CT
c. Citalopram Cotard’s syndrome/Nihilistic delusions is ‘walking c. Laparotomy
d. Clozapine corpse syndrome’, the person think they are dead d. Pelvic XR
e. Imipramine
Ans. The key is D. Pelvic XR. [Ultrasound should be had a similar episode 1y ago and has had episodic cells were seen on the blood film. She was
arranged to locate the device. If ultrasound does back pain and stiffness relieved by exercise and transfused and given IV antibiotics and her
not locate the device and there is no definite diclofenac for four years. What is the SINGLE most condition
history of expulsion then abdominal X-ray should likely cause of his red eye? improved. 3wks later her blood count has
be performed to look for an extrauterine device]. a. Chorioretinitis returned to a similar picture. What is the SINGLE
b. Conjunctivitis most
1621. A pt comes with weight loss and sleep c. Episcleritis likely underlying dx?
disturbance has mild depression. He has a hx of d. Iritis a. ALL
MI. What is the single most appropriate tx? e. Keratitis b. AML
a. Diazepam Ans. The key is D. Iritis. [The symptoms described c. Aplastic anemia
b. ECT are characteristic of ankylosing spondylitis (lower d. CML
c. Imipramine back pain and stiffness which gets better after e. Pernicious anemia
d. Lithium moving around and taking NSAIDS) the extra
e. Antipsychotics articular manifestations of AS is iritis]. Ans. The key is C. Aplastic Anaemia. [The age of
Key: Diazepam the patient and pancytopenic picture give us a
1626. A 40yo divorced man with bipolar affective clinical diagnosis of Aplastic anemia. Normal WBC
1622. A pt. comes back from India and presents disorder attends hospital following an OD of 30 morphology rules out ALL, AML and pernicious
with night sweats and lymphadenopathy. XR: TCA tablets. His new partner has left him and he anaemia while the age rules out CML as a
Cavitation. What investigation should be done has stopped taking his medicine and begun diagnosis].
next? drinking heavily. He appears depressed, feels
a. CT scan hopeless and is ambivalent about being alive. He 1629. An 83yo woman admitted with a chest
b. AFB stain is now fit for discharge from the medical ward and infection becomes confused with impaired
c. Blood culture acknowledges the benefits of previous tx. What is attention and poor concentration. She is restless
d. Bronchoscopy the SINGLE most appropriate next management? and frightened. She is verbally abusive and has
Ans. AFB Stain [The symptoms and arrival from an a. Admission to the psychiatry ward perceptual
endemic area for Pulmonary TB suggests the best b. Arrange psychiatric outpatient follow-up abnormalities. There is no significant prv
course of action would be to go for AFB staining c. Discharge to the care of the general practitioner psychiatric hx. What is the SINGLE most likely dx?
via ZN stain]. d. Referral to local alcohol treatment team a. Delirium
e. Referral to clinical psychologist b. Drug induced psychosis
1623. A 45yo woman has been extensively Ans. The key is A. Admission to the psychiatry c. Lewy body dementia
investigated for a lump she believes to be cancer. ward. [To save the patient from another d. Multi-infarct dementia
She doesn’t think doctors take her seriously and overdose]. e. Psychotic depression
demands another referral. What term best
describes her condition? 1627. A healthy baby boy is born at term to a Ans. The key is A. Delirium. [Delirium or Acute
a. Munchausen syndrome woman who was unwell with confirmed acute hep Confusional States happen in the elderly in
b. Munchausen’s by proxy B response to stressors like acute infections and this
c. Hypochondriasis during pregnancy. The mother is very concerned is most likely brought on by the chest infection
d. Malingering that she may have infected the baby with hep that has developed].
e. Phobia B. What SINGLE preventative intervention should
Ans. The key is C. Hypochondriasis. [Worry about be given to the baby? 1630. A town has a population of 500,000. In a
having a serious illness. This debilitating condition a. Full course of hepatitis B vaccine five year period there are 1250 cases of bladder
is the result of an inaccurate perception of the b. Hepatitis B immunoglobulin alone cancer diagnosed at the only hospital. During the
condition of body or mind despite the absence of c. Hepatitis B vaccine and hepatitis B same period the occupational health department
an actual medical condition]. immunoglobulin diagnosed a further 500 cases. What is the annual
d. Hepatitis B vaccine as single dose incidence per million of bladder cancer in this
1624. A 15yo man presents with bitemporal e. None until hepatitis B status confirmed population?
hemianopia and spade-like hands. What is the Ans. The key is C. Hepatitis B vaccine and hepatitis a. 2100
definite test to confirm the dx? B immunoglobulin. [Babies born to mothers b. 1750
a. Early morning growth hormone infected with hepatitis B have a high risk of c. 1400
b. Insulin tolerance test acquiring infection, which can be prevented by d. 700
c. OGTT with growth hormone measurements vaccination at birth.All babies with seropositive e. 350
d. Random insulin-like growth factor (IGF-1) mothers should have the full primary course of
e. Short ACTH test hepatitis B immunisation and should also have Ans. The key is D. 700.
Ans. The key is C. OGTT with growth hormone HBIG within 24 hours of birth].
measurements. [The bitemporal hemianopia and 1631. A 28yo woman who has had a prv
spade-like hands point towards acromegaly. The pulmonary embolism in pregnancy wishes to
best initial test is insulin like growth factors but 1628. A previously well 15yo girl had an acute discuss
the definitive test that confirms the diagnosis is onset of fever, sweating, bruising and petechiae. contraception. She has menorrhagia but is
OGTT with serial growth hormone A otherwise well. What is the SINGLE most suitable
measurements]. blood count showed: Hgb=63g/L, WBC=1.1mg/L, contraceptive method for this patient?
Neutrophils=0.1, plt=14. No abnormal white a. COCP
1625. A 22yo man has had an acute, painful, red b. Copper IUCD
right eye with blurring of vision for one day. He c. Levonorgestrel intra-uterine system
d. Progestogen implant There is no history of relevant travel outside the b. Leukaemia
e. POP UK. She has two young children. What is the c. Lymphoma
SINGLE most likely dx? d. Sarcoidosis
Ans. The key is C. Levonorgestrel Intra-Uterine a. Psoriasis e. Tuberculosis
System. [The woman has a history of b. Reactive arthritis Ans. The key is A. Infectious Mononucleosis.
thromboembolic disease, which essentially c. Rheumatoid arthritis [Though in infectious mononucleosis lymph nodes
contraindicates COCP. LNG-IUS (Mirena) is the d. Sarcoidosis are usually seen in posterior triangle but can be
hormone releasing device that is most suitable in e. SLE seen in whole body including anterior triangle
this patient with thrombophilia and menorrhagia. Ans. The key is B. Reactive arthritis. It is a wrong also].
key! Correcct key should be E. SLE.
1632. An 8yo girl has had left earache for 2d. The 1637. A 60yo man has had increasing pain in both
earache subsided about 2h ago with the onset of buttocks, thighs and calves on walking for three
a purulent discharge which relieved the pain. Her 1636. A 16yo girl has had an enlarging mass in the months. He has also recently developed
temperature is 39.2C. What is the SINGLE most right side of her neck for the last 6wks. She has impotence. Femoral and distal pulses are absent
appropriate antibiotic? had in both limbs. What is the SINGLE most likely site
a. Amoxicillin no other symptoms. She has a 2 x 2 cm enlarged of arterial obstruction?
b. Ciprofloxacin LN in the anterior triangle of the neck with a. Aorto iliac
c. Clindamycin several smaller associated LN palpable. b. External iliac
d. Erythromycin Oropharyngeal examination shows tonsillar c. Femoropopliteal
e. Flucloxacillin membranes. d. Internal iliac
What is the SINGLE most likely dx? e. Tibial
Ans. The key is A. Amoxicillin. [This is the picture a. Infectious mononucleosis
of Acute Otitis Media which has led to tympanic b. Leukaemia Ans. The key is A. Aorto iliac. [saddle embolus;
membrane perforation. PO Amoxicillin for 7 days c. Lymphoma surgical intervention].
is the treatment of choice]. d. Sarcoidosis Page 718 OHCM 9th edition.
e. Tuberculosis Patient is suufering from leriche’s syndrome.
1633. A 38yo man has disturbing thoughts about Classic triad of:
his house being infected by germs. He is anxious • Pain and claudication of buttock and thighs
about safety and checks the locks of his doors Ans. The key is A. Infectious mononucleosis. (pale cold leg)
repeatedly before going to bed. For the last 8wks [Though in infectious mononucleosis lymph nodes • Erectile dysfunction from aorto iliac
he has been washing his hands every time he are usually seen in posterior triangle but can be occlusive disease
touches the lock, 20-30 times a day. What is the seen in whole body including anterior triangle • Absent femoral and distal pulse.
SINGLE most appropriate management? also].
a. Antidepressant
b. Antipsychotic 1638. A 78yo man has collapsed. He has had a
c. Anxiolytic 1635. A 32yo woman has had a febrile illness and severe headache for 12 hours and had an URTI 3d
d. CBT swelling of the small joints of her hands, feet, ago. He has a temp=39.2C, pulse=122bpm,
e. Psychodynamic psychotherapy wrists BP=84/60mmHg and RR=34bpm but his chest is
and knees for two days. She has a maculopapular clear. He has a GCS=10 and some neck stiffness.
Ans. The key is D. CBT rash and a few palpable, small cervical lymph He has been started on high-flow oxygen. What is
Reason: This scenario describes a case of OCD for Nodes. She was previously well. There is no the SINGLE most appropriate immediate
which the best management is CBT followed by history of relevant travel outside the UK. She has management?
SSRIs or TCAs]. two young children. What is the SINGLE most a. IV antibiotic; CT brain scan
likely dx? b. IV antibiotic; LP
1634. A 65yo man had closure of colostomy a. Psoriasis c. IV fluids; CT brain scan
performed 5d ago. He is not systemically unwell. b. Reactive arthritis d. IV fluids; IV antibiotic
There is c. Rheumatoid arthritis e. IV fluids; LP
a tender, localised fluctuant swelling 4 cm in d. Sarcoidosis
diameter in the wound. What is the SINGLE most e. SLE Ans. The key is D. IV Fluids; IV antibiotic [As his BP
appropriate management? Ans. The key is B. Reactive Arthritis. [The key is is low and has presented with neck stiffness so, IV
a. Abdominal support GMC key]. fluids and V antibiotic should be given, after giving
b. Antibiotics him oxygen].
c. Laparotomy and re-suture wound
d. Local exploration of wound 1636. A 16yo girl has had an enlarging mass in the
e. Observation right side of her neck for the last 6wks. She has 1639. A 16yo boy was brought to hospital in a
had comatose state having taken methadone
Ans. The key is D. Local exploration of wound. no other symptoms. She has a 2 x 2 cm enlarged belonging to his sister. He was given naloxone and
LN in the anterior triangle of the neck with rapidly became alert. Some hours later, he
1635. A 32yo woman has had a febrile illness and several smaller associated LN palpable. gradually becomes semi-conscious again.What is
swelling of the small joints of her hands, feet, Oropharyngeal examination shows tonsillar the SINGLE most likely reason for this patient
wrists and knees for two days. She has a membranes. becoming semi-conscious again in hospital?
maculopapular rash and a few palpable, small What is the SINGLE most likely dx? a. Methadone hepatotoxicity has caused acute
cervical lymph nodes. She was previously well. a. Infectious mononucleosis liver failure
b. Methadone is eliminated from the body more • Most common antibiotic given for 1645. A 32yo woman has had 3 episodes of
slowly than naloxone UNCOMPLICATED STDs is azithromycin and it slurred speech and 2 episodes of transient
c. Naloxone is a partial agonist at the central covers gram negative bacteria which are neisseria weakness of both legs in the past 5yrs. Each
nervous system opioid receptor and chlamydia. episode has resolved in 3m. What is the SINGLE
d. The pt has misused another substance that has • Candida albicans is a fungal infection most likely dx?
caused an intracranial bleed • Gardnerella is a gram variable bacteria and a. Meningioma
e. The pt has misused another substance that is normal commensal. b. Migraine
absorbed more slowly than methadone c. Multiple sclerosis
Ans. The key is B. Methadone is eliminated from 1643. A 48yo man with renal cancer had d. Stroke
the body more slowly than naloxone (short t1/2) radiotherapy for metastatic spinal cord e. Transient ischaemic attack
compression at the 11th thoracic vertebra 4wks
1640. A 27yo woman who takes the COCP has had ago. He has retained sensation but is unable to Ans. The key is C. Multiple sclerosis.
painless vaginal spotting and discharge for 3 days. stand. He has pain in a band around his lower
Her last menstrual period, which lasted four days, trunk controlled by regular oral morphine. He is 1646. An 8yo girl is complying with her asthma
finished 10 days ago. Her last cervical smear two distressed by treatment of low-dose inhaled corticosteroid
years ago was normal. Abdominal and vaginal increasingly frequent episodes of painful muscle prophylaxis and short-acting bronchodilators as
examinations are normal apart from a mild spasms in his right leg. What is the SINGLE most required. Her inhaler technique is good. She
ectropion with contact bleeding. What is the appropriate management of his symptoms? now has a frequent night cough and mild exercise-
SINGLE most appropriate initial inv? a. Amitriptyline induced wheeze. What would be the SINGLE
a. Cervical smear b. Baclofen most appropriate change in her treatment?
b. Colposcopy c. Fentanyl patch a. Add leukotriene antagonist
c. Endocervical swab d. Gabapentin b. Add oral theophylline
d. Endometrial biopsy e. Increase morphine dose c. Add regular long-acting bronchodilator
e. Pelvic US d. Increase dose of inhaled corticosteroid
Ans. The key is B: Baclofen.[Muscle relaxant]. e. Short course of oral corticosteroid
Ans. The key is C. Endocervical swab. [As her
cervical smear and examination of abdomen and 1644. A 4yo girl has had a temp=38.5C for 2days Ans. The key is D. Increase dose of inhaled
vagina are normal, next would be to exclude a and has not wanted to eat her food. Yesterday corticosteroid. This is a wrong key. Correct key is
STD for which Endocervical swab is taken]. she developed a sore throat and small, painful C. Add regular long-acting bronchodilator.
ulcers inside her mouth. Today she has small
1641. A 72yo man being investigated for anaemia blisters on the palms of her hands and soles of her 1647. A 38yo man with longstanding alcohol
is booked for a colonoscopy in 24 hours. What is feet which are painful but not itchy. What is the dependence has vertigo and a tremor every
the SINGLE most appropriate management the SINGLE most likely underlying cause? morning. What is the SINGLE most likely dx?
night before the procedure? a. Coxsackie virus a. Anxiety
b. Herpes simplex virus b. Benign positional vertigo
a. Bisacodyl tablets c. Staphylococcus aureus c. Cerebellar degeneration
b. Glycerine suppository d. Streptococcus pneumonia d. Optic neuritis
c. Lactulose syrup e. Varicella zoster virus e. Temporal lobe epilepsy
d. Magnesium citrate (orally)
e. Senna tablets Ans. The key is A: coxsakie virus Ans. The key is C. Cerebellar degeneration.
Patient is suffering from HFMD (Hand,Foot and [chronic alcohol abuse that leads to temporary or
Ans. The key is D. Magnesium Citrate (Orally) Mouth Disease) permanent cerebellar damage].
• HFMD is due to an infection that usually
1642. A 19yo woman has had progressive bilateral causes a typical illness, including a typical rash. It 1648. An 84yo woman with Alzheimer's dementia
iliac fossa pain and dyspareunia for 3days. She has is most commonly caused by the Coxsackie A16 has recently become incontinent and more
an offensive vaginal discharge and feels unwell virus confused than usual. What is the SINGLE most
and feverish. Her temp=39C. An initial • HFMD most commonly affects children likely dx?
antimicrobial regimen is commenced. What under 10 years of age a. Detrusor overactivity
SINGLE set of organisms are the most appropriate • This might include a high temperature b. Neuropathic bladder
for the antimicrobial regimen to cover? (fever). After this, a sore throat commonly occurs, c. Nocturnal enuresis
a. Neisseria gonorrhoeae and Candida albicans quickly followed by small spots that develop d. UTI
b. Neisseria gonorrhoeae and Candida albicans inside the mouth. These soon progress into small e. Uterine prolapse
and Gardnerellavaginalia mouth ulcers
c. Neisseria gonorrhoeae and Chlamydia • In many cases, spots also develop on the Ans. The key is D. UTI. [In UTI there may be
trachomatis skin. This is typically a day or so after the mouth incontinence of urine and confusion].
d. Neisseria gonorrhoeae and Chlamydia ulcers develop. The spots are small lumps that are
trachomatis and Candida albicans a few millimetres in diameter and usually appear 1649. A 4yo boy complains of pain around his
e. Neisseria gonorrhoeae and Chlamydia on the hands and feet, they are not usually itchy right eye. He is unwell, febrile and also suffers
trachomatis and Gardnerellavaginalis but sometimes they can be a little bit sore. from pain on the right side of his face. What is the
Treatment: most probable dx?
Ans. The key is C. Neisseria gonorrhoeae and • There is no treatment that will take away a. Allergic reaction
Chlamydia trachomatis the virus b. Furuncle
c. Folliculitis
d. Foreign body b. Failure to suppress morning cortisol with
e. Periorbital cellulitis 1653. A pt has loss of sensation on the tip of her dexamethasone
tongue and the inner aspect of the lip. Which c. HTN requiring >2 antihypertensive agents
Ans. The key is E. Periorbital cellulitis. nerve is d. Impaired growth hormone response to glucose
most likely to be involved? loading
1650. A pt presents with irregularly irregular pulse a. Vagus nerve e. Unilateral adrenal enlargement
of 162bpm. What drug is most useful initially? b. Glossopharyngeal nerve Key is E: Unilateral adrenal enlargement
a. Amiodarone c. Lingual nerve Cushing’s disease
b. Digoxin d. Buccal nerve Bilateral adrenal hyperplasia from an ACTH
c. Bisoprolol e. Facial nerve secreting pituitary adenoma.
d. Warfarin Ans. The key is C. Lingual Nerve. Peak age 30-50 years, male female ratio 1:1
e. Heparin A low dose dexamethasone test leads to no
Ans. The key is C. Bisoprolol (most likely its acute 1654. A 51yo woman complains of difficulty change in plasma cortisol but 8 mg may be
AF and 1st line is verapamil/bisoprolol, 2nd line swallowing and also reddish dots on her skin. A enough
digoxin/amiodarone) pic of her hand is seen. What is the most
appropriate term for the condition you would 1658. Which finding, on clinical examination of
***1651. A 59yo man has shown a change in his expect to see? the pulse, suggests a diagnosis of hypertrophic
mood and personality over a 9m period. He has a. Sclerodactyly obstructive cardiomyopathy (HOCM)?
subsequently developed difficulty with memory b. RA a. Irregularly irregular pulse suggesting A-fib
and conc, and then progressive fidgety c. Swan neck deformity b. Pulsusalternans
movements of his limbs and facial musculature. d. Polydactyly c. Pulsusbigeminus
By the time of medical assessment he has frank e. Ulnar deformity d. Pulsusbisferiens
choreiform movements and a mini-mental state Ans. The key is A. sclerodactyly. [Patient is e. Pulsusparadoxus
exam of 21/30. Other exam is normal. He was suffering from crest syndrome, sclerodactyly its Ans. The key is D. Pulsusbisferiens.
adopted and therefore no information on his one of the features]. [Pulsusbisferiens, is a sign where, onpalpation of
famhx is available. He has 3 adult children (27, 30, the pulse, a double peak per cardiac cycle can be
33) of whom the 2 youngest are asymptomatic. 1655. A 37yo female working as a healthcare appreciated. Bisferious means striking twice.
However, the oldest son has recently been inv assistant in a nursing home comes to the ED with Classically, it is detected when aortic insufficiency
by the neurology dept for slightly erratic behavior complaints of severe itching all over her body. On exists in association with aortic stenosis,[1] but
and fidgety restless movements of both legs. asking she replies that she had applied cream may also be found hypertrophic obstructive
Based on the likely clinical dx, which one of the on the body of a resident in the nursing home cardiomyopathy].
following genetic patterns is most likely? who had similar itches. What is the mechanism of
a. AD inheritance with anticipation itching? 1659. A 60yo male is admitted with a 2d hx of
b. AD with variable penetrance a. Allergic reaction lower abdominal pain and marked vomiting. On
c. AR b. Inflammation of keratinocytes examination he has abdominal swelling, guarding
d. X-linked c. Allergic reaction developed due to use of and numerous audible bowel sounds. What is
e. Mitochondrial disorder topical steroid creams the likely dx?
Ans. The key is A. AD inheritance with d. Subcutaneous bleeding a. Gallstone ileus
anticipation. [Patient is suffering from e. None b. Ischemic colitis
Huntingtons disease and that is autosomal Ans. The key is A. allergic reaction. c. Large bowel obstruction
dominant with anticipation which means a genetic d. Sigmoid volvulus
disorder is passed on to the next generation, the 1656. A 65yo pt who had MI 1yr ago now comes e. Small bowel obstruction
symptoms of the genetic disorder become to the ED complaining that his neighbor is Key is D: sigmoid volvulus [History is not
apparent at an earlier age with each generation]. conspiring suggestive of ischemic colitis. There are no bowel
against him. When his son is asked, he denies it sounds heard in gallstone ileus. Small bowel
1652. A 35yo pt has been dx with schizophrenia. and also narrates that sometimes his father says obstruction causes central abdominal pain, So
He mimics the doctors and attendants – doing the that everybody in his office is always talking about given features with lower abdominal pain makes
same physical actions as them. What symptom him, which is not the case. What is the most it more likely to be sigmoid volvulus].
does this pt have? appropriate med?
a. Echopraxia a. TCA 1660. A 17-year-old boy is diagnosed with scabies.
b. Echolalia b. Clozapine Which of the following statements regarding
c. Perseveration c. Olanzapine scabies is correct?
d. Apraxia d. Lorazepam a. Is best treated by salicylate emulsion
e. Anosognosia Ans. The key is C. Olanzapine. [Schizophrenia b. It can be spread by a droplet infection
treated with olanzapine]. c. It causes itchiness in the skin even where there
Ans. The key is A. Echopraxia. [Echopraxia is the is no obvious lesion to be seen
involuntary repetition or imitation of another **1657. You suspect Cushing's disease in a 50yo d. It is caused by Staphylococcus aureus
person's actions. Similar to echolalia, which is the woman who has attended clinic with glycosuria, e. Typically affects the face
involuntary repetition of sounds and language. HTN Ans. The key is C. It causes itchiness in the skin
Echopraxia has long been recognized as a core and a suggestive body habitus. Initial inv point you even where there is no obvious lesion to be seen.
feature of Tourette syndrome, and is considered a towards a dx of Cushing's disease. Which of
complex tic, but it also occurs in autism spectrum the following findings would be against this dx? 1661. An anemic young man is found to have a
disorders, schizophrenia and catatonia]. a. A normal 8am cortisol macrocytosis of 90%. The most likely cause is?
a. Zieve’s syndrome d. Choriocarcinoma
b. Thalassemia minor e. Chorangioma 1667. A 58yo man complains of nose
c. Chronic renal disease f. Vasa previa disfigurement. He has a hx of facial erythema
d. IDA g. Subplacental abruption particularly of the
e. Folate def placenta cheeks and nose. Papules and pustules have been
f. Chronic liver disease h. Subchorionic abruption erupting at intervals over the last 10yrs. He
g. HUS placenta admits to a moderate regular consumption of
h. Cytotoxic chemotherapy i. Placenta accrete alcohol. Exam: noted to have rhinophyma. The
i. Phenytoin j. Placenta previa most likely dx is?
a. Eczema
Ans. The key is E. Folate deficiency. Ans. The key is A. Placental migration. [In 90% of b. Herpes simplex
[Zieve's syndrome is an acute metabolic condition pregnancies, an initial low lying placenta will be c. Epidermolysis bullosa
that can occur during withdrawal from prolonged pulled upwards by the growing uterus and assume d. Dermatomyositis
alcohol abuse. It is defined byhemolytic anemia a normal position in the upper segment. This e. Tinea versicolor
(with spur cells and acanthocytes), phenomenon is referred to as Migration]. f. Pemphigus vulgaris
hyperlipoproteinaemia (excessive blood g. Acne rosacea
lipoprotein), jaundice, and abdominal pain. The 1665. An elderly lady with COPD has chronic SOB. h. Malignant melanoma
underlying cause is liver delipidization]. She is listed for cataract extraction. What is the i. Psoriasis
anaesthetic of choice? j. Atopic dermatitis
1662. An association with HPV is a most a. Facial nerve block
characteristic feature of? b. Bupivacaine infiltration of the Ans. The keyis G. Acne Rosacea. [Acne Rosacea is
a. Torus peri-orbital skin characterised by recurrent episodes of facial
b. Exostosis c. IV midazolam flushing with persistent erythema, telangiectasia,
c. Pleomorphic adenoma d. Peribulbar acupuncture papules and pustules.It is a chronic acneform
d. Verruca vulgaris e. Peribulbar lignocaine disorder of the facial pilosebaceous glands with
e. Fibroma infiltration an increased reactivity of capillaries to heat,
f. Epulis fissuratum f. Topical xylocaine causing flushing and eventually
g. Mucocele g. IV alfentanil telangiectasia.Rhinophyma is an enlarged nose
h. Pyogenic granuloma h. Epidural anesthesia associated with rosacea which occurs almost
i. Parulis i. General anesthesia exclusively in men].
j. Ranula j. Retrobulbar xylocaine Inj
1668. A 60yo man who presented with metastatic
Ans. The key is D.Verruca vulgaris. [It is most Ans. The key is E. Peribulbar lignocaine. [The most adenocarcinoma of unknown source. He
commonly associated with warts or verruca used mode of anaesthesia in ophthalmic feild is developed
vulgaris]. peribulbarlignocaine]. rapidly progressive weakness of his arms and was
found to have a deposit of tumour in his
1663. For the following type of surgery what is the 1666. A 55yo chronic alcoholic with known cervical spine. This was emergently treated with
most likely agent that may cause post-operative hepatic cirrhosis has been on a heavy bout of radiation. He developed considerable nausea
infection -- aorto-iliofemoral reconstruction with alcohol the and vomiting during his therapy and at the end of
a Dacron vascular prosthesis? night before and was brought home by friends the course began to have bloody vomiting.
a. Proteus after falling several times in the pub. While being Following rescusitation with 6 units of blood,
b. E.coli taken up the stairs to his bedroom he falls down what is the next test of choice?
c. Bacteroides fragilis the flight of 5 steps but sustains no obvious a. Apt test
d. Staphylococcus aureus injuiry. His wife calls the ED the next day because b. Neck, chest, abdominal XR
e. Staphylococcus epidermis she could not rouse him in the morning. He is c. 24h esophageal pH probe test
f. C.perfringens brought in in a comatose state and both pupils d. CT abdomen
g. Pseudomonas aeruginosa appear dilated. Skull vault XR appears normal. e. US abdomen
h. Streptococcus fecalis a. Hepatic encephalopathy f. MRI abdomen
i. Streptococcus pneumonia b. Intracerebral hematoma g. Barium swallow
j. Brucella melitensis c. Brain stem injury h. Angiography
d. Extradural hematoma i. Nuclear scan
Ans. The key is D. Staphylococcus epidermidis. e. Chronic subdural hemorrhage j. Endoscopy
[Staphylococcus epidermidis is most common f. Despressed skull fx
cause of infections in prosthesis]. g. Vertibrobasilar ischemia Ans. The key is J. Endoscopy. [Whenever there
h. Acute subdural hematoma hemetemesis endoscopy should be carried out
1664. A primigravida in the 17th week of her i. SAH immediately if the patients condition allows or it
symptomless gestation is found, on US, to have j. Severe migraine attack should be delayed till resucitation ,Underlying
evidence cause for hemetemesis needs to be sorted out.
of placental tissue covering the cervical os. By the Ans. The key is H. Acute subdural hematoma. ( High dose radiation is a cause Ulceration and any
end of her pregnancy she is likely to develop? [Acute subdural hemotoma, typical history of active bleeders must be treated)].
a. Placental migration alcholics, falls and usually debilliated or elderly,
b. Uterine myoma hepatic cirrhosis increases coagulopathy and
c. Uterine rupture chances for bleed].
1669. A pt has fine nail pitting, small yellow- 1671. A pt being sedated with fentanyl develops c. Escherichia coli (Gram -ve)
brown areas of discoloration in the nailbed severe respiratory depression. This is best d. Haemophilus influenza
involving the nails on both hands. These findings a reversed e. Legionella pneumophila
re commonly associated with? using? f. Strep pneumococcus
a. Yellow nail syndrome a. Ethanol g. TB
b. Leukonychia b. Naloxone h. Mycoplasma pneumonia
c. Onychomycosis c. Phyostigmine i. PCP
d. Lichen planus d. Atropine j. Staph aureus
e. Pellagra e. Methylene blue
f. Thallium toxicity f. Diphenhydramine Ans. The key is H. Mycoplasma pneumonia.
g. Contact dermatitis g. Calcium disodium ethylene [Inability to respond to a seven day course of
h. Zinc deficiency diamine tetra-acetic acid amoxicillin suggests atypical pneumonia, patchy
i. Hypoalbuminemia h. Deferoxamine mesylate shadows throughout lung fields and cold
j. Psoriasis i. Flumazenil agglutination points towards mycoplasma].
j. Folic acid
Ans. The key is J. Psoriasis. [Characteristic nail 1674. An 18yo male works in a company where
changes include pitting, discolouration,subungual Ans. The key is B. Naloxone. [Opioid Antagonist, lunches are often catered. One day, the water at
hyperkeratosis, crumbling of the nail plate, and reverses the effects of fentanyl, though it has to the company facility is not working, but they
onycholysis]. be administeres for a longer period of time due manage to have the lunch anyway. 2wks later, he
long half life of fentanyl]. becomes sick. He develops anorexia, nausea,
1670. A young man develops nonfluent, effortful malaise and jaundice. During the course of the
speech with dysarthria. He is able to undertsand 1672. A pt presented with the following blood next 4wks, 7 people who shared in the lunch
speech. He fails to repeat the sentence. What work, become ill with similar symptoms. After a few
would you do next? MCV: Decreased wks, each of the 7 people completely recovers
a. XR skull Serum ferritin: Decreased Total and they replace their caterer. What is a likely dx?
b. Non-contrast CT brain iron binding capacity: Increased a. Pancreatic ca b. Hemochromatosis
c. Contrast CT brain Serum iron: Decreased c. Laennec’s cirrhosis
d. Contrast MRI optic nerves Marrow iron: Absent. d. Hep A
e. 4-vessel cerebral angiogram What is your dx? e. HCC
f. Single vessel cerebral a. Thalassemia trait f. Rotor’s syndrome
angiogram b. Hypoparathyroidism g. Primary biliary cirrhosis
g. Cerebral angiography c. Hereditary sideroblastic anemia h. Gilbert’s syndrome
h. MRI frontal lobe d. Protein energy malnutrition i. Hep B
i. MRI pituitary gland e. Chronic renal failure j. Hemolysis
j. MRI temporal lobe f. Anemia of chronic disease
g. Acute blood loss Ans. The key is D. Hepatitis A. [Symptoms of
Ans. The key is H. MRI Frontal lobe. (Brocas area). h. IDA Hepatitis A range from mild nauseas to liver
[Production (Broca's) dysphasia/aphasia - lesions i. Oral contraceptives failure (very rare).Spread is normally by the
are located in the left pre-central areas. This is a j. Megaloblastic anemia faecal-oral route although there are occasional
non-fluent or expressive aphasia since there are outbreaks through food sources.Hand washing
deficits in speech production, prosody and Ans. The key is H. Iron deficiency Anemia (IDA). and good hygiene around food and drink prevent
syntactic comprehension. Patients will typically [S/S pallor, koilonychia,angular cheilitis, atrophic spread of infection.Increasing age is a direct
exhibit slow and halting speech but with good glossitis, IN marked Anemia ( Cardiac determinant of disease severity].
semantic content. Comprehension is usually good. enlargemnet,Flow Murmurs,ankle oedema and
Unlike Wernicke's aphasia, Broca's patients are heart failure) 1675. A 35yo 1st time donor suddenly passes out
aware of their language difficulties. Prosody is the Inv: FBC : shows microcytic hypochromic anemia, as she is donating blood. Which of the following
study of the meter of verse. Here it means the Serum ferrtitin Level reduced, normal 12-15 steps would be least useful in managing this
rhythm of speech. mcg/L, ( serum ferritin is falsely raised during adverse event?
Sensory (Wernicke's) dysphasia/aphasia - lesions infections), Anisocytosis and poikilocytosis. Total a. Ensure donor is adequately hydrated and has
are located in the left posterior perisylvian region iron binding capacity is increased. not skipped a meal
and primary symptoms are general Treatment: Iron supplememtation with B12 and b. Elevating the donor's legs as this is usually due
comprehension deficits, word retrieval deficits folic acid]. to a vasovagal syncope
and semantic paraphasias. Lesions in this area c. Haemoglobin of the donor meets the minimum
damage the semantic content of language while 1673. A 20yo prv healthy woman presents with requirement for donation
leaving the language production function intact. general malaise, severe cough and breathlessness d. The donation is usually continued along with
The consequence is a fluent or receptive aphasia which has not improved with a seven day course simultaneous normal saline infusion.
in which speech is fluent but lacking in content. of amoxycillin. There is nothing significant to e. The donor should be encouraged to mobilise
Patients lack awareness of their speech find on examination. The x-ray shows patchy after they have recovered
difficulties. Semantics is the meaning of words. shadowing throughout the lung fields. The blood
Semantic paraphrasia is the substitution of a film shows clumping of red cells with suggestion Ans. The key is C. Haemoglobin of the donor
semantically related but incorrect word]. of cold agglutinins. meets the minimum requirement for donation. [If
a. Mycobacterium avium complex syncope then we can continue donation giving
b. Coxiella burnetii simultaneous normal saline to the donor.(option
D). Option C. Haemoglobin of the donour meets Ans. The key is C. Diuresis renography. most frequently affected tissues. PURPURA,
the minimum requirement for donation[as it is LIVEDOID, SUBCUTANEOUS NODULES and
routinely done in every doner prior to donation 1679. Jean is a 72yo woman with recurrent bowel NECROTIC ULCERS. Neurologically,
and therefore this has no impact on syncope!!]. cancer following a hemi-colectomy 2y ago. She is MONONEURITIS MULTIPLEX>...involvemnet of
known to have both local recurrence and liver CNS, GIT,kidneys and heart means higher
1676. An infant is being examined as part of a mets and her pain has been under control on MST mortality.RENAL INVOLVEMENT: hypertension,
routine examination. The child can hold its head 90mg bd. She has had quite severe pain in the Renal failure, GIT:necrosis,perforation.Myalgia].
up and RUQ for the past hour despite having taken her
lifts its chest off a table. He has a palmer and normal dose of MST. You find that she has an 1682. A patient with chronic neutropenia
rooting reflex as well as a social smile. He is not enlarged liver which is hard and irregular. There is develops a chronic cough. A CXR reveals a
afraid of strangers. What is the most likely age of marked localised tenderness over the right lobe of cavitating
this child? her liver. Her abdomen is otherwise soft and intrapulmonary lesion containing a movable
a. neonate non-tender and the bowel sounds are normal. She rounded ball lesion. A likely dx is?
b. 2 months is apyrexial. The tx of choice would be? a. Tuberculosis
c. 6 months a. Oral NSAIDs b. Bronchiectasis
d. one year b. TENS c. Cystic fibrosis
e. one and a half years c. radio therapy to the liver d. Pulmonary hemosiderosis
f. two years d. IM diamorphine e. Mitral stenosis
g. four years e. Paracetamol f. Aspergillosis
h. seven years f. Prednisolone g. Wegener’s granulomatosis
i. ten years g. Physiotherapy h. Goodpasture’s syndrome
j. fourteen year h. epidural anaesthetic i. Pulmonary embolism
i. Pitocin j. Non-SCLC
Ans. The key is C. 6 months. j. Aspirin
Ans. The key is F. Aspergillosis. [Mostly affects
1677. A mother is concerned because her 1m boy Ans. The key is D. IM diamorphine. people with reduced immunity, reduced
has a swelling in his scrotum. He was born neutrophil count is also predilection for
prematurely. On examination the swelling is seen 1680. Titubation is a feature of disease involving aspergillosis].
to transilluminate. The likely cause is? the?
a. Lymphogranuloma Venereum a. Cerebellum 1683. A mother brings her 1yo infant to her
b. Testicular Torsion b. Basal ganglia pediatrician. She describes that following a
c. Hydrocele c. Corpus callosum common cold
d. Epididymitis d. Pons her child's voice has become hoarse and has
e. Seminoma e. Temporal lobe developed a cough that sounds harsh and brassy
f. Mature teratoma f. Occipital lobe and was worse at night. Exam: the child was
g. Varicocele g. Optic chiasma noted to have trouble drawing air into its lungs
h. Lymphoma h. 3rd ventricle between coughs and had trouble drawing air into
i. Orchitis i. Hypothalamus its lungs. There was visible stridor on
j. Spermatocele j. Pituitary gland inhalation. The cause is most likely to be?
a. EBV
Ans. The key is C. Hydrocele. [Transilluminating Ans. The key is A. Cerebellum [Titubations b. Rhinovirus
scrotal swelling is likely a case of hydrocele. 1-2% (head/body nodding) mostly occur due to c. Parainfluenza
neonates present with congenital hydrocele cerebellar lesions]. d. Flavivirus
which disappears by 1-2 years]. e. HIV
1681. A 50yo farmer complains of pain in his left f. Rotavirus
1678. A 2m girl has an ante-natal diagnosis of arm. Exam: he appears to have a neuropathy g. CMV
right hydronephrosis. Postnatal serial US exams affecting isolated nerves in multiple, random h. Kemerovo
revealed areas of his left arm. He also has a palpable i. Creutzfeld-Jacob
increasing dilatation of the right pelvicalyceal purpura and tender nodules on both of his upper j. Rubella
system. No reflux was demonstrated on a MUCG. and lower limbs. A likely diagnosis is?
Appropriate management should include? a. Carpal tunnel syndrome Ans: Parainfuenza. [Parainfluenza virus causes
a. Surgical repair b. Polyarteritis nodosa croup. A rough barking cough with hoarsness and
b. Intermittent catheterization c. Angina Pectoris wheezing, labored breathing, runny nose, fever,
c. Diuresis renography d. Gout cough, decreased appetitie are common
d. Anticholinergic agents e. Cellulitis features].
e. Phenylpropanolamine f. Rheumatoid arthritis
f. Gellhorn pessary g. Erysipelas Investigation: Viral culture of secretions.
g. Biofeedback-assisted behavioral h. Fascitis Treatment: Symptoatic supportive treatment
treatment i. Reiter's Syndrome Antibiotics incase of secondary infection.
h. Oral Estrogen therapy j. Polymyalgia Rheumatica
i. Vaginal Estrogen therapy 1684. INR:Normal, APTT:Elevated, Thrombin
j. Ring pessary Ans. B. Polyarteritis nodosa. time:Elevated, Plt count:Normal, Bleeding time:
[Presentaion:Peripheral nerves and skin are the Normal.
A likely aetiology is? pain. Other s/s are suggestive of AML short Hx, experience visual changes with activity or
a. Waldenström's wbc 100000, petechiae, with lymphadenopathy, movement. The patient reported continued
macroglobulinaemia very high blast cell count and decreasing vision. Goldmann visual fields were
b. Heparin hepatosplenomegaly. Clinically it is very difficult done and showed a central scotoma. A MRI was
c. Sézary cell leukaemia to differentiate between CML and ALL. done at this time and showed inflammation of the
d. Pelger-Hüet anomaly Lymphocytic means it develops from early left optic nerve. A likely diagnosis is?
e. von Willebrand's disease (immature) forms of lymphocytes, a type of white a. Pseudotumor
f. Haemophilia blood cell. This is different from acute myeloid b. Orbital teratoma
g. HIV infection leukemia (AML), which develops in other blood c. Optic neuritis
h. DIC cell types found in the bone marrow]. d. Sarcoidosis
i. Acanthocytosis e. Optic glioma
j. Vit K deficiency 1687. A 63yo male has anal canal carcinoma with f. Lymphangioma
no evidence of spread to the pelvic wall, pelvic g. Rhabdomyosarcoma
Ans: Heparin muscles or lymph nodes. This is typically managed h. Retinal vascular shunts
by? i. Retinoblastoma
1685. An infant has diarrhea for 3d with weight a. Resection of the sigmoid colon j. Mucormycosis
loss from 10 kg to 9 kg. Exam: he is noted to have b. Right hemicolectomy
dry c. Left hemicolectomy Ans. The key is C. Optic neuritis [Possible diagnosis
mucous membranes, poor skin turgor, markedly d. Transverse colectomy is MS. Scotoma, 2yr Hx, age race are suggestive
decreased urine output, and tachycardia. His e. Internal sphincterotomy except more in females than males].
BP=normal and compression-release of the nail f. CT guided drainage
beds shows satisfactory refilling. Appropriate g. Diverticulectomy 1690. A pregnant woman in an early stage of
treatment would include? h. Transverse colostomy labour expresses the wish to have pain relief
a. Plasmapheresis and plasma i. Chemotherapy and during
infusion radiatherapy labour. The anesthetist describes that if the
b. 0.5% Normal Saline j. Abdominal perineal resection patient wishes he can use medication as a local
c. Lactated Ringer's injection anesthetic to block the pain sensations of labour.
d. Packed cells Ans. The key is I. Chemotherapy and radiotherapy. Into which space should the local anaesthetic
e. Whole blood [This is T1 N0 M0 that is stage 1 cancer. be normally injected?
f. Platelets (http://www.cancer.gov/types/anal/hp/anal- a. Anterior pararenal space
g. FFP treatment-pdq#section/_45). b. Aryepiglottic space
h. double strength Normal Saline c. Vestibule space
i. 5% dextrose in 0.5N saline BUT we don’t have tumor size here and sphincter d. Epidural space
solution information. However surgery can be avoided To e. Sub-arachnoid space
j. IV heparin preserve sphincter. Chemo radiation is preferred f. Space of Disse
ohcm 633; 9th ed]. g. Middle ear
Ans. The key is C. Lactated Ringer's injection. h. Posterior pararenal space
1688. A 2m baby develops a life-threatening i. Supraglottic space j. Lesser sac
1686. A 4yo boy has the sudden onset of bone anemia. Blood tests show a normal serum iron,
pain. He begins experiencing bleeding of his ferritin and TIBC. Hemoglobin electrophoresis Ans. The key is D. Epidural space.
gums and frequent bloody noses. His mother reveals a markedly decreased Hemoglobin A
takes him to his pediatrician. Exam: he is pale and content and an increased hemoglobin F content. 1691. A 29yo Afro-Caribbean man presents with a
has This baby's anemia is likely to be secondary to? non-productive cough mild aches in the ankles.
numerous petechiae over his body, with a. Failure of alpha chain production The symptoms have been present for 2m. His ESR
lymphadenopathy and hepatosplenomegaly. He b. Failure of beta chain production is elevated. Ca: 2.69 mmol/l; PO4 -: 1.20 mmol/l;
has c. Deficiency d. Lead poisoning of ALP: 80 iu/L. Serum 25(OH) D: 180 nmol/l
WBC=100,000mm and numerous circulating blast B12
cells. He is admitted to the hospital. A bone e. IDA Normal values for Calcium: 2.12-2.65mmol/l;
marrow biopsy=35% blast cells. Which of the f. Presence of hemoglobin S Phosphate: 0.8-1.45mmol/l; ALP 30-300iu/L;
following is most likely? g. Presence of hemoglobin M Serum 25(OH) D: 20-105nmol/l; Urea: 2.5-
a. Mantle cell lymphoma h. Deficiency of folate 6.7mmol/l; Creatinine: 70-120μmol/l
b. Infectious lymphocytosis i. Bone marrow failure
c. Waldenstrom’s j. Inability to manufacture heme a. Osteoporosis
macroglobulinemia b. Thiazide diuretics
d. CML Ans. The key is B. Failure of beta chain production. c. Skeletal metastases
e. CLL d. Primary hyperparathyroidism
f. Burkitt lymphoma 1689. A 30yo caucasian man presented with a e. Hypoparathyroidism
g. ALL 2wk hx of gradually worsening vision in his left f. Osteomalacia
h. Mycosis fungoides eye. The patient had been seen once by a g. Multiple myeloma
i. Hairy cell leukemia neurologist 2yrs prv for flashes. At that time a h. Paget's disease of bone
j. AML head CT was normal. The patient was lost to i. Sarcoidosis
Ans. The key is AML. [THIS IS AN ACUTE follow up with the neurologist, but the flashes had j. Hyperthyroidism
EMERGENCY IN AML. Leukostatsis causing bone continued for the 2yr period. The patient did not
Ans. The key is I. Sarcoidosis. b. Impaired venous drainage • Can be difficult to diagnose but should
c. Hgb SS disease be suspected whenever a child presents with a
1692. A 22yo has had recent chickenpox. He now d. Steroid use limp or fails to bear weight on the leg.
presents with confusion. He is noted to have low e. Alcoholism • Treatment consists of immobilisation for
urine output and large petechiae all over his body. f. Gaucher's disease a few weeks to protect the limb and to relieve
CXR: a large patch of consolidation is seen. The g. missed fracture pain.
management of choice should be : h. Cushing's disease • Subperiosteal bone formation is usually
a. Ventilatory support i. Radiation apparent on X-rays by two weeks].
b. Open surgical debridement j. Vasculitis
c. Resection of superficial 1698. Which one of the following
petechiae with wide margin Ans. The key is F. Gaucher’s disease. [Couldn’t get electrocardiographic changes is found in
d. Booster vaccine much reliable info about this question!] hypercalcaemia?
e. TENS a. Increased QRS interval
f. Lontophoresis b. Prolonged Q-T interval
g. Nephrostomy 1696. A 7yo boy with frequent episodic asthma is c. Short P-R interval
h. Oral Corticosteroids on tx with sodium cromoglycate. His physician d. Short Q-T interval
i. Brivudin wants Ans. The key is D. Short Q-T interval. [Short Q-T
j. IV acyclovir to add a non-steroid preventer. The mother of the interval secondary to a shortened ST segment].
boy, a teacher, has just read about a nonsteroidal
Ans. The key is J. IV acyclovir. medication which acts on the mast cells, stopping 1699. An elderly male pt with prior hx of
1693. A young girl with a psychiatric hx on med tx them from releasing harmful hematemesis is having hx of long term use of
is brought to the dermatologist by her mother chemicals. Her physician agrees to add this aspirin and
because of recurrent patchy hair loss. Exam: the medication to the boy's drug regimen. Which other drugs, now presents with severe epigastric
hair shafts revealed twisting and fractures. This medication is the physician most likely to add to pain, dysphagia and vomiting. He was
suggests the following pathology: the boy's treatment? connected to vital monitors which were not
a. Infection with Trichophyton a. Inhaled short acting reassuring. What is the management?
tonsurans bronchodilator a. Oral antacids
b. Infection with Microsporum b. SC adrenaline b. IV PPI
canis c. Nedocromil Sodium c. Oral PPI
c. Alopecia areata d. Inhaled long acting d. Endoscopy
d. Telogen Effluvium bronchodilator e. Analgesia
e. Androgenetic Alopecia e. Inhaled sodium cromoglycate Ans. The key is D. Endoscopy. [Long term use of
f. Lichen planus f. Inhaled steroids an nsaid predisposes to peptic ulcers. There may
g. Traction Alopecia g. Inhaled SABA cause considerable bleeding leading to shock].
h. Alopecia totalis h. Oral steroids
i. Trichorrhexis nodosa i. Nebulised bronchodilators 1700. A 68yo man presents with bruising and hx
j. Trichotillomania j. Oral theophylline of falls. He is found to have a mask-like face,
Ans. The key is C. Nedochromil Sodium. pillrolling tremor and shuffling gait. EEG=normal.
Ans. The key is J. Trichotillomania.   Which of the following conditions is he most likely
1697. A 3yo boy is playing with his brother when being treated for?
1694. Syphilis typically causes he falls. He cries immediately and refuses to walk. a. HTN
a. Lymphogranuloma Venereum His b. DM
b. Testicular Torsion mother carries him to hospital. He had a full term c. Psychosis
c. Hydrocele NVD with no neonatal complications. His d. TIA
d. Epididymitis immunisations are up to date. Exam: looks well e. Complex partial seizure
e. Seminoma and well-nourished, no dysmorphic features. He Ans. The key is C. Psychosis. [Antipsychotics can
f. Mature teratoma has slight swelling, warmth and discomfort on the lead to parkinsonism].
g. Varicocele lower 1/3 of the left tibia, and refuses to 1701. A 45yo woman presents with easy
h. Lymphoma weight bear. AP and lateral x rays of the tibia are fatigability, even on no exertion, chronic
i. Orchitis normal. What is the most likely dx? headaches and body aches and severe physical
j. Spermatocele a. Ankle fx and mental exhaustion. She has no underlying
b. Ankle sprain conditions and all inv are non-conclusive. What is
Ans. The key is I. Orchitis. c. Fibular fx the most likely dx?
d. Knee dislocation a. Somatization
1695. A middle aged woman has severe collapse e. Tibial fx b. Chronic fatigure syndrome
of the right femoral head requiring replacement. c. Polymyalgia rheumatic
The Ans. The key is E. Tibial fx. d. GCA e. Depression
removed femoral head is sent for pathology and is [Toddler's fracture Ans. The key is B. Chronic fatigue syndrome.
found to contain enlarged fat cells. The • Undisplaced spiral fractures of the tibial [Chronic fatigue syndrome (CFS) causes persistent
pathologist explains that this is the likely cause of shaft in children under 7 years old often follow fatigue (exhaustion) that affects everyday life and
the patient's femoral head collapse. A likely minimal trauma and may not be visible on initial doesn't go away with sleep or rest].
aetiology is X-ray.[8]
a. Septic emboli
1702. A 23yo male presents to his GP 2wks after a 1706. A 35yo man skidded on a wet road while
RTA concerned about increasing anxiety lethargy riding his motorbike at a speed of 70mph. He has
and headache. At the time he had a CT brain after a large hematoma on temporal scalp, some
banging his head on the steering wheel, which bruises on chest wall and abdomen and a
revealed no abnormality. 6m following this deformed thigh. GCS 11/15. High flow oxygen via
episode his symptoms have resolved. What did his mask given. Most immediate radiological inv
original symptoms likely represent? required during initial resuscitation phase?
a. Conversion disorder a. CXR
b. PTSD b. CT brain
c. Somatization disorder c. CT abdomen
d. GAD d. XR femur
e. Post-concussion syndrome Ans. The key is A. CXR. [CXR may reveal possible
chest structure trauma requiring urgent attention
Ans. The key is E. Post concussion syndrome. [Post which may be life saving while brain lesion need
concussion syndrome is a set of symptoms that time consuming CT and tx and fracture femur may
may continue for weeks, months, or a year or be managed taking more time].
more after a concussion – a minor form of 1707. A 4yo baby has a generalized tonic-clonic
traumatic brain injury]. convulsions and fever of 39C. His mother informs
1703. A 34yo man had a 4mm ureteric stone you that this has happened 3-4 times ebfore.
which he passed in urine. This time he presents What is the most probable dx?
withh 3cm stone in the right kidney. Single most a. Febrile convulsion
appropriate treatment? b. Absence seizures
a. No treatment c. Epilepsy
b. ESWL d. Partial complex seizure
c. Laparotomy Ans. The key is A. Febrile convulsion.
d. Observe
e. Operative stone removal
Ans. The key is E. Operative stone removal.
[Stones < 5mm: pass spontaneously, Inc fluid
intake
Stones 5mm-1cm /pain not resolving: medical
expulsive therapy---> Nifedipine or
Tamsulosin(and/or prednisolone)
Stones 1cm-2cm: ESWL or Ureteroscopy using
dormia basket
Stones > 2cm/large/multiple/complex:
Percutaneous nephrolithotomy].
1704. An 18m girl who has had single UTI is seen
in the OPD. She has fever and vomiting but these
improved with course of trimethoprim.
Subsequently, MCUG showed bilateral
vesicoureteric reflux. Single most appropriate
mgmt?
a. Prophylactic antibiotics
b. Reassure
c. No treatment
d. Ureteric surgery
Ans. The key is D. Ureteric surgery. This is wrong
key. Correct key is prophylactic antibiotics.
[Usually most of the cases of VUR (even though
bilateral) cures with advancing age with antibiotic
prophylaxis and only a minority need surgery].

1705. A 22yo says she has taken about 40 tabs of


paracetamol 3h ago. Her HR=110bpm,
BP=110/80mmHg and RR=22bpm. What’s the
initial management?
a. Activated charcoal
b. N-acetyl cysteine
c. Gastric lavage
d. Wait for 4h paracetamol level
Ans. The key is D. Wait for 4h paracetamol level.

You might also like